You are on page 1of 450

ACJC_H2Maths_2013_Promo_Qn 2

ACJC_H2Maths_2013_Promo_Soln 9
AJC_H2 Math_promo2013_Qn 23
AJC_H2 Math_promo2013_Soln 28
CJC_H2Math_2013_Promo_Qn 38
CJC_H2Maths_2013_Promo_Soln 43
HCI_2013_H2Maths_Promo_Qn 54
HCI_2013_H2Maths_Promo_Soln 61
IJC_H2Maths_2013_Promo_Qn 74
IJC_H2Maths_2013_Promo_Soln 80
JJC_H2Maths_2013_Promo_Qn 105
JJC_H2Maths_2013_Promo_Soln 110
MJC_H2_Maths_2013_Promo_Soln 121
MJC_H2Maths_2013_Promo_Qn 133
NYJC_H2Maths_2013_Promo_Qn 141
NYJC_H2Maths_2013_Promo_Soln 148
RVHS_H2Maths_2013_Promo_Qn 158
RVHS_H2Maths_2013_Promo_Soln 166
TJC_H2Math_promo_Soln 180
TJC_H2Math_promo2013_Qn 203
VJC_H2Maths_2013_Promo_Qn 208
VJC_H2Maths_2013_Promo_Soln 214

sgfreepapers.com 1
ANGLO-CHINESE JUNIOR COLLEGE
MATHEMATICS DEPARTMENT

MATHEMATICS
Higher 2 9740
Paper 1
8 October 2013
JC 1 PROMOTIONAL EXAMINATION
Time allowed: 3 hours
Additional Materials: List of Formulae (MF15)

READ THESE INSTRUCTIONS FIRST

Write your Index number, Form Class, graphic and/or scientific calculator model/s on the cover page.
Write your Index number and full name on all the work you hand in.
Write in dark blue or black pen on your answer scripts.
You may use a soft pencil for any diagrams or graphs.
Do not use paper clips, highlighters, glue or correction fluid.

Answer all the questions.


Give non-exact numerical answers correct to 3 significant figures, or 1 decimal place in the case of
angles in degrees, unless a different level of accuracy is specified in the question.
You are expected to use a graphic calculator.
Unsupported answers from a graphic calculator are allowed unless a question specifically states
otherwise.
Where unsupported answers from a graphic calculator are not allowed in the question, you are
required to present the mathematical steps using mathematical notations and not calculator commands.
You are reminded of the need for clear presentation in your answers.

The number of marks is given in brackets [ ] at the end of each question or part question.
At the end of the examination, fasten all your work securely together

This document consists of 7 printed pages.

Anglo-Chinese Junior College


H2 Mathematics 9740: 2013 JC 1 Promotional Examination
Page 1 of 7

sgfreepapers.com 2
ANGLO-CHINESE JUNIOR COLLEGE
MATHEMATICS DEPARTMENT
JC 1 Promotional Examination 2013

MATHEMATICS 9740
Higher 2 / 100
Paper 1

Index No: Form Class: ___________


Name: _________________________
Calculator model: _____________________

Arrange your answers in the same numerical order.


Place this cover sheet on top of them and tie them together with the string provided.

Question no. Marks


1 /2

2 /3

3 /4

4 /4

5 /5

6 /6

7 /6

8 /7

9 /6

10 /9

11 /9

12 / 11

13 / 13

14 /9

15 /6

Anglo-Chinese Junior College


H2 Mathematics 9740: 2013 JC 1 Promotional Examination
Page 2 of 7

sgfreepapers.com 3
1 The graph of y  f  x  undergoes, in succession, the following transformations:

Step 1: a translation of 1 unit in the negative y-direction; followed by


Step 2: a stretch with scale factor 2 parallel to the x-axis.
3
The equation of the resulting curve is y  ln  2 x  3 , x   . Determine the equation
2
of the graph, y  f  x  . [2]

2 Given that the curve y  ax 3  bx 2  cx  d has turning points at (-4, 258) and (4, 2).
Write and solve a system of simultaneous linear equations satisfied by the constants
a, b, c and d . [3]

3 Differentiate the following with respect to x.

 x
(i) cos 1   , [2]
2

 x  1
3

(ii) ln . [2]
x2  1

4 Find the following integrals:


1
(i) x ln x
dx ; [2]

e2 x
(ii)  4  e 4 x
dx . [2]

3x 2  6 x  10
5 Without the use of a graphing calculator, solve the inequality  2. [3]
x 2  3x  4
3x 2  6 x  10
Deduce the range of values of x such that  2. [2]
x2  3 x  4

[Turn Over

Anglo-Chinese Junior College


H2 Mathematics 9740: 2013 JC 1 Promotional Examination
Page 3 of 7

sgfreepapers.com 4
6 A curve C has parametric equations
x  1  cos , y    sin  ,
where 0    2 ,
dy 1
(i) Show that  cot  and find the gradient of C at the point P where    . [3]
dx 2

(ii) The tangent at P meets the y-axis at A. The tangent at the point Q, where   ,
2
meets the y-axis at B. Find the area of triangle ABP. [3]

7 A right pyramid block has a square base ABCD and its vertical height VM is
( a  x ) where 0  x  a . M is the point where the diagonals AC and BD of the square
meet. This right pyramid block is inscribed in a sphere of fixed radius a so that the
vertices V, A, B, C and D of the block just touch the interior of the sphere with the
vertical height VM passing through the centre O of the sphere.

(i) Show that the length of the side of the square base ABCD is 2( a 2  x 2 ) . [2]

(ii) Hence, find the maximum volume of the block in terms of a. [4]

1
[Volume of a pyramid =  base area  height ]
3

O
B

A C
M

Anglo-Chinese Junior College


H2 Mathematics 9740: 2013 JC 1 Promotional Examination
Page 4 of 7

sgfreepapers.com 5
1
8 The function f is defined by f : x  x  for x  , x  1 .
x
(i) Find f 1  x  and state the domain of f 1 . [3]

(ii) Find fff 1  x  and state its domain and range. [3]

(iii) Show that the composite function f 2 exists. [1]

1 4(k  1)
9 If f  k   , show that f ( k )  f ( k  2)  . [1]
k2 k (k  2) 2
2

2 3 4
Hence, show that the sum to n terms of the series  2 2  2 2   is
        
2
1 3 2
2 4 3 5

15 1 1 
   . [3]

4  4 n  1 2
n  22 

n
k 1 13
Show that  k k  2
k 2
2 2

144
for all values of n  2 . [2]

Use integration by parts to find the exact value of 1  ln x  dx .


e 2
10 (a) [4]

(b) By means of the substitution x  3cos 2   7 sin 2  , where 0    , prove that
2
7 1
3 dx   . [5]
 7  x  x  3 

11 y y  sin x

A2

A1 y  cos x

x
O 
2 
The region bounded by the axes and the curve y  cos x from x  0 to x  is divided
2
into two parts, of areas A1 and A2 , by the curve y  sin x (see diagram). Prove that

A1   2 A . 2 [5]

1
The line y  meets the curve y  sin x and the y-axis at P and Q respectively. The
2
region OPQ, bounded by the arc OP and the lines PQ and QO, is rotated through 4 right
angles about the x-axis to form a solid of revolution of volume V. Find the exact value
of V in terms of  . [4]
Anglo-Chinese Junior College [Turn Over
H2 Mathematics 9740: 2013 JC 1 Promotional Examination
Page 5 of 7

sgfreepapers.com 6
12 The diagram shows the graph of y  f  x  . The curve crosses the axes at the points

 2a , 0  and  0, 2b  . The asymptotes are x  a and y  b . The gradient of the curve at

the point  0, 2b  is 1. y xa

2b
b yb

O a 2a x

On separate diagrams, sketch the graphs of


1
(i) y , [3]
f  x

(ii) y2  f  x , [3]

(iii) y  f  x , [2]

(iv) y  f  x , [3]

giving the equations of any asymptotes and the coordinates of any points of intersection
with the x- and y-axes.

π 
13 (a) In triangle ABC, angle A =     radians, AB = AC = b and BC = a.
2 
a cos 
Show that  . [1]
b    2 
sin  
 4 
  2 
Deduce, for small values of  , a  2b 1   . [4]
 2 8 
1
(b) Given that y  esin 4 x , show that
dy
(i) 1  16 x 2  4y , [1]
dx
2
(ii) 1  16 x  2  16 x
2 d y dy
 16 y . [2]
dx dx
By further differentiation of the result, find the Maclaurin series for y up to and
including the term in x3 . [3]

 7
By choosing a suitable value of x, show that e 6  . [2]
12

Anglo-Chinese Junior College


H2 Mathematics 9740: 2013 JC 1 Promotional Examination
Page 6 of 7

sgfreepapers.com 7
n
1
14 (a) Prove by induction that  (2r  1)
r 1
2
 n(2n  1)(2n  1) .
3
[4]

(b) Use the result in part (a) to


30
(i) evaluate  (2r  3)
r 1
2
, [2]

n
1
(ii) prove that r 2
 nn  12n  1 . [3]
r 1 6

15 A man met with an accident and went into a coma on 10th January 2013. As a result, he
did not pay the bank the outstanding balance of $M for his credit card bill when it is due
for payment on 27th January 2013. On the 27th of each month when the payment for the
credit card bill is due, the bank will charge a 2% interest on any outstanding balance that
is unpaid. After the 2% interest has been added, the bank will still charge an additional
late payment charge of $L monthly.
(a) Express in terms of L and M, his outstanding balance on his credit card on
1st February 2013. [1]
(b) If the man still remains in coma exactly n months later on the day he met with an
accident, show that the accumulated outstanding balance on the man’s credit card is
1.02n M  50 L(1.02n  1) . [3]
(c) Given that M  1000 and L  55 . Find the least value of n when the accumulated
outstanding balance on his credit card first exceeds $2010. [2]

~ End of Paper ~

Anglo-Chinese Junior College


H2 Mathematics 9740: 2013 JC 1 Promotional Examination
Page 7 of 7

sgfreepapers.com 8
Anglo-Chinese Junior College
H2 Mathematics 9740
2013 JC 1 PROMO Solution
Qn Solution
3
1 y  ln  2 x  3 , x  
2
Before Step 2: y  ln  2  2 x   3  ln  4 x  3
Before Step 1: y  ln  4 x  3  1
OR
1 
Resulting curve: y  f  x   1  ln  2 x  3
2 
1   1  
 f  x   ln  4  x   3  1
2   2  
 y  f  x   ln  4 x  3  1
2 Given y  ax 3  bx 2  cx  d
dy
 3ax 2  2bx  c
dx
dy
When x  4,  0, 3a(4) 2  2b(4)  c  0
dx
48a  8b  c  0 (1)

dy
When x  4,  0, 3a(4) 2  2b(4)  c  0
dx
48a  8b  c  0 (2)

When x  4, y  258,


a (4)3  b(4) 2  c(4)  d  258
64a  16b  4c  d  258 (3)

When x  4, y  2,
a(4)3  b(4) 2  c(4)  d  2
64a  16b  4c  d  2 (4)

Using G.C. a  1, b  0, c  48, d  130.


3i 1
d  1  x 
 1  1  x   2 1 1
 cos      cos    
dx   2 2  2  x
2 2
1  
2
1

x
2  4  x  cos
2 1
 
2

sgfreepapers.com 9
d  
3ii
 x  1
3
  d  ln  x  1  
x 2  1  dx   x  1  
ln
dx 
 
d  1 
  ln  x  1  ln  x  1 
dx  2 
1 1
 
x  1 2  x  1

Alternative solution:
d   x 1  3 x 1  x2 1  x 1  2x
3 2 3
1 1 1
ln 2   
dx  x 1   x 1 2  x 1  x 1
3 3 2 2

 
x2 1 x2 1
x 3
 2
2 x 1

4(i) 1
x ln x
dx

1
 
 x  dx 1
  f ( x)  f ( x)   c
n 1

n
using f '( x)dx 
ln x n 1
 2 ln x  c

e 2 x
(ii)
 4  e 4 x
dx

1 2e 2 x f '( x)  f ( x) 
  dx using  dx  sin 1  c
2 22  e 2 x 
2
a 2   f ( x) 
2
 a 

1  e 2 x 
  sin 1  c
2  2 

sgfreepapers.com 10
5 3 x 2  6 x  10
2
x 2  3x  4
3 x 2  6 x  10
20
x 2  3x  4
3 x 2  6 x  10  2 x 2  6 x  8
0
x 2  3x  4
x2  2
0
 x  4  x  1
 x  2  x  2   x  4 x  1  0
+ - + - +
-4  2 1 2

x  4 or  2  x  1 or x  2

3x 2  6 x  10
2
x2  3 x  4
3 x  6 x  10
2

2
x 3 x 4
2

x  4 (n.a.) ;  2  x  1 ; x 2
1  x  1 or x   2 or x  2

6i
x  1  cos y    sin 
dx dy
 sin   1  cos 
d d

dy 1  cos 

dx sin 

2 cos 2
 2
 
2 cos sin
2 2

cos
 2

sin
2

 cot
2

dy
When    ,  0.
dx

sgfreepapers.com 11
ii) Coordinate at A(0,  )

 dy
When   , 1
2 dx
 
y  sin
2 2 1

Equation of tangent: x  1  cos
2

y  x
2
 
Coordinate at B  0, 
 2

 area of triangle ABP


1  
      2
2  2


2

7i) Diagonal DB  2 a 2  x 2

Length of side of square


 
 sin   2 a 2  x 2
4
2
 2 a2  x2
2
 2  a2  x2 

ii) Volume of block , v 


3
 a  x2   x  a 
2 2

Diff. w.r.t.x
  a 2  x 2    x  a  2 x  
dv 2
dx 3
2
  a  x  a  x    x  a  2 x  
3
2
  x  a  a  3 x 
3

dv
For stationary point, 0
dx

2
0  x  a  a  3x 
3
a
x   a (n.a.) x
3

sgfreepapers.com 12
d 2v 2
  x  a  3   a  3x  
dx 2 3 
d 2v a
2
 0 when x 
dx 3

Max. volume of block ,


2  2  a   a  
2

v   a       a
3 
 3   3  
64a 3
 units 3
81

1
8 f :x  x for x  , x  1
(i) x
1
Let y  x   x 2  yx  1  0
x
y  y2  4 y  y2  4
x or x 
2 2
(rejected since x  1 & y  2)
x  x2  4
f 1  x  
2
Df 1   2,  
1
8 fff 1  x   f  x   x 
(ii) x
Domain of fff 1  Df 1  Rf   2,  
5
Range of fff 1  f ( x) : x   2,     ,  
2
1
8 f :x  x for x  , x  1
(iii) x
D f  1,   , R f   2,   Since R f  D f , ff exists.
9 1
Given f (k ) 
k2
f (k )  f (k  2)
1 1
 2
k (k  2) 2
(k  2) 2  k 2

k 2 (k  2) 2
(k 2  4k  4)  k 2

k 2 (k  2) 2
4(k  1)
 2
k (k  2) 2

sgfreepapers.com 13
2 3 4 n 1
 2 2  2 2   2
2
2
(1) (3) (2) (4) (3) (5) (n) (n  2) 2
n
k 1

k 1 k ( k  2)
2 2

1 1
n
1 
  2  
k 1 4  k (k  2) 2 
1 n 1 n
1 
  2   2 
4  k 1 k k 1 ( k  2) 

1 1 1 1 1 1 1 
  2  2  2  2  2   2 
4 1 2 3 4 5 n 
1 1 1 1 1 1 1 
  2  2  2   2   
43 4 5 n (n  1) (n  2) 2 
2

1 1 1 1 1 
  2 2  
4  1 2 (n  1) (n  2) 2 
2

15 1 1 
    
4  4 (n  1) (n  2) 2 
2

n
k 1
k
k 2
2
(k  2) 2
n
k 1 2
  2 2
k 1 k ( k  2)
2 2
(1 )(3 )
15 1 1  2
    
4  4 (n  1) (n  2) 2  9
2

13 1  1 1 
    
144 4  (n  1) (n  2) 2 
2

13 1 1
 (  0 and  0 n    )
144 (n  1) 2
(n  2) 2

e  2 ln x 
e
1  ln x  dx   ln x   x  1  1 x  x  dx
e 2 2
10
(a)  
 e  2 1  ln x  dx
e

 e 1 
 e  2   ln x  x  1  1 x   dx 
e

 x 
 e  2e  2  e  1  e  2
10 x  3cos 2   7 sin 2 
(b)
dx
 6 cos    sin    14sin  cos 
d
 8sin  cos 
when x  3, 3cos 2   7 sin 2   3  sin 2   0    0

when x  7, 3cos 2   7 sin 2   7  cos 2  0   
2

sgfreepapers.com 14
7 1
3 dx
 7  x  x  3 

8sin  cos 
 02 d

 7  3cos 2   7 sin 2  3cos 2   7 sin 2   3 
  

1
 02  8sin  cos   d
 
 4 cos 2   4sin  

2


 
 2 02 1d  2     (proved)
2
   
11 A1   0
4
 sin x  dx    cos x  dx    cos x 
2 4
0
 sin x 2
4 4

 2   2
    1  1  
 2   2 
 2 2

 
A2   0
4
 cos x  sin x  dx  sin x  cos x 04
 2 2
      0  1
 2 2 
 2 1
OR
 
A2   0
2
 cos x  dx  A1  sin x 02   2  2 
 1  0   2  2  
 2 1
OR
2
A2  0 2
sin y  dy    cos y  dy
1 1
2
1

 A1  2  2  2  
2  1  2 A2 (proved)
 1
P , 
 6 2

1 
2
V      06  sin x  dx
2

2 6

2 

24

2
06 1  cos 2 x  dx

2 
sin 2 x  6
  x 
24 2  2  0
2   3 3  2
      
24 2  6 4  8 24

sgfreepapers.com 15
1
y y
12 f  x
(i)

1
y
b
 1 
 0, 
 2b 
O  a, 0  x

x  2a

12 y2  f  x
(ii) y xa

 0, 2b   2a, 0  y b

x
O
y b
 0,  2b 

12 y  f  x y
(iii)

 0,1

O y0 x

xa

12 yf x y
x  a xa
(iv)

 0, 2b 
y b

 2a, 0  O  2a, 0  x

13a) Sine rule

sgfreepapers.com 16
sin A sin B

a b
    
sin     sin   
2  4 2
a b
a cos 

b   
sin   
4 2

a cos 

b   
sin   
4 2
cos 

   
sin cos  cos sin
4 2 4 2
cos 

2  2 
cos  sin
2 2 2 2
For small values of 
2
1
a 2

b    2

 
2   2   2  
1
2  2  2  2 
 
 
2
1
 2
2    2 
1    
2   2 8 

1
  2     2 
a  2b 1   1    
 2   2 8 

  2     2   1 2     2  
2

 2b 1    1   1        ... 
 2 
 2 8  2 2 8  
  2   3 2 
 2b 1   1   
 2  2 8 
  2 
 2b 1   
 2 8 

13b)

sgfreepapers.com 17
1
i) y  esin 4x

ln y  sin 1 4 x
diff. w.r.t x
1 dy 4

y dx 1  16 x 2
dy
1  16 x 2  4y
dx
ii)
dy
1  16 x 2  4y
dx
diff. w.r.t x
d 2 y dy  1 1

1  16 x 2 2
 
dx  2
1  16 x 
2 2
 32 x    4
dy
dx  dx
d2y 16 x dy dy
1  16 x 2 2
 4
dx 1  16 x 2 dx dx
2

1  16 x  dx 2  16 x dy
2 d y

dx
 4 1  16 x 2
dy
dx
2

1  16 x 2  ddx y2  16 x dy
dx
 16 y

1  16 x  ddx y  16 x dy
2

dx
 16 y
2

diff. w.r.t x
3 2 2

1  16 x 2  ddxy3   32 x  ddx y2  16 x ddx y2  16 dy


dx
 16
dy
dx
3 2

1  16 x 2  ddxy3  48 x ddx y2  32 dydx

f 0  1
f '0  4
f ''  0   16
f '''  0   128

64 3
f  x   1  4x  8x2  x  ...
3


 1
e 6
 esin 4x


  sin 1 4 x
6
 
sin     4 x
 6
1
x
8

sgfreepapers.com 18
 2 3
  1   1  64  1 
e  1  4     8         ...
6

 8  8 3  8
1 1 1
 1  
2 8 24
7

12

14 n
1
(a) Let P(n) denote the statement  (2r  1)
r 1
2
 n(2n  1)(2n  1) .
3
When n = 1,
1
LHS   (2r  1) 2  12  1
r 1

1 1
RHS  (1)(2  1)(2  1)  (1)(1)(3)  1
3 3
LHS  RHS
Hence P(1) is true.
Assume P(k) is true for some k    .
k
1
i.e.  (2r  1) 2  k (2k  1)(2k  1) .
r 1 3
We need to show that P(k+1) is true.
k 1
1
i.e.  (2r  1) 2  (k  1)(2k  1)(2k  3)
r 1 3
k 1

 (2r  1)
r 1
2

k
  (2r  1) 2  (2k  1) 2
r 1

1
 k (2k  1)(2k  1)  (2k  1) 2
3
1
 (2k  1)  k (2k  1)  3(2k  1) 
3
1
 (2k  1)  2k 2  5k  3
3
1
 (2k  1)  (k  1)(2k  3) 
3
1
 (k  1)(2k  1)(2k  3)
3
Hence P(k )  P(k  1) is true.
Since P(1) is true and P(k )  P(k  1) is true, by the principle of Mathematical induction,
P(n) is true n   

sgfreepapers.com 19
14 Method 1:
b (i) 30
 (2r  3)2
r 1

 52  7 2  92    632
 12  32  52    632   12  32
32
  (2r  1) 2  10
r 1

1
 (32)(64  1)(64  1)  10
3
1
 (32)(63)(65)  10
3
 43670

Method 2:
Let r  k  2
2r  3  2(k  2)  3  2k  1
When r  1, k  2  1  k  3
When r  30, k  2  30  k  32
30

 (2r  3)
r 1
32
  (2k  1)
k 3
32
  (2r  1) 2  12  32
r 1
32
  (2r  1) 2  10
r 1

1
 (32)(64  1)(64  1)  10
3
1
 (32)(63)(65)  10
3
 43670
14b n
1
(ii) To prove: r 2

r 1
 n(n  1)(2n  1)
6
n
1
Proof:  (2r  1) 2  n(2n  1)(2n  1)
r 1 3
n
1

r 1
(4r 2  4r  1)  n(2n  1)(2n  1)
3

sgfreepapers.com 20
n n n
1
4 r 2  4 r  1  n(2n  1)(2n  1)
r 1 r 1 r 1 3
n
1  1
4 r 2  4   n  n  1   n  n(2n  1)(2n  1)
r 1 2  3
n
1
4 r 2  n(2n  1)(2n  1)  2n(n  1)  n
r 1 3
n
1
4 r 2  n  (2n  1)(2n  1)  6(n  1)  3
r 1 3
n
1

r 1
r 2  n  (4n 2  1)  (6n  6)  3
12
n
1

r 1
r 2  n  4n 2  6n  2 
12
n
1

r 1
r 2  n  2n  2  2n  1
12
n
1

r 1
r 2  n  n  1 2n  1
6

15 Original amount = $M
(a) 2% interest charged = $0.02M
Late payment charge = $L
Total outstanding balance = $(1.02M + L)

No of Outstanding balance left unpaid after 27th of the


(b) months month
later
1 1.02M  L
2 1.022 M  1.02 L  L
3 1.023 M  1.022 L  1.02 L  L
4 1.024 M  1.023 L  1.022 L  1.02 L  L
. .
. .
. .
n 1.02n M  1.02n 1 L    1.022 L  1.02 L  L

Outstanding balance left unpaid n months later


= 1.02n M  1.02n 1 L    1.022 L  1.02 L  L
= 1.02n M  (1.02n 1 L    1.022 L  1.02 L  L)

This is a G.P. with first term a = L, common ratio


r = 1.02 and number of terms is n.

L(1.02n  1)
= 1.02n M 
1.02  1
L(1.02n  1)
= 1.02 M 
n

0.02

sgfreepapers.com 21
100 L(1.02n  1)
= 1.02n M 
2

= 1.02n M  50 L(1.02n  1)

(c) Putting 1.02n M  50 L(1.02n  1)  2010 .


Given M = 1000 and L = 55.
1.02n (1000)  50(55)(1.02n  1)  2010
1.02n (1000)  (2750)(1.02n )  2750  2010
1.02n (1000)  (2750)(1.02n )  4760
1.02n (3750)  4760
476
1.02n 
375
476
log(1.02n )  log( )
375
476
n log(1.02)  log( )
375
476
n  log( ) log(1.02)
375
n  12.04

Since n is a positive integer, n = 13, 14, 15, …

Hence n = 13.

sgfreepapers.com 22
Anderson Junior College
JC1 Promotional Examination 2013
H2 Mathematics (9740)

1 − x2
1. (i)* Find the expansion of in ascending powers of x, up to and including
4− x
the term in x2 . [3]
(ii)* State the set of values of x for which this expansion is valid. [1]
(iii)* Hence, by substituting a suitable value of x, find an approximation for 15
a
in the form , where a and b are integers to be determined. [3]
b

n
2. Evaluate ∑ ( 2−r + 2nr + n2 ) , giving your answer in terms of n. [4]
r =2

3. A curve C is defined by parametric equations


π
x = eθ cos θ , y = e−θ sin θ , ≤θ ≤ 0.for −
2
(i) Sketch the curve C , indicating the axial intercepts in exact form. [2]

(ii) Show that the area bounded by the curve C and the axes is given by
0

∫ −
π
2
(sin 2 θ − sin θ cos θ ) dθ .

Hence determine its exact value. [5]

1
4. A sequence un , n =0,1,2,3,…, is such that u0 = − and
2
1
un +1 = un + ln ( n + 1) − 2
for all n ≥ 0.
4n − 1
1
(i) Prove by mathematical induction that un = ln ( n !) + . [5]
2 ( 2n − 1)
N
 1 
(ii) Hence find ∑ ln ( n + 1) − 4n2 − 1 . [3]
n=0

(iii) Does the series found in (ii) converge? Give a reason for your answer. [1]
N  1 
(iv) Using the series found in (ii), evaluate ∑ ln ( n − 1) − 4 . [2]
n=2 ( n − 2 )2 − 1 

*: Not in topics tested for


SRJC 2014 Promo -1-

sgfreepapers.com 23
5. The curve with equation y = − −2 x is transformed by a translation of 2 units in
the positive x-direction, followed by a reflection in the y-axis.

(i) Find the equation of the resultant curve in the form y = f ( x) and the
coordinates of the points where this curve crosses the x- and y- axes. On a
single diagram, sketch the graph y = f ( x) and its inverse. [5]

(ii) Solve the equation f ( x) = f −1 ( x) , giving your answers in exact form. [3]
−1 x2
(iii) The function g is defined such that f g( x) = − 2 . Find g( x) . [2]
2

x(4 x − 1)
6. Without using a calculator, solve < 3x + 1. [3]
2x −1

Hence, find the solutions of the inequalities


x(4 x − 1)
(a) x − 5 < 3x + 1 < ,
2x −1
cos x (4 cos x + 1)
(b) > 3cos x − 1 for 0 ≤ x ≤ π ,
2 cos x + 1
leaving your answers in exact form. [6]

7. The diagram shows a sketch of the curve y = f ( x ) . The curve cuts the x-axis at
C ( −1, 0 ) , has stationary points at A ( −2, 5 ) and B (1, − 2 ) , and asymptotes x = 0 ,
x = 3 and y = 3 .
y
x=3
y = f(x)

y=3
• 0 x

On separate diagrams, sketch the graphs of


1
(i) y = , [3]
f ( x)
(ii) y = f ' ( x ) , [3]

*: Not in topics tested for


SRJC 2014 Promo -2-

sgfreepapers.com 24
showing, in each case, the asymptotes, the coordinates of the stationary points and
the points of intersection with the axes, whenever possible.
1
8. (a)* Find ∫x2
ln( x + 1) dx . [3]
2
(b)* The diagram shows a shaded region R bounded by the curve ( y − 2) = x +1
and the line y + 2 x = 6.
y

R
x
0

Find the volume generated when R is rotated through 2π radians about the
x-axis, leaving your answer correct to 3 significant figures. [4]

9. The lines l1 and l2 have equations


1  −1 
x −1 y − 2    
= , z =1 and r = 2 + λ  1  , λ ∈
3 a 0  
  1
respectively, where a is a constant.

(i) Given that l1 and l2 intersect at the point N, find N and the value of a. [3]

(ii) Show that the position vector of F, the foot of the perpendicular from the
4 5 1
point P ( 2, 1, 1) to the line l2 is i + j − k . [3]
3 3 3
(iii) Find the position vector of the point P ' , the reflection of P in the line l2 . [2]

(iv) The point Q has coordinates (1, 2, 0). Find the ratio of the area of triangle
NQP to the area of triangle FQP ' . [3]

x2
10. A curve C has equation y = , x ≠ −3λ and λ is a positive constant.
x + 3λ
(i) Find the coordinates of the stationary points of C. [3]
(ii) Draw a sketch of C, labeling clearly, in terms of λ , the asymptotes and the
stationary points. [2]
(iii) Use the graph in (ii), find the number of roots of the equation
x 4 − 2λ x − 6λ 2 = 0 . [3]

*: Not in topics tested for


SRJC 2014 Promo -3-

sgfreepapers.com 25
x2
The function f is defined by f : x  , x ≤ − 6λ .
x + 3λ
(iv) Show that f 2 exists and find the value of f 2 (−6λ ) . [4]
11. Two solid cylinders of the same height are placed at a corner of the wall such that
the vertices A, B, C and D touch the wall. At point E, the two cylinders touch each
other. The diagram below shows a cross section of the cylinders.

R
A •

E
B •r

C D
Let r be the radius of the small cylinder and R be the radius of the big cylinder.
2
(i) Show that R = ( 2 +1 r ) [2]
16π
(ii) Given that the volume of the small cylinder is cm3 , find the exact value
2 +1
of the radius r such that the surface area of the big cylinder is a minimum. [5]

12. Mary has a monthly income of $4000. She is considering applying for a car loan
of $40,000 for 6 years which charges an interest rate of 3.00% per annum,
compounded monthly. Interest is chargeable immediately when the loan sum is
drawn out. The monthly repayment, $ m , is fixed throughout the loan tenure.

(i) Show that the calculated loan balance at the end of the nth loan month, after
the monthly repayment is made, is given by

 401 
n  401 n 
40000   − 400m   − 1 . [3]
 400   400  

(ii) By legislation, banks can approve a car loan only if the monthly repayment
does not exceed 15% of an applicant’s monthly income. Prove that Mary will
not be able to apply for the car loan. [3]

(iii) If the interest rate for all car loans by the banks is compounded monthly, find
the range of interest rates chargeable which will enable Mary to apply for the
*: Not in topics tested for
SRJC 2014 Promo -4-

sgfreepapers.com 26
car loan successfully. Give your answer in the form r % per annum, correct to
1 decimal place. [3]

END OF PAPER

*: Not in topics tested for


SRJC 2014 Promo -5-

sgfreepapers.com 27
Anderson Junior College
JC1 Promotional Examination 2013 _H2 Mathematics (9740)_Solutions
[Questions marked with * are not in the topics tested for 2014 SRJC Promo]
Qn Solutions
1 − x2
= (1 − x 2 ) ( 4 − x )
−1/ 2
1(i)
4− x
−1/2
1  x
= (1 − x 2 )  1 − 
2  4
  1  3  
  − −  2 
2  2   x 
= (1 − x 2 )  1 + + 
1 x
 −  + ... 
2  8 2!  4 
 
 
1  x 3x 2

= (1 − x 2 )  1 + + + ... 
2  8 128 
1 1 125 2
= + x− x + ...
2 16 256

(ii) Expansion is valid for { x : − 4 < x < 4, x ∈ } .

1
(iii) By letting x = ,
4
2
1
1−  
 4  ≈ 1 + 1  1  − 125  1 
   
1 2 16  4  256  16 
4−
4
15
16 ≈ 1987
15 4096
4
1987
15 ≈ where a = 1987 and b = 512
512
or
7680
15 ≈ where a = 7680 and b = 1987
1987

n
2 ∑(2
r =2
−r
+ 2nr + n 2 )
n n n
= ∑ ( 2 ) + ∑ 2nr + ∑ n
r =2
−r

r =2 r =2
2

=
( ) 1 2
2 (1 − ( 1 n −1
2) ) + 2n ⋅ n −1 ( 2 + n ) + n ( n −1)
2
1
1− 2 2
= 1
2 (1 − ( 1 n −1
2 ) ) + n(n −1) [(2 + n) + n]
n
= 12 − ( 12 ) + 2n(n 2 − 1)

sgfreepapers.com 28
π
3(i)* x = eθ cos θ , y = e −θ sin θ , for − ≤θ ≤ 0 y
2
O 1 x
When θ = 0 , x-intercept: (1, 0)
π π
When θ = − , y-intercept: (0, − e 2 )
2

(ii)* Area = −
∫ 0
y dx
0
=− ∫π (e
−θ
{
sin θ ) eθ [cos θ − sin θ ]} dθ

2
0

∫π (sin
2
= θ − sin θ cos θ ) dθ

2
0
 1 − cos 2θ  sin 2θ 
= ∫π  2
−
 2 
 dθ

2
0
θ 1 1 
=  − sin 2θ + cos 2θ 
 2 4 4  −π2
π +2
=
4

1
4(i) Let Pn be the statement un = ln ( n !) + for n ≥ 0, n ∈ .
2 ( 2n − 1)
1
When n = 0 , LHS = u0 = −
2
1 1
RHS = ln ( 0!) + =− Since LHS = RHS, ∴ P0 is true.
2( −1) 2

1
Assume that Pk is true for some k ≥ 0, k ∈ , i.e. uk = ln ( k !) + ,
2 ( 2k − 1)
need to prove that Pk +1 is true, i.e., to show that
1 1
uk +1 = ln ( k + 1) ! + = ln ( k + 1) ! + .
2(2( k + 1) − 1) 2(2k + 1)

LHS of Pk+1
= uk +1
1
= uk + ln ( k + 1) −
4k 2 − 1
1 1
= ln ( k !) + + ln ( k + 1) − 2
2 ( 2k − 1) 4k − 1
1 1
= ln ( k + 1) k ! + −
2 ( 2k − 1) ( 2k − 1)( 2k + 1)
2k − 1
= ln ( k + 1) ! +
2 ( 2k − 1)( 2k + 1)
1
= ln ( k + 1) ! + = RHS of Pk +1
2 ( 2k + 1)

sgfreepapers.com 29
Since P0 is true and Pk is true ⇒ Pk +1 is true,
∴ by the principle of mathematical induction, Pn is true for all non-negative integers n.

1
(ii) un +1 = un + ln ( n + 1) − 2
4n − 1
1
⇒ un +1 − un = ln ( n + 1) − 2
.
4n − 1
N
 1  N
∴ ∑  ln ( n + 1) − 2  = ∑ ( un +1 − un )
n =0  4n − 1  n = 0
= ( u1 − u0
+ u2 − u1
+ u3 − u2
+ u4 − u3
+ 
+ uN +1 − uN )
= uN +1 − u0
1  1
= ln ( N + 1) ! + −− 
2 ( 2 ( N + 1) − 1)  2 
1 1
= ln ( N + 1) ! + +
2 ( 2 N + 1) 2

(iii) N
 1 
∴ ∑  ln ( n + 1) − 2  = ln ( N + 1) ! + 1 1
+ .
n =0  4n − 1  2 ( 2 N + 1) 2
The series is divergent since ln ( N + 1) ! → ∞ when N → ∞ .

(iv) Replace n with n + 2,


N  
1
∑ ln ( n − 1) −
n=2 
2 
4 ( n − 2 ) − 1 

N  1 
= ∑ ln ( n + 2 − 1) − 4 2 
n+ 2=2
 ( n + 2 − 2 ) − 1

N −2
1 
= ∑  ln ( n + 1) − 2 
n =0  4n − 1 
1 1
= ln ( N − 2 + 1) ! + +
2 ( 2( N − 2) + 1) 2
1 1
= ln ( N − 1) ! + +
2 ( 2 N − 3) 2

sgfreepapers.com 30
5(i) y = − −2 x x → x −2
→ y = − 4 − 2 x → x → −x y = − 4 + 2x
Coordinates of points: ( −2,0 ) , ( 0, −2 ) .
y

y = f −1 ( x)

( −2,0) O x

( 0, −2)
y = f ( x)

(ii) From the diagram, the graphs intersect at x = −2, 0 , and where
f ( x) = x ⇒ − 4 + 2 x = x ⇒ x 2 − 2 x − 4 = 0
2 ± 4 + 16
x= = 1± 5
2
Since the graphs intersect where x ≤ 0, solutions for f ( x) = f −1 ( x) are x = −2, 1 − 5, 0 .

x2
(iii) f −1g( x) = −2
2
 x2 
⇒ f ( f −1g( x) ) = f  − 2 
2 
⇒ g( x) = − 4 + x2 − 4 = − x 2 = − x

x(4 x − 1)
6 < 3x + 1
2x −1
4 x 2 − x − (2 x − 1)(3x + 1)
<0
2x −1
−2 x 2 + 1
<0
2x −1
x 2 − 12
>0
2x −1
( x + 12 )( x − 12 )
>0
2x −1
1 1 1
∴ − <x< or x > .
2 2 2
x(4 x − 1) 1 1 1
(a) Solution of 3x + 1 < is x < − or <x< .
2x −1 2 2 2
Also, x − 5 < 3 x + 1 ⇒ x > −3.

1 1 1
Taking the intersection of the solutions, −3 < x < − or <x< .
2 2 2

sgfreepapers.com 31
(b) Replace x with − cos x,
− cos x(−4 cos x − 1)
< − 3cos x + 1
−2 cos x − 1
cos x(4 cos x + 1)
⇒ > 3cos x − 1.
2 cos x + 1
1 1 1
∴ − < − cos x < or − cos x >
2 2 2
1 1 1
− < cos x < or cos x < − .
2 2 2
y

x
O

–1

π 2π 3π
∴ <x< or < x ≤ π.
4 3 4

7(i) 1 y
y=
f ( x)

y = 1/3

O x

7(ii)
y
y = f ′( x )

x=3
y =0
O 1 x

sgfreepapers.com 32
1 1 1 1
8(a)*
∫ x 2
ln( x + 1) dx = − ln( x + 1) + ∫ ⋅
x x x +1
dx

1 1 1 
= − ln( x + 1) + ∫  −  dx
x  x x +1
1
= − ln( x + 1) + ln x − ln( x + 1) + c
x
1 1
Alternative method for ∫ ⋅
x x +1
dx

1 x
=∫ dx = ln +c
1 1 x +1
( x + )2 − ( )2
2 2
2
8(b)* Points of intersection of ( y − 2) = x +1 and y + 2x = 6
2 5
(4 − 2x) = x + 1 ⇒ 4 x 2 − 17 x + 15 = 0 ⇒ x = 3 or x = or GC.
4
2
Also, ( y − 2) = x +1 ⇒ y = 2 ± x +1
5 2 2
3 3
Volume generated = ∫ 4
−1
( )
π 2 + x + 1 dx + ∫ 5 π ( 6 − 2 x ) dx − ∫ π 2 − x + 1 dx
4
2
−1
( )
≈ 78.57254 = 78.6 (3 s.f.)

1 3
9(i) l1 : x − 1 = y − 2 , z = 1 ⇒    
l1 : r =  2  + µ  a  , µ ∈ 
3 a 1  
  0
1  −1 
   
l2 : r =  2  + λ  1  , λ ∈ 
0  
  1
If l1 intersects with l2 ,
 1 + 3µ   1 − λ 
 2 + aµ  =  2 + λ 
   
 1   λ 
   
1 + 3µ = 1 − λ ---------- (1)
2 + aµ = 2 + λ ---------- (2)
1 =λ ---------- (3)
1
Solving for (1) and (3): λ = 1 and µ = −
3
Therefore, point N is (0, 3, 1).

Substitute the values of λ and µ into (2):


 1
2 + a −  = 2 +1
 3
a = −3.

(ii) Let F be the foot of the perpendicular from point P( 2,1,1 ) to the line l2.
1− λ 
Since F lies on l2, OF =  2 + λ  for some λ ∈ 
→

 
 λ 
 

sgfreepapers.com 33
 −1 − λ 

→  → →
 
PF = OF − OP =  1 + λ 
 −1 + λ 
 
 −1 − λ   −1
PF ⊥ l2 ⇒  1 + λ  i 1  = 0
 −1 + λ   1 
   
⇒ 1 + λ + 1 + λ −1 + λ = 0
1
⇒λ =−
3
  1 
1 −  − 3  
  
 43 
 
=  5 3  = i + j − k. (shown)


Thus OF = 2 − 1 4 5 1
 3  3 3 3
   − 1 3 
 −1 
 3 

(iii) Let P ' be the point of reflection of P about the line l2.

→ 


→ 
→ OP '+ OP


PF = FP ' ⇒ By the mid-point theorem, OF = .
2

→ 
→ 

⇒ OP ' = 2 OF − OP
 43   2  23 
     
= 2  53  −  1  =  73 
− 1  1 − 5 
 3    3

(iv) • P(2,1,1)
Q(1,2, 0) F
• N • l2

Note that Q lies on l2.
1 
PF × NQ  
Area of ∆NQP  2 NQ 
=  =  since PF = FP ' .
Area of ∆FQP '  1 FP '× QF   FQ 
2 

1 0  1 


     
NQ =  2  −  3  =  −1 ⇒ NQ = 3
     
 0   1   −1
 1   4 3   − 13   −1
→
   5   1  1  
FQ =  2  −  3  =  3  = 3  1  ⇒ FQ = 1
3
3
   1   1   
0 − 3   3  1
Area of ∆NQP  3 
=  = 3.
Area of ∆FQP '  13 3 
Therefore, the ratio is 3:1.

sgfreepapers.com 34
x2 dy 2 x( x + 3λ ) − x 2 x 2 + 6λ x
10(i) y= ⇒ = =
x + 3λ dx ( x + 3λ )2 ( x + 3λ )2
dy
At stationary point, = 0 ⇒ x = 0 or x = - 6λ. Stationary points: (0, 0), (–6λ, –12λ).
dx

x2 9λ 2
(ii) y= ⇒ y = x − 3λ +
x + 3λ x + 3λ
Asymptotes : x = –3λ, y = x – 3λ
y

y = x – 3λ

–3λ (0,0) 3λ x

–3λ

(–6λ, –12λ)

4 2 x2 2λ
(ii) x − 2 λ x − 6λ = 0 ⇒ = 2
x + 3λ x

By sketching the graph of y = 2 on the diagram, there are 2 points of intersections, hence
x
there are 2 roots to the equation.

(iii) Rf = (–∞,–12λ], and Df = (–∞,–6λ].

Since Rf ⊆ Df, hence f2 exists.

 36λ 2  144λ 2
f 2 (−6λ ) = f   = f ( −12λ ) = = −16λ .
 −6λ + 3λ  −12λ + 3λ

2 2
11(i) ( R − r )2 + ( R − r ) = ( R + r )
2
(R − r) = 1
2
(R + r) 2
R−r 1
⇒ =
R+r 2

R ( ) (
2 −1 = )
2 +1 r

2 +1 2 +1
R= × r
2 −1 2 +1
2
( )
2 +1 r 2
=
2 −1
⇒ R= ( 2 +1 r )
16π
(ii) Volume of small cylinder = V = π r 2 h = .
2 +1

sgfreepapers.com 35
16
h=
r 2
( 2 +1 )
Surface area of big cylinder = A = 2π Rh + 2π R 2 .

2 4
A = 2π ( 2 + 1 rh + 2π) ( 2 +1 r2 )
2
 16  4
= 2π ( 2 +1 r  2 )  + 2π ( )
2 + 1 r2
r

2 +1 ( ) 

32π ( 2 +1 ) + 2π 4
=
r
( 2 + 1 r2 )

dA 4 32π 2 + 1 ( )
dr
= 4π 2 + 1 r − ( r2
)
dA
Let = 0,
dr
4 32π ( 2 +1 )
then 4π ( 2 +1 r = ) r 2

⇒ r3 =
32π ( 2 +1 )
4
4π ( 2 +1 )
8
= 3
( 2 +1 )
2
⇒ r=
2 +1
or 2 ( 2 −1 )
d2 A 4 64π ( 2 +1 )
dr 2
= 4π ( 2 +1 + ) r 3

2
d A
When r = 2 ( 2 −1 ,
dr 2
)
>0.

Hence, r = 2 ( )
2 − 1 gives the minimum surface area of the big cylinder.

3 1
12(i) Monthly interest chargeable = % = %.
12 4
Let monthly repayment amount = $ m .
Loan Loan balance at beginning of loan month Loan Balance at end of loan month
Mth (after monthly repayment)
 401   401 
1 40000   40000  −m
 400   400 
2 2
2  401   401   401   401 
40000   −  m 40000   −  m − m
 400   400   400   400 

  
n n −1 n− 2 n n −1 n−2
 401   401   401   401   401   401   401   401 
n 40000   −   m−  m− … − m 40000   −  m−  m− … − m − m
 400   400   400   400   400   400   400   400 

sgfreepapers.com 36
Loan balance at the end of nth loan month after monthly repayment
n n −1 n −2
 401   401   401   401 
= 40000   −   m −   m− … −  m − m
 400   400   400   400 
  401  n 
n   − 1
= 40000  401  − m   400  
 400   401 
 400 − 1 
 

 401 
n  401 n 
= 40000   − 400m   − 1
 400   400  

 401 
72  401 72 
(ii) Let 40000   − 400m   − 1 = 0
 400   400  
⇒ m = 607.75
15% of $4000 = $600.
Since m = 607.75 > 600 , Mary is not able to take up the car loan.

 72 
(iii) Let 40000a 72 − 600  a − 1  ≤ 0.
 a −1 
 x 72 − 1 
From the GC, using the graph of y = 40000 x 72 − 600  ,
 x −1 
1 < a ≤ 1.0021378.

12 × (1.0021378 – 1) × 100% = 2.56536%


∴ 0% < r % ≤ 2.5% (to 1 decimal place)

sgfreepapers.com 37
CAT
THOLIC JUN
NIOR COLLE
EGE
Gen
neral Certifica
ate of Educa
ation Advanc
ced Level
High
her 2
JC1 Promotiona
al Examinatio
on

MATHEM
M MATICS
S 97
740/01
Pa
aper 1 04 Octo
ober 2013
3 hours
Ad
dditional Ma
aterials: List of Formu
ulae (MF15))

READ THES
SE INSTRUC
CTIONS FIRST

Write
W your naame and claass on all th
he work you u hand in.
Write
W in dark blue or black pen on both
b sides oof the paperr.
Yo
ou may use e a soft pencil for any diagrams
d orr graphs.
Do
o not use sttaples, paper clips, hig
ghlighters, g
glue or corre
ection fluid.

Annswer all th
he questionss.
Give non-exa act numericcal answers s correct too 3 significa
ant figures, or 1 decim mal place inn the case of
anngles in deg
grees, unlesss a differen
nt level of a
accuracy is specified
s in the questioon.
Yoou are expe e a graphic calculator.
ected to use
Unsupported d answers from a gra aphic calcu ulator are allowed
a unless a queestion spec cifically stattes
ottherwise.
Where
W unsup pported ansswers from a graphic ccalculator area not allow wed in a quuestion, you
u are requirred
to
o present thee mathematical steps using
u mathe ematical nootations and not calculaator comma ands.
Yoou are remiinded of the
e need for clear
c presenntation in yo
our answers s.

Att the end o


of the exam
mination, arrrange you r answers in NUMERICAL ORDE ER.
Pllace this co
over sheet in front an
nd fasten a
all your worrk securely
y together.

Th
he number of marks is given in brackets [ ] a
at the end of
o each ques
stion or partt question.

Na
ame: _____
_________
__________
______ Classs: _______
__________
_

Question 1 2 3 4 5 6 7 8 9 10 111 12 13 Tottal

Marks

Total 3 4 5 6 6 6 6 8 8 9 11 13 15
10
00

This document consists of 5 printed pag


ges.

Catho
olic Junior Co
ollege

sgfreepapers.com 38
2

1. Sketch the curve ( y  5) 2  ( x  3) 2  4 , indicating clearly the coordinates of the


turning point(s) and equations of the asymptotes. [3]

1
2. Expand in ascending powers of x, up to and including the term in x2.
3
2x 1
State the range of values of x for which this expansion is valid. [4]

3. The graph of y  f ( x ) , where f ( x ) is a cubic polynomial, passes through the points


1
(1, 6), (2, 15) and has two stationary points at x  and x  2 . Find the equation
3
of the curve and hence, find its x-intercept. [5]

dy
4. (a) Given that tan √ , find . [2]
dx
dy
(b) Given that √ , where 0, 0, find . [4]
dx

5. The parametric equations of a curve are


7
x  t 3 , y  , t  0.
t
(i) Find the equation of the tangent to the curve at the point where t = k,
simplifying your answer. [3]
(ii) Hence find the coordinates of the points X and Y where this tangent
meets the x- and y-axes respectively. [2]
(iii) Hence or otherwise, find the area of the triangle OXY, where O is the
origin. [1]

n
1 3 1 1
6. Prove by the method of differences that r
r 2
2
  
 1 4 2n 2(n  1)
. [4]

n
1
Hence, or otherwise, give a reason why the series r
r 2
2
1
is convergent and state

the sum to infinity. [2]

7. Prove by the method of mathematical induction that


n
sin 2n
 cos[(2r  1) ] 
r 1 2sin 
for all positive integers n. [6]

9740/01/Promo/2013

sgfreepapers.com 39
3

x6 1
8. (a) (i) Without using a calculator, solve the inequality  . [3]
x  3x  4 4  x
2

(ii) Hence, deduce the range of values of x that satisfies


x 6 1

x 3x  4
2
4 x
. [2]
x
(b) Solve the inequality ln( x  6)   . [3]
3

9. Charis Insurance provides an investment linked savings insurance plan with two
options of premium payment, monthly and yearly.
For the monthly premium plan, premiums of $500 are collected on the first day of
each month and an interest of 0.5% per month is earned on the last day of each
month, such that there is $502.50 in the account at the end of the first month and
$1007.51 in the account at the end of the second month.

(i) Show that the total amount in the monthly premium account at the end of n
complete months can be expressed as M (1.005n  1) , where M is an integer
to be found. [4]

For the yearly premium plan, premiums of $6000 are made on the first day of each
year and an interest of 6% per year is earned on the last day of each year.

(ii) Given that the total amount in the yearly premium account at the end of k
complete years is $ 106000 1.06 k  1  , find the number of complete years
it will take for the total amount to first exceed $120 000. [2]

A young couple who just had their first child would like to take up a savings plan for
a period of 20 years to prepare for their child’s university education. A friend of the
couple stated that “0.5% a month is the same as 6% a year since 12 × 0.5 = 6”. With
reference to evidence obtained from the expressions from (i) and (ii), comment on the
validity of the statement. [2]

10. (i) Given that f e , where is a constant, find f 0 , f ′ 0 , f ′′ 0 .


Hence write down the first three terms in the Maclaurin series for f .
Give the coefficients in terms of e and . [5]
(ii) Find the value of such that √2 sin cos sin for all . [2]
(iii) By considering the series in part (i), show that

sin x  e  x 2  x  , where
2 sin( x  )
e 4 is a small angle. [2]

9740/01/Promo/2013

sgfreepapers.com 40
4

11. (a) The diagram below shows the graph of y  f ( x ) . It passes through the
origin O and P (3, 0), and has asymptotes x = 2, y  2 .and y  2 .

On separate diagrams, sketch the graph of

(i) y  f '( x) , [3]

1
(ii) y , [3]
f  x
indicating clearly any asymptote(s) and axial intercept(s).

1
(b) The graph of y  is transformed by a reflection in the y-axis, followed
2x  3
by a translation of 1 unit in the negative x-direction, followed by a stretch with
scale factor 2 parallel to the x-axis.

(i) Find the equation of the new graph in the form y  f  x  . [3]
(ii) Hence, or otherwise, sketch the new graph with any axial intercept(s)
and asymptote(s) indicated clearly. [2]

9740/01/Promo/2013

sgfreepapers.com 41
5

12. Functions f and g are defined by

1
f:x  4  2x2 , x  , 0  x  16
g : x  3x  1 , x

(i) State the range of f. [1]


1 1
(ii) With the aid of a diagram, show that f exists and define f in a similar
form. [4]
1 1
(iii) On the same diagram as in part (ii), sketch the graphs of f and f f ,
indicating their endpoints. [3]
(iv) Explain why the x-coordinates of the point(s) of intersection between the
graphs in part (iii) satisfies the equation x 2  2 x  4  0 . [1]
(v) State whether the composite function fg exists, justifying your answer. [2]
(vi) Find the largest possible domain of g in the form [m, n], m, n   , for which
the composite function fg exists. [2]

13. (a) Relative to the origin O, two points A and B have position vectors given
by a = 4i + 2j + 3k and b = 4i – 2j + 7k respectively. The point P divides
AB in the ratio 3 : 1.

(i) Find the coordinates of P. [2]


(ii) The vector c is a unit vector in the direction of OP .
Write c as a column vector, and give the geometrical meaning
of a  c . [2]
(iii) By using vector cross product, find the exact area of triangle OAP. [3]
x3 z 1
(b) The line l has equation  y3 and the plane p has equation
3 2
3x  y  2 z  0 .

(i) Show that l is perpendicular to p. [2]


(ii) Find the coordinates of the point of intersection of l and p. [3]
(iii) Show that the point C with coordinates  9,1, 7  lies on l.
Find the coordinates of the point C’ which is the mirror image of
C in p. [3]

 End of Paper 

9740/01/Promo/2013

sgfreepapers.com 42
CATHOLIC JUNIOR COLLEGE
H2 MATHEMATICS
JC1 PROMOTIONAL EXAMINATION 2013

Solutions
1 ( y  5) 2  ( x  3) 2  4
( x  3) 2 ( y  5) 2
  1
22 22
Asymptotes:
( y  5) 2  ( x  3) 2
y  5   ( x  3)
y  x  8 or y   x  2
y

y=–x+2 (‐3, 7) y=x+8

(‐3, 3)
x

2 1 1 1
  2 x  1 3  1 1  2x  3
 
3
2x 1
  1  1  
  1      1 
  1      2 x   
3  3 
 2 x   .... 
2

  3 2 
 
 
2 8
 (1  x  x2 )
3 9
1 1
Validity:   x 
2 2
3 Let y  Ax  Bx  Cx  D
3 2

dy
  3 Ax 2  2 Bx  C
dx
A B C  D  6
8 A  4B  2C  D  15
A  2 B  3C  0
12 A  4 B  C  0
Solving,
A  2, B  5, C   4, D  3
y  2 x3  5 x 2  4 x  3

sgfreepapers.com 43
CATHOLIC JUNIOR COLLEGE
H2 MATHEMATICS
JC1 PROMOTIONAL EXAMINATION 2013
When y = 0, x = – 3.26 (3sf)
x-intercept = (– 3.26, 0)
4 (a)
1  12
d
dx

tan 1 x   1

1  ( x )2 2
x

1

2 x (1  x)
(b)

Taking logarithm on both sides,

Differentiating both sides,

5 (i)
d y d y dx  7  7
       3t    4
2

dx dt dt  t 2  3t
7 7
y   4  x  k3 
k 3k
7 28
y  4 x
3k 3k
(ii)
7 7
y   4  x  k3 
k 3k
7 28
y  4 x
3k 3k
y  0, x  4k 3  X is  4k 3 , 0 
28  28 
x  0, y  Y is  0, 
3k  3k 
(iii)
1
Area of OXY   OX  OY 
2
1  28 
  4k 3   
2  3k 
56
 k 2 units 2
3
6 n
1 n
1

r 2 r  1
2

r  2 ( r  1)( r  1)

sgfreepapers.com 44
CATHOLIC JUNIOR COLLEGE
H2 MATHEMATICS
JC1 PROMOTIONAL EXAMINATION 2013
1 n  1 1 
    
2 r 2  r  1 r  1 
1 1 1 
 [   
2 1 3 
1 1
  
2 4
1 1
  
3 5

 1 1 
  
 n  3 n 1
 1 1
  
n2 n
 1 1 
  
 n 1 n 1
1 1 1 1 
 1    
2  2 n (n  1) 
3 1 1
  
4 2n 2(n  1)

n
1 3 1 1
r
r 2
2
  
 1 4 2 n 2( n  1)
.

1 1 3 1 1 3 
1 3
n   ,.
2n
 0,
2(n  1)
 0 , so  
4 2n 2(n  1)
 so
4
r
r 2
2

1 4
7 n
sin 2n
Let Pn be the statement  cos[(2r  1) ] 
r 1 2sin 
for n  Z  , n ≥ 1

When n = 1, L.H.S. = cos 


sin 2 2sin  cos 
R.H.S. =  cos  =L.H.S.
2sin  2sin 
k
sin 2k
Assume Pk is true, i.e.  cos[(2r  1) ]  for some k  Z  , k ≥ 1.
r 1 2sin 
Required to prove Pk+1 is true, i.e.
k 1
sin[2(k  1) ]
 cos[(2r  1) ] 
r 1 2sin 
k
L.H.S. =  cos[(2r  1) ]  u
r 1
k 1

sin 2k
=  cos[(2k  1) ]
2sin 
sin 2k  2 cos[(2k  1) ]sin 
=
2sin 

sgfreepapers.com 45
CATHOLIC JUNIOR COLLEGE
H2 MATHEMATICS
JC1 PROMOTIONAL EXAMINATION 2013
sin 2k  sin[2(k  1) ]  sin 2k
=
2sin 
sin[2(k  1) ]
= = R.H.S.
2sin 
Pk is true  Pk+1 is true.
Hence, by Mathematical Induction, Pn is true for all n  Z  , n ≥ 1.
8 (a)(i)
x6 1

x  3x  4 4  x
2

x6 1
 0
( x  1)( x  4) 4  x
x6 1
 0
( x  1)( x  4) x  4
x  6  ( x  1)
0
( x  1)( x  4)
2x  7
0
( x  1)( x  4)
Using test-point method,
– + – +
-3.5 -1 4
 x  3.5 or  1  x  4

(ii)
x 6 1

2
x 3x  4 4 x
Replace x by x
 x  3.5 or 1  x  4
(no real solution) x 4
-4 x  4

(b)
x
Draw the graphs of y  ln( x  6 ) and y   .
3

sgfreepapers.com 46
CATHOLIC JUNIOR COLLEGE
H2 MATHEMATICS
JC1 PROMOTIONAL EXAMINATION 2013
y
x 6

y  ln( x  6)
x
– 3.1461

x
y
3

Ans:  6  x  3.15
x
Alternative solution: Draw the graph of y  ln( x  6)  .
3
y
x 6

x
y  ln( x  6) 
3
x
– 3.1461

Ans:  6  x  3.15

9 (i)
Total amount after 1 month = 1.005(500)
Total amount after 2 month = 1.005 (500)  1.005(500)
2

Total amount after 3 month


= 1.005 (500)  1.005 (500)  1.005(500)
3 2

Total amount after n months  1.005 (500)  1.005 (500)    1.005(500)


n n1

1.005(500) 1.005n  1

1.005  1
 100500 1.005n  1
M = 100500

(ii)
106000 1.06k  1  120000
Solving, k  12.99
 k  13 complete years.

sgfreepapers.com 47
CATHOLIC JUNIOR COLLEGE
H2 MATHEMATICS
JC1 PROMOTIONAL EXAMINATION 2013

From (i) and (ii), the final amount after 20 years is


100500 1.005240  1  $232175.55 for monthly account
106000 1.0620  1  $233956.36 for yearly account
Hence the statement is invalid as the final total amount differs quite significantly
10 (i)
We are given that .
Differentiating,

Differentiating,

So we have

Hence,

(ii)
Since

we have .
(iii)
Since is a small angle,

then

11 (a)(i)
y  f '( x )

sgfreepapers.com 48
CATHOLIC JUNIOR COLLEGE
H2 MATHEMATICS
JC1 PROMOTIONAL EXAMINATION 2013

(a)(ii)
1
y
f  x

(b)(i)
1
y
2x  3

1 1
y 
2( x)  3 2 x  3

1 1
y 
2( x  1)  3 2 x  1

1 1
y 
1  x 1
2  x   1
2 
(b)(ii)

sgfreepapers.com 49
CATHOLIC JUNIOR COLLEGE
H2 MATHEMATICS
JC1 PROMOTIONAL EXAMINATION 2013

12 (i)
As f is an increasing function,
f(0) = (4) ½ = 2
f(16) = (36) ½ = 6
Range of f, Rf = [2,6]
(ii)
y

y  f  x 16, 6 
2

O x

f is a 1-1 function as the line y = k, 2 k 6 intersects the graph of f exactly once.


(OR: f is a 1-1 function as any line y = k intersects the graph of f at most once.)
Hence f -1 exists.

Let y=f(x) = (4+2x)1/2


y2 = 4+2x
x = (y2 – 4)

f -1(x) = (x2 – 4)]


D f -1 = Rf = [2,6]

Hence f -1 : x (x2 – 4), 2  x  6

sgfreepapers.com 50
CATHOLIC JUNIOR COLLEGE
H2 MATHEMATICS
JC1 PROMOTIONAL EXAMINATION 2013

(iii)

 6,16  16,16 
y  f -1f  x 
yf -1
 x

(16, 6)

2
x

O 2

(iv)
By considering f(x) = x, (4 + 2x)1/2 = x
x2 – 2x – 4 = 0
The x-coordinates of the points of intersection satisfy the
equation x2 – 2x – 4 = 0.

(v)
Rg =
Df = [0, 16]
Rg Df
=> fg does not exist.

(vi)
Consider Rg = Df
3x+1 = 0 => x = -1/3
3x+1 = 16 => x = 5
Hence [- , 5] is the largest possible domain of g for fg to exist.
13 (a)(i)
OA  3OB
OP 
4
 4   4  
1    
  2   3  2  
4   
 3   7  
 4 
 
   1
 6 
 
(a)(ii)

sgfreepapers.com 51
CATHOLIC JUNIOR COLLEGE
H2 MATHEMATICS
JC1 PROMOTIONAL EXAMINATION 2013
4 4
1   1  
c=   1    1
4 2  (1) 2  6 2  6  53  
  6
Geometrically, a  c is the length of projection of the vector a on OP or c.
(a)(iii)
 4   4   15 
     
a  p   2     1    12 
 3   6    12 
     
Area of triangle OAP
1
 ap
2
1
 15 2  ( 12) 2  ( 12) 2
2
1
 513
2

(b)(i)
 3  3 
   
Line l : r   3     1  ,   
1  2 
   
3
 
Plane p : r    1  0
2
 
  3 3
   
Since  1     1 , the normal of the plane p is parallel to the line l, the line l is perpendicular to
  2 2
   
p.

(b)(ii)
 3  3   3 
   
When l intersects p,   3       1  0
 1  2   2 
   
9  9  3    2  4  0
 1
Coordinates of point of intersection = ( 0, -2, -1)

(b)(iii)
  9   3  3 
   
Suppose C with coordinates  9,1, 7  lies on l,  1     3   
  7   1  2 
   

sgfreepapers.com 52
CATHOLIC JUNIOR COLLEGE
H2 MATHEMATICS
JC1 PROMOTIONAL EXAMINATION 2013
 9  3  3
4
Since C satisfies the parametric equations of l with   4 , therefore C lies on l.
We note that C lies on l, l is perpendicular to p and l meets p at (0, -2, -1),
By Ratio Theorem,
 9 
  
1  OC '
 0   
   7 
 2   2
 1 
 
 0   9   9 
      
OC '  2  2    1    5 
 1   7   5 
     

sgfreepapers.com 53
1

HWA CHONG INSTITUTION


JC1 Promotional Examinations
Higher 2

MATHEMATICS 9740
7 October 2013
3 hours

Additional Materials: Answer Paper


List of Formulae (MF15)

READ THESE INSTRUCTIONS FIRST

Write your name and CT group on all the work you hand in.
Write in dark blue or black pen on both sides of the paper.
You may use a soft pencil for any diagrams or graphs.
Do not use staples, paper clips, highlighters, glue or correction fluid.

Answer all the questions.


Give non-exact numerical answers correct to 3 significant figures, or 1 decimal place in
the case of angles in degrees, unless a different level of accuracy is specified in the
question.
You are expected to use a graphic calculator.
Unsupported answers from a graphic calculator are allowed unless a question
specifically states otherwise.
Where unsupported answers from a graphic calculator are not allowed in a question, you
are required to present the mathematical steps using mathematical notations and not
calculator commands.
You are reminded of the need for clear presentation in your answers.

At the end of the examination, fasten all your work securely together.
The number of marks is given in brackets [ ] at the end of each question or part question.

This document consists of 7 printed pages.

© Hwa Chong Institution 9740 / JC1 Promo 2013 [Turn Over

sgfreepapers.com 54
2

1 Sophia has a total saving of $90 million in three accounts A, B and C with $x
million, $y million and $z million respectively. She transfers funds among the
accounts based on the table below.

Percentage of Fund
transferred from To Account A To Account B To Account C
initial amount in
Account A − 37.5% 12.5%
Account B 5% − 5%
Account C 10% 20% −

For instance, 37.5% and 12.5% of the initial amount in Account A are transferred
to Account B and Account C respectively.
As a result of the funds transfer, the amount in Account A decreases by $16 million
and the amount in Account B increases by $19 million.
(i) By considering the amount in Account A, show that
0.5 x  0.05 y  0.1z  16 . [1]
(ii) By forming a system of linear equations, find the values of x, y and z. [3]

It is given that the expansion of  2  px  in ascending powers of x, up to and


q
2

1
including the term in x, is  x . Find the values of p and q.
4
Find, in terms of n, the coefficient of x n in the above expansion. [4]

3 A water tank contains 8000 litres of water initially. At the beginning of each day,
500 litres of water is added to the tank. At the end of each day, 10% of the amount
of water in the tank will be used.
(i) Show that the amount of water in the tank after 3 days is 7051.5 litres. [1]
(ii) Find the least number of days it will take for the water in the tank to be less
than 5000 litres. [3]
(iii) Will the tank ever dry up? Justify your answer. [1]

© Hwa Chong Institution 9740 / JC1 Promo 2013 [Turn Over

sgfreepapers.com 55
3

4 The diagram below shows the graph of y  f  x  . It cuts the axes at the points

 0, 1 , 1.5, 0  and  3, 0  . It has a minimum point at  2.5, 0.5 . The horizontal,


vertical and oblique asymptotes are y  0, x  7 a and y   x  a respectively,
where a is a positive constant. x  7a
y

1
x
O 1.5 3
 2.5, 0.5

y  x  a
On separate diagrams, sketch the graphs of
1
(i) y , [3]
f  x

(ii) y  f ' x , [3]

showing clearly the axial intercepts, the stationary points and the equations of the
asymptotes where applicable.

5 A sequence of real numbers un  , for n    , satisfies the recurrence relation

un1  a a
 , with u1  a , where a and b are fixed non-zero real constants and
un  b b
a  b.
(i) Given that the limit l of the sequence un  exists, find the value of l. [2]

(ii) By expressing un 1 in terms of un , find an expression for un , leaving your

answer in terms of a, b and n. [2]


(iii) Given that the sum to infinity S for the sequence un  exists, state an

inequality satisfied by a and b. Find S in terms of a and b. [2]

© Hwa Chong Institution 9740 / JC1 Promo 2013 [Turn Over

sgfreepapers.com 56
4

By using the substitution u  9  4 x 2 , find x 9  4 x 2 dx .


3
6 (a) [4]
1
(b) Evaluate 0
x 2 tan 1 x dx , giving your answer in exact form. [4]

7 The coordinates of 3 points A, B and C are  2, 0, 1 ,  3, 1, 2  and 1, 2, 4 

respectively.
(a) Find the point D on the x-axis such that there exists a point P on line AB
where C, D and P are collinear. [4]

(b) Find two possible points E on the x-y plane, such that OE is a unit vector

and AOE  150 . [4]

2
8 (i) Express in partial fractions. [2]
r (r  1)  r  3
n
1
(ii) Hence find  2r (r  1)(r  3) .
r 1
[3]


1
(iii) Using the result in part (ii), determine the value of  2r (r  2)(r  3) .
r 5
[3]

9 Prove by mathematical induction that for all n    ,


1
1  1  2   1  2  3  1  2  3  4   ...  1  2  3  ...  n   n  n  1 n  2  .
6
[5]
Hence find, in terms of n,
(i) 3   3  6    3  6  9    3  6  9  12   ...   3  6  9  ...  (6n  3)  , [2]

(ii) 3   3  9    3  9  27   ...   3  9  27  81 ...  3n  . [2]

© Hwa Chong Institution 9740 / JC1 Promo 2013 [Turn Over

sgfreepapers.com 57
5

10 The functions f and g are defined as follows.

f  x  2  x 1 , x  ,
 1 2
 3x 3 , 0  x  2,
g  x  
 1   x  32 , x  2.

(i) Show that f 1 does not exist. [1]


(ii) If the domain of f is restricted to [k , ) such that f 1 exists, state the least

value of k and define f 1 in a similar form. [3]

Use the new domain of f found in part (ii) for the following parts.
(iii) Show algebraically that there is no value of x for which f 1  x   f  x  . [2]

(iv) Find the range of the composite function g f . [2]

(v) Find the value of x such that g f  x   1 . [1]

2x2  3
11 Sketch the graph of y  , showing clearly the axial intercepts, the stationary
x2
points and the equations of the asymptotes where applicable. [3]
2x2  3
(a) Solve the inequality  1. [2]
x2
2sin 2 x  3
Deduce the solution of the inequality  1 , where 0  x  2 . [2]
sin x  2
(b) Describe fully a sequence of transformations which would transform the graph
5 2x2  3
of y  2 x  to the graph of y  . [3]
x x2

© Hwa Chong Institution 9740 / JC1 Promo 2013 [Turn Over

sgfreepapers.com 58
6

12 An art structure, which is a parallelpiped (made of 6 faces of parallelograms) has a


horizontal base OABC, with OA, OC and OD as its three sides and remaining
vertices are B, E, F, and G as shown in the diagram below.

G F
D E
k
j C
B
O
i A

 
It is given that OA  5i and OC  i  7 j . The lines l1 and l2 have equations

given by l1 : r   5    i   7  14  j  6k , where  is a real parameter and



l2 : 3 x  z  15 , y  0 . E and F are on line l1 , and A and E are on line l2 .

(i) Find the position vector of E. [2]


(ii) Find the equation, in scalar product form, of the plane ABFE. [3]

(iii) Find the projection vector of AE onto the base OABC. Hence, or otherwise,
find the area of the projection of the plane ABFE onto the base. [2]
(iv) Find the equation of the line l3 , which is the reflection of line AE about the
base OABC. [2]
(v) An architect wants to add a shelter which has the plane equation
x  ay  bz  c , where a, b and c are unknown constants. He wants the shelter
to meet the plane ABFE at EF. What can be said about the values of a, b and
c? [2]

© Hwa Chong Institution 9740 / JC1 Promo 2013 [Turn Over

sgfreepapers.com 59
7

13 (a) Using differentiation, find the equation of the tangent at the point (2, 1) on

the curve x3  y 3  3  x  y  . [3]

(b) A spherical balloon is inflated such that 0.1 m3 of air is pumped into the
balloon every second. Find the rate of change of its surface area when the
diameter is 1 m . [4]
4 3
[Volume of sphere =  r and surface area of sphere = 4 r 2 .]
3

(c) When designing the floor plan of his new house, Mr Lim wants to build a
triangular garage with 2 adjacent walls of fixed lengths a and b meters and
making an angle of  radians. On the third side of his triangular garage, he
intends to build 4 square-shaped rooms of equal size (see diagram). Find the
value of  when the total area covered by the garage and the 4 rooms is a
maximum. [5]

a θ b

© Hwa Chong Institution 9740 / JC1 Promo 2013 [Turn Over

sgfreepapers.com 60
Suggested Solutions 2013 C1 H2 Math Promotional Examination
Qtn Solutions
1(i) Funds transferred into Account A: 0.05 y  0.1z
Funds transferred from Account A: 0.375 x  0.125 x  0.5 x

So we have 0.5 x   0.05 y  0.1z   16


i.e. 0.5 x  0.05 y  0.1z  16 ----(1)
(ii) Similarly, for Account B, we have
0.375 x  0.1 y  0.2 z  19 ----(2)
We also know x  y  z  90 ----(3)
Solving (1), (2), (3) using GC, we have
x  40, y  20, z  30

 2  px 
2 q

q
q  px 
 2 1  
 2 
  px  
 2 q 1   q     ... 
  2  
 pqx 
 2 q 1   ... 
 2 
1
 x
4
1 1  2 p 
 2 q  ---(1) &    1 ---(2)
4 4 2 
q  2, p  4
 2  4x
2

1
 1 2x
2

4
1
 1  2 2x 
 2 3 2x 2   2 3 4 2x 3 ...
    
4 2! 3! 
x n coefficient
1   2  3 4  ...    n  1   n
    2
4  n! 

1
  1  n  1 2n   1  n  1 2n  2
n n

3(i) Vol of water at end of Day 1


 0.9(8500)
Vol of water at end of Day 2

sgfreepapers.com 61
 0.9  500  0.9(8500)   0.9  500   0.92 (8500)
Vol of water at end of Day 3
 0.9(500)  0.92 (500)  0.93 (8500)
= 7051.5 litres
(ii) Vol of water at end of Day n, V
 0.9(500)  0.92 (500)  ...  0.9n1 (500)  0.9n (8500)
 500  0.9  0.92  ...  0.9n1   0.9n (8500)
 0.9 1  0.9n1  
 500    0.9n (8500)
 1  0.9 
 4500 1  0.9n1   0.9n (8500)
For V < 5000,
4500 1  0.9n 1   0.9n (8500)  5000
From G.C,
n V
18 5025.3
19 4972.8
20 4925.5
Least n  19
Least number of days = 19.
(iii) As n  , V  4500
Therefore, water tank will never dry up.
4i
y

1
Part I y
f  x
1
x
7a
y=0 O
 2.5, 2
Part III

x  1.5 x3
Part II
ii

2.5
y  1
x  7a

sgfreepapers.com 62
5 (i) Since l is the limit,
As n  , un  l , un 1  l
la a
 
l b b
 b l  a   a l  b
 bl  al
 l b  a  0  a  b 
l 0

(ii)
un1  a a

un  b b
 b  un1  a   a  un  b 
 bun 1  aun
a
 un 1  un
b
a
Hence un  is a GP with ratio and since u1  a,
b

n 1
a
un  a  
b

a
(ii) Since S exists, r  1  1
b
a
S
a
1
b
ab

ba

6(i) du
 8x
dx
9  4x2 dx   x2  8x  9  4x2  dx
1
x
3 1/2

8
1  u  9  du  1/2
     u  dx
8  4  dx 
1 9
  u3/2  u1/2 du
32 32
1 3
 u5/2  u3/2  C
80 16
  9  4 x2    9  4 x2   C
1 5/2 3 3/2

80 16

sgfreepapers.com 63
(ii) 1
 1 3  1 
1 1
 1 3  1 
0 x tan x dx   3 x  tan x0  0  3 x 
2 1
2 
dx
 1 x 
1
 1   1  x 
1
  x3  tan1 x    x   dx
 3  0 3 0
1  x2 
1
 1  1 1 
1

  x3  tan1 x   x2  ln 1 x2  
 3  3 2 2  0
 1    1  1 1 
       ln  2 
 3  4  3  2 2 
 1
  1 ln 2
12 6
7(a) 2  5 
   
AB line  r   0     1  ,   
    3
 1  
 2  5 
  
OP    
 1  3 
 
a
  
OD   0 
0
 
C , P, D are collinear.
 
CP  kCD
 2  5  a   a  1
   
  k 2 
 1  3   4 
   
1 5
   1, k  , a  
2 3
 5
 3
  
OD   0 
 0 
 
 
(b) E (a, b, 0)
a A
  
OE   b 
0
 
a 2  b2  1 E
 
OA  OE O
cos150  
OA 1

sgfreepapers.com 64
 2  a
   
 0 b
3  1  0 
 
2 5
3 2a 15
  a or 0.968 (3 s.f.)
2 5 4
15 2 1
 b 1 b  
16 4
 15 1   15 1 
E   , , 0  or E   ,  , 0 
 4 4   4 4 
8 (i)
2 A B C
  
r (r  1)  r  3 r r  1 r  3
2  A  r  1 r  3  B  r  r  3  Cr  r  1
2 1
r  0, A r  1, B  1 r  0, C
3 3
2 2 1 1
   
r (r  1)  r  3 3r r  1 3(r  3)

1 n 2 1 n  2 1 1 
     
4 r 1 r (r  1)  r  3 4 r 1  3r r  1 3(r  3) 
1 2 1 1
 [  
4 3 2 12
2 1 1
  
6 3 15
2 1 1
  
9 4 18
2 1 1
  
12 5 21
2 1 1
  
15 6 24

2 1 1
  
3(n  3) n2 3n
2 1 1
  
3(n  2) n 1 3  n  1
2 1 1
  
3(n  1) n 3 n  2
2 1 1
   ]
3n n 1 3  n  3

sgfreepapers.com 65
17 1 1 1 1 
      
4 18 n  1 3  n  1 3  n  2  3  n  3 
1 7 2 1 1 
   
12  6 n  1 n  2 n  3 

(iii)

1 1
r 5 2 r ( r  2)( r  3)

Replace r by r  3,

1

r  2 2r (r  1)(r  3)

1 1
 
r 1 2r (r  1)(r  3) 2 1 2 4
 1 7 2 1 1  1
 lim      
 
n 12 6 n  1 n  2 n  3   16
 7 1 1 1  1
 lim     
n 72
 6  n  1 12  n  2 12  n  3  16
7 1 5
  
72 16 144
9 (See alternative solution below)
Let P( n) be the statement
1
“ 1+ (1+2) + (1+2+3) + (1+2+3+…+n)  n  n  1 n  2  , n    ”
6
When n  1 , LHS of P(1) = 1,
(1)(2)(3)
RHS of P(1) = 1
6
Since LHS = RHS, P(1) is true.

Assume P(k) is true for some k ∈ Z+ ,


1
i.e. 1+ (1+2) + (1+2+3) + (1+2+3+…+k)  k  k  1 k  2 
6
To show P(k+1) is true,
1
i.e. 1+ (1+2) + (1+2+3) + (1+2+3+…+k+k+1)   k  1 k  2  k  3
6
LHS of P(k+1)
=1+(1+2)+(1+2+3)+(1+2+3+…+k)+(1+2+3+…+k+k+1)
1
 k  k  1 k  2  + 1  2  3  k  k  1
6
1 1
 k  k  1 k  2  +  k  1 k  2 
6 2

sgfreepapers.com 66
1
  k  1 k  2  k  3
6
= RHS of P(k+1)
Since P(1) is true, and P(k) is true => P(k+1) is true, by mathematical induction,
P(n) is true for n ∈ Z+ .

Alternative Solution:
n
1
" Ur  n  n 1 n  2 , where Ur  1 2  3  ...  r,
Let P( n) be the statement r 1 6
nZ "
1
When n  1 , LHS of P(1) = U
r 1
r  U1  1 ,

6
RHS of P(1) = 1
6
Since LHS = RHS, P(1) is true.

Assume P(k) is true for some k ∈ Z+ ,


k
1
i.e.  U r  k  k  1 k  2 
r 1 6

To show P(k+1) is true,


k 1
1
i.e.  U r   k  1 k  2  k  3
r 1 6
LHS of P(k+1)
k 1
 U r
r 1
k
  U r  U k 1
r 1
1
 k  k  1 k  2  + 1  2  3  k  k  1
6
1 1
 k  k  1 k  2  +  k  1 k  2 
6 2
1
 k  k  1 k  2  k  3
6
= RHS of P(k+1)

Since P(1) is true, and P(k) is true => P(k+1) is true, by mathematical induction,
P(n) is true for n ∈ Z+ .

sgfreepapers.com 67
(i)
3   3  6    3  6  9   ...   3  6  9  ...  (6 n  3) 
1 
 3 1  1  2   1  2  3   ...  (1  2  3  ...  (2 n  1))   3   2n  1 2n  2n  1 
6 
 n  2n  1 2n  1

(ii)
3   3  9    3  9  27   ...   3  9  27  81 ...  3n 
 3   31 2    31 2 3   ...   31 2 3... n 
1 1 2   1 2  3  ... 1 2  3... n 
3
n  n 1 n  2 

3 6

10 f  x  2  x  1, x  
(i)

y2
 2,1

The horizontal line y  2 cuts the curve at more than one point, hence f is not
one-to-one and f 1 does not exist.

OR f 1  f  3  2 , hence f is not one-to-one and f 1 does not exist.

(ii) The minimum value is k  2 .

Let y  f  x   2  x  1  x  2  1 x  2 
 x  2   y  1
2

Df 1  Rf  [1, )  f 1  x   2   x  1 , x  1
2

(iii) If there exists a solution for f 1  x   f  x 


 there exists a solution for f 1  x   x
 2   x  1  x
2

 x 2  3x  3  0
2
 3 3
x   0
 2 4
 no solution for x
 f 1  x   f  x  has no solution.

sgfreepapers.com 68
(iv)

 3,1
 2
 0, 
 3
y  g  x

 2, 0   4, 0 

[2, ) 
f
[1, ) 
g
  ,1  Rgf   ,1

(v) g f  x  1
f  x  3
x  2 1  3
x2  2
x2 4
x6

11

y  2x  4

 3.58,14.3

 0.419,1.68
y  1 for part (ii) 

x2
3 axial intercepts
 3  3 
 0,  ,   , 0  OR  0,1.5  ,  1.22, 0 
 2  2 
2 turning points
 0.419,1.68 ,  3.58,14.3
2 asymptotes
x  2, y  2 x  4
(a) Using the graph, the intersections of the curve with the line y = 1 are
 0.5,1 , 1,1 , so the solution is
1
  x  1 or x  2
2

sgfreepapers.com 69
2sin 2 x  3
1
sin x  2
So the solution is
1
  sin x  1 or sin x  2  rej
2

1
2
O
1
 7 1
2  , 
6 2

7 11
 0  x   or   x  2
6 6
(b) 2 x2  3 5
y  2x  4 
x2 x2

Translation of 2 units in the positive x-direction, followed by translation of 8


units in the positive y-direction.

12  5  1
(i)    
lEF : r   14     7  ,   
   0
 6   
l AE : 3x  z  15
x  0 z   15 
 ,y0
1 3
 0  1
   
l AE : r   0     0  ,   
    3
 15   
2
 5  1 7
      
OE   14   2  7    0 
 6  0 6
     
(ii)  1   1   21 
     
n   7    0    3 
      
 0   3   7 

sgfreepapers.com 70
 5   21 
  
 0  .  3   105
 0   7 
  
 21 
 
r.  3   105
 
 7 
(iii) Method 1: E
By Observation, 
Projection vector of AE
1  2
onto  0  =  0  A
0  0
   
2

Method 2: 
Projection of of AE onto normal of floor E
  2   1    1  2
          
AE '    0  . 0    0    0 
  6   0    0   0 
       A

Method 1:

F ' X  7 (Deduce from OC ) B

Area =  AE ' F ' X   2  7  14

A X

Method 2:
1  2  0 
       
Area = AB  AE '   7    0    0   14
 0   0  14 
     
(iv) Let E '' be the reflection of E about and plane OABC. E
7 7
     
OE   0  , OE ''   0  ,
6  6 
   
 2 A
    
AE ''  OE ''  OA   0 
 6 
 

sgfreepapers.com 71
5 1
   
l3 : r   0     0  ,   
    3 
0  
(v) Let  be plane x  ay  bz  c.
EF is //  .
1
 
 7  is  to n  .
0
 
1 1
   1
 7  .  a   0  1  7a  0  a   7
0 b
  
E is on plane  .
7 1
  
 0  .  a   c  7  6b  c.
6 b
  
13 x3  y 3  3x  3 y
(a)
d 3
dx
 x  y 3    3x  3 y 
d
dx
dy dy
3x 2  3 y 2  33
dx dx
dy dy
3x 2  3  3 y 2  3
dx dx
x  1 dy
2

y 2  1 dx
Substitute x  2 and y  1 ,
dy 3
 (undefined)
dx 0
Therefore, the tangent is a vertical line.
Thus, the tangent is x  2 .

(b) Let the radius be r .


dS
We want to find ,
dt
dS dS dV d V
  
dt d r dt d r
  8 r    0.1   4 r 2 
1

5r
1 dS 2
Sub r  into , we get m 2 / s .
2 dt 5

sgfreepapers.com 72
(c) Let the side of each room be x .
By cosine rule,
 4 x   a 2  b 2  2ab cos 
2

1
Total area, A  ab sin   4 x 2
2
A  ab sin    a 2  b 2  2ab cos  
1 1
2 4
1 1 1 1
 ab sin   a 2  b 2  ab cos 
2 4 4 2
dA
To find max area, we let  0.
d
dA d  1 1 2 1 2 1 
  ab sin   a  b  ab cos  
d d  2 4 4 2 
1 1
 ab cos   ab sin 
2 2
1 1
ab cos   ab sin   0
2 2
tan   1
3
  since 0     
4
3
Therefore, stationary point at   .
4
d2 A 1 1
 ab cos   ab sin 
d 2
2 2
2
d A
0
d 2   3
4

Thus, the stationary point is maximum.

sgfreepapers.com 73
INNOVA JUNIOR COLLEGE
JC 1 MID COURSE EXAMINATION
in preparation for General Certificate of Education Advanced Level
Higher 2

CANDIDATE
NAME

CLASS INDEX NUMBER

MATHEMATICS 9740/01
8 October 2013
Additional Materials: Answer Paper
3 hours
Cover Page
List of Formulae (MF15)

READ THESE INSTRUCTIONS FIRST

Do not open this booklet until you are told to do so.

Write your name, class and index number on all the work you hand in.

Write in dark blue or black pen on both sides of the paper. You may use a soft pencil for any
diagrams or graphs.
Do not use staples, paper clips, highlighters, glue or correction fluid.

Answer all the questions.


Give non-exact numerical answers correct to 3 significant figures, or 1 decimal place in the case
of angles in degrees, unless a different level of accuracy is specified in the question.
You are expected to use a graphic calculator.
Unsupported answers from a graphic calculator are allowed unless a question specifically states
otherwise.
Where unsupported answers from a graphic calculator are not allowed in a question, you are
required to present the mathematical steps using mathematical notations and not calculator
commands.

You are reminded of the need for clear presentation in your answers.

At the end of the examination, fasten all your work securely together.

The number of marks is given in brackets [ ] at the end of each question or part question.

This document consists of 6 printed pages.

Innova Junior College [Turn over

sgfreepapers.com 74
2

1 + x2
1* (i) Find the expansion of in ascending powers of x, up to and including the
√ (4 + 2 x)
term in x 2 . [3]

(ii) State the range of values of x for which this expansion is valid. [1]
(iii) Write down the equation of the tangent to the curve
1 + x2
y=
√ (4 + 2 x)
at the point where x = 0. [1]

2 y
B(−1,8) C(0,7)
• •

y=3

• x
A(−3,0) O

x=−4
The diagram shows the graph of y = f ( x ) . There is a maximum point B ( −1,8 ) and the curve
cuts the axes at the points A ( −3, 0 ) and C ( 0, 7 ) . The lines x = − 4 and y = 3 are asymptotes
of the curve.
Sketch, on separate diagrams, the graphs of
(i) y = f ′( x) , [2]

√{f ( 2 x )} ,
(ii) y=− 1 [3]

stating the equations of the asymptotes and the coordinates of the points corresponding to A,
B and C where possible.

IJC/2013/JC19740/01/Oct/13 *: not in topics tested for SRJC 2014 Promo

sgfreepapers.com 75
3

3 (i) Using the method of difference, show that


n
k k 1 1 
∑ ( r + 1)( r + 3) = 2  a − n + 2 − n + 3  ,
r =1
where a is a constant to be determined. [4]


k
(ii) Hence find the range of values of k such that ∑ ( r + 1)( r + 3) is at most 1. [2]
r =1

n
( ) = 1−
r 2r 2n +1
4 (i) Prove by induction that ∑ ( r + 2 )! ( n + 2 )!
for all positive integers n. [5]
r =1

2 n r 2r
( ).
(ii) Hence find an expression in terms of n for ∑ ( r + 2 )! [2]
r =n

5* Find


4
(i) dx , [3]
√ (5 + 4 x − 4 x2 )

∫ (3sin 2θ − secθ )
2
(ii) dθ . [4]

 
6 Referred to the origin O, the points A and B are such that OA = a and OB = b . The point P
on AB is such that AP : PB = 2 : 3 . It is given that a = √ 5 , b = 3 and OP is perpendicular to
AB.

(i) Show that a ⋅ b = −3 . [3]


(ii) Find the size of angle AOB. [2]

(iii) Find the length of projection of OB onto OA. [1]

IJC/2013/JC19740/01/Oct/13 *: not in topics tested for SRJC 2014 Promo [Turn over

sgfreepapers.com 76
4

7 A water tank in the shape of an inverted cone has a height twice that of its radius. Water is
poured into the cone. Given that, when the depth of the water is 10 cm, the volume of water is
increasing at a rate of 10π cm3s −1 , find the rate of increase at this instant of
(i) the slant height of the cone in contact with the water, [5]
(ii) the curved surface area of the cone in contact with the water. [2]
1 2
[The volume of a cone is π r h and the curved surface area is π rl .]
3

8 The equation of a curve is x 2 − 2 xy + 2 y 2 = −12 .

(i) Find the equations of the tangent and normal to the curve at the point P ( 2,4 ) . [5]
(ii) The tangent at P meets the y-axis at A and the normal at P meets the x-axis at B. Find
the area of triangle APB. [3]

9 (a) An arithmetic progression A has first term 3 and the sum of the terms from the
16th term to the 30th term inclusive is 2025. Show that the common difference is 6. [3]

If Sn is the sum of the first n terms of A, show that the sum of the first
n even-numbered terms of A, that is, the second, fourth, sixth, … terms, is given by
 1
 2 +  Sn . [2]
 n

4
(b) A geometric series G has first term 30 and common ratio − . Write down the sum,
5
Sn , of the first n terms of the series. [1]

Find the least value of n for which the magnitude of the difference between Sn and
the sum to infinity of the series is less than 0.004. [3]

A new series is formed by taking the reciprocal of the corresponding terms of G.


Determine if the new series is convergent. [1]

IJC/2013/JC19740/01/Oct/13 *: not in topics tested for SRJC 2014 Promo

sgfreepapers.com 77
5

10* (i) By successively differentiating ln ( 3 + x ) , find the Maclaurin’s series for ln ( 3 + x ) ,


up to and including the term in x3 . [3]

1
(ii) Given that θ is small, find the expansion of ( )
2 − cos 5θ 2 2 in ascending

powers of θ , up to and including the term in θ 4 . [2]

Two particles A and B produce y units of energy when they are x units away from their
original position at x = 0 . The energy produced by particles A and B can be found by the
equations
y = ln ( 3 + x ) and
1
(
y = 2 − cos 5 x 2 2)
respectively, where x ≥ 0 .

(iii) Explain in the context of the question, what is meant by the solution to the
equation
1
( )
ln ( 3 + x ) = 2 − cos 5 x 2 2 . [1]

(iv) Using your answers from parts (i) and (ii), find an estimate for the maximum distance
from the original position such that the difference in energy produced by both
particles is at most 0.4 units. [You may assume that both particles are at the same
distance from the original position.] [2]

11 (i) Find a vector equation of the line through the points A and B with position vectors
3i + 4 j + 5k and −i + 12 j + 9k respectively. [2]

(ii) The perpendicular to this line from the point C with position vector 2i + j − 2k meets
the line at the point N. Find the position vector of N. [3]

(iii) Find a Cartesian equation of the line AC. [2]

(iv) Use a vector product to find the exact area of triangle OAB. [3]

IJC/2013/JC19740/01/Oct/13 *: not in topics tested for SRJC 2014 Promo[Turn over

sgfreepapers.com 78
6

12 A container is made up of an open cylinder of varying height h cm and varying radius r cm,
and a hollow hemispherical lid of varying radius r cm. It costs 5 cents per square centimetre
to manufacture the base, 3 cents per square centimetre to manufacture the curved surface of
the cylinder and 4 cents per square centimetre to manufacture the curved surface of the
hemisphere.

(i) Given that the cylinder is of fixed volume V cm 3 , show that the manufacturing cost
1
 3V  3
of the container is minimum when r is   . [7]
 13π 

(ii) Using the value of r in part (i) and taking V to be 30, find the maximum number of
containers that a person can buy if he has $22. [2]
[The surface area of a sphere is 4π r 2 .]

13 The function f is defined as follows:


1
f:x for x ∈ , x ≠ −2, x ≠ 2.
2
x −4

(i) Sketch the graph of y = f ( x ) . [2]


The function g is defined as follows:
1
g:x for x ∈ , x ≠ a, x ≠ 3, x ≠ b.
x−3
It is given that the function fg exists.
(ii) Find the values of a and b. [2]

(iii) Show that fg ( x ) =


( x − 3 )2 . [2]
( 2 x − 5)( 7 − 2 x )
(iv) Solve the inequality fg ( x ) > 0. [3]
(v) Find the range of fg. [3]
IJC/2013/JC19740/01/Oct/13 *: not in topics tested for SRJC 2014 Promo

sgfreepapers.com 79
2013 H2 Maths MCE_Marking Scheme

1 + x2
1* (i) Find the expansion of in ascending powers of x, up to and including the
√ (4 + 2 x)
term in x 2 . [3]

(ii) State the range of values of x for which this expansion is valid. [1]

(iii) Write down the equation of the tangent to the curve


1 + x2
y=
√ (4 + 2 x)
at the point where x = 0. [1]

1(i) 1 + x2
√ (4 + 2 x)
1
( )
= 1 + x2 ( 4 + 2x )

2

1

1
(
= 1 + x2
2
)  x
1 + 
 2
2

  1 3 
  − ×−  2 
1
2
( 
)


1
2


x
2

= 1 + x 2  1 +  −   +  2
  2!
2  x 
   + ... 
 2  
   
1
(  x 3
)
= 1 + x 2  1 − + x 2 + ... 
2  4 32


1 1 3 1
= − x + x 2 + x 2 + ...
2 8 64 2
1 1 35 2
= − x + x + ...
2 8 64

(ii) x
<1
2
x
−1 < < 1
2
−2 < x < 2
(iii) 1 1
y= − x
2 8

*: Not in topics tested for 2014 SRJC Promo

sgfreepapers.com 80
2

2 y
B(−1,8) C(0,7)
• •

y=3

• x
A(−3,0) O

x=−4
The diagram shows the graph of y = f ( x ) . There is a maximum point B ( −1,8 ) and the curve
cuts the axes at the points A ( −3, 0 ) and C ( 0, 7 ) . The lines x = −4 and y = 3 are asymptotes
of the curve.

Sketch, on separate diagrams, the graphs of


(i) y = f ′( x) , [2]

√{f ( 2 x )} ,
(ii) y=− 1 [3]

stating the equations of the asymptotes and the coordinates of the points corresponding to A,
B and C where possible.

IJC/2013/JC19740/01/Oct/13 *: Not in topics tested for 2014 SRJC Promo

sgfreepapers.com 81
3

2(i)
y

y = f′ (x)

y=0 x

B′(−1,0) O

x = −4

(ii)
y
x
A′(−6,0) O

y = −√3

y = −√f(0.5x)
• • C ′ (0,−√7)
B′(−2,−√8)

IJC/2013/JC19740/01/Oct/13 *: Not in topics tested for 2014 SRJC Promo

sgfreepapers.com 82
4

3 (i) Using the method of difference, show that


n
k k 1 1 
∑ ( r + 1)( r + 3) = 2  a − n + 2 − n + 3  ,
r =1
where a is a constant to be determined. [4]


k
(ii) Hence find the range of values of k such that ∑ ( r + 1)( r + 3) is at most 1. [2]
r =1

3(i) k k 1 1 
=  − 
( r + 1)( r + 3) 2  r +1 r + 3 

n
k k n  1 1 
∑ ( r + 1)( r + 3)
r =1
= ∑  − 
2 r =1  r + 1 r + 3 
k1 1
=  −
22 4
1 1
+ −
3 5
1 1
+ −
4 6
1 1
+ −
5 7
+ …
+ …
1 1
+ −
n −1 n +1
1 1
+ −
n n+2
1 1 
+ − 
n +1 n+3
k5 1 1 
=  − − 
2 6 n+2 n+3
5
a=
6

IJC/2013/JC19740/01/Oct/13 *: Not in topics tested for 2014 SRJC Promo

sgfreepapers.com 83
5

(ii) ∞
k k 5 5k
∑ ( r + 1)( r + 3) = 2  6  = 12
r =1

5k
≤1
12
12
⇒k≤
5

n
( )
r 2r 2n+1
4 (i) Prove by induction that ∑ ( r + 2 )!
r =1
= 1−
( n + 2 )!
for all positive integers n. [5]

2n
( )
r 2r
(ii) Hence find an expression in terms of n for ∑ ( r + 2 )! .
r =n
[2]

4(i) n
( )
r 2r 2n +1
Let Pn denote ∑ ( r + 2 )!
r =1
= 1−
( n + 2 )!
for n ∈+ .

When n = 1,

1
r ( 2r )
LHS = ∑ ( r + 2 )!
r =1

(1) ( 21 )
=
(1 + 2 )!
2
=
3!
1
=
3

21+1
RHS = 1 −
(1 + 2 )!
4
= 1−
3!
2
= 1−
3
1
=
3

Therefore, P1 is true.

Assume Pk is true for some k ∈  + ,


IJC/2013/JC19740/01/Oct/13 *: Not in topics tested for 2014 SRJC Promo

sgfreepapers.com 84
6

k r 2r ( ) 2k +1
i.e. ∑ ( r + 2 )!
r =1
= 1−
( k + 2 )!
.

Want to prove Pk +1 is true,


k +1
( )
r 2r 2k + 2
i.e. ∑ ( r + 2 )!
r =1
=1−
( k + 3 )!
.

k +1
r ( 2r )
LHS = ∑ ( r + 2 )!
r =1
k
r ( 2r )
( k + 1) ( 2k +1 )
= ∑r =1
( r + 2 )! ( k + 3)!
+

2k +1  ( k + 1) ( 2 )
k +1

= 1 −  +
 ( k + 2 ) ! ( k + 3)!
 ( 2k +1 ) ( k + 3) ( k + 1) ( 2k +1 ) 
=1 −  − 
 ( k + 3) ! ( k + 3)! 
 ( 2k +1 ) ( k + 3) − ( k + 1)  
=1 −  
 ( k + 3)! 
 ( 2k +1 ) ( 2 ) 
=1 −  
 ( k + 3)! 
2k +2
=1 −
( k + 3)!
= RHS

Thus Pk is true ⇒ Pk +1 is true.

Since P1 is true, and Pk is true ⇒ Pk +1 is true, by mathematical


induction, Pn is true for all n ∈+ .
(ii) 2n r ( 2r )
∑ ( r + 2 )!
r =n

2n r ( 2r ) n −1 r ( 2r )
=∑
( r + 2 )! ∑

r =1 r =1 ( r + 2 ) !

 22 n +1   2n 
= 1 −   − 1 − 
 ( 2n + 2 ) !  ( n + 1) !
2n 22 n +1
= −
( n + 1)! ( 2n + 2 )!

IJC/2013/JC19740/01/Oct/13 *: Not in topics tested for 2014 SRJC Promo

sgfreepapers.com 85
7

5* Find


4
(i) dx , [3]
√ (5 + 4x − 4x2 )

∫ (3sin 2θ − secθ )
2
(ii) dθ . [4]

5(i) 5 + 4x − 4x2
 5
= −4  x 2 − x − 
 4
 1 1 5
2 2

= −4   x −  −   − 
 2   2  4 

 1 6
2
3  1 
2

= −4   x −  −  = 4  −  x −  
 2  4   2  2  


4
dx
5 + 4 x − 4 x2

∫ ∫
4 4
= dx or dx
4  32 − ( x − )  6 − ( 2 x − 1)
2
1 2
 2 


4
= dx
− ( x − 12 )
3 2
2 2

 x− 1   2x −1 
= 2sin −1  2
+C or 2 sin −1  +C
 3   6 
 2 

(ii) 2
∫ (3sin 2θ − sec θ ) dθ
= ∫ 9sin 2 2θ − 6sin 2θ sec θ + sec2 θ dθ
9
∫ (1 − cos 4θ )dθ − 6∫ 2sin θ cos θ sec θ dθ + ∫ sec θ dθ
2
=
2
9
= ∫ (1 − cos 4θ )dθ − 12 ∫ sin θ dθ + ∫ sec 2θ dθ
2
9 1 
=  θ − sin 4θ  − 12 ( − cos θ ) + tan θ + c
2 4 
9 9
= θ − sin 4θ + 12 cos θ + tan θ + c
2 8

IJC/2013/JC19740/01/Oct/13 *: Not in topics tested for 2014 SRJC Promo

sgfreepapers.com 86
8
 
6 Referred to the origin O, the points A and B are such that OA = a and OB = b . The point P
on AB is such that AP : PB = 2 : 3 . It is given that a = √ 5 , b = 3 and OP is perpendicular to
AB.
(i) Show that a ⋅ b = −3 . [3]
(ii) Find the size of angle AOB. [2]

(iii) Find the exact length of projection of OB onto OA. [1]

6(i)  1
By Ratio Theorem, OP = ( 3a + 2b ) .
5
 
Since OP ⊥ AB, OP ⋅ AB = 0 .
1
( 3a + 2b ) ⋅ ( b − a ) = 0
5
3a ⋅ b − 3a ⋅ a + 2b ⋅ b − 2b ⋅ a = 0
2 2
a ⋅b − 3 a + 2 b = 0
a ⋅ b − 15 + 18 = 0
a ⋅ b = −3
(ii) a ⋅b
cos ∠AOB =
a b
−3
=
3 5
1
= −
5
∠AOB = 116.6 (or 2.03 rad)

(iii) Length of projection of OB onto OA
b ⋅a
=
a
3
=
5

IJC/2013/JC19740/01/Oct/13 *: Not in topics tested for 2014 SRJC Promo

sgfreepapers.com 87
9

7 A water tank in the shape of an inverted cone has a height twice that of its radius. Water is
poured into the cone. Given that, when the depth of the water is 10 cm, the volume of water is
increasing at a rate of 10π cm 3s −1 , find the rate of increase at this instant of
(i) the slant height of the cone in contact with the water, [5]
(ii) the curved surface area of the cone in contact with the water. [2]
1 2
[The volume of a cone is π r h and the curved surface area is π rl .]
3

7(i) Let the radius of the water surface, the depth of the water, the slant height of the water and
the volume of the water at time t seconds be r cm, h cm, l cm and V cm3 respectively.

1 1 2
V = π r 2 h = π r 2 (2r ) = π r 3
3 3 3
dV dV dr dr
= ⋅ = 2π r 2
dt dr dt dt
h = 2r = 10 ⇒ r = 5
dV
When = 10π and r = 5,
dt
dr
10π = 2π (5)2
dt
dr 1
=
dt 5
Using Pythagoras' theorem,
l 2 = ( 2r ) 2 + r 2
l = 5r
dl dl dr dr 1 5
= ⋅ = 5 = 5  = or 0.44721
dt dr dt dt 5 5

5
The rate of increase of the slant height of the cone in contact with the water is cms−1
5
(or 0.447 cms−1) .

IJC/2013/JC19740/01/Oct/13 *: Not in topics tested for 2014 SRJC Promo

sgfreepapers.com 88
10

7(ii) Let the curved surface area of the water at time t seconds be A cm 2 .
A = π rl = π r ( 5r ) = 5π r 2
dA dA dr dr
= ⋅ = 2 5π r
dt dr dt dt

dr 1
When r = 5, = .
dt 5
dA 1
= 2 5π ( 5 )   = 2 5π = 14.0496
dt 5
The rate of increase of the curved surface area of the cone in contact with the water is
2 5π cm 2s −1 (or 14.0 cm2s−1).

IJC/2013/JC19740/01/Oct/13 *: Not in topics tested for 2014 SRJC Promo

sgfreepapers.com 89
11

8 The equation of a curve is x 2 − 2 xy + 2 y 2 = −12 .

(i) Find the equations of the tangent and normal to the curve at the point P ( 2, 4 ) . [5]

(ii) The tangent at P meets the y-axis at A and the normal at P meets the x-axis at B. Find
the area of triangle APB. [3]

8(i) x 2 − 2 xy + 2 y 2 = −12
 dy  dy
2x −  2x + 2 y  + 4 y =0
 dx  dx
dy dy
2x − 2 y = 2x − 4 y
dx dx
dy
2x − 2 y = (2x − 4 y )
dx
dy 2 x − 2 y
=
dx 2 x − 4 y
x− y
=
x − 2y
At P ( 2, 4 ) :
dy 2 − 4
=
dx 2 − 8
1
=
3
Equation of tangent:
1
y−4= ( x − 2)
3
1 10
y= x+
3 3
Gradient of normal = −3
Equation of normal:
y − 4 = −3 ( x − 2 )
y = −3 x + 10

IJC/2013/JC19740/01/Oct/13 *: Not in topics tested for 2014 SRJC Promo

sgfreepapers.com 90
12

8(ii) When tangent meets y -axis at A, x = 0


10
y=
3
 10 
∴ A  0, 
 3
When normal meets x-axis at B, y = 0
3 x = 10
10
x=
3
 10 
∴B ,0
 3 
Area of triangle APB
1
= × AP × BP
2
1 40 160
= × ×
2 9 9
40
= units 2 (or 4.44 units 2 )
9

IJC/2013/JC19740/01/Oct/13 *: Not in topics tested for 2014 SRJC Promo

sgfreepapers.com 91
13

9 (a) An arithmetic progression A has first term 3 and the sum of the terms from the
16th term to the 30th term inclusive is 2025. Show that the common difference is 6. [3]

If Sn is the sum of the first n terms of A, show that the sum of the first
n even-numbered terms of A, that is, the second, fourth, sixth, … terms, is given by
 1
 2 +  Sn . [2]
 n

9(a) S30 − S15 = 2025


30 15
 2 ( 3) + 29d  −  2 ( 3) + 14d  = 2025
2 2
330d = 1980
d=6

n
Sn =  6 + ( n − 1) 6  = 3n 2
2
Sum of 1st n even-numbered terms
n
=  2 ( 3 + 6 ) + ( n − 1)12 
2
n
= [6 + 12n ]
2
1 
= 3n 2  + 2 
n 
 1
=  2 +  Sn
 n

IJC/2013/JC19740/01/Oct/13 *: Not in topics tested for 2014 SRJC Promo

sgfreepapers.com 92
14

4
9(b) A geometric series G has first term 30 and common ratio − . Write down the sum,
5
S n , of the first n terms of the series. [1]

Find the least value of n for which the magnitude of the difference between S n and
the sum to infinity of the series is less than 0.004. [3]

A new series is formed by taking the reciprocal of the corresponding terms of G.


Determine if the new series is convergent. [1]

9(b)   4 n 
30 1 −  −  
  5   50   4  n 
Sn =  = 1 −  −  
 4 3   5 
 
1−  − 
 5
Sn − S∞ < 0.004

50   4   50
n
1 −  −   − < 0.004
3   5  3
 
n
50  4 
  < 0.004
3 5
n
4 3
  < 0.004 ×
5 50
 3 
ln  0.004 × 
n> 
50 
4
ln  
5
n > 37.352
Least value of n is 38.

1 1 1 1 1
New series + + 2 + 3 + 4 +… is a geometric series with
a ar ar ar ar
1 5
common ratio =− .
r 4
1 5
Since = > 1 , the new series is not convergent.
r 4

IJC/2013/JC19740/01/Oct/13 *: Not in topics tested for 2014 SRJC Promo

sgfreepapers.com 93
15

10* (i) By successively differentiating ln ( 3 + x ) , find the Maclaurin’s series for ln ( 3 + x ) , up


to and including the term in x 3 . [3]

1
(ii) Given that θ is small, find the expansion of ( 2 − cos 5θ 2 ) 2 in ascending
powers of θ , up to and including the term in θ 4 . [2]

Two particles A and B produce y units of energy when they are x units away from their
original position at x = 0 . The energy produced by particles A and B can be found by the
equations
y = ln ( 3 + x ) and
1
y = ( 2 − cos 5 x 2 ) 2
respectively, where x ≥ 0 .

(iii) Explain in the context of the question, what is meant by the solution to the
equation
1
ln ( 3 + x ) = ( 2 − cos 5 x 2 ) 2 . [1]

(iv) Using your answers from parts (i) and (ii), find an estimate for the maximum distance
from the original position such that the difference in energy produced by both
particles is at most 0.4 units. [2]
[You may assume that both particles are at the same distance from the original
position.]

IJC/2013/JC19740/01/Oct/13 *: Not in topics tested for 2014 SRJC Promo

sgfreepapers.com 94
16

10(i) Let y = ln(3 + x )


dy −1
= (3 + x )
dx
d2 y −2
2
= − (3 + x )
dx
d3 y −3
= 2 (3 + x )
dx3

When x = 0 ,
dy 1 d2 y 1 d3 y 2
y = ln 3, = , 2
=− , =
dx 3 dx 9 dx 3 27
2 3
x x x
∴ y = ln 3 + − + +…
3 18 81
(ii) Given that θ is small,
1
  2
( 5θ 2 ) 
2
1
( 2 − cos 5θ )
2 2 =  2 − 1 −
  2 
 + …

   
1
 25 2
= 1 + θ 4 + … 
 2 
 1  25
= 1+   θ 4 +…
2 2
25
= 1+ θ 4 +…
4
(iii) The solution (x value) denotes the distance in units
where both particles produce the same number of
units of energy.
(iv) x x 2 x3  25 4 
ln 3 + − + − 1 + x  ≤ 0.4
3 18 81  4 

Or

x x 2 x3  25 4 
−0.4 ≤ ln 3 + − + − 1 + x  ≤ 0.4
3 18 81  4 

From GC, x ≤ 0.57298752 (given x ≥ 0 )

An estimate for the maximum distance is 0.572 units.


(3 s.f.)

IJC/2013/JC19740/01/Oct/13 *: Not in topics tested for 2014 SRJC Promo

sgfreepapers.com 95
17

11 (i) Find a vector equation of the line through the points A and B with position vectors
3i + 4 j + 5k and −i + 12 j + 9k respectively. [2]

(ii) The perpendicular to this line from the point C with position vector 2i + j − 2k meets
the line at the point N. Find the position vector of N. [3]

(iii) Find a Cartesian equation of the line AC. [2]

(iv) Use a vector product to find the exact area of triangle OAB. [3]

IJC/2013/JC19740/01/Oct/13 *: Not in topics tested for 2014 SRJC Promo

sgfreepapers.com 96
18

11(i)  −1   3   −4   −1
        
AB =  12  −  4  =  8  = 4  2 
 9  5  4  1
       
 3  −1
   
l AB : r =  4  + λ  2  , λ ∈ 
5  
  1
 −1   −1
   
or l AB : r =  12  + λ  2  , λ ∈ 
9 1
   
(ii) Since N lies on line AB,
3  −1 
    
ON =  4  + λ  2  for some λ ∈ 
5  
  1
 3  −1  2   1   −1
          
CN =  4  + λ  2  −  1  =  3  + λ  2 
5  1   −2   7  1
         
Since CN ⊥ AB,
 −1 
  
CN ⋅  2  = 0
1
 
 1   −1   −1
     
 3  + λ  2   ⋅  2  = 0
 7     
 1    1 
 1   −1  −1  −1
       
 3⋅ 2  + λ  2 ⋅ 2  = 0
7  1     
    1 1
12 + 6λ = 0
λ = −2
 3   −1  5 
      
ON =  4  − 2  2  =  0 
 5  1   3
     

IJC/2013/JC19740/01/Oct/13 *: Not in topics tested for 2014 SRJC Promo

sgfreepapers.com 97
19

11(iii)  2   3   −1  1
        
AC =  1  −  4  =  −3  = −  3 
 −2   5   −7   
      7

Cartesian eqn of line AC:


y −4 z −5
x−3 = =
3 7
y −1 z + 2
or x−2 = =
3 7
(iv) Area of triangle OAB
1  
= OA × OB
2
 3   −1
1    
=
2  4  ×  12 
 5  9 
   
 −24 
1  
=
2  −32 
 40 
 
 −3 
= × 8  −4 
1
2  5
 
= 4 9 + 16 + 25
= 4 50
= 20 2

IJC/2013/JC19740/01/Oct/13 *: Not in topics tested for 2014 SRJC Promo

sgfreepapers.com 98
20

12 A container is made up of an open cylinder of varying height h cm and varying radius r cm,
and a hollow hemispherical lid of varying radius r cm. It costs 5 cents per square centimetre
to manufacture the base, 3 cents per square centimetre to manufacture the curved surface of
the cylinder and 4 cents per square centimetre to manufacture the curved surface of the
hemisphere.

(i) Given that the cylinder is of fixed volume V cm3 , show that the manufacturing cost
1
 3V  3
of the container is minimum when r is   . [7]
 13π 

(ii) Using the value of r in part (i) and taking V to be 30, find the maximum number of
containers that a person can buy if he has $22. [2]
[The surface area of a sphere is 4π r 2 .]

12(i) V = π r 2 h
V
∴h =
π r2

IJC/2013/JC19740/01/Oct/13 *: Not in topics tested for 2014 SRJC Promo

sgfreepapers.com 99
21

Let C cents be the manufacturing cost of the container.


C = 4 ( 2π r 2 ) + 3 ( 2π rh ) + 5 (π r 2 )
 V 
=13π r 2 + 6π r  2 
πr 
6V
=13π r 2 +
r
dC
= 13π ( 2r ) + 6V ( −r −2 )
dr
6V
= 26π r − 2
r
dC
Let =0
dr
6V
26π r − 2 = 0
r
26π r = 6V
3

6V
r3 =
26π
3V
=
13π
3V
r=3
13π
2
dC
= 26π − 6V ( −2r −3 )
dr 2
12V
= 26π + 3
r
12V
= 26π +
 3V 
 
 13π 
= 26π + 52π
= 78π > 0
Hence, the manufacturing cost is minimum
3V
when r = 3 . [Shown]
13π

IJC/2013/JC19740/01/Oct/13 *: Not in topics tested for 2014 SRJC Promo

sgfreepapers.com 100
22

(ii) 6V
C = 13π r 2 +
r
2
 3V  6V
= 13π  3  +
 13π  3
3V
13π
2
 90  3 180
= 13π   +
 13π 
1
 90  3
 
 13π 
= 207.48 cents
= $2.0748

22
2.0748
= 10.603
∴ Maximum number of containers he can buy is 10.

13 The function f is defined as follows:


1
f:x 2 for x ∈ , x ≠ −2, x ≠ 2.
x −4

(i) Sketch the graph of y = f ( x ) . [2]

The function g is defined as follows:


1
g:x for x ∈ , x ≠ a , x ≠ 3, x ≠ b.
x−3

It is given that the function fg exists.

(ii) Find the values of a and b. [2]


( x − 3)
2

(iii) Show that fg ( x ) = . [2]


( 2 x − 5)( 7 − 2 x )
(iv) Solve the inequality fg ( x ) > 0. [3]
(v) Find the range of fg. [3]

13(i)
y
y = f(x)

IJC/2013/JC19740/01/Oct/13 *: Not in topics tested for 2014 SRJC Promo


−2 2 y=0
−1/4

sgfreepapers.com 101
23

(ii) For fg to exist, Rg ⊆ Df .


Hence, g ( x ) cannot take the values – 2 and 2.
1 5
= −2 ⇒ x =
x −3 2
1 7
= 2⇒ x =
x −3 2
5 7
The values of a and b are and .
2 2

(iii) 1
f g(x) = 2
 1 
  −4
 x −3
1
=
1 − 4 ( x − 3)
2

( x − 3)
2

( x − 3)
2

=
12 −  2 ( x − 3) 
2

( x − 3)
2

=
1 + 2 ( x − 3)  1 − 2 ( x − 3) 

( x − 3)
2

= (shown)
( 2 x − 5)( 7 − 2 x )
(iv) ( x − 3)
2

>0
( 7 − 2 x )( 2 x − 5)
– + + –

IJC/2013/JC19740/01/Oct/13 *: Not in topics tested for 2014 SRJC Promo

sgfreepapers.com 102
24

2.5 3 3.5

Solving,
5 7
< x < 3 or 3 < x <
2 2
5 7
or <x< , x≠3
2 2

(v) Sketching the graph of y = g(x),


y
y = g(x)

(7/2, 2)
° x
(5/2, −2)°
O 3

x=3

Rg = { y ∈ : y ≠ −2, 0, 2}
Referring to the graph of y = f(x) in part (i),
 1 
Rfg =  y ∈ : y < − or y > 0 
 4 

OR

Sketch the graph of y = fg(x).

O 5/2 3 7/2 x
o y = − 1/4

y = fg(x)

x = 5/2 x = 7/2

IJC/2013/JC19740/01/Oct/13 *: Not in topics tested for 2014 SRJC Promo

sgfreepapers.com 103
25

From the graph of y = fg(x),


 1 
Rfg =  y ∈  : y < − or y > 0  .
 4 

IJC/2013/JC19740/01/Oct/13 *: Not in topics tested for 2014 SRJC Promo

sgfreepapers.com 104
2

1* Expand

(1 + 2 x ) 4 + 3x

in ascending powers of x , up to and including the term in x 2 . [3]

Determine the range of values of x for which the expansion is valid. [1]

2n − 1 A B
2 (i) Given that can be written in the form + , find the values of the
(n − 1)2 n 2 (n − 1)2 n2
constants A and B. [2]

N
2r − 1
(ii) Hence find ∑ (r − 1)2 r 2 . [3]
r =2

N
2r + 1
(iii) Using your answer in (ii), find ∑ r (r + 1) 2
2
. [2]
r =1

3 Machines A and B are used to cut metal bars of length 30m into pieces of decreasing lengths.

(i) The lengths of all the pieces cut by machine A form an arithmetic progression with
common difference d m. If the total length of the first 25 pieces cut is 25m and the length
of the 25th piece is 0.5m, find the value of d. [3]

(ii) The length of the first piece cut by machine B is 2m and the lengths of all the pieces cut
form a geometric progression. The 25th piece cut by machine B has length 0.5m. Find the
maximum number of pieces of metal bars cut. [4]

4 A sequence u1, u2 , u3 ,… is given by


4 + 2un
u1 = 1 and un +1 = for n ≥ 1 .
5
(i) Find the values of u2 and u3 . [2]

(ii) It is given that un → l as n → ∞ . Showing your working, find the exact value of l. [2]

(iii) For this value of l, use the method of mathematical induction to prove that
n −1
1 2 [4]
un = l −   for n ≥ 1 .
3 5 

*: Not in the topics tested in 2014 SRJC Promo

sgfreepapers.com 105
3

x 2 − 3x + 3
5 The curve C has equation y = .
1− x

(i) Find the equations of the asymptotes of C. [2]

(ii) Prove using an algebraic method, that y cannot lie between two certain values (to be
determined). [3]

(iii) Sketch the curve C clearly labeling all asymptotes, turning points and axial intercepts. [3]

6 The diagram shows the graph of y = f(x). It has a vertical asymptotes at x = 1 and x = ‒1. It has
a stationary point of inflexion at the origin.
x = ‒1 y x=1

x
O

Sketch on separate diagrams, the graphs of


(i) y = f (2 − x) , [3]
(ii) y = − f ( x) , [2]

(iii) y = f '( x) . [2]

*: Not in the topics tested in 2014 SRJC Promo


[Turn over

sgfreepapers.com 106
4

7 (a) Show that x 2 − 3 x + 5 is always positive and solve the inequality

x2 − 3x + 5
< 0. [4]
(4 − x)( x − 2)

( x + 2)2 − 3 x − 1 [2]
Hence find the solution for the inequality <0.
x(2 − x)

(b) A factory produces 3 brands of drinks, A, B and C. The total price of 1 litre of A, 1 litre
of B and 2 litres of C is $9. The total price of 1 litre of B and 1 litre of C is $3.50. The
total price of 2.5 litres of B and 2 litres of C is twice the price of 1 litre of A.
Write down and solve the equations to find the price of each litre of A, B and C. [4]

8 The functions f and g are defined by


f : x  3ln ( x 2 + 1) , 0 ≤ x ≤ 2,
g : x  e x + 1, x ≥ 0.

(i) Find f −1 ( x) , stating the domain of f −1 . [3]

(ii) Sketch the graphs of y = f ( x) and y = f −1 ( x ) on a single diagram. State the geometrical

relationship between the graphs and hence state the number of solutions to f ( x ) = f −1 ( x) . [4]

(iii) Show that gf exists, define it in a similar form and find its range. [4]

*: Not in the topics tested in 2014 SRJC Promo

sgfreepapers.com 107
5

9 (a)

A closed cylindrical can with base radius r and height h has a fixed volume V.
(i) Show that the total surface area of the can, A, is given by
2V [1]
A = 2π r 2 + .
r
(ii) Find h in terms of r when the minimum surface area is achieved. [4]

(b)

Wall
2m
y

A ladder of length 2 m, leaning against the wall, slips in such a way that x increases at
a rate of 0.02 ms −1 . Find the rate of decrease of y at the instant when x is 1 m. [4]

*: Not in the topics tested in 2014 SRJC Promo [Turn over

sgfreepapers.com 108
6

10 (a) The curve C is defined by


x = e 3t , y = t2 , where t ≥ 0 .
dy dy [3]
(i) Find in terms of t and determine the value of t for which is zero.
dx dx

(ii) Sketch the graph of C. [2]

(b) The equation of a curve C is x 2 − 2 xy + 2 y 2 = k , where k is a constant.

dy [3]
Find in terms of x and y.
dx

Given that C has two points for which the tangents are parallel to the line y = x , find
the range of values of k. [3]

Given that k = 4 , find the exact coordinates of each point on the curve C at which the
tangent is parallel to the y-axis. [4]

11* (a) Find


[3]
∫x e
2 x
(i) dx ,
π
(ii) ∫0
3
sin 2 2 x dx , leaving your answer in exact form. [3]

(b) Using the substitution u = 3 x − 1 , find

∫ (3x − 1)
9x [3]
2
dx .

(c) Given that x + 1 = A ( 2 x − 4 ) + B for all values of x , find the constants A and B.

Hence, find


x +1 [5]
2
dx .
x − 4 x + 13

[End of Paper]

*: Not in the topics tested in 2014 SRJC Promo

sgfreepapers.com 109
Jurong Junior College
2013 JC1 H2 Mathematics Promo Solutions
(Qn 1 and 11 are not in topics tested for 2014 SRJC Promo)
Qn Solution
1 (1 + 2 x ) 4 + 3x
1
 3x  2
= (1 + 2 x ) 2 1 + 
 4 
 3x 9 x2 
= 2 (1 + 2 x ) 1 + − + ... 
 8 128 
19 x 87 x 2
= 2+ + ...
4 64
Validity:
3x
<1
4
4 4
− <x<
3 3

Qn Solution
2 2n − 1 A B
(i) 2 2
= 2
+ 2
(n − 1) n (n − 1) n
An 2 + B(n − 1) 2
=
(n − 1) 2 n 2

2n − 1 = An 2 + B (n − 1)2
When n = 0, B = −1.
When n = 1, A = 1.

2n − 1 1 1
∴ 2 2
= 2
− 2
(n − 1) n (n − 1) n
N
2r − 1 N  1 1
(ii) ∑ (r − 1)2 r 2 = ∑  2
− 
r2 
r =2 r =2  (r − 1)
 1 1
=  2− 2
 1 2
1 1
+ 2− 2
2 3
1 1
+ 2− 2
3 4
+ ...
1 1  1
+ 2
− 2  = 1− 2
( N − 1) N  N

sgfreepapers.com 110
N N +1
2r + 1 2r − 1

r =1
2
r (r + 1) 2
= ∑ 2 2
r = 2 ( r − 1) r

1
= 1−
( N + 1)2

Qn Solution
3 (i) S 25 = 25

25
[ a + 0.5] = 25
2
⇒ a = 1.5

a + 24d = 0.5
1
Subst a = 1.5, d = − = 0.0417 (to 3 s.f)
24
(ii) GP a = 2

ar 24 = 0.5
2r 24 = 0.5
1
r 24 =
4
1
r = 24 =0.94387 (to 5 s.f)
4
S n ≤ 30
  1 n 
2 1 −  24  
  4 
  ≤ 30
 1
1 −  24 
 4
n
 1
1 −  24  ≤ 0.84195
 4
n
 1
 24  ≥ 0.15805
 4
ln 0.15805
n≤
1
ln 24
4
n ≤ 31.931
Therefore maximum number of pieces cut = 31.

sgfreepapers.com 111
Alternative Solution
S n ≤ 30
2 1 − ( 0.94387 ) 
n
  ≤ 30
1 − ( 0.94387 )
n
1 − ( 0.94387 ) ≤ 0.84195
( 0.94387 )n ≥ 0.15805
ln 0.15805
n≤
ln 0.94387
n ≤ 31.9

Therefore maximum number of pieces cut = 31.

Qn Solution
4 4 + 2(1) 6
(i) u2 = = = 1.2
5 5
6
4 + 2( )
u3 = 5 = 32 = 1.28
5 25
(ii) As n → ∞, un → l , un+1 → l .
4 + 2l
l=
5
4
l=
3
n −1
4 1 2
(iii) Let Pn be the statement un = −   for all n ≥ 1 .
3 3 5 
.
LHS of P1 = u1 = 1 (by defn)
1−1
RHS of P1 = 4 − 1  2  =
3
=1
3 3 5 3
∴ P1 is true.
k −1
4 1 2
Assume that Pk is true for some k ≥ 1 , ie uk = −  
3 3 5
k
4 1 2
We want to prove Pk +1 , ie uk +1 = −  
3 3 5 

LHS of Pk +1 = uk +1
4 + 2uk
=
5
4 2 4 1  2  
k −1

= +  −   
5 5  3 3  5  
3

sgfreepapers.com 112
k −1
12 8 1  2  2 
= + −   
15 15 3  5  5 
k
4 1 2
= −  
3 3 5 
= RHS of Pk +1
∴ Pk is true ⇒ Pk +1 is true.
∴ By Mathematical Induction, Pn is true for all n ≥ 1 .

5 i)
Asymptotes:
By Long Division,
x 2 − 3x + 3 1
y= = 2− x+
1− x 1− x
Asymptotes: x = 1, y = 2 − x
ii)
x 2 − 3x + 3
y=
1− x
y (1 − x ) = x 2 − 3 x + 3
x 2 + ( y − 3) x + 3 − y = 0
For no solutions, Discriminant < 0
2
( y − 3) − 4 (3 − y ) < 0
(y 2
− 6 y + 9 ) − (12 − 4 y ) < 0
y2 − 2 y − 3 < 0
( y − 3)( y + 1) < 0
∴−1 < y < 3
iii)
y

0 x

( 2, −1)
y = 2− x

x =1

sgfreepapers.com 113
6 i) y x=1 x=3

0 x
2

ii) x = ‒1 y x=1

x
0

iii) x = ‒1 y x=1

x
0

sgfreepapers.com 114
Qn Solution
7 2 2
2  3  3
(a) x − 3 x + 5 =  x −  −  −  + 5
 2  2
2
 3  11
=x−  +
 2 4
2
 3
Since  x −  ≥ 0 for all real values of x, ∴ x 2 − 3 x + 5
 2
is always positive.
x2 − 3x + 5
<0
(4 − x)( x − 2)

Since x 2 − 3 x + 5 is always positive, (4 − x)( x − 2) < 0

− + −

2 4

∴ x < 2 or x > 4 -------(1)

( x + 2)2 − 3 x − 1
<0
x(2 − x)
Replace x in eqn (1) with (x+2),
∴ x+2< 2 or x+2>4
⇒ x<0 or x>2

(b) Let the price of 1 litre of A, B and C be a, b and c


respectively.

Given that
a + b + 2c = 9
b + c = 3.50
2.5b + 2c = 2a ⇒ 2a − 2.5b − 2c = 0

Using GC, a = $4, b = $2, c = $1.50 .

sgfreepapers.com 115
Qn Solution
8 i)
y = 3ln ( x 2 + 1)
y
x = ± e 3 −1
y
3
x = e −1 since 0 ≤ x ≤ 2
x
∴ f −1 ( x ) = e 3 − 1, 0 ≤ x ≤ 3ln 5
ii) y

3ln5 y = f(x)

−1
2 y = f (x)

O 2 3ln5 x

They are reflections about y = x and


there are 2 solutions.
iii)
Rf = [ 0,3ln 5]
Dg = [ 0, ∞ )
Rf ⊆ Dg
∴ gf exists
3
gf ( x ) = ( x 2 + 1) + 1, 0≤ x≤2
Rgf = [ 2,126]

sgfreepapers.com 116
Qn Solution
9 (i) V = π r 2 h
(a) V
h= 2
πr

A = 2π r 2 + 2π rh
 V 
= 2π r 2 + 2π r  2 
πr 
2V
= 2π r 2 + (shown)
r
dA 2V
(ii) For min A, = 4π r − 2 = 0
dr r
3
4π r = 2V
1
 V 3
r = 
 2π 
d2 A 4V
2
= 4π + 3 > 0
dr r
Thus, A is minimum.
Substitute V = π r 2 h ,
1
 π r 2h 3
r = 
 2π 
r 2h
r3 =
2
h = 2r
(b) y = 22 − x 2
= 4 − x2
dy 1
= ( −2 x )
dx 2 4 − x 2
x
=−
4 − x2
dy dy dx
= ×
dt dx dt
x
=− × (0.02)
4 − x2
1
=− × (0.02)
4 − 12
= −0.011547
= −0.0115
∴ y decreases at a rate of 0.0115 ms-1.

sgfreepapers.com 117
Qn Solution
10(a) dx
(i) x = e 3t ⇒ = 3e3t
dt
d y
y = t2 ⇒ = 2t
dt
dy 2t
∴ = 3t
dx 3e
dy
When = 0,
dx
2t
=0
3e3t
t=0
(ii)

(b) x 2 − 2 xy + 2 y 2 = k … (1)
Differentiate throughout w.r.t. x.
 dy  dy
2x − 2  x + y  + 4 y =0
 dx  dx
dy y−x
=
dx 2 y − x
dy
For tangents which are parallel to the line y = x , = 1.
dx
y−x
=1
2y − x
y − x = 2y − x
y=0
Subst. y = 0 into (1):
x 2 − 2 x(0) + 2(0) 2 = k
x2 = k
Given that there are 2 tangents parallel to the line y = x ,
k >0

sgfreepapers.com 118
dy
For tangents which are parallel to the y-axis, is undefined.
dx
2y − x = 0
x = 2y
Subst. x = 2 y and k = 4 into (1):
(2 y )2 − 2(2 y ) y + 2 y 2 = 4
y=± 2
x = ±2 2
The coordinates are −2 2, − 2 ( ) and (2 )
2, 2 .

Qn Solution
11(a)
(i) ∫ x 2 e x d x = x 2 e x − 2 ∫ xe x d x

= x 2 e x − 2  xe x − ∫ e x dx 

= x 2 e x − 2  xe x − e x  + c
= ex ( x2 − 2 x + 2) + c
(ii) π
1 π3
∫ sin 2 x dx = ∫ 1 − cos 4 x dx
3 2
0 2 0
π
1 1 3
=  x − sin 4 x 
2 4 0
1 π 1 4π 
=  − sin
23 4 3 
1 π 3
=  + 
23 8 
(b) 9x u +1
∫ ( 3 x − 1) 2
dx = ∫ 2 du
u
1
=∫ + u −2 du
u
1
= ln u − + c
u
1
= ln 3 x − 1 − +c
3x − 1

10

sgfreepapers.com 119
(c) x + 1 = A ( 2 x − 4) + B
= 2 Ax − 4 A + B
By comparing coefficients,
1
2A =1⇒ A =
2
−4 A + B = 1 ⇒ B = 3

x +1
∫x 2
− 4 x + 13
dx

1
( 2x − 4) + 3
=∫22 dx
x − 4 x + 13
1 2x − 4 1
= ∫ 2 dx + 3∫ dx
2 x − 4 x + 13 ( x − 2) 2 + 32
1 1  x−2
= ln x 2 − 4 x + 13 + 3   tan −1  +c
2 3  3 
1  x−2
= ln ( x 2 − 4 x + 13) + tan −1  +c
2  3 

11

sgfreepapers.com 120
2013 MJC H2 MATH (9740) JC 1 PROMOTIONAL EXAM – MARKING SCHEME

Qn Solution
1 Inequalities
2 2
 1 1
x − x +7 =  x −  +7 − 
2

 2 2
2
 1  27
=x−  +
 2 4
2 2
 1  1  27
Since  x −  ≥ 0 for all real values of x, x−  + > 0 (shown).
 2  2 4

3 −1
> , x ≠ −1, x ≠ 2
( x − 2)
2
x +1
3 1
+ >0
( x − 2)
2
x +1
3 ( x + 1) + ( x 2 − 4 x + 4 )
>0
( x + 1)( x − 2 )
2

x2 − x + 7
>0
( x + 1)( x − 2 )
2

2
 1  27
> 0 and ( x − 2 ) > 0 for all x ∈  \ {2}
2
Since x 2 − x + 7 =  x −  +
 2 4
⇒ ( x + 1) > 0
∴ x > −1 , x ≠ 2

Alternatively

2
 1  27
Since x 2 − x + 7 =  x −  + > 0 for all real values of x,
 2 4
1
>0
( x + 1)( x − 2 )
2

− + +

−1 −2

∴ x > −1 , x ≠ 2

MJC/2013 JC1 Promotional Exam Marking Scheme/H2 Math (9740)/Math Dept Page 1 of 12

sgfreepapers.com 121
Qn Solution
2 Techniques of Differentiation
2 t −t 2
x = sin −1 (1 − t ) y=e
1
dx 1 dy 1
= ( −1) =e 2t −t 2
( 2t − t ) ( 2 − 2t )
2 −2

dt 1 − (1 − t ) dt 2
2

dx
=−
1 dy e
=
2 t −t 2
(1 − t )
dt 2t − t 2 dt 2t − t 2
dy
d y dt
∴ = =e 2t −t 2
( t − 1)
dx dx
dt

Qn Solution
3 SLE
(i)
At A, b + c = a + d .
At B, a + b + c = 48.
At C, a + c = 2b.
At D, d = b + 2a.
After simplifying,
− a + b + c − d = 0.
a + b + c = 48.
a − 2b + c = 0.
2a + b − d = 0.
Using GC, a = 8, b = 16, c = 24 and d = 32.
(ii) Total amount collected = $0.50 ( 2c + b )
= $0.50 ( 48 + 16 )
= $32

MJC/2013 JC1 Promotional Exam Marking Scheme/H2 Math (9740)/Math Dept Page 2 of 12

sgfreepapers.com 122
Qn Solution
4 Vectors I

(i) 2 Q 1 B
A
3
1
P
C
a
b

O

OC = kb
Using Ratio Theorem,
 a + 3kb
OP =
4
 a + 2b
OQ =
3

(ii) Given that O, P and Q are collinear,


 
OP = λ OQ for some λ ∈ 
1 3k 1 2 
a + b = λ  a + b
4 4 3 3 

Since a and b are non-zero and non-parallel vectors,


1 λ 3 2
= ------ (1) and k = λ ------ (2)
4 3 4 3

3
From (1): λ = ------ (3)
4

Substitute (3) into (2)

2  3  4 
k =   
3  4  3 
2
=
3

2
∴k =
3

Alternatively,
Given that O, P and Q are collinear,
 
OQ = λ OP for some λ ∈ 

MJC/2013 JC1 Promotional Exam Marking Scheme/H2 Math (9740)/Math Dept Page 3 of 12

sgfreepapers.com 123
1 2 1 3k 
a + b = λ  a + b
3 3 4 4 

Since a and b are non-zero and non-parallel vectors,


1 1 2  3k 
= λ   ------ (1) and = λ   ------ (2)
3 4 3  4 
4
From (1): λ = ------ (3)
3
Substitute (3) into (2)

2 4 
k=  
3  3λ 
2  4  3  2
=    =
3  3  4  3

2
∴k =
3

Qn Solution
5* Maclaurin’s Series and Binomial Theorem [Not in topics tested for SRJC 2014 Promo]
(i) e x sin 2 x
 ( 2 x ) + ... 
3
 x 2 x3
=  1 + x + + + ...   2 x −
 2! 3!   3! 

 x 2 x3  8x3 
=  1 + x + + + ...  2 x − + ... 
 2 6  6 
3
8x
= 2x − + 2 x 2 + x3 + ...
6
1
= 2 x + 2 x 2 − x 3 + ...
3
(ii) x
e sin 2 x −
1
= e x sin 2 x  ( 4 − x ) 2
4− x
1

 1  −1  x  2
=  2 x + 2 x 2 − x3 + ...  ( 4 ) 2 1 − 
 3   4
  1  3  
1    1  x   − 2  − 2   x  2 
=  2 x + 2 x 2 − x3 + ...   1 +  −  −  +    −
1
  + ... 
2 3    2  4  2!  4 
 
 
 x 3  x  
2
1 1
=  2 x + 2 x 2 − x3 + ...   1 + +   + ... 
2 3   8 8  16  
1 2x2 3x 3 x3 2 x 3 
=  2x + + 2 x2 + − + ... 
2 8 64 3 8 
9 x 2 7 x3
= x+ − + ..
8 384

MJC/2013 JC1 Promotional Exam Marking Scheme/H2 Math (9740)/Math Dept Page 4 of 12

sgfreepapers.com 124
Qn Solution
6 Graphing Techniques 1
(i) y

y = − 5x y = 5x
( 0, 2 )

O x

y2 = 5x2 + 4
( 0, −2 )

(ii) 5x2 + 4 = h2 1 − x2( )


(
y 2 = h2 1 − x2 )
y 2 + x 2 h2 = h 2
y2
2
+ x2 = 1
h
y

y = − 5x y = 5x2 + 4 y = 5x

( 0, 2 )
( 0, h )

( 0, −h )
−1 O 1 x
y2
x2 + =1
h2
( 0, −2 )

y2
Graph to be inserted is x 2 + =1.
h2
From the graphs, 0 < h < 2 .

Qn Solution
7 Application of Differentiation (Tangent/ Normal)

MJC/2013 JC1 Promotional Exam Marking Scheme/H2 Math (9740)/Math Dept Page 5 of 12

sgfreepapers.com 125
x2
y=
x −1
dy 2 x ( x − 1) − x
2
=
( x − 1)
2
dx
x2 − 2 x
=
( x − 1)
2

8
Since gradient of tangent at A is
9
x2 − 2x 8
=
( x − 1)
2
9
Using GC, x = 4 or x = −2
Since x2 < x1 , x coordinate at point B is x2 = −2
4
Sub x2 = −2 into C we have y2 = −
3
 4
∴ coordinates of B is  −2, − 
 3
9
Since gradient of normal at B is −
8

 4 9
y −  −  = − ( x − ( −2 ) ) − − − −(*)
 3 8
9 43
y=− x−
8 12

Qn Solution
8 Transformation of graphs
(a) x −1
y= 2
3x − 5
↓ 1. Replace y by − y
x −1
−y = 2
3x − 5
↓ 2. Replace y by y − 1
x −1
1− y =
3x 2 − 5
↓ 3. Replace x by 2x
2. 2 x − 1
1− y =
12 x 2 − 5

The transformations are in the following order:


1. Reflection in the x-axis.
2. Translation of 1 unit in the positive y-direction.
3. Scaling parallel to the x-axis by factor ½.
(or 3-1-2, 1-3-2, 1-3-2)

Alternatively,
The transformations are in the following order:
MJC/2013 JC1 Promotional Exam Marking Scheme/H2 Math (9740)/Math Dept Page 6 of 12

sgfreepapers.com 126
1. Translation of 1 unit in the negative y-direction.
2. Reflection in the x-axis.
3. Scaling parallel to the x-axis by factor ½.
(or 1-3-2)
(b)(i)
y

y = f '( x)

y=0
O (3, 0) x

(b)(ii) x=0
y

y2 = f ( x )

y= 2

(−6, 0)
 3 O 3  (6, 0) x
 − ,0  ,0
 2  2 
y=− 2

x=0

Qn Solution
9 Mathematical Induction (RR) and MOD
(i) 1
Let Pn be the statement un = 2 for n ∈+ .
2n
1
When n = 1, LHS = u1 =
2
1 1
RHS = 2
= = LHS
2 (1) 2
∴ P1 is true.

Assume Pk is true for some k ∈+ ,


1
i.e. uk = 2 -------------------- (*)
2k

MJC/2013 JC1 Promotional Exam Marking Scheme/H2 Math (9740)/Math Dept Page 7 of 12

sgfreepapers.com 127
1
To prove Pk+1 is also true, i.e. uk +1 = 2
.
2 ( k + 1)
2(k + 1) − 1
LHS = uk +1 = uk − 2
(from the recurrence relation)
2k 2 ( k + 1)
2k + 1
= uk −
2k 2
( k + 1)2
1 2k + 1
= 2
− from (*)
2k 2k 2
( k + 1)2
2
=
( k + 1) − 2k − 1
2
2k 2 ( k + 1)
k2
= 2
2k 2 ( k + 1)
1
= 2
= RHS
2 ( k + 1)

Thus Pk is true ⇒ Pk +1 is true .


Since P1 is true, and Pk is true ⇒ Pk +1 is true , by Mathematical Induction, Pn is true
for all n ∈+ .

(ii) N
2n + 1 N

∑ 2n = ∑ ( un − un +1 )
( n + 1)
2 2
n =1 n =1

= u1 − u2
+ u 2 − u3

+ u N − u N +1
= u1 − u N +1

1 1 1 1 
−= = 1 − 
2 2 ( N + 1)2 2  ( N + 1)2 
 
N +1
(iii) N
2n + 3 2n + 1

n = 0 2 ( n + 1) ( n + 2 )
2 2
=∑ 2
n =1 2 n ( n + 1)
2

1 1
= −
2 2 ( N + 2 )2

MJC/2013 JC1 Promotional Exam Marking Scheme/H2 Math (9740)/Math Dept Page 8 of 12

sgfreepapers.com 128
Qn Solution
10 Vectors
(i) 2 2
      
AB = OB − OA =  −1 ⇒ a direction vector for the line is  −1
1 1
   
 1 2
   
vector equation of the line AB : r = 1 + λ  −1 , λ ∈ 
 1  
  1

To determine whether point C lies on the line:


 1
 2 = 1 + 2λ ⇒ λ =
 2  1 2 2
      
Let  1  = 1 + λ  −1 . Then 1 = 1 − λ ⇒ λ = 0
 5  1   5 = 1 + λ ⇒ λ = 4
    1 

Since the values of λ are inconsistent, i.e. no value of λ satisfies all the equations,
hence shown that point C does not lie on the line AB .
(ii) Let N be the foot of the perpendicular from C to line AB
 1 2
   
line AB : r = 1 + λ  −1 , λ ∈ 
 1  
  1
 1 + 2λ 
  
Since N lies on line AB then ON =  1 − λ  for some λ ∈ .
 1+ λ 
 

 1 + 2λ   2   −1 + 2λ 
        
CN = ON − OC =  1 − λ  −  1  =  −λ 
 1 + λ   5   −4 + λ 
     
→ →  −1 + 2λ   2 
  
CN ⊥ line AB, CN ⋅ d = 0 ⇒  −λ  .  −1 = 0
 −4 + λ   1 
  
⇒ −2 + 4 λ + λ − 4 + λ = 0 ⇒ λ = 1

Therefore, the position vector of the foot of the perpendicular from point C to line AB.
 1 + 2 (1)   3 
    
ON =  1 − (1)  =  0 
 1 + (1)   2 
   
 
Since ON = OB , the angle ABC is 90 degrees.
(iii)
The position vector of C ' , the reflection of point C in the line AB

MJC/2013 JC1 Promotional Exam Marking Scheme/H2 Math (9740)/Math Dept Page 9 of 12

sgfreepapers.com 129
 
 OC + OC '
ON =
2
  
OC ' = 2ON − OC
3 2
   
= 2 0 −  1
2 5
   
4
 
=  −1 
 −1 
 

Qn Solution
11 AP GP
(a) Let T1, T3, T6, be the first, third and sixth term of an arithmetic series with first term a and
common difference d.

T1 = a, T3 = a + 2d , T6 = a + 5d
a + 5d a + 2d
=
a + 2d a
a ( a + 5d ) = ( a + 2d )
2

a 2 + 5ad = a 2 + 4ad + 4d 2
ad = 4d 2

Since d ≠ 0 ⇒ a = 4d
T a + 2d 6 d 3
Common ratio r = 3 = = =
T1 a 4d 2
Since r > 1 , the geometric progression is not convergent.

15
S15 = [ 2a + 14d ]
2
15
= [ 2(4d ) + 14d ]
2
= 165d
165
= a
4
(b) 9
a = 2, r =
10
a
S∞ =
1− r
2
=
9
1−
10
= 20

MJC/2013 JC1 Promotional Exam Marking Scheme/H2 Math (9740)/Math Dept Page 10 of 12

sgfreepapers.com 130
Sn ≥ 15
2   9 n 
1 −  ≥ 15
9    
1 −   10  
10
  9 n 
1 −    ≥ 0.75
  10  
n
9
  ≤ 0.25
 10 
n ≥ 13.158
The minimum number of days required is 14 days.

Qn Solution
12 Applications of Differentiation
(i) h
sin α = ∴ PQ = h cosec α
PQ
QR = k − PQ − RS
= k − 2 PQ
= k − 2h cosec α (shown)

h
A= ( QR + PS )
2
h h 
=  2QR + 2 
2 tan α 
= h ( k − 2h cosec α + h cot α )
= hk + h 2 (cot α − 2 cosec α ) (shown)

(ii)
A = hk + h 2 (cot α − 2 cosec α )
dA
= h 2 (−cosec 2 α + 2cosec α cot α )

= h 2cosec α (−cosec α + 2 cot α )

dA
When = 0, h 2cosec α (−cosec α + 2 cot α ) = 0

Since h 2 cosec α ≠ 0,
−cosec α + 2 cot α = 0
−1 + 2 cos α
=0
sin α
−1 + 2 cos α = 0
1
cos α =
2
π
α=
3

MJC/2013 JC1 Promotional Exam Marking Scheme/H2 Math (9740)/Math Dept Page 11 of 12

sgfreepapers.com 131
π 
− π π 
+

α   3  
3 3
dA

Alternatively
dA
= h 2 (−cosec 2 α + 2cosec α cot α )

d2 A
= h 2 (2cosec 2 α cot α − 2cosec3 α − 2cosec α cot 2 α )
dα 2
= 2h 2 cosec α (cosec α cot α − cosec 2 α − cot 2 α )

π
When α = ,
3
d2 A π π π π π
= 2h 2 cosec (cosec cot − cosec 2 − cot 2 )
dα 2
3 3 3 3 3
2  2  1   2   1  
2 2

= 2h 2   −  −  
3  3  3   3   3  
4 2 2 4 1
= h  − − <0
3  3 3 3
4 2
=− h <0
3
π
α= gives max A
3

π
When α =
3
Max A = hk + h 2 (cot α − 2 cosec α )
π π
= hk + h 2 (cot − 2 cosec )
3 3
 1  2 
= hk + h 2  −2  
 3  3 
= hk − 3h 2

MJC/2013 JC1 Promotional Exam Marking Scheme/H2 Math (9740)/Math Dept Page 12 of 12

sgfreepapers.com 132
MERIDIAN JUNIOR COLLEGE
JC1 Promotional Examination
Higher 2

___________________________________________________________________

H2 Mathematics 9740/01
Paper 1 08 October 2013

2 Hours 30 Minutes
Additional Materials: Writing paper
List of Formulae (MF 15)
___________________________________________________________________
READ THESE INSTRUCTIONS FIRST

Write your name and civics group on all the work you hand in.
Write in dark blue or black pen on both sides of the paper.
You may use a soft pencil for any diagrams or graphs.
Do not use staples, paper clips, highlighters, glue or correction fluid.

Answer all the questions.


Give non-exact numerical answers correct to 3 significant figures, or 1 decimal place in the case of
angles in degrees, unless a different level of accuracy is specified in the question.
You are expected to use a graphic calculator.
Unsupported answers from a graphic calculator are allowed unless a question specifically states
otherwise.
Where unsupported answers from a graphic calculator are not allowed in a question, you are
required to present the mathematical steps using mathematical notations and not calculator
commands.
You are reminded of the need for clear presentation in your answers.

At the end of the examination, fasten all your work securely together.
The number of marks is given in brackets [ ] at the end of each question or part question.

___________________________________________________________________
This document consists of 6 printed pages and 2 blank page

[Turn Over
MJC/2013 JC1 Promotional Examination/9740/01

sgfreepapers.com 133
2
BLANK PAGE

*: Not in topics tested for SRJC 2014 Promotional Exam


MJC/2013 JC1 Promotional Examination/9740/01

sgfreepapers.com 134
3

1 Show that x 2 − x + 7 is always positive for all real values of x. [1]


Hence, using an algebraic method, solve the inequality
3 1
2
>− . [3]
( x − 2) x +1

2t −t 2 dy
2 The parametric equations of a curve C are x = sin −1 (1 − t ), y = e . Find in terms of t.
dx
[4]

3 The diagram below shows the traffic flow of vehicles in four traffic junctions A, B, C and D.
Each arrow indicates the direction of the vehicles entering or leaving the junction. The
unknown constants a, b, c and d indicate the number of vehicles entering or leaving a particular
junction. It is given that the total number of vehicles entering a traffic junction must be equal to
the total number of vehicles leaving that same junction. There are 48 vehicles leaving junction
B.

A D

B C

(i) Determine the values of a , b, c and d . [3]

(ii) The shaded region indicates the presence of an Electronic Road Pricing (ERP) gantry
located at that road. It is known that each gantry charges a fixed price of $0.50 per
vehicle. How much revenue will be collected in total by the gantries in these regions?
[1]

[Turn Over
MJC/2013 JC1 Promotional Examination/9740/01

sgfreepapers.com 135
4

 
4 Referred to the origin O, the points A and B are such that OA = a and OB = b, where a and b
 
are non-zero and non-parallel vectors. The point C lies on OB such that OC = kOB , where k
is a constant. P is on AC such that AP : PC = 3 : 1, and Q is on AB such that AQ : AB = 2 : 3.

 
(i) Find OP and OQ in terms of a, b and k . [2]

(ii) Given that O, P and Q are collinear, find the value of k . [3]

5 (i)* Obtain the series expansion for e x sin 2 x , up to and including the term in x 3 . [3]

e x sin 2 x
(ii)* Hence deduce the first three non-zero terms in the series expansion of . [3]
4− x

6 The curve C has equation y 2 = 5 x 2 + 4 .

(i) Sketch C, indicating clearly the axial intercepts, the equations of the asymptotes and the
coordinates of the stationary points. [3]

(ii) Hence by inserting a suitable graph, determine the range of values of h, where h is a

( )
positive constant, such that the equation 5 x 2 + 4 = h2 1 − x 2 has no real roots. [3]

7 The curve C has equation


x2
y= .
x −1
Points A ( x1, y1 ) and B ( x2 , y2 ) lie on curve C such that the tangent at A is parallel to tangent at

8 16
B where x2 < x1 . Given further that the equation of tangent at A is y = x + , find the
9 9
coordinates of B, and hence find the equation of normal at point B. [6]

*: Not in topics tested for SRJC 2014 Promotional Exam


MJC/2013 JC1 Promotional Examination/9740/01

sgfreepapers.com 136
5

x −1
8 (a) State a sequence of transformations which transform the graph of y = to the
3x 2 − 5
2x − 1
graph of 1 − y = . [3]
12 x 2 − 5

(b) The diagram below shows the graph of y = f ( x ) .

x
O

Sketch, on separate clearly labeled diagrams, the graphs of

(i) y = f '( x ) , [2]

(ii) y2 = f ( x ) . [3]

MJC/2013 JC1 Promotional Examination/9740/01

sgfreepapers.com 137
6

1
9 A sequence u1 , u2 , u3 , is such that u1 = and
2
2n + 1
un+1 = un − , for all n ≥ 1.
2n 2
( n + 1)2

1
(i) Use the method of mathematical induction to prove that un = 2
for n ∈  + . [4]
2n

N
2n + 1
(ii) Hence find ∑ 2n ( n + 1)
n =1
2 2
. [3]

N
2n + 3
(iii) Use your answer to part (ii) to find ∑ 2 ( n + 1) ( n + 2)
n =0
2 2
. [2]

10 Referred to the origin O, the position vectors of two points A and B are given by i + j + k and
3 i + 2 k respectively. Also, the position vector of C is given by 2i + j + 5k .

(i) Find a vector equation of the line AB and show that point C does not lie on the line. [3]

(ii) Find the position vector of the foot of the perpendicular from point C to line AB.
Hence write down the size of angle ABC. [5]

(iii) Find the position vector of C ' , the reflection of point C in the line AB. [2]

*: Not in topics tested for SRJC 2014 Promotional Exam


MJC/2013 JC1 Promotional Examination/9740/01

sgfreepapers.com 138
7

11 (a) The first, third and sixth terms of an arithmetic progression with non-zero common
difference d and first term a, are three consecutive terms of a geometric progression.
Determine if the geometric series is convergent, justifying your answer. Find also the
sum of the first 15 terms of the arithmetic progression in terms of a. [5]

(b) A pile driver is used to drive piles into the soil at a new condominium site. On the first
day, the depth piled into the soil is 2 m. On each subsequent day, the depth piled into the
9
soil is of the depth piled into the soil on the previous day. Find the maximum
10
theoretical depth that can possibly be piled into the soil. Find the minimum number of
days required to drive the piles to a depth of at least 15m into the soil. [5]

12 A student wants to construct a model of a roof structure of fixed height h cm from a


rectangular piece of cardboard of width k cm. The cardboard is to be bent in such a way that
the cross-section PQRS is as shown in the diagram, with PQ + QR + RS = k and with PQ and
RS each inclined to the horizontal at an angle α .

Q R

h h

P S

(i) Show that QR = k − 2h cosec α and that the area A cm 2 of the cross-section PQRS is

given by A = hk + h2 (cot α − 2 cosec α ) . [3]

(ii) Use differentiation to find, in terms of k and h, the maximum value of A as α varies. [5]

MJC/2013 JC1 Promotional Examination/9740/01

sgfreepapers.com 139
8
BLANK PAGE

*: Not in topics tested for SRJC 2014 Promotional Exam


MJC/2013 JC1 Promotional Examination/9740/01

sgfreepapers.com 140
NANYANG JUNIOR COLLEGE
JC1 PROMOTIONAL EXAMINATION
Higher 2

MATHEMATICS 9740/01

Paper 1 1st October 2013

3 Hours

Additional Materials: Cover Sheet


Answer Papers
List of Formulae (MF15)

READ THESE INSTRUCTIONS FIRST

Write your name and class on all the work you hand in.
Write in dark blue or black pen on both sides of the paper.
You may use a soft pencil for any diagrams or graphs.
Do not use staples, paper clips, highlighters, glue or correction fluid.

Answer all the questions.


Give non-exact numerical answers correct to 3 significant figures, or 1 decimal place in the case of angles in
degrees, unless a different level of accuracy is specified in the question.
You are expected to use a graphic calculator.
Unsupported answers from a graphic calculator are allowed unless a question specifically states otherwise.
Where unsupported answers from a graphic calculator are not allowed in a question, you are required to
present the mathematical steps using mathematical notations and not calculator commands.
You are reminded of the need for clear presentation in your answers.

At the end of the examination, fasten all your work securely together.
The number of marks is given in brackets [ ] at the end of each question or part question.

This document consists of 7 printed pages.

NYJC 2013 JC1 Promotional Examination 9740/01 1


sgfreepapers.com 141
1 Solve the inequality x 2 − 2 x − 3 > x + 1 . [4]

2 Differentiate the following expressions with respect to x, simplifying your answers as far as possible:
2
(a) tan −1   , [3]
x

1+ x
(b) ln . [3]
1− x

1 1
3 A sequence u1 , u2 , u3 , ... is such that u1 = and un+1 = un + + 2− n , for n ∈  + .
4 n ( n + 1)

9 1 − n+1
(i) Prove by mathematical induction that un = − −2 for n ∈  + . [5]
4 n
(ii) Explain why {un } is convergent. [1]

9
(iii) Show that u n is less than for n ∈  + . [1]
4

4 Show that r !( r 2 + 1) = ( r + 2)! − 3( r + 1)! + 2r ! where r ∈  + . [1]


Hence, using method of difference, show that the sum of the first n terms of the series

( 5)( 2!) + (10)( 3!) + (17 )( 4!) + is ( n + 2)!( n + 1) − 2 . [4]


n
Using the above result, explain why ∑ r !(r
r =1
2
) is less than ( n + 1) !n . [2]

NYJC 2013 JC1 Promotional Examination 9740/01 2

sgfreepapers.com 142
5 (a) The points A and B relative to the origin O have position vectors i + 2 j − 2k and − 4i + 5 j + 2k
AP λ
respectively. The point P lies on line AB such that = .
PB 1 − λ

(i) Show that OP = (1 − 5λ ) i + ( 2 + 3λ ) j + ( 4λ − 2 ) k . [1]

(ii) Given further that C is a point with position vector −5i + α j − 2k and that O, P and C
are collinear, find the values of λ and α . [3]

(b) The equations of three planes π 1 , π 2 , π 3 are


π 1 : 2 x − 2 y + z = −4 ,
π 2 : 2x + 3 y − 4z = 1 ,
π3 : β x − 3y + z = γ ,
respectively.

(i) The planes π 1 and π 2 intersect in a line l. Find a vector equation of l. [1]
(ii) Hence, find the values of β and γ such that there are infinitely many points of
intersection between π 1 , π 2 and π 3 . [2]

y2
6 The curve C 1 has equation x 2 − = 1 . The curve C 2 has parametric equations
4

x = a sin t , y = a cos t , where 0 ≤ t ≤ 2π and a > 0 .

(i) Write down the Cartesian equation of C 2 . Sketch C1 and C 2 on the same diagram, stating the
exact coordinates of any points of intersection with the axes and the equations of any
asymptotes. [5]

(ii) State the range of values of a such that there are 4 points of intersection between C1 and C 2 .
Show algebraically, that the x-coordinates of the points of intersection satisfy the equation
5x2 = 4 + a2 . [2]

(iii) Explain geometrically why there are only 2 values for the x-coordinates when there are 4 points
of intersection between C1 and C 2 . Find the exact values of x if a = 3 . [2]

[Turn Over
NYJC 2013 JC1 Promotional Examination 9740/01 3

sgfreepapers.com 143
7 The function f is defined by
1
f : x → x 2 − , x ∈ , 1 ≤ x < 2 .
x
(i) Show, by differentiation, that f is strictly increasing. [2]
(ii) State the range of f. [1]
Solve the equation f ( x ) = f ( x ) , giving your answer to two decimal places.
−1
(iii) [2]

The function g is defined by


π
g: x → 1 + sin x, x ∈ , 0 ≤ x < .
2
(iv) Only one of the composite functions fg and gf exists. Give a definition (including the domain)
of the composite that exists, and explain why the other composite does not exist. [3]
(v) For the composite function which exists, state its range. [1]

8 The equation of a curve is


2
y ( x + 2 ) + 2 y 2 ( x + 2 ) − 12 x = 0 , where x and y are positive variables.

dy 1
(i) Show that the value of is when x = 2 . [5]
dx 16
(ii) Find the equation of the normal to the curve at the point where x = 2 . [2]
(iii) Given that the normal in (ii) meets the line x = 2 at the point P and the line x = 0 at the
point S. Find the exact area of triangle OSP, where O is the origin. [2]

NYJC 2013 JC1 Promotional Examination 9740/01 4

sgfreepapers.com 144
9 There are 16 boys and 10 girls in a JC1 class. It so happens that within the class, the heights of all the
girls form a geometric progression, while the heights of all the boys form an arithmetic progression.
The two shortest students in the class, a boy and a girl, both have a height of 150.0 cm, while the
tallest boy in the class has a height of 180.0 cm. The fourth shortest girl in the class has a height of
157.5 cm.
1
(i) Show that the common ratio r between the heights of the girls is 1.05 3 and find the height of
the tallest girl in the class, giving your answer in cm correct to one decimal place. [2]
(ii) Find the number of girls in the class taller than 164.0 cm. [3]
(iii) Find the average height of the girls in the class, giving your answer in cm correct to one
decimal place. [3]
(iv) Find the average height of the entire class, giving your answer in cm correct to one decimal
place. [2]

10 The position vectors of the points A, B and C with respect to the origin O are a, b and a − 2b
respectively. Plane π contains the point A and has b as its normal vector. If the angle between
vectors a and b is 60o and a = 2 b , find in terms of b,
(i) the length of projection of a onto b , [2]
(ii) the distance between point C and the plane π . [3]

Given that a = i + 5j + 2k and b = i + 2j − k ,


(iii) find the position vector of the foot of perpendicular from point C to the plane π , [5]
(iv) show that the position vector of the point of the reflection of point C in the plane π is
3i + 9 j . [2]

[Turn Over
NYJC 2013 JC1 Promotional Examination 9740/01 5

sgfreepapers.com 145
11 The graphs of y = f ' ( x ) and y 2 = f ( x ) are shown in the diagrams below.

y y

y2 = f(x)

x x
–1 0 1 –1 0 1 2

y=
y= f 'f( x’(x)
) –2

(a) On separate diagrams, sketch the graphs of


(i) y = f ' (1 − x ) , [3]

(ii) y = f ( x) , [4]

showing clearly the x-intercepts and asymptotes (if any).

(b) State the set of values of x for which the graph of y = f ( x ) is concave upwards. [2]

NYJC 2013 JC1 Promotional Examination 9740/01 6

sgfreepapers.com 146
12 (a) The curve C has parametric equations
2
x = θ 2 + 4θ , y= , for θ > 0 .
θ
A point P(x, y) moves on the curve C in such a way that the x-coordinate of P decreases at a
constant rate of 4 units per second. Find the rate at which the y-coordinate of P is changing
when x = 4 . [4]

(b)
A B

D 4 C

The diagram above shows the floor plan of a storeroom. The floor plan consists of a square
ABCD of side 4 units from which a quadrant of a circle with centre A and radius 3 units has
been removed. The owner intends to store a rectangular crate with one corner of the base at C,
and the opposite corner of the base at P against the curved wall. The base of the crate has area
π
y unit2 and angle DAP is θ radians, where 0 ≤ θ ≤ .
4

dy
Show that = 3 ( sin θ − cos θ )( 4 − 3sin θ − 3cos θ ) . [2]

Hence, find the least possible value of y. [5]

-----END OF PAPER-----
[Turn Over
NYJC 2013 JC1 Promotional Examination 9740/01 7

sgfreepapers.com 147
2013 NYJC JC1 Promo 9740/1 Solutions

Qn
1
y y=x+1

y = ( x + 1)( x − 3)

O x
–1 2 3 4

x < −1 or − 1 < x < 2 or x > 4 .

2 d −2
 2  2( − x ) −2
(a) [tan −1  ] = = 2
dx  x  1 + ( 2 )2 x + 4
x

d 1+ x d 1
(b) (ln ) = [ ( ln(1 + x ) − ln(1 − x ) )]
dx 1− x dx 2
1 1 −1 1 1
= ( − )= 2
or
2 1+ x 1− x 1− x (1 + x )(1 − x )
Alternative Solution
 
d  1+ x   1  1 1 − x  1 − x + 1 + x  
 In  =   .  
dx  1− x   1+ x  2 1 + x  (1 − x )2  
   
 1− x 
1  1− x  2 
=   
2  1 + x   (1 − x )2 
 
1
=
(1 − x ) (1 + x)

3 9 1
(i) Let Pn denote the proposition un = − − 2 − n +1 for all n ∈ Z + .
4 n
1
For n = 1 , LHS = u1 =
4
9 1 −1+1 9 1
RHS = − − 2 = − 1 − 1 = = LHS.
4 1 4 4
∴ P1 is true.
Page 1 of 10

sgfreepapers.com 148
2013 NYJC JC1 Promo 9740/1 Solutions
Qn
9 1
Assume that Pk is true for some k ∈ Z + , i.e. , uk =− − 2− k +1 .
4 k
9 1
−2 ( )
− k +1 +1
To prove that that Pk+1 is true, i.e., uk +1 = −
4 k +1
For n = k + 1,
1
LHS = uk +1 = uk + + 2− k
k ( k + 1)
9 1 1
= − − 2− k +1 + + 2− k
4 k k ( k + 1)
9 1 1 
= −  −  − 2
4  k k ( k + 1) 
−k
( ) ( 2 − 1)
9 k + 1−1
= − − 2− k −1+1
4 k ( k + 1)
9 1
−2 ( )
− k +1 +1
−=
4 k +1
Hence Pk+1 is true
Since P1 is true and Pk is true ⇒ Pk+1 is true, hence by Mathematical Induction, Pn is true for
all n ∈ Z + .

1 9
(ii) As n → ∞ , → 0 , 2− n → 0 , hence un → , i.e. {un } is convergent
n 4

1 9 1 9
(iii) Since > 0 , 2− n > 0 for n ≥ 1 , un = − − 2− n+1 <
n 4 n 4

4 (r + 2)!− 3(r + 1)!+ 2r ! = r !( (r + 2)(r + 1) − 3(r + 1) + 2 )


= r !(r 2 + 3r + 2 − 3r − 3 + 2)
( )
= r ! r 2 + 1 (Shown)

n +1 n +1

∑ r !(r
r =2
2
+ 1) = ∑ [ (r + 2)!− 3(r + 1)!+ 2r !]
r =2
= 4!− 3(3!) + 2(2!)
+ 5!− 3(4!) + 2(3!)
+ 6!− 3(5!) + 2(4 !)

+ (n + 1) !− 3(n)!+ 2(n − 1)!
+ (n + 2) !− 3(n + 1)!+ 2(n)!
Page 2 of 10

sgfreepapers.com 149
2013 NYJC JC1 Promo 9740/1 Solutions
Qn
+ (n + 3) !− 3(n + 2)!+ 2(n + 1)!
= (n + 3)!− 2(n + 2)!− 3!+ 2(2!)
= (n + 2)!(n + 3 − 2) − 2
= (n + 2)!(n + 1) − 2 (Shown)

∑ r !(r
r =1
2
+ 1) = (n + 1)!(n) − 2 + (1!)(11 + 1) = (n + 1)!n

Since r !(r 2 ) < r !(r 2 + 1) for r ∈  +


n n
Therefore ∑ r !(r 2 ) < ∑ r !(r 2 + 1) = (n + 1)!n
r =1 r =1

 
5a  (1 − λ ) OA + λ OB
OP =
1− λ + λ
= (1 − λ )( i + 2 j − 2k ) + λ ( −4i + 5 j + 2k )
(1 − 5λ ) i + ( 2 + 3λ ) j + ( 4λ − 2 ) k
 
OP = µ OC
 1 − 5λ   −5 
   
 2 + 3λ  = µ  α 
 4λ − 2   −2 
   

2 1
Solving, λ = , µ= , α = 16
5 5

b π 1 : 2 x − 2 y + z = −4 ,
π 2 : 2x + 3 y − 4z = 1 ,
π3 : β x − 3y + z = γ .

 −1  1
Line of intersection of π 1 and π 2 , l: r =  1  + λ  2  , λ ∈  .
 
0  2
   

For infinite points of intersection between 3 planes, l is on π 3 .

Page 3 of 10

sgfreepapers.com 150
2013 NYJC JC1 Promo 9740/1 Solutions
Qn
1  β 
  
 2 i −3  = 0 ⇒β =4
 2  1 
  

 −1   β 
  
 1 i −3  = γ ⇒ γ = −7
01
  

6(i) x = a sin t , y = a cos t


x y
sin t = , cos t =
a a

sin 2 t + cos 2 t = 1
2 2
x  y
  +  =1
a a

x2 + y 2 = a2

y = −2 x y = 2x
y

C1 C2

(-a,0) (-1,0) (1,0)

x
( a, 0 )

(ii) a >1
y2
x2 − = 1 .........(1)
4

x2 + y2 = a2
y 2 = a 2 − x 2 .......(2)

Page 4 of 10

sgfreepapers.com 151
2013 NYJC JC1 Promo 9740/1 Solutions
Qn
 a2 − x2 
x2 −   =1
 4 
4x2 − a2 + x2 = 4

(iii) 5x2 = 4 + a2 (shown)


The points of intersection between the 2 curves are symmetrical about the x-axis, thus there are
only 2 values for the x-coordinates.

5 x 2 = 13
13
x=±
5

7(i) 1
f ′ ( x) = 2x + > 0 for 1 ≤ x < 2 ⇒ f is strictly increasing.
x2

(ii) Since f is strictly increasing, its minimum and maximum values correspond to the minimum and
maximum x values. Thus
 1  7
Rf = 1 − 1, 4 −  =  0,  .
 2  2

(iii) f ( x ) = f −1 ( x ) ⇒ f ( x ) = x
1
⇒ x2 − = x
x
⇒ x − x2 − 1 = 0
3

⇒ x = 1.47.

(iv) Since Rg = [1, 2 ) = Df, fg exists.


 7  π
Since Rf = 0,  ⊄ 0,  = Dg, gf does not exist.
 2  2

2 1
fg(x) = f( sin x + 1 ) = ( sin x + 1) − .
sin x + 1
 π
Dfg = Dg = 0,  .
 2

2 1 π
(v) fg : x → ( sin x + 1) − , x ∈ , 0 ≤ x < .
sin x + 1 2
 7
Rfg = 0,  .
 2

Page 5 of 10

sgfreepapers.com 152
2013 NYJC JC1 Promo 9740/1 Solutions
Qn
8(i) ( x + 2)
2
y + 2 ( x + 2 ) y 2 − 12 x = 0
Differentiating wrt x,
dy 2 dy
( x + 2 ) + 2 y ( x + 2 ) + 4 y ( x + 2 ) + 2 y 2 − 12 = 0 -------(1)
dx dx

When x = 2 , 16 y + 8 y 2 − 24 = 0
y2 + 2 y − 3 = 0
( y + 3)( y − 1) = 0
y = −3(rejected ∵ y > 0) or y =1

Subst (2 , 1) into equation (1),


dy dy
16 + 8 + 16 + 2 − 12 = 0
dx dx
dy
32 =2
dx
dy 1
=
dx 16

(ii) Equation of normal: y − 1 = −16 ( x − 2 )


y = −16 x + 33

(iii) Points P and S has coordinates (2 , 1) and (0 , 33) respectively


1
Area of triangle OSP = × 33 × 2 = 33
2

9(i) Let un denote the height of the nth shortest girl in the class in cm, and r denote the common ratio
between the heights of the girls.
Then un = ar n −1 where u1 = a = 150.0 and u4 = ar 3 = 157.5
1
157.5
⇒r = 3
= 1.05 ⇒ r = 1.05 3
150.0
Also, u10 = ar 9 = a (r 3 )3 = (150.0)(1.05)3 = 173.6 (to 1 d.p.)
∴ The height of the tallest girl is 173.6 cm.

ii un > 164.0
n −1
⇒ (150.0)(1.05) 3 − 164.0 > 0
Using GC,
n n−1
(150.0)(1.05) 3 − 164.0
6 -1.29
7 1.38
8 4.09

Page 6 of 10

sgfreepapers.com 153
2013 NYJC JC1 Promo 9740/1 Solutions
Qn
Hence n ≥ 7 .
Since there are 10 girls in the class, the number of girls who are taller than 164.0 cm is 10-7+1=4.
Thus there are 4 girls in the class taller than 164.0 cm.

iii Average height of girls


1 1 a (1 − r10 )
= S10 =
10 10 1 − r
10
(150.0)(1 − 1.05 3 )
= 1
10(1 − 1.05 3 )
= 161.57
= 161.6 cm (to 1 d.p.)

Average height of boys


iv 1
= S16
16
1 16
= × (150.0 + 180.0)
16 2
= 165.0 cm
Average height of class
16(165.0) + 10(161.57)
=
16 + 10
= 163.7 cm (to 1 d.p.)
10(i) length of projection = aib = a b cos 60°

1
=2b = b
2

(ii) distance between C and the plane =


( a − 2b − a ) ⋅ (b) =
−2b ⋅ b
b b

2
−2 b
=
b
=2b

 1   1   −1 
(iii)      
c =  5  − 2 2  =  1 
 2   −1   4 
     

Page 7 of 10

sgfreepapers.com 154
2013 NYJC JC1 Promo 9740/1 Solutions
Qn
 1  1  1 
π : r ⋅  2  =  5  ⋅  2  = 9,
 −1   2   −1
     

 −1 1
   
l :r =  1 +λ 2 
4  −1 
   

 −1 + λ   1 
   
 1 + 2λ  ⋅  2  = 9
 4 − λ   −1 
   
− 1 + λ + 2 + 4λ − 4 + λ = 9 ⇒ λ = 2

 −1   1   1 
     
(iv) position vector of the foot of perpendicular from c to plane =  1  + 2  2  =  5 
 4   −1   2 
     

 1   −1  3 
     
position vector of point of reflection of C in plane = 2  5  −  1  =  9 
 2  4  0
     

11a

x= 0 x=2

Page 8 of 10

sgfreepapers.com 155
2013 NYJC JC1 Promo 9740/1 Solutions
Qn
y

y = f(x)
4

–1 0 1 x
2

b (−∞, −1) ∪ ( −1,1)

2
12a x = θ 2 + 4θ , y=
θ
dy dy dx
= ⋅
dt dx dt
 dy dθ  dx
= ⋅ ⋅
 dθ dx  dt
−2 1
= 2⋅ ⋅ ( −4 )
θ 2θ + 4
4
= 2
θ (θ + 2 )
When x = 4 , θ 2 + 4θ = 4 ⇒ θ = 0.82843 since θ > 0
dy 4
= 2 = 2.06 units/sec
dt θ (θ + 2 )

Rate of change of y-coordinate is 2.06 units/sec.

Page 9 of 10

sgfreepapers.com 156
2013 NYJC JC1 Promo 9740/1 Solutions
Qn
12b y = ( 4 − 3cosθ ) ( 4 − 3sinθ )

dy
= ( 4 − 3sin θ )( 3sin θ ) + ( 4 − 3cos θ )( −3cos θ )

= 3  4sin θ − 3sin 2 θ − 4 cos θ + 3cos 2 θ 

= 3 3 ( cos 2 θ − sin 2 θ ) + 4sin θ − 4 cos θ 


= 3 3 ( cos θ − sin θ )( cos θ + sin θ ) + 4 ( sin θ − cos θ ) 
= 3 ( sin θ − cos θ )( 4 − 3sin θ − 3cos θ )

dy
=0

3 ( sin θ − cos θ )( 4 − 3sin θ − 3cos θ ) = 0
sin θ − cos θ = 0 or 4 − 3sin θ − 3cos θ = 0
4
sin θ + cos θ =
π 3
θ= or
4 π
θ = 0.44556 θ =1.1252 (rej 0 ≤ θ ≤ )
4
2
d y
= 3 ( sin θ − cos θ )( 3sin θ − 3cos θ ) + 3 ( sin θ + cos θ )( 4 − 3sin θ − 3cos θ )
dθ 2
π d2y
When θ = , < 0 ⇒ y is max
4 dθ 2
d2y
When θ = 0.44556 , > 0 ⇒ y is min
dθ 2

Min y = ( 4 − 3cos 0.44556 )( 4 − 3sin 0.44556 ) =3.50

Page 10 of 10

sgfreepapers.com 157
1

Name ( ) Class

RIVER VALLEY HIGH SCHOOL


2013 Year 5 Promotional Examination
Higher 2

MATHEMATICS 9740/01
Paper 1 19 September 2013
3 hours
Additional Materials: Answer Paper
List of Formulae (MF15)
Cover Page

READ THESE INSTRUCTIONS FIRST

Do not open this booklet until you are told to do so.


Write your name, class and index number in the space at the top of this page.

Write your name and class on all the work you hand in.
Write in dark blue or black pen on both sides of the paper.
You may use a soft pencil for any diagrams or graphs.
Do not use staples, paper clips, highlighters, glue or correction fluid.

Answer all the questions.


Give non-exact numerical answers correct to 3 significant figures, or 1 decimal place in the
case of angles in degrees, unless a different level of accuracy is specified in the question.
You are expected to use a graphic calculator.
Where unsupported answers from a graphic calculator are not allowed in a question, you are
required to present the mathematical steps using mathematical notations and not calculator
commands.
You are reminded of the need for clear presentation in your answers.
Up to 2 marks may be deducted for poor presentation in your answers.

At the end of the examination, place the cover page on top of your answer paper and fasten
all your work securely together.

The number of marks is given in brackets [ ] at the end of each question or part question.

This document consists of 7 printed pages and 1 blank page.

©RIVER VALLEY HIGH SCHOOL 9740/01/2013

sgfreepapers.com 158
2

1
Let f (xx)   x  3
 9  4x 

1. (i) 2 . Find thee series exp
pansion off f ( x ) in asscending
m in x 2 .
powerss of x, up to and includiing the term [3]
9 9
(ii) Denotee the answer to part (i)) by g( x ) . Find,
F for   x  , tthe set of values
v of
4 4
x for which
w the vaalue of g( x ) is within 0.2 of f (xx ) . [2]

2. Thhe graphs oof y 2  f ( x) and y  f  x  are giv


ven below.

D
Deduce the graphs
g of
(i) y  f ( x) , [3]
(ii) y  f '(( x) , [2]
cllearly indicaating any assymptotes, iintersection
ns with the axes
a and staationary poin
nts.

©RIVER
R VALLEY HIGH
H SCHOO
OL 9740/01/2
2013

sgfreepapers.com 159
3

3. The diagram shows the sketch of the curve C, ( y  1) 2  x x 2  1 , with the vertex at
1,1 .

(i) Write down the equation of the graph when C is translated 1 unit in the negative
y-direction. [1]
(ii) The shaded region R, bounded by C and the vertical line, x  a , is rotated
through π radians about the line y  1 . By using the substitution u  x 2  1 , or
otherwise, find the exact volume obtained in terms of a . [5]

4. (a) A theme park sells day passes at different prices depending on the age of the
customer. The age categories are senior citizens (ages 60 and above), adult (ages
13 to 59) and child (ages 4 to 12). Three tour groups visited the theme park on the
same day. The numbers in each category for each group together with the total
cost of the day passes for each group are given as follows.

Group Senior Citizens Adult Child Total Cost


1 2 19 9 $196.40
2 0 10 3 $90.20
3 1 7 4 $77.00

Write down and solve equations to find the cost of a day pass for each of the
age category. [3]

4 x2  4 x  1
(b) Without using a GC, solve  0. [4]
x2  2 x  8

©RIVER VALLEY HIGH SCHOOL 9740/01/2013

sgfreepapers.com 160
4

5. The cross section of an open container consists of a semicircle, a rectangle ABCD and
an isosceles triangle CED as shown in the diagram below. Given that AD  BC  x cm ,
5
AB  DC  FE  y cm , DE  CE and the height of the container is cm .
3
A x cm D

y cm F E

B C

The interior vertical walls of this container, ADECB, need to be painted. The time
needed to paint the walls will be 1 minute per 10 cm2 for the straight parts and 1 minute
per 8 cm2 for the semicircular part. Given that a total time of 200 minutes is required to
paint all the walls, find, by differentiation, the values of x and y which gives a
maximum cross-sectional area, giving your answers correct to the nearest integers. [7]

π π
6. It is given that the curve y3  tan 1 y  ln(cos x) , where   x  , passes through the
2 2
origin.
dy
(i) Show that  3 y 4  3 y 2  1
  1  y 2  tan x . [2]
dx
(ii) Find the Maclaurin series for y , up to and including the term x 2 . [3]
π
dy

4
(iii) Hence, find an approximation to the value of dx , in terms of  . [2]
0 dx

7. In a particular river in Brazil, a sudden surge in the number of piranhas (a type of fish
known for their sharp teeth and a voracious appetite for meat) is observed and has
affected the livelihood of the villagers living along the river. A group of fishermen is
engaged to catch these piranhas and the piranhas are caught at a rate inversely
proportional to the number of piranhas left. Furthermore, due to aggressive nature, the
number of piranhas is reduced at a rate of one-tenth of the piranhas remaining.

(i) If x (in thousands) is the number of piranhas remaining at time t (in days) after the
group of fishermen is deployed to catch the piranhas, show that x 2  10k  Ae 0.2t
where k is a positive constant. [4]
(ii) If there are 5000 piranhas at the start of the deployment of the fishermen and after
5 days, the number of piranhas remaining is 3000. Calculate the number of days
required to remove all the piranhas. [3]

©RIVER VALLEY HIGH SCHOOL 9740/01/2013

sgfreepapers.com 161
5

8. (a) Five out of the six digits, 0, 1, 2, 3, 4 and 5 are chosen and arranged randomly to
form a five-digit number. No digit is repeated.

Find the number of five-digit numbers that are


(i) greater than 10000, [2]
(ii) greater than 10000 and even. [3]

(b) An ice-cream shop has 4 different flavours of ice-cream, vanilla, chocolate,


strawberry and durian and 3 different toppings containing peanuts, raisins and
berries. Assuming Peter decides to visit the ice-cream shop and make a selection
of at least 1 flavour and at least 1 topping, find how many different selections can
he make? [3]

9. (a) The function f and g are defined by

f : x  x2  6 x  11, x 3
1
g:x  , x  k , where k is a positive constant.
x2

(i) Show that the inverse function of f exists. [1]


(ii) Find f 1  x  and state the domain of f 1 . [3]
(iii) State the greatest value of k for which the composite function gf exists and
find the range of gf for this value of k. [3]

(b) Given that h is a one-one function, determine, with reasons, if hh 1 exists. [2]

10. (a) The sum, Sn of the first n terms of a sequence u1 , u2 , u3 ,  is given by


1
Sn  ln a nb 2
 n n
2

, where 0  a  1, b  1 .
(i) Find un in terms of a and b . [2]
(ii) Prove that the sequence is an arithmetic progression. [2]
(iii) Given that 0  ab  1 when n  7 , find the sum of the negative terms of
n 1

the sequence. [1]


1 
(b) By considering sin  n  sin    , show, using the method of differences,
2 
 1 
cos  N    
N
1 1   2 
 sin  n   cot    
2 2  1 
. [4]
n 1
2sin   
2 

©RIVER VALLEY HIGH SCHOOL 9740/01/2013

sgfreepapers.com 162
6

11. (a) A and B are events such that P  B   0.3 , P  A '  B '  0.9 and
P  A  B '  0.45 .
(i) P  A , [2]
(ii) P  A ' B  . [2]

(b) In a cooking school, all students must take a theory and practical test. It is
reported that 95% of the students pass the theory test. Of those who pass, 85%
also pass the practical test. Of those who fail the theory test, 60% pass the
practical test.

Draw a tree diagram to show the above information. [2]

Find the probability that a student, randomly chosen from the cooking school,
(i) passes the practical test, [1]
(ii) passes the theory test, given that he fails the practical test. [2]

12. A curve C has parametric equations


x  et , y  t 2 .
(i) Sketch the curve C. [2]

The normal to C at point A with coordinates  e 2 , 4  is denoted by l .


(ii) Find the Cartesian equation of l , expressing y in terms of x. [3]
(iii) Find the exact area of the region bounded by l , C and the x-axis. Express your
a
answer in the form 2  be 2  c where a, b and c are constants to be determined.
e
[5]

©RIVER VALLEY HIGH SCHOOL 9740/01/2013

sgfreepapers.com 163
7

13. It is thought that the pH value of water may affect the size of pearl in pearl oyster
farming. A pearl farmer wished to investigate whether there was any correlation
between the pH value of the water and the size of the pearl cultivated. The size of the
pearls and the pH value of the water where the oysters are cultivated are shown in the
table below.

pH value of
7.7 7.8 7.9 8.0 8.1 8.2 8.3
water, x
Size of
pearl, y (in 6.82 7.28 7.61 7.79 7.91 8.02 8.05
cm)

(i) Draw a scatter diagram to illustrate the data, labeling the axes clearly. [2]
(ii) Comment on whether a linear model would be appropriate. [1]

It is thought that the size of pearl can be modeled by one of the formulae
y  a  bx 2 or y  c  dx
2

where a, b, c and d are constants.

(iii) Find, correct to 4 decimal places, the value of the product moment correlation
coefficient between
(a) x 2 and y,
(b) x and y 2 . [2]
(iv) Use your answer to parts (i) and (iii) to explain which of y  a  bx 2
or
y 2  c  dx is the better model. [2]
(v) The pearl farmer will like to have pearls which are exactly 8.00 cm. Find the
equation of a suitable regression line, and use it to find the required pH value of
the water, correct to 1 decimal place. Comment on the reliability of your answer.
[4]

END OF PAPER

©RIVER VALLEY HIGH SCHOOL 9740/01/2013

sgfreepapers.com 164
8

Blank Page

©RIVER VALLEY HIGH SCHOOL 9740/01/2013

sgfreepapers.com 165
2013 Y
Year 5 H22 Maths Promotion
P nal Exam
mination Marking
M S
Scheme

1(i) 1
f ( x )   x  3 9  4 x 

2

1
1 
 4 
 2
  x  3 9 1  x 
2

 9 
  1  1  3  
         2 
1 2   4   2  2   4 
  x  3 1     x     x   
3  1  9  2  9  
 
1  2 2 2 
  x  3 1  x  x  
3  9 27 
5 4 2
 1 x  x
9 27
(ii) 0.2  f ( x)  g( x)  0.22 or f  x   g  x   0.2
2
y

0
0.2
f g
x

0.2

U
Using GC,
 x  R, 1.877  x  1.25

2(i)

(ii)

sgfreepapers.com 166
0

3 (i) G
Graph to be translated
t 1 unit in neggative y- direction
 y  f  x 1  y 1  f  x
R
Replace y with
w y 1,
( y  1  1) 2  x x 2  1
y 2  x x2 1
(ii) V
Volume obtaained
a
   x x 2  1 dx
1

a 2 1
u
  xu   du
 x
0 u  x2 1
2
a 1
du x
 0
u 2 du 
dx u
a 2 1 dx u
 u3  
   du x
 3 0
 3

3
a 2
 1 2

4(a) L
Let x, y andd z be the cost of a dayy pass for a senior, ad
dult and
chhild respecttively.

2 x  19 y  9 z  196.4
10 y  3 z  90.2
x  7 y  4 z  77

U
Using GC,

sgfreepapers.com 167
x  3.60
y  7.40
z  5.40

Thus, the cost of a day pass for a senior is $3.60, for an adult is
$7.40 and for a child is $5.40.

(b) 4x2  4 x  1
0
x2  2 x  8
Let y  x
4 y2  4 y 1
0
y2  2 y  8
 2 y  1  0
2

 y  2  y  4 

Since  2 y  1  0,  2 y  1  0 satisfy the inequality


2 2

1
y
2
1
x 
2
1 1
x  or x  
2 2

 y  2  y  4  0
y4
y  2
or x 4
x  2
x4 or x  4
(no solution)

Answer: x  4 or x4

Alternatively(Method 2),

2
4 x  4 x 1
2
0
x  2 x 8

sgfreepapers.com 168
When x  0 ,
4x2  4x  1
 0 and x  0
x2  2x  8
 2 x  1  0
2
+ -2 - -1/2- 0 - 4 +
 x  2  x  4 
x  2 or x  4 and x  0
1
x or x  4
2 -2 -1/2 0 4

Or when x  0 ,
4x2  4x  1
 0 and x  0
x2  2x  8
 2 x  1  0
2

 x  2  x  4 
x  2 or x  4 and x  0
1
x or x  4
2

1 1
Answer: x  or x   or x  4 or x4
2 2

Alternatively( Method 3),

2
4 x  4 x 1
2
0
x  2 x 8

 2 x  1  0
2

 x  2  x  4 
1 1
 2 x  1
2
 0 satisfy the inequality  x  x
2 2
 x  2   0 for all values of x,
 x  4   0  x  4  x  4 or x  4
1 1
Answer: x  or x   or x  4 or x4
2 2

5 2
 y 5y
side of triangle     y 2 
2 2

sgfreepapers.com 169
 y 5 
 x  x  2  y
  2   2   5   200
  
10 8  3
 
 
x 5y  y
   120
5 10 16
x 5y  y
 120  
5 10 16
5 y 5 y
x  600  
2 16

Cross sectional area, W


2
 y 1 1
       xy  y 2
 2 2 2
y 2
y 2
 5 y 5 y 
   y  600   
8 2  2 16 
 y2 y2 5 y 2 5 y 2
   600 y  
8 2 2 16
3 y 2 y 2 5 y2
 600 y   
16 2 2

For maximum W,
dW
0
dy
3 y
600   y  5y  0
8
 3 
y  1  5   60
 8 
y  248.533  249
x  78.1347  78

d 2W 3
2
  1  5  2.414
dy 8

y  249 and x  78 will result in a maximum cross sectional


area.

sgfreepapers.com 170
6(i) y 3  tan 1 y  ln(cos x)
Differentiating both sides w.r.t x,
dy 1 dy  sin x
3 y2  
dx 1  y 2 dx cos x
dy
dx
 
3 y 2 1  y 2   1   1  y 2  tan x

 3 y 4  3 y 2  1 ddyx   1  y 2  tan x (shown)


(ii) Differentiating both sides w.r.t x,
2
 dy  d2 y
 
d
  x
12 y 3
 6 y  
d x 2 
3 y 4  3 y 2  1

 dy 
  1  y 2  sec 2 x   2 y  tan x
 dx 

When x  0,
dy d2 y
y  0,  0,  1
dx dx 2
0 1 1
 y  0  x  x2     x2  
1! 2! 2

(iii) π
π dy  x2  4
 dx    
4
0 dx  2 0

 4
2
π
 0
2
π2
 or  0.03125 2
32

7 (i) Due to the fishermen catching the fishes,


dx 1

dt x
dx k
 ,where k is a positive constant
dt x

Due to the aggressive nature of the fishes,


dx
 0.1x
dt

Rate of change of fishes,

sgfreepapers.com 171
dx k
   0.1x
dt x
k  0.1x 2

x
x
 k  0.1x 2 dx   1 dt
1 0.2 x

0.2 k  0.1x 2
dx   1 dt

1
ln k  0.1x 2  t  c
0.2
ln k  0.1x 2  0.2t  c1
k  0.1x 2  e 0.2t  c1
k  0.1x 2  ec1 e 0.2t
x 2  10k  10ec1 e 0.2t
x 2  10k  Ae 0.2t

Alternatively,
x
 k  0.1x 2 dx   1 dt
1 0.2 x

0.2 k  0.1x 2
dx   1 dt

1
ln  k  0.1x 2   t  c since k  0.1x 2  0
0.2
ln  k  0.1x 2   0.2t  c1
k  0.1x 2  e0.2t  c1
x 2  10k  Ae 0.2t

(ii) When t  0 , x  5
25  10k  A

When t  5 , x  3
9  10k  Ae1

Solving,
A  25.3116 and k  0.0311627

When x  0 ,
t  21.986

Number of days required = 22

sgfreepapers.com 172
8(a) No. of five-digit numbers greater than 10000
(i)  5 5 4  3 2
 600

Alternatively,
No restrictions – case where 0 is the first digit
6
P5  5 P4  600
(ii) Method 1
Case 1: First digit is 1 or 3 or 5 (odd)
3  4  3  2  3  216

Case 2: First digit is 2 or 4 (even)


2  4  3  2  2  96

No. of five-digit numbers greater than 10000 and even


 216  96
 312
Method 2
Case 1: Last digit is 2 or 4
4  4  3  2  2  192

Case 2: Last digit is 0


5  4  3  2  1  120

No. of five-digit numbers greater than 10000 and even


 192  120
 312

Method 3
No. of five digit numbers greater than 10000 – No. of five digit
numbers greater than 10000 that are odd
 600  4  4  3  2  3
 312

(b) Total number of selections


  24  1   23  1  105

Alternatively,
Method 2: Listing 12 Cases
C
4
1 C2  4C3  4C4    3C1  3C2  3C3   105
4

Method 3: Complement

sgfreepapers.com 173
No restrictions – 0 flavors or 0 toppings
 24  23   23  24  1
 128  23
 105

9(i)

Any horizontal line, y  k , k  2 cuts the graph of y  f ( x ) at


most once.
(ii) Let y  f ( x)  x 2  6 x  11
y  ( x  3) 2  2
( x  3) 2  y  2
x  3 y  2
Since x  3 , x  3  y  2
 f 1 ( x)  3  x  2, x  2

Df 1  Rf   2,  
(iii) For gf to exist, Rf  Dg i.e.  2,     k ,  
 greatest value of k  2

Df Rf Rgf
 
 3,   2,   0, 1 4 
 1
Rgf   0, 
 4
Alternative method,
gf  x   g  x 2  6 x  11
1
 ,x 3
 x 2  6 x  11
2

sgfreepapers.com 174
y  1
 3, 
 4
x
0 y  gf  x 

 1
Rgf   0, 
 4
(b) Since h is a one-one function, h 1 exists.

Since Rh 1  Dh , the rule for composite function, Rh 1  Dh is


fulfilled. Therefore hh 1 exists.

10 un  Sn  Sn 1
(a)(i) 1
 n n
2 1
 n1  n1
2

 ln a n b 2  ln a n 1b 2

 ln ab 2
1
 n  n  n 3n  2  
2 2

 ln ab n 1
(ii) un  un 1  ln ab n 1  ln ab n 11
 ln b
Since ln b is a constant, the sequence is an AP.
(iii) For n  7 , 0  ab n 1  1  ln ab n 1  0
Therefore, sum of negative terms is S6  ln a 6b 2
1
 6 6  ln a 6b15
2

(b) Using factor formula,

 1  1   1   1  
sin  n  sin      cos  n      cos  n     
2  2   2   2 
1   1   1  
sin  n    cos  n      cos  n     
1   2   2 
2sin    
 2 

10

sgfreepapers.com 175
N
1 N
  1   1  
 sin  n   1 
  cos  n      cos  n     
 2   2  
n 1
2sin    n 1 
2 
 3  
 cos  cos 
 2 2 
 5 3 
  cos  cos 
 2 2 
 7 5 
  cos  cos 
1  2 2 

 1    
2sin     
2  
 1   3 
  cos  N      cos  N     
  2   2  
 
  1   1  
  cos  N  2     cos  N  2    
     

1   1  
  cos  N      cos 
1   2  2
2sin    
2 
  1 
cos cos  N    
2   2 

1  1 
2sin    2sin   
2  2 
 1 
cos  N    
1 1   2 
 cot     (shown)
2 2  1 
2sin   
2 

11

sgfreepapers.com 176
11 P  A  B  =11  P  A  B   1  0.9  0.1
(a)(i)
P  A =P  A  B   P  A  B   0.445  0.1  0.5
55

(ii) P  A ' B   P  B   P  A  B 
 0.3  0.1
 0.2
(b)

(i) P  passes thee practical test


t 

 0.05  0.6  0.95  0.85


5  0.8375
(ii) P  passes thee theory testt | he fails thhe practical test 
0.95  0.155

1  0.83755
 0.877

12
(i)

12

sgfreepapers.com 177
(ii) dy dx
 2t ,  et
dt dt
dy
 2te  t
dx
At point A, t  2,
A
1 e 2
ggradient of nnormal  
2  2  e 2 4
E
Equation of linel l,
2
e
y  4    x  e2 
4
e 2
116  e 4
y  x
4 4
(iii)

R
Required areea
 area of triangle + areaa under curvve C
t 2
1  16  e 4 2 dx
   2
4  e   y dt

2  e  t  0 dt
2
 16 
 2  2    t 2 e t dt
e  0
32  2 t 2 
2
 2    t e   2  t e t dt 
 
e  0
0 
32  t 2 2 t 
 2  4e  2  te    e dt 
2

e 
0
0 

32
e 2 
 4e 2  2 2e 2   et 
2

0 
32
 2  4e 2  4e 2  2e 2  2
e
32
 2  2e 2  2
e

13

sgfreepapers.com 178
13(i) y

8.05

6.82
x
0 7.7 8.3

(ii) The scatter diagram shows y is increasing at a decreasing rate


and hence a linear model is not appropriate.
(iii) (a) r  0.9358

(b) r  0.9464
(iv) Since the product moment correlation coefficient between x
and y 2 is closer to 1 compared to that between x 2 and y and y
increases as x increases but at a decreasing rate, hence
y 2  c  dx is the better model.
(v) Using the GC,
y 2  176.23  29.347 x

When y  8.00 , x  8.2

From (iii), r  0.9464 is close to 1. Since y  8.00 is within


the data range of y and x is the independent variable, hence
the answer is reliable.

14

sgfreepapers.com 179
TEMASEK JUNIOR COLLEGE, SINGAPORE
JC One
Promotion Examination 2013
Higher 2

MATHEMATICS 9740

Solutions

TJC/MA9740/JC1Promo2013

sgfreepapers.com 180
2

1 Find the general solution of the following differential equation

1 dy 1
  0, where x  1. [4]
1  x dx 1  x 2
Solution:
 1  dy 1
   0
 1  x  dx 1  x
2

dy 1 x

dx 1  x2
dy 1 x
 
dx 1  x 1  x2
2

1 1 2x
y   dx   dx
1 x 2
2 1  x2
  tan1 x  ln 1  x2   c
1
2
1
(or  tan1 x  ln 1  x2  c )
2

TJC/MA9740/JC1Promo2013

sgfreepapers.com 181
3

2 (i) The first three terms of a sequence are given by u1  19 , u2  34 , u3  52 . Given


that un is a quadratic polynomial in n, find un in terms of n. [4]

(ii) Find the smallest value of n for which un is greater than 200. [2]

Solution:
(i) Let un  an 2  bn  c where a, b, c are constants.

When n  1 , a  b  c  19 ----- (1)


When n  2 , 4 a  2b  c  34 ----- (2)
When n  3 , 9 a  3b  c  52 ----- (3)

Using GC to solve the system of equations, we get


3 21
a , b , c7
2 2
3 21
 un  n 2  n  7
2 2

(ii)
Method I: For un  200 ,
3 2 21
n  n  7  200
2 2
 n  15.4 or n  8.37 (3sf)

 the smallest value of n is 9.

Method II:

For un  200 ,
By GC
U 8  187  200
U 9  223  200

 The smallest value of n is 9.

TJC/MA9740/JC1Promo2013

sgfreepapers.com 182
4

3 A wire of length L cm is cut into two pieces. One piece is used to form a circle while
the other piece is used to form an equilateral triangle.
Show that, with the total area of the circle and triangle being the smallest, the
3
proportion of the length of the smaller piece to the length of the bigger piece is .
9
[6]
Solution:
Let one of the pieces be x cm and use it for form the circle.
So the other piece is L-x and it’s used to for the equilateral triangle.

L-x x

x
For area of circle (radius r): 2 r  x  r 
2
2
 x  x2
Therefore area is    
 2  4

For area of equilateral triangle:

1 Lx  
2
3
 sin     L  x
2
Area = 
2 3   3  36
 L  x
2
x2 3 3 2
  L  x 
2
Hence total area, A = [the other form x also accepted]
4 36 4 36
dA x 3
   L  x
dx 2 18

TJC/MA9740/JC1Promo2013

sgfreepapers.com 183
5

Method I:
dA
For max/min,  0,
dx
1x 2 3 x 3 x 3
   L  x  0    L  x    1
2 36 2 18  L  x 9

3
Hence the ratio of the length of the smaller piece to the length of the bigger piece is
9
(shown)

d2 A 1 3
And    0  A is minimum.
dx 2
2 18

Method II:
dA
For max/min,  0,
dx
2x 2 3 x 3
   L  x   0    L  x   0 --- (*)
4 36 2 18
 1 3 3 3 L
 x     L x
 2 18  18 9  3

d2 A 1 3 3 L
   0  A is minimum at x 
dx 2
2 18 9  3

x 3 x 3 x 3
From (*)  ( L  x)  0   ( L  x)   ( 1)
2 18 2 18 Lx 9

3
Hence the ratio of the length of the smaller piece to the length of the bigger piece is
9
(shown)

TJC/MA9740/JC1Promo2013

sgfreepapers.com 184
6

4
y
y   x  1 , x  1
2

y  4x  4
4

y  x 1
2
R

x
1 1 3

The shaded region R in the diagram above is bounded by the y-axis, the line y   x  1
and the curves y   x  1 for x  1 and y  4 x  4 .
2

Find the volume of the solid of revolution formed when R is rotated completely about
the y-axis. [6]

Solution:

2
 y2  4 
 
 2
4 4


2
Required volume =  1  y dy    dy  1 1
0 2  4  3
 17.26666709  54.24483447  54.2 unit2

TJC/MA9740/JC1Promo2013

sgfreepapers.com 185
7

5 Given that y  ln  2  tan 1 x  , show that


2

1  x  ddxy2  2 x ddyx  1  x2   ddyx   0 .


2
2
[3]

Hence find the Maclaurin's expansion for y, up to and including the term in x2 . [3]

Solution:
y  ln  2  tan 1 x   e y  2  tan 1 x

Differentiate wrt x
dy dy
 1  x 2   e y    (1)
1
 ey 
dx 1  x 2
dx
Differentiate (1) wrt x
2
d2y dy dy  dy 
 1  x  2  2 x
2
 e  y   1  x 2    [From (1)]
dx dx dx  dx 
2
d2 y dy  dy 
 1  x  2  2 x  1  x 2     0
2

dx dx  dx 
dy 1 d 2 y 1
When x  0, y  ln 2,  , 2 
dx 2 dx 4
1  1
   
 y  ln 2   2
x
4 2 1 1
x    ln 2  x  x 2
1! 2! 2 8

TJC/MA9740/JC1Promo2013

sgfreepapers.com 186
8

n
r 9 1 3 n
6 Prove by mathematical induction 3
r 1
r 1
     for all positive integers
4 3n1  4 2 
of n. [5]
Hence show that
1 2 3 4 9
 2  3  4 . [2]
4 4 4 4 16

Solution:

n
r 9 1 3 n
Let P(n) be the statement 3 r 1
r 1
     for n  1, 2, 3, 4,
4 3n1  4 2 
1
r 9 3 1
When n = 1, LHS =  r 1 = 1 ; RHS =     1
r 1 3 4 4 2
So P(1) is true.
k
r 9 1 3 k
Assume P(k) is true for some k    , i.e.  r 1   k 1   
r 1 3 4 3 4 2
k 1
r 9 1  3 k 1 
To show P(k + 1) is true i.e.  r 1   k   
r 1 3 4 3 4 2 
k 1
r k
r k 1
LHS =  r 1 =  r 1 +
r 1 3 r 1 3 3k
 9 1  3 k  k  1
=   k 1      k
 4 3  4 2  3
9 1  9 3k 
=  k    k  1
4 3 4 2 

9 1  9 3k  2k  2  9 1  9 k  2  9 1  5 k 
=  k  =  k   =    
4 3 4 2  4 3 4 2  4 3k  4 2 

9 1  3 k 1 
=  k    = RHS
4 3 4 2 

So P ( k+1) is true.
Since P(1) is true, and P(k) is true  P(k + 1) is true.
n
r 9 1 3 n
 By mathematical induction, P(n) is true for all n    , ie. 3
r 1
r 1
    
4 3n1  4 2 

n

r 9
Since  r 1 
r 1 3 4

1 2 3 r 1  r 1  r 9
Hence  2  3     r   r 1   r 1  (deduced)
4 4 4 r 1 4 4 r 1 4 4 r 1 3 16

TJC/MA9740/JC1Promo2013

sgfreepapers.com 187
9

7 Functions f and g are defined by


2x  2
f:x , for x  , x  1 ,
x2
g : x  2  x , for x  , x  2 .

(i) Given that f has an inverse, show that the composite function gf 1 exists. Find

gf 1 and state its range. [5]

(ii) Find the value(s) of x such that f  x   f 1


 x . [2]

Solution:
(i) R f 1 = Df =  ,1

D g   , 2

Since R f 1  D g , the composite function gf 1 exists. (Shown)

2x  2
Let y .
x2
 xy  2 y  2 x  2

 xy  2 x  2 y  2

 x  y  2  2 y  2

2y  2
 x
y2
2x  2
 f 1
( x)  .
x2

 2x  2 
gf 1  x  = g  
 x2  y

 2x  2   2  2
= 2   22  
 x2   x2 x2

Dgf 1 = Df 1  R f   0, 2 

2
So, gf 1: x   , x  , 0  x  2 x
x2

TJC/MA9740/JC1Promo2013

sgfreepapers.com 188
10

For range of gf 1 :
M1 - By mapping method
g

1 2 x

Thus, R gf 1 = 1,   .
y
M2 - By direct sketching method
Dgf 1  Df 1 = R f   0, 2 

Therefore R gf 1 = 1,  

1
x
0 2

y
(ii)

From the graph,


f  x   f 1  x 

 0  x 1

x
1

TJC/MA9740/JC1Promo2013

sgfreepapers.com 189
11

8 Prove that

  r  1 r  2  
ln    ln   r  1 r    2 ln   r  r  1   ln   r  1 r  2   . [2]
 r  r  1 

Hence, find in terms of n,

 1 4   25   3 6    n  1 n  2     n  n  3 
ln    ln    ln      ln    ln   ,
 23   3 4   45    n  n  1    n  1 n  2  

leaving your answer as a single logarithmic function. [5]

Solution:

(i) RHS  ln   r  1 r    2 ln   r  r  1   ln   r  1 r  2  


  r  1 r  r  1 r  2  
 ln  
  r 
2
 r  1
2

 
  r  1 r  2  
 ln    LHS
  r  r  1 

(ii)
 1 4   25   3 6    n  1 n  2     n  n  3 
ln    ln    ln      ln    ln  
 23   3 4   45    n  n  1    n  1 n  2  
n1 
  ln
 r 1 r  2   n1 ln r 1 r  2ln r r  1   ln r 1 r  2 
         
r 2  r  r  1  r2 
 ln 1 2   2ln  2  3  ln  3 4 
 ln  2  3  2ln  3 4   ln  4  5
 ln  3 4   2ln  4  5  ln  5 6 
 ln  4  5  2ln  5 6   ln  6  7 

 ln  n  2  n  1  2ln  n  1 n   ln  n  n  1
 ln  n  1 n   2ln  n  n  1  ln  n  1 n  2 
 ln  n  n  1  2ln  n  1 n  2   ln  n  2  n  3
 ln 1 2   ln  2  3  ln  n  1 n  2   ln  n  2  n  3 [

 2  n  2  n  3   n3 
 ln    ln  
 6  n  1 n  2    3  n  1 

TJC/MA9740/JC1Promo2013

sgfreepapers.com 190
12

9 Jessie wishes to take up a loan of $20,000 on the 1st day of the Year 2014. She intends
to pay an instalment of $300 on the 1st day of each month, beginning from February
2014. She sources out two banks, XYZ Bank and ABC Bank, which offer such loans.
The two banks have different ways of charging interest. XYZ Bank charges a monthly
interest of 0.5% on the outstanding amount owed at the end of each month, while ABC
Bank charges a fixed interest of $60 at the end of each month until the loan is repaid.

(a) If Jessie takes up the loan from XYZ Bank, show that the outstanding loan at the
end of February 2014 after the interest has been added will be $19899. [2]

Hence, find the number of months Jessie will take to repay her loan. [4]

(b) Which bank should Jessie take a loan from if she wishes to clear her loan as soon
as possible? Justify your answers. [3]

Solution:

kth Outstanding loan at the beginning of kth Outstanding loan at the


month month from 2014 end of kth month from
2014
1 20000 1.005  20000 
2 1.005  20000   300 1.0052  20000   300 1.005

3 1.0052  20000   300 1.005  300


 
 20000   300 1.005  300 1.005 
n 1 n2 n 3
n 1.005
   300 1.005   300 1.005   300
2

(a) Outstanding loan at the end of February 2014  1.0052  20000   300 1.005  $19899
[Shown]
Hence
Let 1.005n 1  20000   300 1.005     300 1.005   300  0
n2

 1.005n 1  20000   300 1  1.005   1.005     1.005  0


2 n2

 
11.005 n 1  1 
 1.005 n 1
 20000   300  0
 1.005  1 

 1.005n 1  20000   60000 1.005   1  0


n 1

 
 40000 1.005 
n 1
 60000
60000
ln
  n  1  40000  n  82.29558565
ln 1.005 
 Jessie will repay her loan on the 1 day of 83 month. Therefore, she will take 82
st rd

months to repay her loan.


TJC/MA9740/JC1Promo2013

sgfreepapers.com 191
13

(b)
Method I:
For Bank ABC,

kth Outstanding loan at the beginning of kth month Outstanding loan at


month from 2014 the end of kth month
from 2014
1 20000 20000  60
2 20000  60  300 20000  60  300  60
3 20000  60  300  60  300  20000  60  2   300  2 
 20000  240  2 

 
n 20000  240  n  1

For 20000  240  n  1  0  n  84.33333


 Jessie will repay her loan on the 1 day of 85 month if she takes up bank ABC.
st th

Hence, she should take the loan from bank XYZ.

Method II:
When n = 83, 20000  240  83  1  320  0
 Jessie will not be able to clear her loan by the 83 month if she takes up bank ABC.
rd

Hence, she should take the loan from bank XYZ.

TJC/MA9740/JC1Promo2013

sgfreepapers.com 192
14

10 A curve C is given parametrically by the equations

x  2cos3  , y  2sin 3 
 
where    .
2 2

Show that the normal at the point with parameter  has equation
y sin   x cos   2  sin 4   cos 4   . [4]

The normal at the point Q where   , cuts C again at the point P, where   p .
6
Show that sin 3 p  3 cos3 p  1  0 and hence find the coordinates of P. [5]
Solution:
x  2cos3  , y  2sin 3 
dx dy
 3  2  cos 2    sin    3  2  sin 2  cos 
dt dt
 6sin  cos2   6sin 2  cos 

dy d y d t
     tan 
d x dt dx
 Gradient of normal to the curve  cot 

Eqn. of normal to the curve at  2 cos3  , 2sin 3   :


y  2sin 3  cos 

x  2cos3  sin 
 y sin   2sin 4   x cos   2 cos 4 
 y sin   x cos   2  sin 4   cos 4   (shown)

Eqn. of normal to the curve at Q , i.e.   :
6
 3    1 4  3 
4
1
y    x    2       
2  2    2   2  

 y  3x  2

When the normal to the curve at Q cuts C again at P, i.e.   p ,


2sin 3 p  3  2 cos3 p   2
 sin 3 p  3 cos3 p  1  0 (shown)
 p  0.7445633 or 0.52359878  rejected, pointQ 

 The coordinates of P is  0.795, 0.622  . (3sf)

TJC/MA9740/JC1Promo2013

sgfreepapers.com 193
15

11 A sequence of real numbers x1 , x 2 , x3 ,... satisfies the recurrence relation

2( xn2  xn )
xn1   1 , x1  k , where k  1 .
3

(a) When k = 5, state the value of x 9 and describe the behavior of the sequence. [2]

(b) Prove algebraically that, if the sequence converges, then it converges to


either 1 or 3. [3]

(c) State a value of k such that the sequence converges to 1. [1]

(d) When k  2 , state the integer m such that m  xn  m  1 for all integers n  1 . [1]
x 1
Hence, by considering n 1 , show that x n 1  x n for all integers n  1 . [3]
xn  1
Solution:
(a) x9  3.44
The sequence converges to 3 decreasingly.

(b) If the sequence converges to l. So when n   , xn1  l and xn  l .


Solving, we have
2(l 2  l )
l  1  3(l  1)2  2l 2  2l  l 2  4l  3  0  l  1 or l  3 .
3
Hence, if the sequence converges, then it converges to either 1 or 3. [Proven]

(c) The sequence converges to 1 when k = 1

(d) From GC, m = 2.

Method I:

xn 1  1 3
 xn  xn 
2 2
2 xn 2 1
   1
xn  1 xn  1 3  xn  1 3 xn  1
1 1
2  xn  3   .
xn  1 2
2 1 2 3
 1  1
3 xn  1 3 2
x 1
 n 1  1  xn 1  xn
xn  1

xn 1  1 3
 xn  xn 
2 2
2 xn
Or  
xn  1 xn  1 3  xn  1

2 xn 2 xn
Now xn  3  2 xn  3  3 xn   1  xn  1  1
3  3 xn 3  xn  1

TJC/MA9740/JC1Promo2013

sgfreepapers.com 194
16

Method II:

xn1  1 3
 xn  xn 
2 2
2 xn
 
xn  1 xn  1 3  xn  1
2x
From the graph of y  , when 2  x  3, y  1
3  x  1

1
x
0 2 3

Since 2  xn  3
xn 1  1 2 xn
  1  xn1  xn
xn  1 3  xn  1

TJC/MA9740/JC1Promo2013

sgfreepapers.com 195
17


1  1 
12 (a) Find ln   dx , leaving your answer in exact form. [4]
1 x2  x2 

t
(b) Using the substitution u  t , find  t  1 dt . [6]

Solution:
(a) Method I (simplify using Laws of Log before integration):


e
1  1 
ln   dx
1 x2  x2 


e
 2 x 2 ln x dx
1
 1 

e
e
 
 2    x ln x    x 1 dx 
1

 1
1 x 
 

e
 2   e1  0   x 2 dx 
 1 


 2 e1    x 1  
e

1

 
 2 e 1   e 1  1  2 2e 1  1  
 4e1  2

Method II (apply By Parts formula without simplification):


e
1  1 
ln   dx
1 x2  x2 
 
1 1  2 

e


e
 1  1 
   ln  2          dx
 x  x  1 1 x  1   x3 
 
 x2 


e
 1  1  2
   ln  2   ln1  dx
 e e   1 x2


e
 1  2
    2   0   dx
 e  1 x2
e
2  2
  
e  x 1
2  2 
    2
e  e 
4
 2
e

TJC/MA9740/JC1Promo2013

sgfreepapers.com 196
18

(b) u t  t  u2
dt
Diff. wrt u,  2u
du
t
 t  1 dt
u u2
  2  2u  du  2 2 du
u 1 u 1
 1 
 2 1  2  du
 u 1 
 1 u 1 
 2 u  ln C
 2 u  1 
t 1
 2 t  ln C
t 1

TJC/MA9740/JC1Promo2013

sgfreepapers.com 197
19

k 2 1
13 It is given that f  x    x  1  where k  1 .
x 1

(i) Show by differentiation that the graph of y  f  x  has no turning points. [3]

(ii) On separate diagrams, draw sketches of the graphs of

(a) y  f  x , [4]

(b) y  f ' x . [2]

You should indicate where possible, numerically or in terms of k, any asymptotes


and axial intercepts for each of the curves.

(iii) Find in terms of k, the range of x that satisfies the inequality


k f ( x)   x  k   x  k 
2
[4]
Solution:
k 2 1 k 2 1
(i) f  x  x 1   f   x   1 
x 1  x  12
Since k  1,  k 2  1  0
k 2 1
Since  x  1 is also always > 0, 1 
2
0
 x  12
 f '( x )  0 for all x  
 y  f  x  has no turning points.

Hence y  f  x  has no turning point.

TJC/MA9740/JC1Promo2013

sgfreepapers.com 198
20

k 2 1
(ii)(a) When x = 0, y  1   k 2
1
k 1
2
When y = 0, x 1 0
x 1
k 2  1   x  1 x  1
k 2 1  x2 1
x  k
y

(ii)(b)
y

TJC/MA9740/JC1Promo2013

sgfreepapers.com 199
21

(iii)
Methodd 1:
k f ( x)   x  k   x  k 
2

 f ( x) 
 xk x  k 
2

Sketcch the curvees y  f ( x) and y


 x k x k
2

k
Case 1:
y

x
k α k

 k2 y  f ( x)

To findd α and , seet

k 2 1  x  k   x  k 
2
x 1 
x 1 k


x  k
2 2

x k x  k
2

x 1 k
 x  k  1 
  x  k  x  k    0
 k x  1
 x 2   k  1 x  2k 
  x  k  x  k   0
 k  x  1 
 x   k or o x   k  1 x  2k  0
2

 k  1   k  12  8k
 x   k or
o x
2

 
 k  1  k  6k  1
2
nd  
an
 k  1  k 2  6k  1
2 2

 k  x  1 or
 k  1 
k 2  6k  1
x
 k  1  k 2  6k  1 or x  k
2 2
This case is valid if k  6k  1  0 , i.e.  k  3  8  0 , i.e. k  3  2 2 (ssince k > 1)
2 2

TJC/MA97
740/JC1Promo
o2013

sgfreepapers.com 200
22

Case 2 ( 1  k  3  2 2 ):
y y
 x  k 2  x  k 
x 1 k
y  x 1

x
k k

k2

From the diagram, we have


k  x  1 or x  k .

TJC/MA9740/JC1Promo2013

sgfreepapers.com 201
23

Method 2:
 k 2 1 
  x k x  k
2
k   x 1 
 x 1 
  x2  k 2 
  x k x  k
2
k
 x 1 
k
 x  k  x  k   
 x  k   0
 x 1 

 x 2   k  1  2k 
 x  k  x  k     0
 x  1 
 
 x  k  x  k  x  1 x   k  1  2k  0 ,
2
x 1

 
Case 1( x 2   k  1  2k can be factorized, i.e. when  k  1  4 1 2k   0 ,
2

i.e. k 2  6k  1  0 ,
6  36  4
i.e. k  ,
2
i.e. k  3  2 2 )

We have
   k  1  k  1 2  8k   
        x     k  1   k  1  8k
2
  0
 x  k  x  k  x  1  x        
  2 2
    

 k  x  1 or
 k  1  k 2  6k  1
x
 k  1  k 2  6k  1
or x  k
2 2

Case 2 ( 1  k  3  2 2 )

Since x 2   k  1  2k  0 ,
  x  k  x  k  x  1  0
k  x  1 or x  k

TJC/MA9740/JC1Promo2013

sgfreepapers.com 202
TEMASEK JUNIOR COLLEGE, SINGAPORE
JC One
Promotion Examination 2013
Higher 2

MATHEMATICS 9740
4 October 2013
Additional Materials: Answer paper 3 hours
List of Formulae (MF15)

READ THESE INSTRUCTIONS FIRST

Write your Civics Group and Name on all the work that you hand in.
Write in dark blue or black pen on both sides of the paper.
You may use a soft pencil for any diagrams or graphs.
Do not use staples, paper clips, highlighters, glue or correction fluid.

Answer all the questions.


Give non-exact numerical answers correct to 3 significant figures, or 1 decimal place in the case of
angles in degrees, unless a different level of accuracy is specified in the question.

You are expected to use a graphic calculator.

Unsupported answers from a graphic calculator are allowed unless a question specifically states
otherwise.

Where unsupported answers from a graphic calculator are not allowed in a question, you are required to
present the mathematical steps using mathematical notations and not calculator commands.

You are reminded of the need for clear presentation in your answers.

The number of marks is given in brackets [ ] at the end of each question or part question.

At the end of the examination, fasten all your work securely together.

This document consists of 5 printed pages.

© TJC 2013 [ Turn over

TJC/MA9740/JC1Promo2013

sgfreepapers.com 203
2

1 Find the general solution of the following differential equation

1 dy 1
  0, where x  1. [4]
1  x dx 1  x 2

2 (i) The first three terms of a sequence are given by u1  19 , u2  34 , u3  52 . Given


that un is a quadratic polynomial in n, find un in terms of n. [4]

(ii) Find the smallest value of n for which un is greater than 200. [2]

3 A wire of length L cm is cut into two pieces. One piece is used to form a circle while
the other piece is used to form an equilateral triangle.
Show that, with the total area of the circle and triangle being the smallest, the ratio of
3
the length of the smaller piece to the length of the bigger piece is .
9
[6]

4
y
y   x  1 , x  1
2

y  4x  4
4

y  x 1
2
R

x
1 1 3

The shaded region R in the diagram above is bounded by the y-axis, the line
y   x  1 and the curves y   x  1 for x  1 and y  4 x  4 .
2

Find the volume of the solid of revolution formed when R is rotated completely about
the y-axis. [6]

TJC/MA9740/JC1Promo2013

sgfreepapers.com 204
3

5 Given that y  ln  2  tan 1 x  , show that


2

1  x  ddxy2  2 x ddyx  1  x2   ddyx   0 .


2
2
[3]

Hence find the Maclaurin's expansion for y, up to and including the term in x2 . [3]

n
r 9 1 3 n
6 Prove by mathematical induction 3
r 1
r 1
     for all positive integers
4 3n1  4 2 
of n. [5]
Hence show that
1 2 3 4 9
 2  3  4 . [2]
4 4 4 4 16

7 Functions f and g are defined by


2x  2
f:x , for x  , x  1 ,
x2
g : x  2  x , for x  , x  2 .

(i) Given that f has an inverse, show that the composite function gf 1 exists. Find

gf 1 and state its range. [5]

(ii) Find the value(s) of x such that f  x   f 1


 x . [2]

8 Prove that

  r  1 r  2  
ln    ln   r  1 r    2 ln   r  r  1   ln   r  1 r  2   . [2]
 r  r  1 

Hence, find in terms of n,

 1 4   25   3 6    n  1 n  2     n  n  3 
ln    ln    ln      ln    ln   ,
 23   3 4   45    n  n  1    n  1 n  2  

leaving your answer as a single logarithmic function. [5]

TJC/MA9740/JC1Promo2013

sgfreepapers.com 205
4

9 Jessie wishes to take up a loan of $20,000 on the 1st day of the Year 2014. She intends
to pay an instalment of $300 on the 1st day of each month, beginning from February
2014. She sources out two banks, XYZ Bank and ABC Bank, which offer such loans.
The two banks have different ways of charging interest. XYZ Bank charges a monthly
interest of 0.5% on the outstanding amount owed at the end of each month, while ABC
Bank charges a fixed interest of $60 at the end of each month until the loan is repaid.

(a) If Jessie takes up the loan from XYZ Bank, show that the outstanding loan at the
end of February 2014 after the interest has been added will be $19899. [2]

Hence, find the number of months Jessie will take to repay her loan. [4]

(b) Which bank should Jessie take a loan from if she wishes to clear her loan as soon
as possible? Justify your answers. [3]

10 A curve C is given parametrically by the equations

x  2cos3  , y  2sin 3 

 
where    .
2 2

Show that the normal at the point with parameter  has equation

y sin   x cos   2  sin 4   cos 4   . [4]


The normal at the point Q where   , cuts C again at the point P, where   p .
6
Show that sin 3 p  3 cos3 p  1  0 and hence find the coordinates of P. [5]

11 A sequence of real numbers x1 , x 2 , x3 ,... satisfies the recurrence relation

2( xn2  xn )
xn1   1 , x1  k , where k  1 .
3

(a) When k = 5, state the value of x 9 and describe the behavior of the sequence. [2]

(b) Prove algebraically that, if the sequence converges, then it converges to


either 1 or 3. [3]

(c) State a value of k such that the sequence converges to 1. [1]

(d) When k  2 , state the integer m such that m  xn  m  1 for all integers n  1 . [1]
x 1
Hence, by considering n 1 , show that x n 1  x n for all integers n  1 . [3]
xn  1
TJC/MA9740/JC1Promo2013

sgfreepapers.com 206
5


e
1  1 
12 (a) Find ln   dx , leaving your answer in exact form. [4]
1 x2  x2 

t
(b) Using the substitution u  t , find  t  1 dt . [6]

k 2 1
13 It is given that f  x    x  1  where k  1 .
x 1

(i) Show by differentiation that the graph of y  f  x  has no turning points. [3]

(ii) On separate diagrams, draw sketches of the graphs of

(a) y  f  x , [4]

(b) y  f ' x . [2]

You should indicate where possible, numerically or in terms of k, any asymptotes


and axial intercepts for each of the curves.

(iii) Find in terms of k, the range of x that satisfies the inequality


k f ( x)   x  k   x  k 
2
[4]

***End of Paper***

TJC/MA9740/JC1Promo2013

sgfreepapers.com 207
VICTORIA JUNIOR COLLEGE
PROMOTIONAL EXAMINATION

MATHEMATICS 9740
(HIGHER 2)
Monday 8 am -11 am
23 September 2013 3 hours

Additional materials: Answer Paper


List of Formulae (MF15)

READ THESE INSTRUCTIONS FIRST

Write your name and CT group on all the work you hand in.
Write in dark blue or black pen on both sides of the paper.
You may use a soft pencil for any diagrams or graphs.
Do not use staples, paper clips, highlighters, glue or correction fluid.

Answer all the questions.


Give non-exact numerical answers correct to 3 significant figures, or 1 decimal place in the case of angles in
degrees, unless a different level of accuracy is specified in the question.
You are expected to use a graphic calculator.
Unsupported answers from a graphic calculator are allowed unless a question specifically states otherwise.
Where unsupported answers from a graphic calculator are not allowed, you are required to present the
mathematical steps using mathematical notations and not calculator commands.
You are reminded of the need for clear presentation in your answers.

At the end of the examination, fasten all your work securely together.
The number of marks is given in brackets [ ] at the end of each question or part question.

This document consists of 6 printed pages

© VJC 2013 VICTORIA JUNIOR COLLEGE [Turn over

sgfreepapers.com 208
1 A sequence with its first four terms given is shown below.

1, (1  2), (1  2  2 2 ), (1  2  22  23 ), 

Show that the n th term of this sequence is 2 n  1. [2]

Find the sum of the first n terms of the sequence. [3]

2 A sequence of positive real numbers x1 , x2 , x3 , ... satisfies the relation

3  xn
xn 1  for n  1.
2 xn  3
(i) Given that the sequence converges to  , find the exact value of  . [3]

(ii) By using a graphical approach, prove that

xn 1  xn if 0  xn   .
[2]

3 A curve is defined by the parametric equations

x  2 at 2 , y  3at ,

where a is a non–zero constant.

 17a 
Given that B is the point  ,0  , find the coordinates of the points on the curve which
 4 
are nearest to B. [5]

4 (i) Given that f  r    r  1 r 2 , show that f  r  1  f  r   r  3r  1 . [1]

N
(ii) Use the method of differences to find  r  3r  1
r 1
in terms of N. Hence find the
N
r  3r  1
limit of 
r 1 N3
as N approaches infinity. [3]

N
(iii) Use your first answer in part (ii) to find   r  1 3r  2 
r 3
in the form

aN 3  bN 2  cN  d , where a , b, c and d are constants to be found. [2]

sgfreepapers.com 209
d  x  2 1
5 (a) (i) Prove that  2  2  2 . [2]
dx  x  1   x  1 2
x 1


1
1
(ii) Find the exact value of dx. [3]
x 2  1
2
0

e2 x
 1 e
1 1
(b) Find the constant A such that  A . Hence find dx. [3]
1  e2 x 1  e2 x 2x

1 1
6 (i) Find the expansion of  in ascending powers of x, up to and
1  x2 (1  x) 2
2
including the term in x . [3]

1
Let y  sin 1 ( x)  .
(1  x)
(ii) By successively differentiating y, find the Maclaurin’s series for y, up to and
including the term in x3 . [4]

(iii) Show that the same result in part (i) can be obtained by using your answer in part
(ii). [2]

7 A sequence u0 , u1 , u2 , … is such that u0  b and un 1  run  a , for all n ൒ 0, where

a , b and r are constants.

(a) For the case where r  1 ,


1 rn
(i) prove by induction that un  r b  a for n  0,
n
[4]
1 r

(ii) write down the set of values of r for which the sequence u0 , u1 , u2 , . . .

converges, and state the limit of this sequence. [2]

N
(b) For the case where r  1 , find u1 , u2 , u3 , and hence find u
n 0
n in terms of a, b, N.

N 1
Give your answer in the form  k2b  Na  , where k1 and k2 are integers to be
k1
determined. [3]

[Turn over
3

sgfreepapers.com 210
8 y

x
O

x
The above diiagram show
ws a sketch of the curv quation y 
ve C with eq , x  0.
ex
(aa) (i) Finnd the exactt coordinate s of the max
ximum poin
nt on C. [3]

hat ln x  x – 1 for all x > 0.


(ii) Hennce show th [2]

b)
(b A partiicle is consttrained to m
move along C, starting from the orrigin O, such that its
x-coorddinate increeases at a coonstant rate. The particcle took 2 sseconds to reach
r the
 4  a
point  4, 4  . Wh
hen it is at thhe point  a, a  , the y-coordinat
y te of the paarticle is
 e   e 
decreassing at a raate of 0.25 uunit per seco
ond. Find a given that a < 2. [4]

9 (aa) m, S n 1 , off the first n  1 terms of


The sum o a sequencce u1 , u2 , u 3 , … is giv
ven by

S n 1  8n 2  19n  11 .
(i) Finnd un and sh
how that thee sequence is an arithm
metic progreession. [4]
(ii) Finnd the least value of n, such that sum of the first
fi n terms is at least 4000
4 less
thaan the sum of
o the next n terms. [3]

b)
(b

4m

A frog falls into a mud ddy drain wwith a slantt wall meassuring 4m iin length. Itt tries to
esscape from the drain by b leaping ssuccessively y on the slaant wall. T
Though it can cover
0..7 m in its first leap, the
t wall is so slippery y that for suubsequent aattempts it can only
coover 4/5 thee distance of
o its previoous leap. Determine if the frog wiill be able to
t escape
foorm the draiin, justifying your answwer. [3]

sgfreepapers.com 211
y
10 (i) y  f ( x)
 4, 1 
 
 3, 2   2
x
y0 O  3, 0 

x  2 x2

The diagram ows the grraph of y  f  x  . It has a non--stationary point of


m above sho
innflexion  0, 0  , an inteersection wiith the x-axis at  3, 0  , a minimum
m point  3,
3 2  and
 1
a maximum point  4,  . The veertical asym he graph aree x  2 an
mptotes of th nd x  2 .
 2
The horizonttal asymptotte is y  0 .

Skketch the grraph of y  f  2x  , m


making cleaar the main relevant
r feaatures and the shape
off the graph near the points where y  0 . [3]

((ii)

ws the graphh of y  g  x  . The inteersections oof the graph with the


Thhe diagram above show
axxes have cooordinates  0, 1 , 1, 0  and  3, 0  . The asymptotes of thee graph are the lines
x  2 and y   x  2 .

Skketch the grraph of y  g '  x  , makking clear th


he main releevant featurres. [3]

((iii) The funnction h is defined


d as
. g( x) for x  2,
h(xx)  
 f ( x) for x  2.
Skketch the grraphs of
(aa) y = h(x)), [1]
1
(bb) y  , making clear
c the maain relevant features. [4]
h( x)
urn over
[Tu
5

sgfreepapers.com 212
11 The function f is defined as follows.
4
f :x x for x  , x  0.
x
(i) Find f 1 ( x ) . [3]
(ii) Show that f ( x)  0 . [1]
(iii) Solve the inequality f 1 ( x)  6 , giving your answer in exact form. [2]
(iv) Sketch the graph of y  f 1f ( x ) . [1]

Functions h and g are defined by


4
h: x  x  for x   , x   2 , x  0 , x  2,
x
1
g : x  1 for x  , x  0.
x
( x 2  x  4)
(v) Show that gh( x)   . [1]
( x 2  4)
(vi) Solve the inequality gh( x )  0 , giving your answer in an exact form. [3]

 x  1
2
y2
12 The curve C1 has equation  4.
4 9
Sketch C1, making clear the main relevant features, and state the set of values that x can
take. [4]

Another curve C2 is defined by the parametric equations


2
x , y  3 t ln t , where t  1.
t 1
2

Use a non-graphical method to determine the set of possible values of x. [2]

Sketch the curve C2, labelling all axial intercepts and asymptotes (if any) clearly. [2]

Hence, without solving the equation, state the number of real roots to the equation
2
 2 
 
2
9 2  1  4 3 t ln t  144 ,
 t 1 

explaining your reason(s) clearly. [2]


Given that k > 0, state the smallest integer value of k such that the equation
2
 2 
 
2
9 2  k  1  4 3 t ln t  144
 t 1 
has exactly one real root which is positive. [2]

sgfreepapers.com 213
Victoria Junior College
Mathematics H2 (9740) – JC 1 Promotional Examination 2013
Solutions

1. (i) The nth term


 1  2  22  ...  2n 1

1  2n

1 2
 2n  1

Sn    2r  1   2r 1
n n n
 ii 
r 1 r 1 r 1

2(1  2n )
 n
1 2
 2n1  n  2

2. (i) As n   , xn   and xn1   .


3

2  3
2  4  3  0
2

4  16  24

4
  1  12 10

Since xn  0 for all n,   1  12 10 .

3 x 1 9
(ii) Sketch y    and y  x .
2x  3 2 2(2 x  3)
y
y=x

(0,1) 3 x
y
(3, 0) 2x  3
o
x
y
1 
2

3
x
2

sgfreepapers.com 214
3 x
2. When 0  x   , the graph of y  is above the graph of
2x  3
3 x
y  x.   x.
2x  3

3  xn
Hence for 0  xn   ,  xn
2 xn  3

 xn 1  xn .

3 (i) Let A be a point on the curve.


2
 17 a 
AB    2at 2    0  3at 
2 2

 4 
2
289a
  4a 2t 4  17 a 2t 2  9a 2t 2
16
289a 2
 4 a 2 t 4  8a 2 t 2 
16
289a 2
AB  4a 2t 4  8a 2t 2 
16

Let S  AB .

dS 16a 2t 3  16a 2t

dt 289a 2
2 4a 2t 4  8a 2t 2 
16

dS
Let  0 , then
dt

16a 2 t 3  16a 2t
0
289a 2
2 4 a t  8a t 
2 4 2 2

16

16a 2t 3  16a 2t  0  t (t 2  1)  0

 t  0 or t  1 or t  1
17 a
At t  0 , S  AB  .
4
15a
At t  1 , S  AB  (nearer)
4
Hence, substitute t  1 (which correspond to points nearest to B) into
x and y .
The coordinates are: (2a, 3a) and (2a, -3a).

sgfreepapers.com 215
4. (i)

f(r  1)  f(r )
 r (r  1)2  (r  1)r 2
 r (r  1)2  (r  1)r 
 r  r 2  2r  1  r 2  r 
 r (3r  1)
N
 ii   r(3r  1)
r 1
N
   f(r  1)  f(r) 
r 1

 f(2)  f(1) +
f(3)  f(2) +

f(N )  f(N  1) +
f(N  1)  f( N ) +
 f(N  1)  f(1)
 N ( N  1)2  0
 N ( N  1)2

r  3r  1 N ( N  1) 2  N  1  
2 2
N
1

r 1 N 3

N 3

 N  
  1   .
N

1 N
r  3r  1
As N   ,
N
 0.  the limit of 
r 1 N3
is 1.

N
 iii   ( r  1)(3r  2)
r 3

 2  7  3  10  ...  ( N  1)(3N  2)
N

 r(3r  1)
r 1

 1  4   2  7  ...  ( N  1)(3N  2)   N (3N  1)


N N
  ( r  1)(3r  2)   r (3r  1)  4  N (3N  1)
r 3 r 1

 N ( N  1) 2  4  N (3N  1)
 N 3  2 N 2  N  4  3N 2  N
 N3  N2  4

sgfreepapers.com 216
d  x  x 2  1  x (2 x )
5. (a)  i   
dx  x 2  1   x 2  1
2

1  x2

x  1
2 2

2  1  x2

x  1
2 2

2 1  x2
 
x  1 x  1
2 2 2 2

2 1
 
x  1 x 1
2 2 2

 1 

1 1
2  x 
 ii    2 dx   2 
  x 2  1 x  1  x  10
2
0
 


1
1 1 1
2 dx   tan 1 x  
x 2  1
2
0
0 2


1 
1
1
2 dx  
x 2  1
2
0 2 4


1 
1
1
dx  
x 2  1
2
0 4 8

e2 x
(b) RHS = A+
1  e2 x
A  Ae2 x  e 2 x

1  e2 x
Comparing the numerator to that of the LHS,

A  Ae2 x  e2 x  1
 A 1

1  e2 x 
 1  e2 x dx   1  1  e2 x  dx
1
 x  ln 1  e2 x  C
2

sgfreepapers.com 217
6 (i)

1 1 1
  (1  x 2 ) 2  (1  x) 2
1  x2 (1  x) 2

 1   (2)(3) 2 
 1  x 2     1  2 x  x  
 2   2! 
5
 2x  x2  
2
dy 1
 ii    (1  x ) 2
dx 1 x 2

d y  1
2
3
    (1  x 2 ) 2 ( 2 x )  2(1  x ) 3
 2
2
dx
3
 x (1  x 2 ) 2
 2(1  x ) 3
d3 y 3  3 5
 (1  x 2 ) 2  x    (1  x 2 ) 2 ( 2 x )  6(1  x ) 4
 2
3
dx

When x = 0,

y 1
dy
 1 1  0
dx
d2 y
 02  2
dx 2
d3 y
 1  0  6  5
dx3
5 3
Hence, y  1  x 2  x 
6

1 5
(iii) y  sin 1 ( x )   1  x2  x3  
(1  x ) 6

Differentiating both sides w.r.t x,


1 1 5
 = 2 x  x 2   (verified).
1 x 2 (1  x) 2
2

sgfreepapers.com 218
7. (a)(i)

1 rn
Let Pn be the statement: un  r nb  a for n  0.
1 r
Consider P0 :
L.H.S. of P0 = u0 = b

1 r0
R.H.S. of P0 = r 0b  a b
1 r

 P0 is true.

Assume Pk is true for some k  0.

1 rk
i.e. uk  r b  a
k
.
1 r

Consider Pk  1 :

1  r k 1
R.H.S. of Pk  1 = r k 1b  a
1 r

L.H.S. of Pk  1 = uk  1
 ruk  a
 1  rk 
 r  rkb  a a
 1 r 
k 1
ar 1  r k  a (1  r )
 r b 
1 r 1 r
k 1
ar  ar  a  ar
 r k 1b 
1 r
a 1  r k 1 
 r k 1b 
1 r

 Pk is true  Pk  1 is true.

 P0 is true
Hence, 
 Pk is true  Pk 1 is true.

1 rn
By induction, un  r nb  a for n  0.
1 r

sgfreepapers.com 219
7  ii  The sequence converges for r   : 1  r  1.
a
The limit of the sequence is .
1 r

(b)

u0  b
u1  b  a
u2  b  2a
u3  b  3a
 
uN  b  Na
N
N
  un   N  1 b   a  Na 
n 0 2
N
  N  1 b  1  N  a
2
N 1
=  2b  Na 
2

x
8(a)(i) y 
ex
dy e x  xe x

dx e2 x
1 x
 x
e
dy
 0  x 1
dx

 1
Substitute x  1 into y . Maximum point is 1,  .
 e

(ii) For x  0,
1 x 1
y  i.e. x 
e e e
Since ln is an increasing function,
 x
ln  x   ln  e 1 
e 
 ln x  ln e x  1
 ln x  x  1
 ln x  x  1

sgfreepapers.com 220
8(b) The particle took 2 seconds to move from x  0 to x  4 ,

dx
so  2.
dt

At x  a ,

dy dy dt
 
d x d t dx
1 1
 0.25  
2 8

 a  dy 1  a
At  a , a  ,  a
 e  dx e
1 a 1
 a  1 x
e 8 y
ex
From GC, a  1.65 (reject 2.45 as a  2 ).
y y

1 x
y
ex

x
o
1
y
8

9  a  i  Replacing n with n  1,
Sn  8(n  1) 2  19(n  1)  11
 8n 2  16n  8  19n  19  11
 8n 2  3n

un  Sn  Sn 1
  8n 2  3n   8n 2  19n  11
 16n  11
un  un 1  16n  11  16( n  1)  11
 16
Since the difference between 2 consecutive
terms is a constant, the sequence is an AP.

sgfreepapers.com 221
 ii   S2n  Sn   Sn  4000
8(2n) 2
 3(2n )   2  8n 2  3n   4000
32n 2  6n  16n 2  6n  4000
n 2  250
 n  15.8 (reject as n    ) or n  15.8
Thus, least n is 16.

(b) The distance covered by frog is a GP with a =0.7 and r = 0.8


Total distance covered after n leaps is given by

0.7 1  0.8n 
Sn 
1  0.8
 3.5 1  0.8n 
As n  , (0.8) n  0  Sn  3.5, that is, S   3.5
Since S  4, the frog will never be able to escape from the drain.

10 (i) y

1
(2, )
2
( 3 , 2)
2 y  f 2x
y=0 (0, 0) x
( 3 , 0)
2
x  1

(ii) y

x
o
y  1
y  g '( x )

x2

(iii) (a)

y x2
(0,1) 1
(4, )
2 y  h( x)
o (1,0) x
(3, 0) y0

y  x  2

sgfreepapers.com 222
10(b)

y
1
y
h( x)
(0,1) (4, 2)
(2, 0)
x
y0 o

x 1 x3
4 x2  4
11 (i) y  x  y
x x
x  xy  4  0
2

y  y 2  16
x
2
y  y 2  16
Since x  0 , x 
2
1 1 1 1 2
 f 1 ( y )  y  y 2  16  f 1 ( x )  x  x  16 .
2 2 2 2
4 4
(ii) f '( x )  1  2 . Since 2  0 for all real x<0, f '( x )  1
x x
Hence f '( x )  0.

(iii) Since f is an increasing function,


f 1 ( x )  6  f  f 1 ( x )   f ( 6)
4 16
x  6  x
6 3
y
(iv)

o x
yx
y  f 1f(x)

1
(v) gh( x )  g  h(x )   1
x 4 2

x
x x  ( x 2  4) ( x 2  x  4)
 2 1   
x 4 x2  4 x2  4

10

sgfreepapers.com 223
11(vi) Test Point method:

x2  x  4  0  x 
1
2

1  17 
     ( x 2  x  4)
Sign of 
2
1
(1  17) 2
1
(1  17)
x2  4
2 2
1 1
 2  x  (1  17 ) or 2  x  (1  17)
2 2
Alternatively, use graphs:
x  2
y x2

1  1 
 (1  17), 0   (1  17), 0 
2  2 
o
x
y  1 ( x 2  x  4)
y
‘ x2  4

1 1
 2  x  (1  17 ) or 2  x  (1  17)
2 2

12 y
3
y  ( x  1)
2

(1, 0)
o x
( 3, 0) (5, 0)
 x  1
2
y2
 4
4 9
3
y   ( x  1)
2
 x  1  x  1  y 2  1
2 2
y2
 4
4 9 42 62
 the set of values of x =  x   : x  3 or x  5
1 1
t 2  1  t2 1  2  0  
t 1 2
2

2
0  1, that is, 0  x  1
t 1 2

 the set of values of x =  x   : 0  x  1

11

sgfreepapers.com 224
12
x0
2
y x , y  3 t ln t , t  1.
t 1
2

(1, 0)
x
o

2
 2 
 
2
9 2  1  4 3 t ln t  144  (1)
 t 1 
2
 2 
 1
 
2
 2
 t  1   3 t ln t 4
4 9

 x  1
2
y2 2
Since C1 :   4 and C2 : x  2 , y  3 t ln t ,
4 9 t 1
the number of roots of the above equation can then be found by the
number of intersections between C1 and C2. However, since C1 is
only defined for x  3 or x  5 and C2 is defined for 0  x  1 ,
there is no point of intersection.
2
 2 
 
2
Hence 9  2  1  4 3 t ln t  144 has no real root.
 t 1 
2
 2 
 
2
9 2  k  1  4 3 t ln t  144
 t  1 

Since x is replaced with x + k in the equation of C1, C1 is


translated k units in the negative x-direction. Hence smallest
integer value of k is 5.
OR
Since x is replaced with x - k in the equation of C2, C2 is
translated k units in the positive x-direction. Hence smallest
integer value of k is 5.

12

sgfreepapers.com 225
ACJC_H2Maths_2013_Promo_Qn 2
ACJC_H2Maths_2013_Promo_Soln 9
AJC_H2 Math_promo2013_Qn 23
AJC_H2 Math_promo2013_Soln 28
CJC_H2Math_2013_Promo_Qn 38
CJC_H2Maths_2013_Promo_Soln 43
HCI_2013_H2Maths_Promo_Qn 54
HCI_2013_H2Maths_Promo_Soln 61
IJC_H2Maths_2013_Promo_Qn 74
IJC_H2Maths_2013_Promo_Soln 80
JJC_H2Maths_2013_Promo_Qn 105
JJC_H2Maths_2013_Promo_Soln 110
MJC_H2_Maths_2013_Promo_Soln 121
MJC_H2Maths_2013_Promo_Qn 133
NYJC_H2Maths_2013_Promo_Qn 141
NYJC_H2Maths_2013_Promo_Soln 148
RVHS_H2Maths_2013_Promo_Qn 158
RVHS_H2Maths_2013_Promo_Soln 166
TJC_H2Math_promo_Soln 180
TJC_H2Math_promo2013_Qn 203
VJC_H2Maths_2013_Promo_Qn 208
VJC_H2Maths_2013_Promo_Soln 214

sgfreepapers.com 1
ANGLO-CHINESE JUNIOR COLLEGE
MATHEMATICS DEPARTMENT

MATHEMATICS
Higher 2 9740
Paper 1
8 October 2013
JC 1 PROMOTIONAL EXAMINATION
Time allowed: 3 hours
Additional Materials: List of Formulae (MF15)

READ THESE INSTRUCTIONS FIRST

Write your Index number, Form Class, graphic and/or scientific calculator model/s on the cover page.
Write your Index number and full name on all the work you hand in.
Write in dark blue or black pen on your answer scripts.
You may use a soft pencil for any diagrams or graphs.
Do not use paper clips, highlighters, glue or correction fluid.

Answer all the questions.


Give non-exact numerical answers correct to 3 significant figures, or 1 decimal place in the case of
angles in degrees, unless a different level of accuracy is specified in the question.
You are expected to use a graphic calculator.
Unsupported answers from a graphic calculator are allowed unless a question specifically states
otherwise.
Where unsupported answers from a graphic calculator are not allowed in the question, you are
required to present the mathematical steps using mathematical notations and not calculator commands.
You are reminded of the need for clear presentation in your answers.

The number of marks is given in brackets [ ] at the end of each question or part question.
At the end of the examination, fasten all your work securely together

This document consists of 7 printed pages.

Anglo-Chinese Junior College


H2 Mathematics 9740: 2013 JC 1 Promotional Examination
Page 1 of 7

sgfreepapers.com 2
ANGLO-CHINESE JUNIOR COLLEGE
MATHEMATICS DEPARTMENT
JC 1 Promotional Examination 2013

MATHEMATICS 9740
Higher 2 / 100
Paper 1

Index No: Form Class: ___________


Name: _________________________
Calculator model: _____________________

Arrange your answers in the same numerical order.


Place this cover sheet on top of them and tie them together with the string provided.

Question no. Marks


1 /2

2 /3

3 /4

4 /4

5 /5

6 /6

7 /6

8 /7

9 /6

10 /9

11 /9

12 / 11

13 / 13

14 /9

15 /6

Anglo-Chinese Junior College


H2 Mathematics 9740: 2013 JC 1 Promotional Examination
Page 2 of 7

sgfreepapers.com 3
1 The graph of y  f  x  undergoes, in succession, the following transformations:

Step 1: a translation of 1 unit in the negative y-direction; followed by


Step 2: a stretch with scale factor 2 parallel to the x-axis.
3
The equation of the resulting curve is y  ln  2 x  3 , x   . Determine the equation
2
of the graph, y  f  x  . [2]

2 Given that the curve y  ax 3  bx 2  cx  d has turning points at (-4, 258) and (4, 2).
Write and solve a system of simultaneous linear equations satisfied by the constants
a, b, c and d . [3]

3 Differentiate the following with respect to x.

 x
(i) cos 1   , [2]
2

 x  1
3

(ii) ln . [2]
x2  1

4 Find the following integrals:


1
(i) x ln x
dx ; [2]

e2 x
(ii)  4  e 4 x
dx . [2]

3x 2  6 x  10
5 Without the use of a graphing calculator, solve the inequality  2. [3]
x 2  3x  4
3x 2  6 x  10
Deduce the range of values of x such that  2. [2]
x2  3 x  4

[Turn Over

Anglo-Chinese Junior College


H2 Mathematics 9740: 2013 JC 1 Promotional Examination
Page 3 of 7

sgfreepapers.com 4
6 A curve C has parametric equations
x  1  cos , y    sin  ,
where 0    2 ,
dy 1
(i) Show that  cot  and find the gradient of C at the point P where    . [3]
dx 2

(ii) The tangent at P meets the y-axis at A. The tangent at the point Q, where   ,
2
meets the y-axis at B. Find the area of triangle ABP. [3]

7 A right pyramid block has a square base ABCD and its vertical height VM is
( a  x ) where 0  x  a . M is the point where the diagonals AC and BD of the square
meet. This right pyramid block is inscribed in a sphere of fixed radius a so that the
vertices V, A, B, C and D of the block just touch the interior of the sphere with the
vertical height VM passing through the centre O of the sphere.

(i) Show that the length of the side of the square base ABCD is 2( a 2  x 2 ) . [2]

(ii) Hence, find the maximum volume of the block in terms of a. [4]

1
[Volume of a pyramid =  base area  height ]
3

O
B

A C
M

Anglo-Chinese Junior College


H2 Mathematics 9740: 2013 JC 1 Promotional Examination
Page 4 of 7

sgfreepapers.com 5
1
8 The function f is defined by f : x  x  for x  , x  1 .
x
(i) Find f 1  x  and state the domain of f 1 . [3]

(ii) Find fff 1  x  and state its domain and range. [3]

(iii) Show that the composite function f 2 exists. [1]

1 4(k  1)
9 If f  k   , show that f ( k )  f ( k  2)  . [1]
k2 k (k  2) 2
2

2 3 4
Hence, show that the sum to n terms of the series  2 2  2 2   is
        
2
1 3 2
2 4 3 5

15 1 1 
   . [3]

4  4 n  1 2
n  22 

n
k 1 13
Show that  k k  2
k 2
2 2

144
for all values of n  2 . [2]

Use integration by parts to find the exact value of 1  ln x  dx .


e 2
10 (a) [4]

(b) By means of the substitution x  3cos 2   7 sin 2  , where 0    , prove that
2
7 1
3 dx   . [5]
 7  x  x  3 

11 y y  sin x

A2

A1 y  cos x

x
O 
2 
The region bounded by the axes and the curve y  cos x from x  0 to x  is divided
2
into two parts, of areas A1 and A2 , by the curve y  sin x (see diagram). Prove that

A1   2 A . 2 [5]

1
The line y  meets the curve y  sin x and the y-axis at P and Q respectively. The
2
region OPQ, bounded by the arc OP and the lines PQ and QO, is rotated through 4 right
angles about the x-axis to form a solid of revolution of volume V. Find the exact value
of V in terms of  . [4]
Anglo-Chinese Junior College [Turn Over
H2 Mathematics 9740: 2013 JC 1 Promotional Examination
Page 5 of 7

sgfreepapers.com 6
12 The diagram shows the graph of y  f  x  . The curve crosses the axes at the points

 2a , 0  and  0, 2b  . The asymptotes are x  a and y  b . The gradient of the curve at

the point  0, 2b  is 1. y xa

2b
b yb

O a 2a x

On separate diagrams, sketch the graphs of


1
(i) y , [3]
f  x

(ii) y2  f  x , [3]

(iii) y  f  x , [2]

(iv) y  f  x , [3]

giving the equations of any asymptotes and the coordinates of any points of intersection
with the x- and y-axes.

π 
13 (a) In triangle ABC, angle A =     radians, AB = AC = b and BC = a.
2 
a cos 
Show that  . [1]
b    2 
sin  
 4 
  2 
Deduce, for small values of  , a  2b 1   . [4]
 2 8 
1
(b) Given that y  esin 4 x , show that
dy
(i) 1  16 x 2  4y , [1]
dx
2
(ii) 1  16 x  2  16 x
2 d y dy
 16 y . [2]
dx dx
By further differentiation of the result, find the Maclaurin series for y up to and
including the term in x3 . [3]

 7
By choosing a suitable value of x, show that e 6  . [2]
12

Anglo-Chinese Junior College


H2 Mathematics 9740: 2013 JC 1 Promotional Examination
Page 6 of 7

sgfreepapers.com 7
n
1
14 (a) Prove by induction that  (2r  1)
r 1
2
 n(2n  1)(2n  1) .
3
[4]

(b) Use the result in part (a) to


30
(i) evaluate  (2r  3)
r 1
2
, [2]

n
1
(ii) prove that r 2
 nn  12n  1 . [3]
r 1 6

15 A man met with an accident and went into a coma on 10th January 2013. As a result, he
did not pay the bank the outstanding balance of $M for his credit card bill when it is due
for payment on 27th January 2013. On the 27th of each month when the payment for the
credit card bill is due, the bank will charge a 2% interest on any outstanding balance that
is unpaid. After the 2% interest has been added, the bank will still charge an additional
late payment charge of $L monthly.
(a) Express in terms of L and M, his outstanding balance on his credit card on
1st February 2013. [1]
(b) If the man still remains in coma exactly n months later on the day he met with an
accident, show that the accumulated outstanding balance on the man’s credit card is
1.02n M  50 L(1.02n  1) . [3]
(c) Given that M  1000 and L  55 . Find the least value of n when the accumulated
outstanding balance on his credit card first exceeds $2010. [2]

~ End of Paper ~

Anglo-Chinese Junior College


H2 Mathematics 9740: 2013 JC 1 Promotional Examination
Page 7 of 7

sgfreepapers.com 8
Anglo-Chinese Junior College
H2 Mathematics 9740
2013 JC 1 PROMO Solution
Qn Solution
3
1 y  ln  2 x  3 , x  
2
Before Step 2: y  ln  2  2 x   3  ln  4 x  3
Before Step 1: y  ln  4 x  3  1
OR
1 
Resulting curve: y  f  x   1  ln  2 x  3
2 
1   1  
 f  x   ln  4  x   3  1
2   2  
 y  f  x   ln  4 x  3  1
2 Given y  ax 3  bx 2  cx  d
dy
 3ax 2  2bx  c
dx
dy
When x  4,  0, 3a(4) 2  2b(4)  c  0
dx
48a  8b  c  0 (1)

dy
When x  4,  0, 3a(4) 2  2b(4)  c  0
dx
48a  8b  c  0 (2)

When x  4, y  258,


a (4)3  b(4) 2  c(4)  d  258
64a  16b  4c  d  258 (3)

When x  4, y  2,
a(4)3  b(4) 2  c(4)  d  2
64a  16b  4c  d  2 (4)

Using G.C. a  1, b  0, c  48, d  130.


3i 1
d  1  x 
 1  1  x   2 1 1
 cos      cos    
dx   2 2  2  x
2 2
1  
2
1

x
2  4  x  cos
2 1
 
2

sgfreepapers.com 9
d  
3ii
 x  1
3
  d  ln  x  1  
x 2  1  dx   x  1  
ln
dx 
 
d  1 
  ln  x  1  ln  x  1 
dx  2 
1 1
 
x  1 2  x  1

Alternative solution:
d   x 1  3 x 1  x2 1  x 1  2x
3 2 3
1 1 1
ln 2   
dx  x 1   x 1 2  x 1  x 1
3 3 2 2

 
x2 1 x2 1
x 3
 2
2 x 1

4(i) 1
x ln x
dx

1
 
 x  dx 1
  f ( x)  f ( x)   c
n 1

n
using f '( x)dx 
ln x n 1
 2 ln x  c

e 2 x
(ii)
 4  e 4 x
dx

1 2e 2 x f '( x)  f ( x) 
  dx using  dx  sin 1  c
2 22  e 2 x 
2
a 2   f ( x) 
2
 a 

1  e 2 x 
  sin 1  c
2  2 

sgfreepapers.com 10
5 3 x 2  6 x  10
2
x 2  3x  4
3 x 2  6 x  10
20
x 2  3x  4
3 x 2  6 x  10  2 x 2  6 x  8
0
x 2  3x  4
x2  2
0
 x  4  x  1
 x  2  x  2   x  4 x  1  0
+ - + - +
-4  2 1 2

x  4 or  2  x  1 or x  2

3x 2  6 x  10
2
x2  3 x  4
3 x  6 x  10
2

2
x 3 x 4
2

x  4 (n.a.) ;  2  x  1 ; x 2
1  x  1 or x   2 or x  2

6i
x  1  cos y    sin 
dx dy
 sin   1  cos 
d d

dy 1  cos 

dx sin 

2 cos 2
 2
 
2 cos sin
2 2

cos
 2

sin
2

 cot
2

dy
When    ,  0.
dx

sgfreepapers.com 11
ii) Coordinate at A(0,  )

 dy
When   , 1
2 dx
 
y  sin
2 2 1

Equation of tangent: x  1  cos
2

y  x
2
 
Coordinate at B  0, 
 2

 area of triangle ABP


1  
      2
2  2


2

7i) Diagonal DB  2 a 2  x 2

Length of side of square


 
 sin   2 a 2  x 2
4
2
 2 a2  x2
2
 2  a2  x2 

ii) Volume of block , v 


3
 a  x2   x  a 
2 2

Diff. w.r.t.x
  a 2  x 2    x  a  2 x  
dv 2
dx 3
2
  a  x  a  x    x  a  2 x  
3
2
  x  a  a  3 x 
3

dv
For stationary point, 0
dx

2
0  x  a  a  3x 
3
a
x   a (n.a.) x
3

sgfreepapers.com 12
d 2v 2
  x  a  3   a  3x  
dx 2 3 
d 2v a
2
 0 when x 
dx 3

Max. volume of block ,


2  2  a   a  
2

v   a       a
3 
 3   3  
64a 3
 units 3
81

1
8 f :x  x for x  , x  1
(i) x
1
Let y  x   x 2  yx  1  0
x
y  y2  4 y  y2  4
x or x 
2 2
(rejected since x  1 & y  2)
x  x2  4
f 1  x  
2
Df 1   2,  
1
8 fff 1  x   f  x   x 
(ii) x
Domain of fff 1  Df 1  Rf   2,  
5
Range of fff 1  f ( x) : x   2,     ,  
2
1
8 f :x  x for x  , x  1
(iii) x
D f  1,   , R f   2,   Since R f  D f , ff exists.
9 1
Given f (k ) 
k2
f (k )  f (k  2)
1 1
 2
k (k  2) 2
(k  2) 2  k 2

k 2 (k  2) 2
(k 2  4k  4)  k 2

k 2 (k  2) 2
4(k  1)
 2
k (k  2) 2

sgfreepapers.com 13
2 3 4 n 1
 2 2  2 2   2
2
2
(1) (3) (2) (4) (3) (5) (n) (n  2) 2
n
k 1

k 1 k ( k  2)
2 2

1 1
n
1 
  2  
k 1 4  k (k  2) 2 
1 n 1 n
1 
  2   2 
4  k 1 k k 1 ( k  2) 

1 1 1 1 1 1 1 
  2  2  2  2  2   2 
4 1 2 3 4 5 n 
1 1 1 1 1 1 1 
  2  2  2   2   
43 4 5 n (n  1) (n  2) 2 
2

1 1 1 1 1 
  2 2  
4  1 2 (n  1) (n  2) 2 
2

15 1 1 
    
4  4 (n  1) (n  2) 2 
2

n
k 1
k
k 2
2
(k  2) 2
n
k 1 2
  2 2
k 1 k ( k  2)
2 2
(1 )(3 )
15 1 1  2
    
4  4 (n  1) (n  2) 2  9
2

13 1  1 1 
    
144 4  (n  1) (n  2) 2 
2

13 1 1
 (  0 and  0 n    )
144 (n  1) 2
(n  2) 2

e  2 ln x 
e
1  ln x  dx   ln x   x  1  1 x  x  dx
e 2 2
10
(a)  
 e  2 1  ln x  dx
e

 e 1 
 e  2   ln x  x  1  1 x   dx 
e

 x 
 e  2e  2  e  1  e  2
10 x  3cos 2   7 sin 2 
(b)
dx
 6 cos    sin    14sin  cos 
d
 8sin  cos 
when x  3, 3cos 2   7 sin 2   3  sin 2   0    0

when x  7, 3cos 2   7 sin 2   7  cos 2  0   
2

sgfreepapers.com 14
7 1
3 dx
 7  x  x  3 

8sin  cos 
 02 d

 7  3cos 2   7 sin 2  3cos 2   7 sin 2   3 
  

1
 02  8sin  cos   d
 
 4 cos 2   4sin  

2


 
 2 02 1d  2     (proved)
2
   
11 A1   0
4
 sin x  dx    cos x  dx    cos x 
2 4
0
 sin x 2
4 4

 2   2
    1  1  
 2   2 
 2 2

 
A2   0
4
 cos x  sin x  dx  sin x  cos x 04
 2 2
      0  1
 2 2 
 2 1
OR
 
A2   0
2
 cos x  dx  A1  sin x 02   2  2 
 1  0   2  2  
 2 1
OR
2
A2  0 2
sin y  dy    cos y  dy
1 1
2
1

 A1  2  2  2  
2  1  2 A2 (proved)
 1
P , 
 6 2

1 
2
V      06  sin x  dx
2

2 6

2 

24

2
06 1  cos 2 x  dx

2 
sin 2 x  6
  x 
24 2  2  0
2   3 3  2
      
24 2  6 4  8 24

sgfreepapers.com 15
1
y y
12 f  x
(i)

1
y
b
 1 
 0, 
 2b 
O  a, 0  x

x  2a

12 y2  f  x
(ii) y xa

 0, 2b   2a, 0  y b

x
O
y b
 0,  2b 

12 y  f  x y
(iii)

 0,1

O y0 x

xa

12 yf x y
x  a xa
(iv)

 0, 2b 
y b

 2a, 0  O  2a, 0  x

13a) Sine rule

sgfreepapers.com 16
sin A sin B

a b
    
sin     sin   
2  4 2
a b
a cos 

b   
sin   
4 2

a cos 

b   
sin   
4 2
cos 

   
sin cos  cos sin
4 2 4 2
cos 

2  2 
cos  sin
2 2 2 2
For small values of 
2
1
a 2

b    2

 
2   2   2  
1
2  2  2  2 
 
 
2
1
 2
2    2 
1    
2   2 8 

1
  2     2 
a  2b 1   1    
 2   2 8 

  2     2   1 2     2  
2

 2b 1    1   1        ... 
 2 
 2 8  2 2 8  
  2   3 2 
 2b 1   1   
 2  2 8 
  2 
 2b 1   
 2 8 

13b)

sgfreepapers.com 17
1
i) y  esin 4x

ln y  sin 1 4 x
diff. w.r.t x
1 dy 4

y dx 1  16 x 2
dy
1  16 x 2  4y
dx
ii)
dy
1  16 x 2  4y
dx
diff. w.r.t x
d 2 y dy  1 1

1  16 x 2 2
 
dx  2
1  16 x 
2 2
 32 x    4
dy
dx  dx
d2y 16 x dy dy
1  16 x 2 2
 4
dx 1  16 x 2 dx dx
2

1  16 x  dx 2  16 x dy
2 d y

dx
 4 1  16 x 2
dy
dx
2

1  16 x 2  ddx y2  16 x dy
dx
 16 y

1  16 x  ddx y  16 x dy
2

dx
 16 y
2

diff. w.r.t x
3 2 2

1  16 x 2  ddxy3   32 x  ddx y2  16 x ddx y2  16 dy


dx
 16
dy
dx
3 2

1  16 x 2  ddxy3  48 x ddx y2  32 dydx

f 0  1
f '0  4
f ''  0   16
f '''  0   128

64 3
f  x   1  4x  8x2  x  ...
3


 1
e 6
 esin 4x


  sin 1 4 x
6
 
sin     4 x
 6
1
x
8

sgfreepapers.com 18
 2 3
  1   1  64  1 
e  1  4     8         ...
6

 8  8 3  8
1 1 1
 1  
2 8 24
7

12

14 n
1
(a) Let P(n) denote the statement  (2r  1)
r 1
2
 n(2n  1)(2n  1) .
3
When n = 1,
1
LHS   (2r  1) 2  12  1
r 1

1 1
RHS  (1)(2  1)(2  1)  (1)(1)(3)  1
3 3
LHS  RHS
Hence P(1) is true.
Assume P(k) is true for some k    .
k
1
i.e.  (2r  1) 2  k (2k  1)(2k  1) .
r 1 3
We need to show that P(k+1) is true.
k 1
1
i.e.  (2r  1) 2  (k  1)(2k  1)(2k  3)
r 1 3
k 1

 (2r  1)
r 1
2

k
  (2r  1) 2  (2k  1) 2
r 1

1
 k (2k  1)(2k  1)  (2k  1) 2
3
1
 (2k  1)  k (2k  1)  3(2k  1) 
3
1
 (2k  1)  2k 2  5k  3
3
1
 (2k  1)  (k  1)(2k  3) 
3
1
 (k  1)(2k  1)(2k  3)
3
Hence P(k )  P(k  1) is true.
Since P(1) is true and P(k )  P(k  1) is true, by the principle of Mathematical induction,
P(n) is true n   

sgfreepapers.com 19
14 Method 1:
b (i) 30
 (2r  3)2
r 1

 52  7 2  92    632
 12  32  52    632   12  32
32
  (2r  1) 2  10
r 1

1
 (32)(64  1)(64  1)  10
3
1
 (32)(63)(65)  10
3
 43670

Method 2:
Let r  k  2
2r  3  2(k  2)  3  2k  1
When r  1, k  2  1  k  3
When r  30, k  2  30  k  32
30

 (2r  3)
r 1
32
  (2k  1)
k 3
32
  (2r  1) 2  12  32
r 1
32
  (2r  1) 2  10
r 1

1
 (32)(64  1)(64  1)  10
3
1
 (32)(63)(65)  10
3
 43670
14b n
1
(ii) To prove: r 2

r 1
 n(n  1)(2n  1)
6
n
1
Proof:  (2r  1) 2  n(2n  1)(2n  1)
r 1 3
n
1

r 1
(4r 2  4r  1)  n(2n  1)(2n  1)
3

sgfreepapers.com 20
n n n
1
4 r 2  4 r  1  n(2n  1)(2n  1)
r 1 r 1 r 1 3
n
1  1
4 r 2  4   n  n  1   n  n(2n  1)(2n  1)
r 1 2  3
n
1
4 r 2  n(2n  1)(2n  1)  2n(n  1)  n
r 1 3
n
1
4 r 2  n  (2n  1)(2n  1)  6(n  1)  3
r 1 3
n
1

r 1
r 2  n  (4n 2  1)  (6n  6)  3
12
n
1

r 1
r 2  n  4n 2  6n  2 
12
n
1

r 1
r 2  n  2n  2  2n  1
12
n
1

r 1
r 2  n  n  1 2n  1
6

15 Original amount = $M
(a) 2% interest charged = $0.02M
Late payment charge = $L
Total outstanding balance = $(1.02M + L)

No of Outstanding balance left unpaid after 27th of the


(b) months month
later
1 1.02M  L
2 1.022 M  1.02 L  L
3 1.023 M  1.022 L  1.02 L  L
4 1.024 M  1.023 L  1.022 L  1.02 L  L
. .
. .
. .
n 1.02n M  1.02n 1 L    1.022 L  1.02 L  L

Outstanding balance left unpaid n months later


= 1.02n M  1.02n 1 L    1.022 L  1.02 L  L
= 1.02n M  (1.02n 1 L    1.022 L  1.02 L  L)

This is a G.P. with first term a = L, common ratio


r = 1.02 and number of terms is n.

L(1.02n  1)
= 1.02n M 
1.02  1
L(1.02n  1)
= 1.02 M 
n

0.02

sgfreepapers.com 21
100 L(1.02n  1)
= 1.02n M 
2

= 1.02n M  50 L(1.02n  1)

(c) Putting 1.02n M  50 L(1.02n  1)  2010 .


Given M = 1000 and L = 55.
1.02n (1000)  50(55)(1.02n  1)  2010
1.02n (1000)  (2750)(1.02n )  2750  2010
1.02n (1000)  (2750)(1.02n )  4760
1.02n (3750)  4760
476
1.02n 
375
476
log(1.02n )  log( )
375
476
n log(1.02)  log( )
375
476
n  log( ) log(1.02)
375
n  12.04

Since n is a positive integer, n = 13, 14, 15, …

Hence n = 13.

sgfreepapers.com 22
Anderson Junior College
JC1 Promotional Examination 2013
H2 Mathematics (9740)

1 − x2
1. (i)* Find the expansion of in ascending powers of x, up to and including
4− x
the term in x2 . [3]
(ii)* State the set of values of x for which this expansion is valid. [1]
(iii)* Hence, by substituting a suitable value of x, find an approximation for 15
a
in the form , where a and b are integers to be determined. [3]
b

n
2. Evaluate ∑ ( 2−r + 2nr + n2 ) , giving your answer in terms of n. [4]
r =2

3. A curve C is defined by parametric equations


π
x = eθ cos θ , y = e−θ sin θ , ≤θ ≤ 0.for −
2
(i) Sketch the curve C , indicating the axial intercepts in exact form. [2]

(ii) Show that the area bounded by the curve C and the axes is given by
0

∫ −
π
2
(sin 2 θ − sin θ cos θ ) dθ .

Hence determine its exact value. [5]

1
4. A sequence un , n =0,1,2,3,…, is such that u0 = − and
2
1
un +1 = un + ln ( n + 1) − 2
for all n ≥ 0.
4n − 1
1
(i) Prove by mathematical induction that un = ln ( n !) + . [5]
2 ( 2n − 1)
N
 1 
(ii) Hence find ∑ ln ( n + 1) − 4n2 − 1 . [3]
n=0

(iii) Does the series found in (ii) converge? Give a reason for your answer. [1]
N  1 
(iv) Using the series found in (ii), evaluate ∑ ln ( n − 1) − 4 . [2]
n=2 ( n − 2 )2 − 1 

*: Not in topics tested for


SRJC 2014 Promo -1-

sgfreepapers.com 23
5. The curve with equation y = − −2 x is transformed by a translation of 2 units in
the positive x-direction, followed by a reflection in the y-axis.

(i) Find the equation of the resultant curve in the form y = f ( x) and the
coordinates of the points where this curve crosses the x- and y- axes. On a
single diagram, sketch the graph y = f ( x) and its inverse. [5]

(ii) Solve the equation f ( x) = f −1 ( x) , giving your answers in exact form. [3]
−1 x2
(iii) The function g is defined such that f g( x) = − 2 . Find g( x) . [2]
2

x(4 x − 1)
6. Without using a calculator, solve < 3x + 1. [3]
2x −1

Hence, find the solutions of the inequalities


x(4 x − 1)
(a) x − 5 < 3x + 1 < ,
2x −1
cos x (4 cos x + 1)
(b) > 3cos x − 1 for 0 ≤ x ≤ π ,
2 cos x + 1
leaving your answers in exact form. [6]

7. The diagram shows a sketch of the curve y = f ( x ) . The curve cuts the x-axis at
C ( −1, 0 ) , has stationary points at A ( −2, 5 ) and B (1, − 2 ) , and asymptotes x = 0 ,
x = 3 and y = 3 .
y
x=3
y = f(x)

y=3
• 0 x

On separate diagrams, sketch the graphs of


1
(i) y = , [3]
f ( x)
(ii) y = f ' ( x ) , [3]

*: Not in topics tested for


SRJC 2014 Promo -2-

sgfreepapers.com 24
showing, in each case, the asymptotes, the coordinates of the stationary points and
the points of intersection with the axes, whenever possible.
1
8. (a)* Find ∫x2
ln( x + 1) dx . [3]
2
(b)* The diagram shows a shaded region R bounded by the curve ( y − 2) = x +1
and the line y + 2 x = 6.
y

R
x
0

Find the volume generated when R is rotated through 2π radians about the
x-axis, leaving your answer correct to 3 significant figures. [4]

9. The lines l1 and l2 have equations


1  −1 
x −1 y − 2    
= , z =1 and r = 2 + λ  1  , λ ∈
3 a 0  
  1
respectively, where a is a constant.

(i) Given that l1 and l2 intersect at the point N, find N and the value of a. [3]

(ii) Show that the position vector of F, the foot of the perpendicular from the
4 5 1
point P ( 2, 1, 1) to the line l2 is i + j − k . [3]
3 3 3
(iii) Find the position vector of the point P ' , the reflection of P in the line l2 . [2]

(iv) The point Q has coordinates (1, 2, 0). Find the ratio of the area of triangle
NQP to the area of triangle FQP ' . [3]

x2
10. A curve C has equation y = , x ≠ −3λ and λ is a positive constant.
x + 3λ
(i) Find the coordinates of the stationary points of C. [3]
(ii) Draw a sketch of C, labeling clearly, in terms of λ , the asymptotes and the
stationary points. [2]
(iii) Use the graph in (ii), find the number of roots of the equation
x 4 − 2λ x − 6λ 2 = 0 . [3]

*: Not in topics tested for


SRJC 2014 Promo -3-

sgfreepapers.com 25
x2
The function f is defined by f : x  , x ≤ − 6λ .
x + 3λ
(iv) Show that f 2 exists and find the value of f 2 (−6λ ) . [4]
11. Two solid cylinders of the same height are placed at a corner of the wall such that
the vertices A, B, C and D touch the wall. At point E, the two cylinders touch each
other. The diagram below shows a cross section of the cylinders.

R
A •

E
B •r

C D
Let r be the radius of the small cylinder and R be the radius of the big cylinder.
2
(i) Show that R = ( 2 +1 r ) [2]
16π
(ii) Given that the volume of the small cylinder is cm3 , find the exact value
2 +1
of the radius r such that the surface area of the big cylinder is a minimum. [5]

12. Mary has a monthly income of $4000. She is considering applying for a car loan
of $40,000 for 6 years which charges an interest rate of 3.00% per annum,
compounded monthly. Interest is chargeable immediately when the loan sum is
drawn out. The monthly repayment, $ m , is fixed throughout the loan tenure.

(i) Show that the calculated loan balance at the end of the nth loan month, after
the monthly repayment is made, is given by

 401 
n  401 n 
40000   − 400m   − 1 . [3]
 400   400  

(ii) By legislation, banks can approve a car loan only if the monthly repayment
does not exceed 15% of an applicant’s monthly income. Prove that Mary will
not be able to apply for the car loan. [3]

(iii) If the interest rate for all car loans by the banks is compounded monthly, find
the range of interest rates chargeable which will enable Mary to apply for the
*: Not in topics tested for
SRJC 2014 Promo -4-

sgfreepapers.com 26
car loan successfully. Give your answer in the form r % per annum, correct to
1 decimal place. [3]

END OF PAPER

*: Not in topics tested for


SRJC 2014 Promo -5-

sgfreepapers.com 27
Anderson Junior College
JC1 Promotional Examination 2013 _H2 Mathematics (9740)_Solutions
[Questions marked with * are not in the topics tested for 2014 SRJC Promo]
Qn Solutions
1 − x2
= (1 − x 2 ) ( 4 − x )
−1/ 2
1(i)
4− x
−1/2
1  x
= (1 − x 2 )  1 − 
2  4
  1  3  
  − −  2 
2  2   x 
= (1 − x 2 )  1 + + 
1 x
 −  + ... 
2  8 2!  4 
 
 
1  x 3x 2

= (1 − x 2 )  1 + + + ... 
2  8 128 
1 1 125 2
= + x− x + ...
2 16 256

(ii) Expansion is valid for { x : − 4 < x < 4, x ∈ } .

1
(iii) By letting x = ,
4
2
1
1−  
 4  ≈ 1 + 1  1  − 125  1 
   
1 2 16  4  256  16 
4−
4
15
16 ≈ 1987
15 4096
4
1987
15 ≈ where a = 1987 and b = 512
512
or
7680
15 ≈ where a = 7680 and b = 1987
1987

n
2 ∑(2
r =2
−r
+ 2nr + n 2 )
n n n
= ∑ ( 2 ) + ∑ 2nr + ∑ n
r =2
−r

r =2 r =2
2

=
( ) 1 2
2 (1 − ( 1 n −1
2) ) + 2n ⋅ n −1 ( 2 + n ) + n ( n −1)
2
1
1− 2 2
= 1
2 (1 − ( 1 n −1
2 ) ) + n(n −1) [(2 + n) + n]
n
= 12 − ( 12 ) + 2n(n 2 − 1)

sgfreepapers.com 28
π
3(i)* x = eθ cos θ , y = e −θ sin θ , for − ≤θ ≤ 0 y
2
O 1 x
When θ = 0 , x-intercept: (1, 0)
π π
When θ = − , y-intercept: (0, − e 2 )
2

(ii)* Area = −
∫ 0
y dx
0
=− ∫π (e
−θ
{
sin θ ) eθ [cos θ − sin θ ]} dθ

2
0

∫π (sin
2
= θ − sin θ cos θ ) dθ

2
0
 1 − cos 2θ  sin 2θ 
= ∫π  2
−
 2 
 dθ

2
0
θ 1 1 
=  − sin 2θ + cos 2θ 
 2 4 4  −π2
π +2
=
4

1
4(i) Let Pn be the statement un = ln ( n !) + for n ≥ 0, n ∈ .
2 ( 2n − 1)
1
When n = 0 , LHS = u0 = −
2
1 1
RHS = ln ( 0!) + =− Since LHS = RHS, ∴ P0 is true.
2( −1) 2

1
Assume that Pk is true for some k ≥ 0, k ∈ , i.e. uk = ln ( k !) + ,
2 ( 2k − 1)
need to prove that Pk +1 is true, i.e., to show that
1 1
uk +1 = ln ( k + 1) ! + = ln ( k + 1) ! + .
2(2( k + 1) − 1) 2(2k + 1)

LHS of Pk+1
= uk +1
1
= uk + ln ( k + 1) −
4k 2 − 1
1 1
= ln ( k !) + + ln ( k + 1) − 2
2 ( 2k − 1) 4k − 1
1 1
= ln ( k + 1) k ! + −
2 ( 2k − 1) ( 2k − 1)( 2k + 1)
2k − 1
= ln ( k + 1) ! +
2 ( 2k − 1)( 2k + 1)
1
= ln ( k + 1) ! + = RHS of Pk +1
2 ( 2k + 1)

sgfreepapers.com 29
Since P0 is true and Pk is true ⇒ Pk +1 is true,
∴ by the principle of mathematical induction, Pn is true for all non-negative integers n.

1
(ii) un +1 = un + ln ( n + 1) − 2
4n − 1
1
⇒ un +1 − un = ln ( n + 1) − 2
.
4n − 1
N
 1  N
∴ ∑  ln ( n + 1) − 2  = ∑ ( un +1 − un )
n =0  4n − 1  n = 0
= ( u1 − u0
+ u2 − u1
+ u3 − u2
+ u4 − u3
+ 
+ uN +1 − uN )
= uN +1 − u0
1  1
= ln ( N + 1) ! + −− 
2 ( 2 ( N + 1) − 1)  2 
1 1
= ln ( N + 1) ! + +
2 ( 2 N + 1) 2

(iii) N
 1 
∴ ∑  ln ( n + 1) − 2  = ln ( N + 1) ! + 1 1
+ .
n =0  4n − 1  2 ( 2 N + 1) 2
The series is divergent since ln ( N + 1) ! → ∞ when N → ∞ .

(iv) Replace n with n + 2,


N  
1
∑ ln ( n − 1) −
n=2 
2 
4 ( n − 2 ) − 1 

N  1 
= ∑ ln ( n + 2 − 1) − 4 2 
n+ 2=2
 ( n + 2 − 2 ) − 1

N −2
1 
= ∑  ln ( n + 1) − 2 
n =0  4n − 1 
1 1
= ln ( N − 2 + 1) ! + +
2 ( 2( N − 2) + 1) 2
1 1
= ln ( N − 1) ! + +
2 ( 2 N − 3) 2

sgfreepapers.com 30
5(i) y = − −2 x x → x −2
→ y = − 4 − 2 x → x → −x y = − 4 + 2x
Coordinates of points: ( −2,0 ) , ( 0, −2 ) .
y

y = f −1 ( x)

( −2,0) O x

( 0, −2)
y = f ( x)

(ii) From the diagram, the graphs intersect at x = −2, 0 , and where
f ( x) = x ⇒ − 4 + 2 x = x ⇒ x 2 − 2 x − 4 = 0
2 ± 4 + 16
x= = 1± 5
2
Since the graphs intersect where x ≤ 0, solutions for f ( x) = f −1 ( x) are x = −2, 1 − 5, 0 .

x2
(iii) f −1g( x) = −2
2
 x2 
⇒ f ( f −1g( x) ) = f  − 2 
2 
⇒ g( x) = − 4 + x2 − 4 = − x 2 = − x

x(4 x − 1)
6 < 3x + 1
2x −1
4 x 2 − x − (2 x − 1)(3x + 1)
<0
2x −1
−2 x 2 + 1
<0
2x −1
x 2 − 12
>0
2x −1
( x + 12 )( x − 12 )
>0
2x −1
1 1 1
∴ − <x< or x > .
2 2 2
x(4 x − 1) 1 1 1
(a) Solution of 3x + 1 < is x < − or <x< .
2x −1 2 2 2
Also, x − 5 < 3 x + 1 ⇒ x > −3.

1 1 1
Taking the intersection of the solutions, −3 < x < − or <x< .
2 2 2

sgfreepapers.com 31
(b) Replace x with − cos x,
− cos x(−4 cos x − 1)
< − 3cos x + 1
−2 cos x − 1
cos x(4 cos x + 1)
⇒ > 3cos x − 1.
2 cos x + 1
1 1 1
∴ − < − cos x < or − cos x >
2 2 2
1 1 1
− < cos x < or cos x < − .
2 2 2
y

x
O

–1

π 2π 3π
∴ <x< or < x ≤ π.
4 3 4

7(i) 1 y
y=
f ( x)

y = 1/3

O x

7(ii)
y
y = f ′( x )

x=3
y =0
O 1 x

sgfreepapers.com 32
1 1 1 1
8(a)*
∫ x 2
ln( x + 1) dx = − ln( x + 1) + ∫ ⋅
x x x +1
dx

1 1 1 
= − ln( x + 1) + ∫  −  dx
x  x x +1
1
= − ln( x + 1) + ln x − ln( x + 1) + c
x
1 1
Alternative method for ∫ ⋅
x x +1
dx

1 x
=∫ dx = ln +c
1 1 x +1
( x + )2 − ( )2
2 2
2
8(b)* Points of intersection of ( y − 2) = x +1 and y + 2x = 6
2 5
(4 − 2x) = x + 1 ⇒ 4 x 2 − 17 x + 15 = 0 ⇒ x = 3 or x = or GC.
4
2
Also, ( y − 2) = x +1 ⇒ y = 2 ± x +1
5 2 2
3 3
Volume generated = ∫ 4
−1
( )
π 2 + x + 1 dx + ∫ 5 π ( 6 − 2 x ) dx − ∫ π 2 − x + 1 dx
4
2
−1
( )
≈ 78.57254 = 78.6 (3 s.f.)

1 3
9(i) l1 : x − 1 = y − 2 , z = 1 ⇒    
l1 : r =  2  + µ  a  , µ ∈ 
3 a 1  
  0
1  −1 
   
l2 : r =  2  + λ  1  , λ ∈ 
0  
  1
If l1 intersects with l2 ,
 1 + 3µ   1 − λ 
 2 + aµ  =  2 + λ 
   
 1   λ 
   
1 + 3µ = 1 − λ ---------- (1)
2 + aµ = 2 + λ ---------- (2)
1 =λ ---------- (3)
1
Solving for (1) and (3): λ = 1 and µ = −
3
Therefore, point N is (0, 3, 1).

Substitute the values of λ and µ into (2):


 1
2 + a −  = 2 +1
 3
a = −3.

(ii) Let F be the foot of the perpendicular from point P( 2,1,1 ) to the line l2.
1− λ 
Since F lies on l2, OF =  2 + λ  for some λ ∈ 
→

 
 λ 
 

sgfreepapers.com 33
 −1 − λ 

→  → →
 
PF = OF − OP =  1 + λ 
 −1 + λ 
 
 −1 − λ   −1
PF ⊥ l2 ⇒  1 + λ  i 1  = 0
 −1 + λ   1 
   
⇒ 1 + λ + 1 + λ −1 + λ = 0
1
⇒λ =−
3
  1 
1 −  − 3  
  
 43 
 
=  5 3  = i + j − k. (shown)


Thus OF = 2 − 1 4 5 1
 3  3 3 3
   − 1 3 
 −1 
 3 

(iii) Let P ' be the point of reflection of P about the line l2.

→ 


→ 
→ OP '+ OP


PF = FP ' ⇒ By the mid-point theorem, OF = .
2

→ 
→ 

⇒ OP ' = 2 OF − OP
 43   2  23 
     
= 2  53  −  1  =  73 
− 1  1 − 5 
 3    3

(iv) • P(2,1,1)
Q(1,2, 0) F
• N • l2

Note that Q lies on l2.
1 
PF × NQ  
Area of ∆NQP  2 NQ 
=  =  since PF = FP ' .
Area of ∆FQP '  1 FP '× QF   FQ 
2 

1 0  1 


     
NQ =  2  −  3  =  −1 ⇒ NQ = 3
     
 0   1   −1
 1   4 3   − 13   −1
→
   5   1  1  
FQ =  2  −  3  =  3  = 3  1  ⇒ FQ = 1
3
3
   1   1   
0 − 3   3  1
Area of ∆NQP  3 
=  = 3.
Area of ∆FQP '  13 3 
Therefore, the ratio is 3:1.

sgfreepapers.com 34
x2 dy 2 x( x + 3λ ) − x 2 x 2 + 6λ x
10(i) y= ⇒ = =
x + 3λ dx ( x + 3λ )2 ( x + 3λ )2
dy
At stationary point, = 0 ⇒ x = 0 or x = - 6λ. Stationary points: (0, 0), (–6λ, –12λ).
dx

x2 9λ 2
(ii) y= ⇒ y = x − 3λ +
x + 3λ x + 3λ
Asymptotes : x = –3λ, y = x – 3λ
y

y = x – 3λ

–3λ (0,0) 3λ x

–3λ

(–6λ, –12λ)

4 2 x2 2λ
(ii) x − 2 λ x − 6λ = 0 ⇒ = 2
x + 3λ x

By sketching the graph of y = 2 on the diagram, there are 2 points of intersections, hence
x
there are 2 roots to the equation.

(iii) Rf = (–∞,–12λ], and Df = (–∞,–6λ].

Since Rf ⊆ Df, hence f2 exists.

 36λ 2  144λ 2
f 2 (−6λ ) = f   = f ( −12λ ) = = −16λ .
 −6λ + 3λ  −12λ + 3λ

2 2
11(i) ( R − r )2 + ( R − r ) = ( R + r )
2
(R − r) = 1
2
(R + r) 2
R−r 1
⇒ =
R+r 2

R ( ) (
2 −1 = )
2 +1 r

2 +1 2 +1
R= × r
2 −1 2 +1
2
( )
2 +1 r 2
=
2 −1
⇒ R= ( 2 +1 r )
16π
(ii) Volume of small cylinder = V = π r 2 h = .
2 +1

sgfreepapers.com 35
16
h=
r 2
( 2 +1 )
Surface area of big cylinder = A = 2π Rh + 2π R 2 .

2 4
A = 2π ( 2 + 1 rh + 2π) ( 2 +1 r2 )
2
 16  4
= 2π ( 2 +1 r  2 )  + 2π ( )
2 + 1 r2
r

2 +1 ( ) 

32π ( 2 +1 ) + 2π 4
=
r
( 2 + 1 r2 )

dA 4 32π 2 + 1 ( )
dr
= 4π 2 + 1 r − ( r2
)
dA
Let = 0,
dr
4 32π ( 2 +1 )
then 4π ( 2 +1 r = ) r 2

⇒ r3 =
32π ( 2 +1 )
4
4π ( 2 +1 )
8
= 3
( 2 +1 )
2
⇒ r=
2 +1
or 2 ( 2 −1 )
d2 A 4 64π ( 2 +1 )
dr 2
= 4π ( 2 +1 + ) r 3

2
d A
When r = 2 ( 2 −1 ,
dr 2
)
>0.

Hence, r = 2 ( )
2 − 1 gives the minimum surface area of the big cylinder.

3 1
12(i) Monthly interest chargeable = % = %.
12 4
Let monthly repayment amount = $ m .
Loan Loan balance at beginning of loan month Loan Balance at end of loan month
Mth (after monthly repayment)
 401   401 
1 40000   40000  −m
 400   400 
2 2
2  401   401   401   401 
40000   −  m 40000   −  m − m
 400   400   400   400 

  
n n −1 n− 2 n n −1 n−2
 401   401   401   401   401   401   401   401 
n 40000   −   m−  m− … − m 40000   −  m−  m− … − m − m
 400   400   400   400   400   400   400   400 

sgfreepapers.com 36
Loan balance at the end of nth loan month after monthly repayment
n n −1 n −2
 401   401   401   401 
= 40000   −   m −   m− … −  m − m
 400   400   400   400 
  401  n 
n   − 1
= 40000  401  − m   400  
 400   401 
 400 − 1 
 

 401 
n  401 n 
= 40000   − 400m   − 1
 400   400  

 401 
72  401 72 
(ii) Let 40000   − 400m   − 1 = 0
 400   400  
⇒ m = 607.75
15% of $4000 = $600.
Since m = 607.75 > 600 , Mary is not able to take up the car loan.

 72 
(iii) Let 40000a 72 − 600  a − 1  ≤ 0.
 a −1 
 x 72 − 1 
From the GC, using the graph of y = 40000 x 72 − 600  ,
 x −1 
1 < a ≤ 1.0021378.

12 × (1.0021378 – 1) × 100% = 2.56536%


∴ 0% < r % ≤ 2.5% (to 1 decimal place)

sgfreepapers.com 37
CAT
THOLIC JUN
NIOR COLLE
EGE
Gen
neral Certifica
ate of Educa
ation Advanc
ced Level
High
her 2
JC1 Promotiona
al Examinatio
on

MATHEM
M MATICS
S 97
740/01
Pa
aper 1 04 Octo
ober 2013
3 hours
Ad
dditional Ma
aterials: List of Formu
ulae (MF15))

READ THES
SE INSTRUC
CTIONS FIRST

Write
W your naame and claass on all th
he work you u hand in.
Write
W in dark blue or black pen on both
b sides oof the paperr.
Yo
ou may use e a soft pencil for any diagrams
d orr graphs.
Do
o not use sttaples, paper clips, hig
ghlighters, g
glue or corre
ection fluid.

Annswer all th
he questionss.
Give non-exa act numericcal answers s correct too 3 significa
ant figures, or 1 decim mal place inn the case of
anngles in deg
grees, unlesss a differen
nt level of a
accuracy is specified
s in the questioon.
Yoou are expe e a graphic calculator.
ected to use
Unsupported d answers from a gra aphic calcu ulator are allowed
a unless a queestion spec cifically stattes
ottherwise.
Where
W unsup pported ansswers from a graphic ccalculator area not allow wed in a quuestion, you
u are requirred
to
o present thee mathematical steps using
u mathe ematical nootations and not calculaator comma ands.
Yoou are remiinded of the
e need for clear
c presenntation in yo
our answers s.

Att the end o


of the exam
mination, arrrange you r answers in NUMERICAL ORDE ER.
Pllace this co
over sheet in front an
nd fasten a
all your worrk securely
y together.

Th
he number of marks is given in brackets [ ] a
at the end of
o each ques
stion or partt question.

Na
ame: _____
_________
__________
______ Classs: _______
__________
_

Question 1 2 3 4 5 6 7 8 9 10 111 12 13 Tottal

Marks

Total 3 4 5 6 6 6 6 8 8 9 11 13 15
10
00

This document consists of 5 printed pag


ges.

Catho
olic Junior Co
ollege

sgfreepapers.com 38
2

1. Sketch the curve ( y  5) 2  ( x  3) 2  4 , indicating clearly the coordinates of the


turning point(s) and equations of the asymptotes. [3]

1
2. Expand in ascending powers of x, up to and including the term in x2.
3
2x 1
State the range of values of x for which this expansion is valid. [4]

3. The graph of y  f ( x ) , where f ( x ) is a cubic polynomial, passes through the points


1
(1, 6), (2, 15) and has two stationary points at x  and x  2 . Find the equation
3
of the curve and hence, find its x-intercept. [5]

dy
4. (a) Given that tan √ , find . [2]
dx
dy
(b) Given that √ , where 0, 0, find . [4]
dx

5. The parametric equations of a curve are


7
x  t 3 , y  , t  0.
t
(i) Find the equation of the tangent to the curve at the point where t = k,
simplifying your answer. [3]
(ii) Hence find the coordinates of the points X and Y where this tangent
meets the x- and y-axes respectively. [2]
(iii) Hence or otherwise, find the area of the triangle OXY, where O is the
origin. [1]

n
1 3 1 1
6. Prove by the method of differences that r
r 2
2
  
 1 4 2n 2(n  1)
. [4]

n
1
Hence, or otherwise, give a reason why the series r
r 2
2
1
is convergent and state

the sum to infinity. [2]

7. Prove by the method of mathematical induction that


n
sin 2n
 cos[(2r  1) ] 
r 1 2sin 
for all positive integers n. [6]

9740/01/Promo/2013

sgfreepapers.com 39
3

x6 1
8. (a) (i) Without using a calculator, solve the inequality  . [3]
x  3x  4 4  x
2

(ii) Hence, deduce the range of values of x that satisfies


x 6 1

x 3x  4
2
4 x
. [2]
x
(b) Solve the inequality ln( x  6)   . [3]
3

9. Charis Insurance provides an investment linked savings insurance plan with two
options of premium payment, monthly and yearly.
For the monthly premium plan, premiums of $500 are collected on the first day of
each month and an interest of 0.5% per month is earned on the last day of each
month, such that there is $502.50 in the account at the end of the first month and
$1007.51 in the account at the end of the second month.

(i) Show that the total amount in the monthly premium account at the end of n
complete months can be expressed as M (1.005n  1) , where M is an integer
to be found. [4]

For the yearly premium plan, premiums of $6000 are made on the first day of each
year and an interest of 6% per year is earned on the last day of each year.

(ii) Given that the total amount in the yearly premium account at the end of k
complete years is $ 106000 1.06 k  1  , find the number of complete years
it will take for the total amount to first exceed $120 000. [2]

A young couple who just had their first child would like to take up a savings plan for
a period of 20 years to prepare for their child’s university education. A friend of the
couple stated that “0.5% a month is the same as 6% a year since 12 × 0.5 = 6”. With
reference to evidence obtained from the expressions from (i) and (ii), comment on the
validity of the statement. [2]

10. (i) Given that f e , where is a constant, find f 0 , f ′ 0 , f ′′ 0 .


Hence write down the first three terms in the Maclaurin series for f .
Give the coefficients in terms of e and . [5]
(ii) Find the value of such that √2 sin cos sin for all . [2]
(iii) By considering the series in part (i), show that

sin x  e  x 2  x  , where
2 sin( x  )
e 4 is a small angle. [2]

9740/01/Promo/2013

sgfreepapers.com 40
4

11. (a) The diagram below shows the graph of y  f ( x ) . It passes through the
origin O and P (3, 0), and has asymptotes x = 2, y  2 .and y  2 .

On separate diagrams, sketch the graph of

(i) y  f '( x) , [3]

1
(ii) y , [3]
f  x
indicating clearly any asymptote(s) and axial intercept(s).

1
(b) The graph of y  is transformed by a reflection in the y-axis, followed
2x  3
by a translation of 1 unit in the negative x-direction, followed by a stretch with
scale factor 2 parallel to the x-axis.

(i) Find the equation of the new graph in the form y  f  x  . [3]
(ii) Hence, or otherwise, sketch the new graph with any axial intercept(s)
and asymptote(s) indicated clearly. [2]

9740/01/Promo/2013

sgfreepapers.com 41
5

12. Functions f and g are defined by

1
f:x  4  2x2 , x  , 0  x  16
g : x  3x  1 , x

(i) State the range of f. [1]


1 1
(ii) With the aid of a diagram, show that f exists and define f in a similar
form. [4]
1 1
(iii) On the same diagram as in part (ii), sketch the graphs of f and f f ,
indicating their endpoints. [3]
(iv) Explain why the x-coordinates of the point(s) of intersection between the
graphs in part (iii) satisfies the equation x 2  2 x  4  0 . [1]
(v) State whether the composite function fg exists, justifying your answer. [2]
(vi) Find the largest possible domain of g in the form [m, n], m, n   , for which
the composite function fg exists. [2]

13. (a) Relative to the origin O, two points A and B have position vectors given
by a = 4i + 2j + 3k and b = 4i – 2j + 7k respectively. The point P divides
AB in the ratio 3 : 1.

(i) Find the coordinates of P. [2]


(ii) The vector c is a unit vector in the direction of OP .
Write c as a column vector, and give the geometrical meaning
of a  c . [2]
(iii) By using vector cross product, find the exact area of triangle OAP. [3]
x3 z 1
(b) The line l has equation  y3 and the plane p has equation
3 2
3x  y  2 z  0 .

(i) Show that l is perpendicular to p. [2]


(ii) Find the coordinates of the point of intersection of l and p. [3]
(iii) Show that the point C with coordinates  9,1, 7  lies on l.
Find the coordinates of the point C’ which is the mirror image of
C in p. [3]

 End of Paper 

9740/01/Promo/2013

sgfreepapers.com 42
CATHOLIC JUNIOR COLLEGE
H2 MATHEMATICS
JC1 PROMOTIONAL EXAMINATION 2013

Solutions
1 ( y  5) 2  ( x  3) 2  4
( x  3) 2 ( y  5) 2
  1
22 22
Asymptotes:
( y  5) 2  ( x  3) 2
y  5   ( x  3)
y  x  8 or y   x  2
y

y=–x+2 (‐3, 7) y=x+8

(‐3, 3)
x

2 1 1 1
  2 x  1 3  1 1  2x  3
 
3
2x 1
  1  1  
  1      1 
  1      2 x   
3  3 
 2 x   .... 
2

  3 2 
 
 
2 8
 (1  x  x2 )
3 9
1 1
Validity:   x 
2 2
3 Let y  Ax  Bx  Cx  D
3 2

dy
  3 Ax 2  2 Bx  C
dx
A B C  D  6
8 A  4B  2C  D  15
A  2 B  3C  0
12 A  4 B  C  0
Solving,
A  2, B  5, C   4, D  3
y  2 x3  5 x 2  4 x  3

sgfreepapers.com 43
CATHOLIC JUNIOR COLLEGE
H2 MATHEMATICS
JC1 PROMOTIONAL EXAMINATION 2013
When y = 0, x = – 3.26 (3sf)
x-intercept = (– 3.26, 0)
4 (a)
1  12
d
dx

tan 1 x   1

1  ( x )2 2
x

1

2 x (1  x)
(b)

Taking logarithm on both sides,

Differentiating both sides,

5 (i)
d y d y dx  7  7
       3t    4
2

dx dt dt  t 2  3t
7 7
y   4  x  k3 
k 3k
7 28
y  4 x
3k 3k
(ii)
7 7
y   4  x  k3 
k 3k
7 28
y  4 x
3k 3k
y  0, x  4k 3  X is  4k 3 , 0 
28  28 
x  0, y  Y is  0, 
3k  3k 
(iii)
1
Area of OXY   OX  OY 
2
1  28 
  4k 3   
2  3k 
56
 k 2 units 2
3
6 n
1 n
1

r 2 r  1
2

r  2 ( r  1)( r  1)

sgfreepapers.com 44
CATHOLIC JUNIOR COLLEGE
H2 MATHEMATICS
JC1 PROMOTIONAL EXAMINATION 2013
1 n  1 1 
    
2 r 2  r  1 r  1 
1 1 1 
 [   
2 1 3 
1 1
  
2 4
1 1
  
3 5

 1 1 
  
 n  3 n 1
 1 1
  
n2 n
 1 1 
  
 n 1 n 1
1 1 1 1 
 1    
2  2 n (n  1) 
3 1 1
  
4 2n 2(n  1)

n
1 3 1 1
r
r 2
2
  
 1 4 2 n 2( n  1)
.

1 1 3 1 1 3 
1 3
n   ,.
2n
 0,
2(n  1)
 0 , so  
4 2n 2(n  1)
 so
4
r
r 2
2

1 4
7 n
sin 2n
Let Pn be the statement  cos[(2r  1) ] 
r 1 2sin 
for n  Z  , n ≥ 1

When n = 1, L.H.S. = cos 


sin 2 2sin  cos 
R.H.S. =  cos  =L.H.S.
2sin  2sin 
k
sin 2k
Assume Pk is true, i.e.  cos[(2r  1) ]  for some k  Z  , k ≥ 1.
r 1 2sin 
Required to prove Pk+1 is true, i.e.
k 1
sin[2(k  1) ]
 cos[(2r  1) ] 
r 1 2sin 
k
L.H.S. =  cos[(2r  1) ]  u
r 1
k 1

sin 2k
=  cos[(2k  1) ]
2sin 
sin 2k  2 cos[(2k  1) ]sin 
=
2sin 

sgfreepapers.com 45
CATHOLIC JUNIOR COLLEGE
H2 MATHEMATICS
JC1 PROMOTIONAL EXAMINATION 2013
sin 2k  sin[2(k  1) ]  sin 2k
=
2sin 
sin[2(k  1) ]
= = R.H.S.
2sin 
Pk is true  Pk+1 is true.
Hence, by Mathematical Induction, Pn is true for all n  Z  , n ≥ 1.
8 (a)(i)
x6 1

x  3x  4 4  x
2

x6 1
 0
( x  1)( x  4) 4  x
x6 1
 0
( x  1)( x  4) x  4
x  6  ( x  1)
0
( x  1)( x  4)
2x  7
0
( x  1)( x  4)
Using test-point method,
– + – +
-3.5 -1 4
 x  3.5 or  1  x  4

(ii)
x 6 1

2
x 3x  4 4 x
Replace x by x
 x  3.5 or 1  x  4
(no real solution) x 4
-4 x  4

(b)
x
Draw the graphs of y  ln( x  6 ) and y   .
3

sgfreepapers.com 46
CATHOLIC JUNIOR COLLEGE
H2 MATHEMATICS
JC1 PROMOTIONAL EXAMINATION 2013
y
x 6

y  ln( x  6)
x
– 3.1461

x
y
3

Ans:  6  x  3.15
x
Alternative solution: Draw the graph of y  ln( x  6)  .
3
y
x 6

x
y  ln( x  6) 
3
x
– 3.1461

Ans:  6  x  3.15

9 (i)
Total amount after 1 month = 1.005(500)
Total amount after 2 month = 1.005 (500)  1.005(500)
2

Total amount after 3 month


= 1.005 (500)  1.005 (500)  1.005(500)
3 2

Total amount after n months  1.005 (500)  1.005 (500)    1.005(500)


n n1

1.005(500) 1.005n  1

1.005  1
 100500 1.005n  1
M = 100500

(ii)
106000 1.06k  1  120000
Solving, k  12.99
 k  13 complete years.

sgfreepapers.com 47
CATHOLIC JUNIOR COLLEGE
H2 MATHEMATICS
JC1 PROMOTIONAL EXAMINATION 2013

From (i) and (ii), the final amount after 20 years is


100500 1.005240  1  $232175.55 for monthly account
106000 1.0620  1  $233956.36 for yearly account
Hence the statement is invalid as the final total amount differs quite significantly
10 (i)
We are given that .
Differentiating,

Differentiating,

So we have

Hence,

(ii)
Since

we have .
(iii)
Since is a small angle,

then

11 (a)(i)
y  f '( x )

sgfreepapers.com 48
CATHOLIC JUNIOR COLLEGE
H2 MATHEMATICS
JC1 PROMOTIONAL EXAMINATION 2013

(a)(ii)
1
y
f  x

(b)(i)
1
y
2x  3

1 1
y 
2( x)  3 2 x  3

1 1
y 
2( x  1)  3 2 x  1

1 1
y 
1  x 1
2  x   1
2 
(b)(ii)

sgfreepapers.com 49
CATHOLIC JUNIOR COLLEGE
H2 MATHEMATICS
JC1 PROMOTIONAL EXAMINATION 2013

12 (i)
As f is an increasing function,
f(0) = (4) ½ = 2
f(16) = (36) ½ = 6
Range of f, Rf = [2,6]
(ii)
y

y  f  x 16, 6 
2

O x

f is a 1-1 function as the line y = k, 2 k 6 intersects the graph of f exactly once.


(OR: f is a 1-1 function as any line y = k intersects the graph of f at most once.)
Hence f -1 exists.

Let y=f(x) = (4+2x)1/2


y2 = 4+2x
x = (y2 – 4)

f -1(x) = (x2 – 4)]


D f -1 = Rf = [2,6]

Hence f -1 : x (x2 – 4), 2  x  6

sgfreepapers.com 50
CATHOLIC JUNIOR COLLEGE
H2 MATHEMATICS
JC1 PROMOTIONAL EXAMINATION 2013

(iii)

 6,16  16,16 
y  f -1f  x 
yf -1
 x

(16, 6)

2
x

O 2

(iv)
By considering f(x) = x, (4 + 2x)1/2 = x
x2 – 2x – 4 = 0
The x-coordinates of the points of intersection satisfy the
equation x2 – 2x – 4 = 0.

(v)
Rg =
Df = [0, 16]
Rg Df
=> fg does not exist.

(vi)
Consider Rg = Df
3x+1 = 0 => x = -1/3
3x+1 = 16 => x = 5
Hence [- , 5] is the largest possible domain of g for fg to exist.
13 (a)(i)
OA  3OB
OP 
4
 4   4  
1    
  2   3  2  
4   
 3   7  
 4 
 
   1
 6 
 
(a)(ii)

sgfreepapers.com 51
CATHOLIC JUNIOR COLLEGE
H2 MATHEMATICS
JC1 PROMOTIONAL EXAMINATION 2013
4 4
1   1  
c=   1    1
4 2  (1) 2  6 2  6  53  
  6
Geometrically, a  c is the length of projection of the vector a on OP or c.
(a)(iii)
 4   4   15 
     
a  p   2     1    12 
 3   6    12 
     
Area of triangle OAP
1
 ap
2
1
 15 2  ( 12) 2  ( 12) 2
2
1
 513
2

(b)(i)
 3  3 
   
Line l : r   3     1  ,   
1  2 
   
3
 
Plane p : r    1  0
2
 
  3 3
   
Since  1     1 , the normal of the plane p is parallel to the line l, the line l is perpendicular to
  2 2
   
p.

(b)(ii)
 3  3   3 
   
When l intersects p,   3       1  0
 1  2   2 
   
9  9  3    2  4  0
 1
Coordinates of point of intersection = ( 0, -2, -1)

(b)(iii)
  9   3  3 
   
Suppose C with coordinates  9,1, 7  lies on l,  1     3   
  7   1  2 
   

sgfreepapers.com 52
CATHOLIC JUNIOR COLLEGE
H2 MATHEMATICS
JC1 PROMOTIONAL EXAMINATION 2013
 9  3  3
4
Since C satisfies the parametric equations of l with   4 , therefore C lies on l.
We note that C lies on l, l is perpendicular to p and l meets p at (0, -2, -1),
By Ratio Theorem,
 9 
  
1  OC '
 0   
   7 
 2   2
 1 
 
 0   9   9 
      
OC '  2  2    1    5 
 1   7   5 
     

sgfreepapers.com 53
1

HWA CHONG INSTITUTION


JC1 Promotional Examinations
Higher 2

MATHEMATICS 9740
7 October 2013
3 hours

Additional Materials: Answer Paper


List of Formulae (MF15)

READ THESE INSTRUCTIONS FIRST

Write your name and CT group on all the work you hand in.
Write in dark blue or black pen on both sides of the paper.
You may use a soft pencil for any diagrams or graphs.
Do not use staples, paper clips, highlighters, glue or correction fluid.

Answer all the questions.


Give non-exact numerical answers correct to 3 significant figures, or 1 decimal place in
the case of angles in degrees, unless a different level of accuracy is specified in the
question.
You are expected to use a graphic calculator.
Unsupported answers from a graphic calculator are allowed unless a question
specifically states otherwise.
Where unsupported answers from a graphic calculator are not allowed in a question, you
are required to present the mathematical steps using mathematical notations and not
calculator commands.
You are reminded of the need for clear presentation in your answers.

At the end of the examination, fasten all your work securely together.
The number of marks is given in brackets [ ] at the end of each question or part question.

This document consists of 7 printed pages.

© Hwa Chong Institution 9740 / JC1 Promo 2013 [Turn Over

sgfreepapers.com 54
2

1 Sophia has a total saving of $90 million in three accounts A, B and C with $x
million, $y million and $z million respectively. She transfers funds among the
accounts based on the table below.

Percentage of Fund
transferred from To Account A To Account B To Account C
initial amount in
Account A − 37.5% 12.5%
Account B 5% − 5%
Account C 10% 20% −

For instance, 37.5% and 12.5% of the initial amount in Account A are transferred
to Account B and Account C respectively.
As a result of the funds transfer, the amount in Account A decreases by $16 million
and the amount in Account B increases by $19 million.
(i) By considering the amount in Account A, show that
0.5 x  0.05 y  0.1z  16 . [1]
(ii) By forming a system of linear equations, find the values of x, y and z. [3]

It is given that the expansion of  2  px  in ascending powers of x, up to and


q
2

1
including the term in x, is  x . Find the values of p and q.
4
Find, in terms of n, the coefficient of x n in the above expansion. [4]

3 A water tank contains 8000 litres of water initially. At the beginning of each day,
500 litres of water is added to the tank. At the end of each day, 10% of the amount
of water in the tank will be used.
(i) Show that the amount of water in the tank after 3 days is 7051.5 litres. [1]
(ii) Find the least number of days it will take for the water in the tank to be less
than 5000 litres. [3]
(iii) Will the tank ever dry up? Justify your answer. [1]

© Hwa Chong Institution 9740 / JC1 Promo 2013 [Turn Over

sgfreepapers.com 55
3

4 The diagram below shows the graph of y  f  x  . It cuts the axes at the points

 0, 1 , 1.5, 0  and  3, 0  . It has a minimum point at  2.5, 0.5 . The horizontal,


vertical and oblique asymptotes are y  0, x  7 a and y   x  a respectively,
where a is a positive constant. x  7a
y

1
x
O 1.5 3
 2.5, 0.5

y  x  a
On separate diagrams, sketch the graphs of
1
(i) y , [3]
f  x

(ii) y  f ' x , [3]

showing clearly the axial intercepts, the stationary points and the equations of the
asymptotes where applicable.

5 A sequence of real numbers un  , for n    , satisfies the recurrence relation

un1  a a
 , with u1  a , where a and b are fixed non-zero real constants and
un  b b
a  b.
(i) Given that the limit l of the sequence un  exists, find the value of l. [2]

(ii) By expressing un 1 in terms of un , find an expression for un , leaving your

answer in terms of a, b and n. [2]


(iii) Given that the sum to infinity S for the sequence un  exists, state an

inequality satisfied by a and b. Find S in terms of a and b. [2]

© Hwa Chong Institution 9740 / JC1 Promo 2013 [Turn Over

sgfreepapers.com 56
4

By using the substitution u  9  4 x 2 , find x 9  4 x 2 dx .


3
6 (a) [4]
1
(b) Evaluate 0
x 2 tan 1 x dx , giving your answer in exact form. [4]

7 The coordinates of 3 points A, B and C are  2, 0, 1 ,  3, 1, 2  and 1, 2, 4 

respectively.
(a) Find the point D on the x-axis such that there exists a point P on line AB
where C, D and P are collinear. [4]

(b) Find two possible points E on the x-y plane, such that OE is a unit vector

and AOE  150 . [4]

2
8 (i) Express in partial fractions. [2]
r (r  1)  r  3
n
1
(ii) Hence find  2r (r  1)(r  3) .
r 1
[3]


1
(iii) Using the result in part (ii), determine the value of  2r (r  2)(r  3) .
r 5
[3]

9 Prove by mathematical induction that for all n    ,


1
1  1  2   1  2  3  1  2  3  4   ...  1  2  3  ...  n   n  n  1 n  2  .
6
[5]
Hence find, in terms of n,
(i) 3   3  6    3  6  9    3  6  9  12   ...   3  6  9  ...  (6n  3)  , [2]

(ii) 3   3  9    3  9  27   ...   3  9  27  81 ...  3n  . [2]

© Hwa Chong Institution 9740 / JC1 Promo 2013 [Turn Over

sgfreepapers.com 57
5

10 The functions f and g are defined as follows.

f  x  2  x 1 , x  ,
 1 2
 3x 3 , 0  x  2,
g  x  
 1   x  32 , x  2.

(i) Show that f 1 does not exist. [1]


(ii) If the domain of f is restricted to [k , ) such that f 1 exists, state the least

value of k and define f 1 in a similar form. [3]

Use the new domain of f found in part (ii) for the following parts.
(iii) Show algebraically that there is no value of x for which f 1  x   f  x  . [2]

(iv) Find the range of the composite function g f . [2]

(v) Find the value of x such that g f  x   1 . [1]

2x2  3
11 Sketch the graph of y  , showing clearly the axial intercepts, the stationary
x2
points and the equations of the asymptotes where applicable. [3]
2x2  3
(a) Solve the inequality  1. [2]
x2
2sin 2 x  3
Deduce the solution of the inequality  1 , where 0  x  2 . [2]
sin x  2
(b) Describe fully a sequence of transformations which would transform the graph
5 2x2  3
of y  2 x  to the graph of y  . [3]
x x2

© Hwa Chong Institution 9740 / JC1 Promo 2013 [Turn Over

sgfreepapers.com 58
6

12 An art structure, which is a parallelpiped (made of 6 faces of parallelograms) has a


horizontal base OABC, with OA, OC and OD as its three sides and remaining
vertices are B, E, F, and G as shown in the diagram below.

G F
D E
k
j C
B
O
i A

 
It is given that OA  5i and OC  i  7 j . The lines l1 and l2 have equations

given by l1 : r   5    i   7  14  j  6k , where  is a real parameter and



l2 : 3 x  z  15 , y  0 . E and F are on line l1 , and A and E are on line l2 .

(i) Find the position vector of E. [2]


(ii) Find the equation, in scalar product form, of the plane ABFE. [3]

(iii) Find the projection vector of AE onto the base OABC. Hence, or otherwise,
find the area of the projection of the plane ABFE onto the base. [2]
(iv) Find the equation of the line l3 , which is the reflection of line AE about the
base OABC. [2]
(v) An architect wants to add a shelter which has the plane equation
x  ay  bz  c , where a, b and c are unknown constants. He wants the shelter
to meet the plane ABFE at EF. What can be said about the values of a, b and
c? [2]

© Hwa Chong Institution 9740 / JC1 Promo 2013 [Turn Over

sgfreepapers.com 59
7

13 (a) Using differentiation, find the equation of the tangent at the point (2, 1) on

the curve x3  y 3  3  x  y  . [3]

(b) A spherical balloon is inflated such that 0.1 m3 of air is pumped into the
balloon every second. Find the rate of change of its surface area when the
diameter is 1 m . [4]
4 3
[Volume of sphere =  r and surface area of sphere = 4 r 2 .]
3

(c) When designing the floor plan of his new house, Mr Lim wants to build a
triangular garage with 2 adjacent walls of fixed lengths a and b meters and
making an angle of  radians. On the third side of his triangular garage, he
intends to build 4 square-shaped rooms of equal size (see diagram). Find the
value of  when the total area covered by the garage and the 4 rooms is a
maximum. [5]

a θ b

© Hwa Chong Institution 9740 / JC1 Promo 2013 [Turn Over

sgfreepapers.com 60
Suggested Solutions 2013 C1 H2 Math Promotional Examination
Qtn Solutions
1(i) Funds transferred into Account A: 0.05 y  0.1z
Funds transferred from Account A: 0.375 x  0.125 x  0.5 x

So we have 0.5 x   0.05 y  0.1z   16


i.e. 0.5 x  0.05 y  0.1z  16 ----(1)
(ii) Similarly, for Account B, we have
0.375 x  0.1 y  0.2 z  19 ----(2)
We also know x  y  z  90 ----(3)
Solving (1), (2), (3) using GC, we have
x  40, y  20, z  30

 2  px 
2 q

q
q  px 
 2 1  
 2 
  px  
 2 q 1   q     ... 
  2  
 pqx 
 2 q 1   ... 
 2 
1
 x
4
1 1  2 p 
 2 q  ---(1) &    1 ---(2)
4 4 2 
q  2, p  4
 2  4x
2

1
 1 2x
2

4
1
 1  2 2x 
 2 3 2x 2   2 3 4 2x 3 ...
    
4 2! 3! 
x n coefficient
1   2  3 4  ...    n  1   n
    2
4  n! 

1
  1  n  1 2n   1  n  1 2n  2
n n

3(i) Vol of water at end of Day 1


 0.9(8500)
Vol of water at end of Day 2

sgfreepapers.com 61
 0.9  500  0.9(8500)   0.9  500   0.92 (8500)
Vol of water at end of Day 3
 0.9(500)  0.92 (500)  0.93 (8500)
= 7051.5 litres
(ii) Vol of water at end of Day n, V
 0.9(500)  0.92 (500)  ...  0.9n1 (500)  0.9n (8500)
 500  0.9  0.92  ...  0.9n1   0.9n (8500)
 0.9 1  0.9n1  
 500    0.9n (8500)
 1  0.9 
 4500 1  0.9n1   0.9n (8500)
For V < 5000,
4500 1  0.9n 1   0.9n (8500)  5000
From G.C,
n V
18 5025.3
19 4972.8
20 4925.5
Least n  19
Least number of days = 19.
(iii) As n  , V  4500
Therefore, water tank will never dry up.
4i
y

1
Part I y
f  x
1
x
7a
y=0 O
 2.5, 2
Part III

x  1.5 x3
Part II
ii

2.5
y  1
x  7a

sgfreepapers.com 62
5 (i) Since l is the limit,
As n  , un  l , un 1  l
la a
 
l b b
 b l  a   a l  b
 bl  al
 l b  a  0  a  b 
l 0

(ii)
un1  a a

un  b b
 b  un1  a   a  un  b 
 bun 1  aun
a
 un 1  un
b
a
Hence un  is a GP with ratio and since u1  a,
b

n 1
a
un  a  
b

a
(ii) Since S exists, r  1  1
b
a
S
a
1
b
ab

ba

6(i) du
 8x
dx
9  4x2 dx   x2  8x  9  4x2  dx
1
x
3 1/2

8
1  u  9  du  1/2
     u  dx
8  4  dx 
1 9
  u3/2  u1/2 du
32 32
1 3
 u5/2  u3/2  C
80 16
  9  4 x2    9  4 x2   C
1 5/2 3 3/2

80 16

sgfreepapers.com 63
(ii) 1
 1 3  1 
1 1
 1 3  1 
0 x tan x dx   3 x  tan x0  0  3 x 
2 1
2 
dx
 1 x 
1
 1   1  x 
1
  x3  tan1 x    x   dx
 3  0 3 0
1  x2 
1
 1  1 1 
1

  x3  tan1 x   x2  ln 1 x2  
 3  3 2 2  0
 1    1  1 1 
       ln  2 
 3  4  3  2 2 
 1
  1 ln 2
12 6
7(a) 2  5 
   
AB line  r   0     1  ,   
    3
 1  
 2  5 
  
OP    
 1  3 
 
a
  
OD   0 
0
 
C , P, D are collinear.
 
CP  kCD
 2  5  a   a  1
   
  k 2 
 1  3   4 
   
1 5
   1, k  , a  
2 3
 5
 3
  
OD   0 
 0 
 
 
(b) E (a, b, 0)
a A
  
OE   b 
0
 
a 2  b2  1 E
 
OA  OE O
cos150  
OA 1

sgfreepapers.com 64
 2  a
   
 0 b
3  1  0 
 
2 5
3 2a 15
  a or 0.968 (3 s.f.)
2 5 4
15 2 1
 b 1 b  
16 4
 15 1   15 1 
E   , , 0  or E   ,  , 0 
 4 4   4 4 
8 (i)
2 A B C
  
r (r  1)  r  3 r r  1 r  3
2  A  r  1 r  3  B  r  r  3  Cr  r  1
2 1
r  0, A r  1, B  1 r  0, C
3 3
2 2 1 1
   
r (r  1)  r  3 3r r  1 3(r  3)

1 n 2 1 n  2 1 1 
     
4 r 1 r (r  1)  r  3 4 r 1  3r r  1 3(r  3) 
1 2 1 1
 [  
4 3 2 12
2 1 1
  
6 3 15
2 1 1
  
9 4 18
2 1 1
  
12 5 21
2 1 1
  
15 6 24

2 1 1
  
3(n  3) n2 3n
2 1 1
  
3(n  2) n 1 3  n  1
2 1 1
  
3(n  1) n 3 n  2
2 1 1
   ]
3n n 1 3  n  3

sgfreepapers.com 65
17 1 1 1 1 
      
4 18 n  1 3  n  1 3  n  2  3  n  3 
1 7 2 1 1 
   
12  6 n  1 n  2 n  3 

(iii)

1 1
r 5 2 r ( r  2)( r  3)

Replace r by r  3,

1

r  2 2r (r  1)(r  3)

1 1
 
r 1 2r (r  1)(r  3) 2 1 2 4
 1 7 2 1 1  1
 lim      
 
n 12 6 n  1 n  2 n  3   16
 7 1 1 1  1
 lim     
n 72
 6  n  1 12  n  2 12  n  3  16
7 1 5
  
72 16 144
9 (See alternative solution below)
Let P( n) be the statement
1
“ 1+ (1+2) + (1+2+3) + (1+2+3+…+n)  n  n  1 n  2  , n    ”
6
When n  1 , LHS of P(1) = 1,
(1)(2)(3)
RHS of P(1) = 1
6
Since LHS = RHS, P(1) is true.

Assume P(k) is true for some k ∈ Z+ ,


1
i.e. 1+ (1+2) + (1+2+3) + (1+2+3+…+k)  k  k  1 k  2 
6
To show P(k+1) is true,
1
i.e. 1+ (1+2) + (1+2+3) + (1+2+3+…+k+k+1)   k  1 k  2  k  3
6
LHS of P(k+1)
=1+(1+2)+(1+2+3)+(1+2+3+…+k)+(1+2+3+…+k+k+1)
1
 k  k  1 k  2  + 1  2  3  k  k  1
6
1 1
 k  k  1 k  2  +  k  1 k  2 
6 2

sgfreepapers.com 66
1
  k  1 k  2  k  3
6
= RHS of P(k+1)
Since P(1) is true, and P(k) is true => P(k+1) is true, by mathematical induction,
P(n) is true for n ∈ Z+ .

Alternative Solution:
n
1
" Ur  n  n 1 n  2 , where Ur  1 2  3  ...  r,
Let P( n) be the statement r 1 6
nZ "
1
When n  1 , LHS of P(1) = U
r 1
r  U1  1 ,

6
RHS of P(1) = 1
6
Since LHS = RHS, P(1) is true.

Assume P(k) is true for some k ∈ Z+ ,


k
1
i.e.  U r  k  k  1 k  2 
r 1 6

To show P(k+1) is true,


k 1
1
i.e.  U r   k  1 k  2  k  3
r 1 6
LHS of P(k+1)
k 1
 U r
r 1
k
  U r  U k 1
r 1
1
 k  k  1 k  2  + 1  2  3  k  k  1
6
1 1
 k  k  1 k  2  +  k  1 k  2 
6 2
1
 k  k  1 k  2  k  3
6
= RHS of P(k+1)

Since P(1) is true, and P(k) is true => P(k+1) is true, by mathematical induction,
P(n) is true for n ∈ Z+ .

sgfreepapers.com 67
(i)
3   3  6    3  6  9   ...   3  6  9  ...  (6 n  3) 
1 
 3 1  1  2   1  2  3   ...  (1  2  3  ...  (2 n  1))   3   2n  1 2n  2n  1 
6 
 n  2n  1 2n  1

(ii)
3   3  9    3  9  27   ...   3  9  27  81 ...  3n 
 3   31 2    31 2 3   ...   31 2 3... n 
1 1 2   1 2  3  ... 1 2  3... n 
3
n  n 1 n  2 

3 6

10 f  x  2  x  1, x  
(i)

y2
 2,1

The horizontal line y  2 cuts the curve at more than one point, hence f is not
one-to-one and f 1 does not exist.

OR f 1  f  3  2 , hence f is not one-to-one and f 1 does not exist.

(ii) The minimum value is k  2 .

Let y  f  x   2  x  1  x  2  1 x  2 
 x  2   y  1
2

Df 1  Rf  [1, )  f 1  x   2   x  1 , x  1
2

(iii) If there exists a solution for f 1  x   f  x 


 there exists a solution for f 1  x   x
 2   x  1  x
2

 x 2  3x  3  0
2
 3 3
x   0
 2 4
 no solution for x
 f 1  x   f  x  has no solution.

sgfreepapers.com 68
(iv)

 3,1
 2
 0, 
 3
y  g  x

 2, 0   4, 0 

[2, ) 
f
[1, ) 
g
  ,1  Rgf   ,1

(v) g f  x  1
f  x  3
x  2 1  3
x2  2
x2 4
x6

11

y  2x  4

 3.58,14.3

 0.419,1.68
y  1 for part (ii) 

x2
3 axial intercepts
 3  3 
 0,  ,   , 0  OR  0,1.5  ,  1.22, 0 
 2  2 
2 turning points
 0.419,1.68 ,  3.58,14.3
2 asymptotes
x  2, y  2 x  4
(a) Using the graph, the intersections of the curve with the line y = 1 are
 0.5,1 , 1,1 , so the solution is
1
  x  1 or x  2
2

sgfreepapers.com 69
2sin 2 x  3
1
sin x  2
So the solution is
1
  sin x  1 or sin x  2  rej
2

1
2
O
1
 7 1
2  , 
6 2

7 11
 0  x   or   x  2
6 6
(b) 2 x2  3 5
y  2x  4 
x2 x2

Translation of 2 units in the positive x-direction, followed by translation of 8


units in the positive y-direction.

12  5  1
(i)    
lEF : r   14     7  ,   
   0
 6   
l AE : 3x  z  15
x  0 z   15 
 ,y0
1 3
 0  1
   
l AE : r   0     0  ,   
    3
 15   
2
 5  1 7
      
OE   14   2  7    0 
 6  0 6
     
(ii)  1   1   21 
     
n   7    0    3 
      
 0   3   7 

sgfreepapers.com 70
 5   21 
  
 0  .  3   105
 0   7 
  
 21 
 
r.  3   105
 
 7 
(iii) Method 1: E
By Observation, 
Projection vector of AE
1  2
onto  0  =  0  A
0  0
   
2

Method 2: 
Projection of of AE onto normal of floor E
  2   1    1  2
          
AE '    0  . 0    0    0 
  6   0    0   0 
       A

Method 1:

F ' X  7 (Deduce from OC ) B

Area =  AE ' F ' X   2  7  14

A X

Method 2:
1  2  0 
       
Area = AB  AE '   7    0    0   14
 0   0  14 
     
(iv) Let E '' be the reflection of E about and plane OABC. E
7 7
     
OE   0  , OE ''   0  ,
6  6 
   
 2 A
    
AE ''  OE ''  OA   0 
 6 
 

sgfreepapers.com 71
5 1
   
l3 : r   0     0  ,   
    3 
0  
(v) Let  be plane x  ay  bz  c.
EF is //  .
1
 
 7  is  to n  .
0
 
1 1
   1
 7  .  a   0  1  7a  0  a   7
0 b
  
E is on plane  .
7 1
  
 0  .  a   c  7  6b  c.
6 b
  
13 x3  y 3  3x  3 y
(a)
d 3
dx
 x  y 3    3x  3 y 
d
dx
dy dy
3x 2  3 y 2  33
dx dx
dy dy
3x 2  3  3 y 2  3
dx dx
x  1 dy
2

y 2  1 dx
Substitute x  2 and y  1 ,
dy 3
 (undefined)
dx 0
Therefore, the tangent is a vertical line.
Thus, the tangent is x  2 .

(b) Let the radius be r .


dS
We want to find ,
dt
dS dS dV d V
  
dt d r dt d r
  8 r    0.1   4 r 2 
1

5r
1 dS 2
Sub r  into , we get m 2 / s .
2 dt 5

sgfreepapers.com 72
(c) Let the side of each room be x .
By cosine rule,
 4 x   a 2  b 2  2ab cos 
2

1
Total area, A  ab sin   4 x 2
2
A  ab sin    a 2  b 2  2ab cos  
1 1
2 4
1 1 1 1
 ab sin   a 2  b 2  ab cos 
2 4 4 2
dA
To find max area, we let  0.
d
dA d  1 1 2 1 2 1 
  ab sin   a  b  ab cos  
d d  2 4 4 2 
1 1
 ab cos   ab sin 
2 2
1 1
ab cos   ab sin   0
2 2
tan   1
3
  since 0     
4
3
Therefore, stationary point at   .
4
d2 A 1 1
 ab cos   ab sin 
d 2
2 2
2
d A
0
d 2   3
4

Thus, the stationary point is maximum.

sgfreepapers.com 73
INNOVA JUNIOR COLLEGE
JC 1 MID COURSE EXAMINATION
in preparation for General Certificate of Education Advanced Level
Higher 2

CANDIDATE
NAME

CLASS INDEX NUMBER

MATHEMATICS 9740/01
8 October 2013
Additional Materials: Answer Paper
3 hours
Cover Page
List of Formulae (MF15)

READ THESE INSTRUCTIONS FIRST

Do not open this booklet until you are told to do so.

Write your name, class and index number on all the work you hand in.

Write in dark blue or black pen on both sides of the paper. You may use a soft pencil for any
diagrams or graphs.
Do not use staples, paper clips, highlighters, glue or correction fluid.

Answer all the questions.


Give non-exact numerical answers correct to 3 significant figures, or 1 decimal place in the case
of angles in degrees, unless a different level of accuracy is specified in the question.
You are expected to use a graphic calculator.
Unsupported answers from a graphic calculator are allowed unless a question specifically states
otherwise.
Where unsupported answers from a graphic calculator are not allowed in a question, you are
required to present the mathematical steps using mathematical notations and not calculator
commands.

You are reminded of the need for clear presentation in your answers.

At the end of the examination, fasten all your work securely together.

The number of marks is given in brackets [ ] at the end of each question or part question.

This document consists of 6 printed pages.

Innova Junior College [Turn over

sgfreepapers.com 74
2

1 + x2
1* (i) Find the expansion of in ascending powers of x, up to and including the
√ (4 + 2 x)
term in x 2 . [3]

(ii) State the range of values of x for which this expansion is valid. [1]
(iii) Write down the equation of the tangent to the curve
1 + x2
y=
√ (4 + 2 x)
at the point where x = 0. [1]

2 y
B(−1,8) C(0,7)
• •

y=3

• x
A(−3,0) O

x=−4
The diagram shows the graph of y = f ( x ) . There is a maximum point B ( −1,8 ) and the curve
cuts the axes at the points A ( −3, 0 ) and C ( 0, 7 ) . The lines x = − 4 and y = 3 are asymptotes
of the curve.
Sketch, on separate diagrams, the graphs of
(i) y = f ′( x) , [2]

√{f ( 2 x )} ,
(ii) y=− 1 [3]

stating the equations of the asymptotes and the coordinates of the points corresponding to A,
B and C where possible.

IJC/2013/JC19740/01/Oct/13 *: not in topics tested for SRJC 2014 Promo

sgfreepapers.com 75
3

3 (i) Using the method of difference, show that


n
k k 1 1 
∑ ( r + 1)( r + 3) = 2  a − n + 2 − n + 3  ,
r =1
where a is a constant to be determined. [4]


k
(ii) Hence find the range of values of k such that ∑ ( r + 1)( r + 3) is at most 1. [2]
r =1

n
( ) = 1−
r 2r 2n +1
4 (i) Prove by induction that ∑ ( r + 2 )! ( n + 2 )!
for all positive integers n. [5]
r =1

2 n r 2r
( ).
(ii) Hence find an expression in terms of n for ∑ ( r + 2 )! [2]
r =n

5* Find


4
(i) dx , [3]
√ (5 + 4 x − 4 x2 )

∫ (3sin 2θ − secθ )
2
(ii) dθ . [4]

 
6 Referred to the origin O, the points A and B are such that OA = a and OB = b . The point P
on AB is such that AP : PB = 2 : 3 . It is given that a = √ 5 , b = 3 and OP is perpendicular to
AB.

(i) Show that a ⋅ b = −3 . [3]


(ii) Find the size of angle AOB. [2]

(iii) Find the length of projection of OB onto OA. [1]

IJC/2013/JC19740/01/Oct/13 *: not in topics tested for SRJC 2014 Promo [Turn over

sgfreepapers.com 76
4

7 A water tank in the shape of an inverted cone has a height twice that of its radius. Water is
poured into the cone. Given that, when the depth of the water is 10 cm, the volume of water is
increasing at a rate of 10π cm3s −1 , find the rate of increase at this instant of
(i) the slant height of the cone in contact with the water, [5]
(ii) the curved surface area of the cone in contact with the water. [2]
1 2
[The volume of a cone is π r h and the curved surface area is π rl .]
3

8 The equation of a curve is x 2 − 2 xy + 2 y 2 = −12 .

(i) Find the equations of the tangent and normal to the curve at the point P ( 2,4 ) . [5]
(ii) The tangent at P meets the y-axis at A and the normal at P meets the x-axis at B. Find
the area of triangle APB. [3]

9 (a) An arithmetic progression A has first term 3 and the sum of the terms from the
16th term to the 30th term inclusive is 2025. Show that the common difference is 6. [3]

If Sn is the sum of the first n terms of A, show that the sum of the first
n even-numbered terms of A, that is, the second, fourth, sixth, … terms, is given by
 1
 2 +  Sn . [2]
 n

4
(b) A geometric series G has first term 30 and common ratio − . Write down the sum,
5
Sn , of the first n terms of the series. [1]

Find the least value of n for which the magnitude of the difference between Sn and
the sum to infinity of the series is less than 0.004. [3]

A new series is formed by taking the reciprocal of the corresponding terms of G.


Determine if the new series is convergent. [1]

IJC/2013/JC19740/01/Oct/13 *: not in topics tested for SRJC 2014 Promo

sgfreepapers.com 77
5

10* (i) By successively differentiating ln ( 3 + x ) , find the Maclaurin’s series for ln ( 3 + x ) ,


up to and including the term in x3 . [3]

1
(ii) Given that θ is small, find the expansion of ( )
2 − cos 5θ 2 2 in ascending

powers of θ , up to and including the term in θ 4 . [2]

Two particles A and B produce y units of energy when they are x units away from their
original position at x = 0 . The energy produced by particles A and B can be found by the
equations
y = ln ( 3 + x ) and
1
(
y = 2 − cos 5 x 2 2)
respectively, where x ≥ 0 .

(iii) Explain in the context of the question, what is meant by the solution to the
equation
1
( )
ln ( 3 + x ) = 2 − cos 5 x 2 2 . [1]

(iv) Using your answers from parts (i) and (ii), find an estimate for the maximum distance
from the original position such that the difference in energy produced by both
particles is at most 0.4 units. [You may assume that both particles are at the same
distance from the original position.] [2]

11 (i) Find a vector equation of the line through the points A and B with position vectors
3i + 4 j + 5k and −i + 12 j + 9k respectively. [2]

(ii) The perpendicular to this line from the point C with position vector 2i + j − 2k meets
the line at the point N. Find the position vector of N. [3]

(iii) Find a Cartesian equation of the line AC. [2]

(iv) Use a vector product to find the exact area of triangle OAB. [3]

IJC/2013/JC19740/01/Oct/13 *: not in topics tested for SRJC 2014 Promo[Turn over

sgfreepapers.com 78
6

12 A container is made up of an open cylinder of varying height h cm and varying radius r cm,
and a hollow hemispherical lid of varying radius r cm. It costs 5 cents per square centimetre
to manufacture the base, 3 cents per square centimetre to manufacture the curved surface of
the cylinder and 4 cents per square centimetre to manufacture the curved surface of the
hemisphere.

(i) Given that the cylinder is of fixed volume V cm 3 , show that the manufacturing cost
1
 3V  3
of the container is minimum when r is   . [7]
 13π 

(ii) Using the value of r in part (i) and taking V to be 30, find the maximum number of
containers that a person can buy if he has $22. [2]
[The surface area of a sphere is 4π r 2 .]

13 The function f is defined as follows:


1
f:x for x ∈ , x ≠ −2, x ≠ 2.
2
x −4

(i) Sketch the graph of y = f ( x ) . [2]


The function g is defined as follows:
1
g:x for x ∈ , x ≠ a, x ≠ 3, x ≠ b.
x−3
It is given that the function fg exists.
(ii) Find the values of a and b. [2]

(iii) Show that fg ( x ) =


( x − 3 )2 . [2]
( 2 x − 5)( 7 − 2 x )
(iv) Solve the inequality fg ( x ) > 0. [3]
(v) Find the range of fg. [3]
IJC/2013/JC19740/01/Oct/13 *: not in topics tested for SRJC 2014 Promo

sgfreepapers.com 79
2013 H2 Maths MCE_Marking Scheme

1 + x2
1* (i) Find the expansion of in ascending powers of x, up to and including the
√ (4 + 2 x)
term in x 2 . [3]

(ii) State the range of values of x for which this expansion is valid. [1]

(iii) Write down the equation of the tangent to the curve


1 + x2
y=
√ (4 + 2 x)
at the point where x = 0. [1]

1(i) 1 + x2
√ (4 + 2 x)
1
( )
= 1 + x2 ( 4 + 2x )

2

1

1
(
= 1 + x2
2
)  x
1 + 
 2
2

  1 3 
  − ×−  2 
1
2
( 
)


1
2


x
2

= 1 + x 2  1 +  −   +  2
  2!
2  x 
   + ... 
 2  
   
1
(  x 3
)
= 1 + x 2  1 − + x 2 + ... 
2  4 32


1 1 3 1
= − x + x 2 + x 2 + ...
2 8 64 2
1 1 35 2
= − x + x + ...
2 8 64

(ii) x
<1
2
x
−1 < < 1
2
−2 < x < 2
(iii) 1 1
y= − x
2 8

*: Not in topics tested for 2014 SRJC Promo

sgfreepapers.com 80
2

2 y
B(−1,8) C(0,7)
• •

y=3

• x
A(−3,0) O

x=−4
The diagram shows the graph of y = f ( x ) . There is a maximum point B ( −1,8 ) and the curve
cuts the axes at the points A ( −3, 0 ) and C ( 0, 7 ) . The lines x = −4 and y = 3 are asymptotes
of the curve.

Sketch, on separate diagrams, the graphs of


(i) y = f ′( x) , [2]

√{f ( 2 x )} ,
(ii) y=− 1 [3]

stating the equations of the asymptotes and the coordinates of the points corresponding to A,
B and C where possible.

IJC/2013/JC19740/01/Oct/13 *: Not in topics tested for 2014 SRJC Promo

sgfreepapers.com 81
3

2(i)
y

y = f′ (x)

y=0 x

B′(−1,0) O

x = −4

(ii)
y
x
A′(−6,0) O

y = −√3

y = −√f(0.5x)
• • C ′ (0,−√7)
B′(−2,−√8)

IJC/2013/JC19740/01/Oct/13 *: Not in topics tested for 2014 SRJC Promo

sgfreepapers.com 82
4

3 (i) Using the method of difference, show that


n
k k 1 1 
∑ ( r + 1)( r + 3) = 2  a − n + 2 − n + 3  ,
r =1
where a is a constant to be determined. [4]


k
(ii) Hence find the range of values of k such that ∑ ( r + 1)( r + 3) is at most 1. [2]
r =1

3(i) k k 1 1 
=  − 
( r + 1)( r + 3) 2  r +1 r + 3 

n
k k n  1 1 
∑ ( r + 1)( r + 3)
r =1
= ∑  − 
2 r =1  r + 1 r + 3 
k1 1
=  −
22 4
1 1
+ −
3 5
1 1
+ −
4 6
1 1
+ −
5 7
+ …
+ …
1 1
+ −
n −1 n +1
1 1
+ −
n n+2
1 1 
+ − 
n +1 n+3
k5 1 1 
=  − − 
2 6 n+2 n+3
5
a=
6

IJC/2013/JC19740/01/Oct/13 *: Not in topics tested for 2014 SRJC Promo

sgfreepapers.com 83
5

(ii) ∞
k k 5 5k
∑ ( r + 1)( r + 3) = 2  6  = 12
r =1

5k
≤1
12
12
⇒k≤
5

n
( )
r 2r 2n+1
4 (i) Prove by induction that ∑ ( r + 2 )!
r =1
= 1−
( n + 2 )!
for all positive integers n. [5]

2n
( )
r 2r
(ii) Hence find an expression in terms of n for ∑ ( r + 2 )! .
r =n
[2]

4(i) n
( )
r 2r 2n +1
Let Pn denote ∑ ( r + 2 )!
r =1
= 1−
( n + 2 )!
for n ∈+ .

When n = 1,

1
r ( 2r )
LHS = ∑ ( r + 2 )!
r =1

(1) ( 21 )
=
(1 + 2 )!
2
=
3!
1
=
3

21+1
RHS = 1 −
(1 + 2 )!
4
= 1−
3!
2
= 1−
3
1
=
3

Therefore, P1 is true.

Assume Pk is true for some k ∈  + ,


IJC/2013/JC19740/01/Oct/13 *: Not in topics tested for 2014 SRJC Promo

sgfreepapers.com 84
6

k r 2r ( ) 2k +1
i.e. ∑ ( r + 2 )!
r =1
= 1−
( k + 2 )!
.

Want to prove Pk +1 is true,


k +1
( )
r 2r 2k + 2
i.e. ∑ ( r + 2 )!
r =1
=1−
( k + 3 )!
.

k +1
r ( 2r )
LHS = ∑ ( r + 2 )!
r =1
k
r ( 2r )
( k + 1) ( 2k +1 )
= ∑r =1
( r + 2 )! ( k + 3)!
+

2k +1  ( k + 1) ( 2 )
k +1

= 1 −  +
 ( k + 2 ) ! ( k + 3)!
 ( 2k +1 ) ( k + 3) ( k + 1) ( 2k +1 ) 
=1 −  − 
 ( k + 3) ! ( k + 3)! 
 ( 2k +1 ) ( k + 3) − ( k + 1)  
=1 −  
 ( k + 3)! 
 ( 2k +1 ) ( 2 ) 
=1 −  
 ( k + 3)! 
2k +2
=1 −
( k + 3)!
= RHS

Thus Pk is true ⇒ Pk +1 is true.

Since P1 is true, and Pk is true ⇒ Pk +1 is true, by mathematical


induction, Pn is true for all n ∈+ .
(ii) 2n r ( 2r )
∑ ( r + 2 )!
r =n

2n r ( 2r ) n −1 r ( 2r )
=∑
( r + 2 )! ∑

r =1 r =1 ( r + 2 ) !

 22 n +1   2n 
= 1 −   − 1 − 
 ( 2n + 2 ) !  ( n + 1) !
2n 22 n +1
= −
( n + 1)! ( 2n + 2 )!

IJC/2013/JC19740/01/Oct/13 *: Not in topics tested for 2014 SRJC Promo

sgfreepapers.com 85
7

5* Find


4
(i) dx , [3]
√ (5 + 4x − 4x2 )

∫ (3sin 2θ − secθ )
2
(ii) dθ . [4]

5(i) 5 + 4x − 4x2
 5
= −4  x 2 − x − 
 4
 1 1 5
2 2

= −4   x −  −   − 
 2   2  4 

 1 6
2
3  1 
2

= −4   x −  −  = 4  −  x −  
 2  4   2  2  


4
dx
5 + 4 x − 4 x2

∫ ∫
4 4
= dx or dx
4  32 − ( x − )  6 − ( 2 x − 1)
2
1 2
 2 


4
= dx
− ( x − 12 )
3 2
2 2

 x− 1   2x −1 
= 2sin −1  2
+C or 2 sin −1  +C
 3   6 
 2 

(ii) 2
∫ (3sin 2θ − sec θ ) dθ
= ∫ 9sin 2 2θ − 6sin 2θ sec θ + sec2 θ dθ
9
∫ (1 − cos 4θ )dθ − 6∫ 2sin θ cos θ sec θ dθ + ∫ sec θ dθ
2
=
2
9
= ∫ (1 − cos 4θ )dθ − 12 ∫ sin θ dθ + ∫ sec 2θ dθ
2
9 1 
=  θ − sin 4θ  − 12 ( − cos θ ) + tan θ + c
2 4 
9 9
= θ − sin 4θ + 12 cos θ + tan θ + c
2 8

IJC/2013/JC19740/01/Oct/13 *: Not in topics tested for 2014 SRJC Promo

sgfreepapers.com 86
8
 
6 Referred to the origin O, the points A and B are such that OA = a and OB = b . The point P
on AB is such that AP : PB = 2 : 3 . It is given that a = √ 5 , b = 3 and OP is perpendicular to
AB.
(i) Show that a ⋅ b = −3 . [3]
(ii) Find the size of angle AOB. [2]

(iii) Find the exact length of projection of OB onto OA. [1]

6(i)  1
By Ratio Theorem, OP = ( 3a + 2b ) .
5
 
Since OP ⊥ AB, OP ⋅ AB = 0 .
1
( 3a + 2b ) ⋅ ( b − a ) = 0
5
3a ⋅ b − 3a ⋅ a + 2b ⋅ b − 2b ⋅ a = 0
2 2
a ⋅b − 3 a + 2 b = 0
a ⋅ b − 15 + 18 = 0
a ⋅ b = −3
(ii) a ⋅b
cos ∠AOB =
a b
−3
=
3 5
1
= −
5
∠AOB = 116.6 (or 2.03 rad)

(iii) Length of projection of OB onto OA
b ⋅a
=
a
3
=
5

IJC/2013/JC19740/01/Oct/13 *: Not in topics tested for 2014 SRJC Promo

sgfreepapers.com 87
9

7 A water tank in the shape of an inverted cone has a height twice that of its radius. Water is
poured into the cone. Given that, when the depth of the water is 10 cm, the volume of water is
increasing at a rate of 10π cm 3s −1 , find the rate of increase at this instant of
(i) the slant height of the cone in contact with the water, [5]
(ii) the curved surface area of the cone in contact with the water. [2]
1 2
[The volume of a cone is π r h and the curved surface area is π rl .]
3

7(i) Let the radius of the water surface, the depth of the water, the slant height of the water and
the volume of the water at time t seconds be r cm, h cm, l cm and V cm3 respectively.

1 1 2
V = π r 2 h = π r 2 (2r ) = π r 3
3 3 3
dV dV dr dr
= ⋅ = 2π r 2
dt dr dt dt
h = 2r = 10 ⇒ r = 5
dV
When = 10π and r = 5,
dt
dr
10π = 2π (5)2
dt
dr 1
=
dt 5
Using Pythagoras' theorem,
l 2 = ( 2r ) 2 + r 2
l = 5r
dl dl dr dr 1 5
= ⋅ = 5 = 5  = or 0.44721
dt dr dt dt 5 5

5
The rate of increase of the slant height of the cone in contact with the water is cms−1
5
(or 0.447 cms−1) .

IJC/2013/JC19740/01/Oct/13 *: Not in topics tested for 2014 SRJC Promo

sgfreepapers.com 88
10

7(ii) Let the curved surface area of the water at time t seconds be A cm 2 .
A = π rl = π r ( 5r ) = 5π r 2
dA dA dr dr
= ⋅ = 2 5π r
dt dr dt dt

dr 1
When r = 5, = .
dt 5
dA 1
= 2 5π ( 5 )   = 2 5π = 14.0496
dt 5
The rate of increase of the curved surface area of the cone in contact with the water is
2 5π cm 2s −1 (or 14.0 cm2s−1).

IJC/2013/JC19740/01/Oct/13 *: Not in topics tested for 2014 SRJC Promo

sgfreepapers.com 89
11

8 The equation of a curve is x 2 − 2 xy + 2 y 2 = −12 .

(i) Find the equations of the tangent and normal to the curve at the point P ( 2, 4 ) . [5]

(ii) The tangent at P meets the y-axis at A and the normal at P meets the x-axis at B. Find
the area of triangle APB. [3]

8(i) x 2 − 2 xy + 2 y 2 = −12
 dy  dy
2x −  2x + 2 y  + 4 y =0
 dx  dx
dy dy
2x − 2 y = 2x − 4 y
dx dx
dy
2x − 2 y = (2x − 4 y )
dx
dy 2 x − 2 y
=
dx 2 x − 4 y
x− y
=
x − 2y
At P ( 2, 4 ) :
dy 2 − 4
=
dx 2 − 8
1
=
3
Equation of tangent:
1
y−4= ( x − 2)
3
1 10
y= x+
3 3
Gradient of normal = −3
Equation of normal:
y − 4 = −3 ( x − 2 )
y = −3 x + 10

IJC/2013/JC19740/01/Oct/13 *: Not in topics tested for 2014 SRJC Promo

sgfreepapers.com 90
12

8(ii) When tangent meets y -axis at A, x = 0


10
y=
3
 10 
∴ A  0, 
 3
When normal meets x-axis at B, y = 0
3 x = 10
10
x=
3
 10 
∴B ,0
 3 
Area of triangle APB
1
= × AP × BP
2
1 40 160
= × ×
2 9 9
40
= units 2 (or 4.44 units 2 )
9

IJC/2013/JC19740/01/Oct/13 *: Not in topics tested for 2014 SRJC Promo

sgfreepapers.com 91
13

9 (a) An arithmetic progression A has first term 3 and the sum of the terms from the
16th term to the 30th term inclusive is 2025. Show that the common difference is 6. [3]

If Sn is the sum of the first n terms of A, show that the sum of the first
n even-numbered terms of A, that is, the second, fourth, sixth, … terms, is given by
 1
 2 +  Sn . [2]
 n

9(a) S30 − S15 = 2025


30 15
 2 ( 3) + 29d  −  2 ( 3) + 14d  = 2025
2 2
330d = 1980
d=6

n
Sn =  6 + ( n − 1) 6  = 3n 2
2
Sum of 1st n even-numbered terms
n
=  2 ( 3 + 6 ) + ( n − 1)12 
2
n
= [6 + 12n ]
2
1 
= 3n 2  + 2 
n 
 1
=  2 +  Sn
 n

IJC/2013/JC19740/01/Oct/13 *: Not in topics tested for 2014 SRJC Promo

sgfreepapers.com 92
14

4
9(b) A geometric series G has first term 30 and common ratio − . Write down the sum,
5
S n , of the first n terms of the series. [1]

Find the least value of n for which the magnitude of the difference between S n and
the sum to infinity of the series is less than 0.004. [3]

A new series is formed by taking the reciprocal of the corresponding terms of G.


Determine if the new series is convergent. [1]

9(b)   4 n 
30 1 −  −  
  5   50   4  n 
Sn =  = 1 −  −  
 4 3   5 
 
1−  − 
 5
Sn − S∞ < 0.004

50   4   50
n
1 −  −   − < 0.004
3   5  3
 
n
50  4 
  < 0.004
3 5
n
4 3
  < 0.004 ×
5 50
 3 
ln  0.004 × 
n> 
50 
4
ln  
5
n > 37.352
Least value of n is 38.

1 1 1 1 1
New series + + 2 + 3 + 4 +… is a geometric series with
a ar ar ar ar
1 5
common ratio =− .
r 4
1 5
Since = > 1 , the new series is not convergent.
r 4

IJC/2013/JC19740/01/Oct/13 *: Not in topics tested for 2014 SRJC Promo

sgfreepapers.com 93
15

10* (i) By successively differentiating ln ( 3 + x ) , find the Maclaurin’s series for ln ( 3 + x ) , up


to and including the term in x 3 . [3]

1
(ii) Given that θ is small, find the expansion of ( 2 − cos 5θ 2 ) 2 in ascending
powers of θ , up to and including the term in θ 4 . [2]

Two particles A and B produce y units of energy when they are x units away from their
original position at x = 0 . The energy produced by particles A and B can be found by the
equations
y = ln ( 3 + x ) and
1
y = ( 2 − cos 5 x 2 ) 2
respectively, where x ≥ 0 .

(iii) Explain in the context of the question, what is meant by the solution to the
equation
1
ln ( 3 + x ) = ( 2 − cos 5 x 2 ) 2 . [1]

(iv) Using your answers from parts (i) and (ii), find an estimate for the maximum distance
from the original position such that the difference in energy produced by both
particles is at most 0.4 units. [2]
[You may assume that both particles are at the same distance from the original
position.]

IJC/2013/JC19740/01/Oct/13 *: Not in topics tested for 2014 SRJC Promo

sgfreepapers.com 94
16

10(i) Let y = ln(3 + x )


dy −1
= (3 + x )
dx
d2 y −2
2
= − (3 + x )
dx
d3 y −3
= 2 (3 + x )
dx3

When x = 0 ,
dy 1 d2 y 1 d3 y 2
y = ln 3, = , 2
=− , =
dx 3 dx 9 dx 3 27
2 3
x x x
∴ y = ln 3 + − + +…
3 18 81
(ii) Given that θ is small,
1
  2
( 5θ 2 ) 
2
1
( 2 − cos 5θ )
2 2 =  2 − 1 −
  2 
 + …

   
1
 25 2
= 1 + θ 4 + … 
 2 
 1  25
= 1+   θ 4 +…
2 2
25
= 1+ θ 4 +…
4
(iii) The solution (x value) denotes the distance in units
where both particles produce the same number of
units of energy.
(iv) x x 2 x3  25 4 
ln 3 + − + − 1 + x  ≤ 0.4
3 18 81  4 

Or

x x 2 x3  25 4 
−0.4 ≤ ln 3 + − + − 1 + x  ≤ 0.4
3 18 81  4 

From GC, x ≤ 0.57298752 (given x ≥ 0 )

An estimate for the maximum distance is 0.572 units.


(3 s.f.)

IJC/2013/JC19740/01/Oct/13 *: Not in topics tested for 2014 SRJC Promo

sgfreepapers.com 95
17

11 (i) Find a vector equation of the line through the points A and B with position vectors
3i + 4 j + 5k and −i + 12 j + 9k respectively. [2]

(ii) The perpendicular to this line from the point C with position vector 2i + j − 2k meets
the line at the point N. Find the position vector of N. [3]

(iii) Find a Cartesian equation of the line AC. [2]

(iv) Use a vector product to find the exact area of triangle OAB. [3]

IJC/2013/JC19740/01/Oct/13 *: Not in topics tested for 2014 SRJC Promo

sgfreepapers.com 96
18

11(i)  −1   3   −4   −1
        
AB =  12  −  4  =  8  = 4  2 
 9  5  4  1
       
 3  −1
   
l AB : r =  4  + λ  2  , λ ∈ 
5  
  1
 −1   −1
   
or l AB : r =  12  + λ  2  , λ ∈ 
9 1
   
(ii) Since N lies on line AB,
3  −1 
    
ON =  4  + λ  2  for some λ ∈ 
5  
  1
 3  −1  2   1   −1
          
CN =  4  + λ  2  −  1  =  3  + λ  2 
5  1   −2   7  1
         
Since CN ⊥ AB,
 −1 
  
CN ⋅  2  = 0
1
 
 1   −1   −1
     
 3  + λ  2   ⋅  2  = 0
 7     
 1    1 
 1   −1  −1  −1
       
 3⋅ 2  + λ  2 ⋅ 2  = 0
7  1     
    1 1
12 + 6λ = 0
λ = −2
 3   −1  5 
      
ON =  4  − 2  2  =  0 
 5  1   3
     

IJC/2013/JC19740/01/Oct/13 *: Not in topics tested for 2014 SRJC Promo

sgfreepapers.com 97
19

11(iii)  2   3   −1  1
        
AC =  1  −  4  =  −3  = −  3 
 −2   5   −7   
      7

Cartesian eqn of line AC:


y −4 z −5
x−3 = =
3 7
y −1 z + 2
or x−2 = =
3 7
(iv) Area of triangle OAB
1  
= OA × OB
2
 3   −1
1    
=
2  4  ×  12 
 5  9 
   
 −24 
1  
=
2  −32 
 40 
 
 −3 
= × 8  −4 
1
2  5
 
= 4 9 + 16 + 25
= 4 50
= 20 2

IJC/2013/JC19740/01/Oct/13 *: Not in topics tested for 2014 SRJC Promo

sgfreepapers.com 98
20

12 A container is made up of an open cylinder of varying height h cm and varying radius r cm,
and a hollow hemispherical lid of varying radius r cm. It costs 5 cents per square centimetre
to manufacture the base, 3 cents per square centimetre to manufacture the curved surface of
the cylinder and 4 cents per square centimetre to manufacture the curved surface of the
hemisphere.

(i) Given that the cylinder is of fixed volume V cm3 , show that the manufacturing cost
1
 3V  3
of the container is minimum when r is   . [7]
 13π 

(ii) Using the value of r in part (i) and taking V to be 30, find the maximum number of
containers that a person can buy if he has $22. [2]
[The surface area of a sphere is 4π r 2 .]

12(i) V = π r 2 h
V
∴h =
π r2

IJC/2013/JC19740/01/Oct/13 *: Not in topics tested for 2014 SRJC Promo

sgfreepapers.com 99
21

Let C cents be the manufacturing cost of the container.


C = 4 ( 2π r 2 ) + 3 ( 2π rh ) + 5 (π r 2 )
 V 
=13π r 2 + 6π r  2 
πr 
6V
=13π r 2 +
r
dC
= 13π ( 2r ) + 6V ( −r −2 )
dr
6V
= 26π r − 2
r
dC
Let =0
dr
6V
26π r − 2 = 0
r
26π r = 6V
3

6V
r3 =
26π
3V
=
13π
3V
r=3
13π
2
dC
= 26π − 6V ( −2r −3 )
dr 2
12V
= 26π + 3
r
12V
= 26π +
 3V 
 
 13π 
= 26π + 52π
= 78π > 0
Hence, the manufacturing cost is minimum
3V
when r = 3 . [Shown]
13π

IJC/2013/JC19740/01/Oct/13 *: Not in topics tested for 2014 SRJC Promo

sgfreepapers.com 100
22

(ii) 6V
C = 13π r 2 +
r
2
 3V  6V
= 13π  3  +
 13π  3
3V
13π
2
 90  3 180
= 13π   +
 13π 
1
 90  3
 
 13π 
= 207.48 cents
= $2.0748

22
2.0748
= 10.603
∴ Maximum number of containers he can buy is 10.

13 The function f is defined as follows:


1
f:x 2 for x ∈ , x ≠ −2, x ≠ 2.
x −4

(i) Sketch the graph of y = f ( x ) . [2]

The function g is defined as follows:


1
g:x for x ∈ , x ≠ a , x ≠ 3, x ≠ b.
x−3

It is given that the function fg exists.

(ii) Find the values of a and b. [2]


( x − 3)
2

(iii) Show that fg ( x ) = . [2]


( 2 x − 5)( 7 − 2 x )
(iv) Solve the inequality fg ( x ) > 0. [3]
(v) Find the range of fg. [3]

13(i)
y
y = f(x)

IJC/2013/JC19740/01/Oct/13 *: Not in topics tested for 2014 SRJC Promo


−2 2 y=0
−1/4

sgfreepapers.com 101
23

(ii) For fg to exist, Rg ⊆ Df .


Hence, g ( x ) cannot take the values – 2 and 2.
1 5
= −2 ⇒ x =
x −3 2
1 7
= 2⇒ x =
x −3 2
5 7
The values of a and b are and .
2 2

(iii) 1
f g(x) = 2
 1 
  −4
 x −3
1
=
1 − 4 ( x − 3)
2

( x − 3)
2

( x − 3)
2

=
12 −  2 ( x − 3) 
2

( x − 3)
2

=
1 + 2 ( x − 3)  1 − 2 ( x − 3) 

( x − 3)
2

= (shown)
( 2 x − 5)( 7 − 2 x )
(iv) ( x − 3)
2

>0
( 7 − 2 x )( 2 x − 5)
– + + –

IJC/2013/JC19740/01/Oct/13 *: Not in topics tested for 2014 SRJC Promo

sgfreepapers.com 102
24

2.5 3 3.5

Solving,
5 7
< x < 3 or 3 < x <
2 2
5 7
or <x< , x≠3
2 2

(v) Sketching the graph of y = g(x),


y
y = g(x)

(7/2, 2)
° x
(5/2, −2)°
O 3

x=3

Rg = { y ∈ : y ≠ −2, 0, 2}
Referring to the graph of y = f(x) in part (i),
 1 
Rfg =  y ∈ : y < − or y > 0 
 4 

OR

Sketch the graph of y = fg(x).

O 5/2 3 7/2 x
o y = − 1/4

y = fg(x)

x = 5/2 x = 7/2

IJC/2013/JC19740/01/Oct/13 *: Not in topics tested for 2014 SRJC Promo

sgfreepapers.com 103
25

From the graph of y = fg(x),


 1 
Rfg =  y ∈  : y < − or y > 0  .
 4 

IJC/2013/JC19740/01/Oct/13 *: Not in topics tested for 2014 SRJC Promo

sgfreepapers.com 104
2

1* Expand

(1 + 2 x ) 4 + 3x

in ascending powers of x , up to and including the term in x 2 . [3]

Determine the range of values of x for which the expansion is valid. [1]

2n − 1 A B
2 (i) Given that can be written in the form + , find the values of the
(n − 1)2 n 2 (n − 1)2 n2
constants A and B. [2]

N
2r − 1
(ii) Hence find ∑ (r − 1)2 r 2 . [3]
r =2

N
2r + 1
(iii) Using your answer in (ii), find ∑ r (r + 1) 2
2
. [2]
r =1

3 Machines A and B are used to cut metal bars of length 30m into pieces of decreasing lengths.

(i) The lengths of all the pieces cut by machine A form an arithmetic progression with
common difference d m. If the total length of the first 25 pieces cut is 25m and the length
of the 25th piece is 0.5m, find the value of d. [3]

(ii) The length of the first piece cut by machine B is 2m and the lengths of all the pieces cut
form a geometric progression. The 25th piece cut by machine B has length 0.5m. Find the
maximum number of pieces of metal bars cut. [4]

4 A sequence u1, u2 , u3 ,… is given by


4 + 2un
u1 = 1 and un +1 = for n ≥ 1 .
5
(i) Find the values of u2 and u3 . [2]

(ii) It is given that un → l as n → ∞ . Showing your working, find the exact value of l. [2]

(iii) For this value of l, use the method of mathematical induction to prove that
n −1
1 2 [4]
un = l −   for n ≥ 1 .
3 5 

*: Not in the topics tested in 2014 SRJC Promo

sgfreepapers.com 105
3

x 2 − 3x + 3
5 The curve C has equation y = .
1− x

(i) Find the equations of the asymptotes of C. [2]

(ii) Prove using an algebraic method, that y cannot lie between two certain values (to be
determined). [3]

(iii) Sketch the curve C clearly labeling all asymptotes, turning points and axial intercepts. [3]

6 The diagram shows the graph of y = f(x). It has a vertical asymptotes at x = 1 and x = ‒1. It has
a stationary point of inflexion at the origin.
x = ‒1 y x=1

x
O

Sketch on separate diagrams, the graphs of


(i) y = f (2 − x) , [3]
(ii) y = − f ( x) , [2]

(iii) y = f '( x) . [2]

*: Not in the topics tested in 2014 SRJC Promo


[Turn over

sgfreepapers.com 106
4

7 (a) Show that x 2 − 3 x + 5 is always positive and solve the inequality

x2 − 3x + 5
< 0. [4]
(4 − x)( x − 2)

( x + 2)2 − 3 x − 1 [2]
Hence find the solution for the inequality <0.
x(2 − x)

(b) A factory produces 3 brands of drinks, A, B and C. The total price of 1 litre of A, 1 litre
of B and 2 litres of C is $9. The total price of 1 litre of B and 1 litre of C is $3.50. The
total price of 2.5 litres of B and 2 litres of C is twice the price of 1 litre of A.
Write down and solve the equations to find the price of each litre of A, B and C. [4]

8 The functions f and g are defined by


f : x  3ln ( x 2 + 1) , 0 ≤ x ≤ 2,
g : x  e x + 1, x ≥ 0.

(i) Find f −1 ( x) , stating the domain of f −1 . [3]

(ii) Sketch the graphs of y = f ( x) and y = f −1 ( x ) on a single diagram. State the geometrical

relationship between the graphs and hence state the number of solutions to f ( x ) = f −1 ( x) . [4]

(iii) Show that gf exists, define it in a similar form and find its range. [4]

*: Not in the topics tested in 2014 SRJC Promo

sgfreepapers.com 107
5

9 (a)

A closed cylindrical can with base radius r and height h has a fixed volume V.
(i) Show that the total surface area of the can, A, is given by
2V [1]
A = 2π r 2 + .
r
(ii) Find h in terms of r when the minimum surface area is achieved. [4]

(b)

Wall
2m
y

A ladder of length 2 m, leaning against the wall, slips in such a way that x increases at
a rate of 0.02 ms −1 . Find the rate of decrease of y at the instant when x is 1 m. [4]

*: Not in the topics tested in 2014 SRJC Promo [Turn over

sgfreepapers.com 108
6

10 (a) The curve C is defined by


x = e 3t , y = t2 , where t ≥ 0 .
dy dy [3]
(i) Find in terms of t and determine the value of t for which is zero.
dx dx

(ii) Sketch the graph of C. [2]

(b) The equation of a curve C is x 2 − 2 xy + 2 y 2 = k , where k is a constant.

dy [3]
Find in terms of x and y.
dx

Given that C has two points for which the tangents are parallel to the line y = x , find
the range of values of k. [3]

Given that k = 4 , find the exact coordinates of each point on the curve C at which the
tangent is parallel to the y-axis. [4]

11* (a) Find


[3]
∫x e
2 x
(i) dx ,
π
(ii) ∫0
3
sin 2 2 x dx , leaving your answer in exact form. [3]

(b) Using the substitution u = 3 x − 1 , find

∫ (3x − 1)
9x [3]
2
dx .

(c) Given that x + 1 = A ( 2 x − 4 ) + B for all values of x , find the constants A and B.

Hence, find


x +1 [5]
2
dx .
x − 4 x + 13

[End of Paper]

*: Not in the topics tested in 2014 SRJC Promo

sgfreepapers.com 109
Jurong Junior College
2013 JC1 H2 Mathematics Promo Solutions
(Qn 1 and 11 are not in topics tested for 2014 SRJC Promo)
Qn Solution
1 (1 + 2 x ) 4 + 3x
1
 3x  2
= (1 + 2 x ) 2 1 + 
 4 
 3x 9 x2 
= 2 (1 + 2 x ) 1 + − + ... 
 8 128 
19 x 87 x 2
= 2+ + ...
4 64
Validity:
3x
<1
4
4 4
− <x<
3 3

Qn Solution
2 2n − 1 A B
(i) 2 2
= 2
+ 2
(n − 1) n (n − 1) n
An 2 + B(n − 1) 2
=
(n − 1) 2 n 2

2n − 1 = An 2 + B (n − 1)2
When n = 0, B = −1.
When n = 1, A = 1.

2n − 1 1 1
∴ 2 2
= 2
− 2
(n − 1) n (n − 1) n
N
2r − 1 N  1 1
(ii) ∑ (r − 1)2 r 2 = ∑  2
− 
r2 
r =2 r =2  (r − 1)
 1 1
=  2− 2
 1 2
1 1
+ 2− 2
2 3
1 1
+ 2− 2
3 4
+ ...
1 1  1
+ 2
− 2  = 1− 2
( N − 1) N  N

sgfreepapers.com 110
N N +1
2r + 1 2r − 1

r =1
2
r (r + 1) 2
= ∑ 2 2
r = 2 ( r − 1) r

1
= 1−
( N + 1)2

Qn Solution
3 (i) S 25 = 25

25
[ a + 0.5] = 25
2
⇒ a = 1.5

a + 24d = 0.5
1
Subst a = 1.5, d = − = 0.0417 (to 3 s.f)
24
(ii) GP a = 2

ar 24 = 0.5
2r 24 = 0.5
1
r 24 =
4
1
r = 24 =0.94387 (to 5 s.f)
4
S n ≤ 30
  1 n 
2 1 −  24  
  4 
  ≤ 30
 1
1 −  24 
 4
n
 1
1 −  24  ≤ 0.84195
 4
n
 1
 24  ≥ 0.15805
 4
ln 0.15805
n≤
1
ln 24
4
n ≤ 31.931
Therefore maximum number of pieces cut = 31.

sgfreepapers.com 111
Alternative Solution
S n ≤ 30
2 1 − ( 0.94387 ) 
n
  ≤ 30
1 − ( 0.94387 )
n
1 − ( 0.94387 ) ≤ 0.84195
( 0.94387 )n ≥ 0.15805
ln 0.15805
n≤
ln 0.94387
n ≤ 31.9

Therefore maximum number of pieces cut = 31.

Qn Solution
4 4 + 2(1) 6
(i) u2 = = = 1.2
5 5
6
4 + 2( )
u3 = 5 = 32 = 1.28
5 25
(ii) As n → ∞, un → l , un+1 → l .
4 + 2l
l=
5
4
l=
3
n −1
4 1 2
(iii) Let Pn be the statement un = −   for all n ≥ 1 .
3 3 5 
.
LHS of P1 = u1 = 1 (by defn)
1−1
RHS of P1 = 4 − 1  2  =
3
=1
3 3 5 3
∴ P1 is true.
k −1
4 1 2
Assume that Pk is true for some k ≥ 1 , ie uk = −  
3 3 5
k
4 1 2
We want to prove Pk +1 , ie uk +1 = −  
3 3 5 

LHS of Pk +1 = uk +1
4 + 2uk
=
5
4 2 4 1  2  
k −1

= +  −   
5 5  3 3  5  
3

sgfreepapers.com 112
k −1
12 8 1  2  2 
= + −   
15 15 3  5  5 
k
4 1 2
= −  
3 3 5 
= RHS of Pk +1
∴ Pk is true ⇒ Pk +1 is true.
∴ By Mathematical Induction, Pn is true for all n ≥ 1 .

5 i)
Asymptotes:
By Long Division,
x 2 − 3x + 3 1
y= = 2− x+
1− x 1− x
Asymptotes: x = 1, y = 2 − x
ii)
x 2 − 3x + 3
y=
1− x
y (1 − x ) = x 2 − 3 x + 3
x 2 + ( y − 3) x + 3 − y = 0
For no solutions, Discriminant < 0
2
( y − 3) − 4 (3 − y ) < 0
(y 2
− 6 y + 9 ) − (12 − 4 y ) < 0
y2 − 2 y − 3 < 0
( y − 3)( y + 1) < 0
∴−1 < y < 3
iii)
y

0 x

( 2, −1)
y = 2− x

x =1

sgfreepapers.com 113
6 i) y x=1 x=3

0 x
2

ii) x = ‒1 y x=1

x
0

iii) x = ‒1 y x=1

x
0

sgfreepapers.com 114
Qn Solution
7 2 2
2  3  3
(a) x − 3 x + 5 =  x −  −  −  + 5
 2  2
2
 3  11
=x−  +
 2 4
2
 3
Since  x −  ≥ 0 for all real values of x, ∴ x 2 − 3 x + 5
 2
is always positive.
x2 − 3x + 5
<0
(4 − x)( x − 2)

Since x 2 − 3 x + 5 is always positive, (4 − x)( x − 2) < 0

− + −

2 4

∴ x < 2 or x > 4 -------(1)

( x + 2)2 − 3 x − 1
<0
x(2 − x)
Replace x in eqn (1) with (x+2),
∴ x+2< 2 or x+2>4
⇒ x<0 or x>2

(b) Let the price of 1 litre of A, B and C be a, b and c


respectively.

Given that
a + b + 2c = 9
b + c = 3.50
2.5b + 2c = 2a ⇒ 2a − 2.5b − 2c = 0

Using GC, a = $4, b = $2, c = $1.50 .

sgfreepapers.com 115
Qn Solution
8 i)
y = 3ln ( x 2 + 1)
y
x = ± e 3 −1
y
3
x = e −1 since 0 ≤ x ≤ 2
x
∴ f −1 ( x ) = e 3 − 1, 0 ≤ x ≤ 3ln 5
ii) y

3ln5 y = f(x)

−1
2 y = f (x)

O 2 3ln5 x

They are reflections about y = x and


there are 2 solutions.
iii)
Rf = [ 0,3ln 5]
Dg = [ 0, ∞ )
Rf ⊆ Dg
∴ gf exists
3
gf ( x ) = ( x 2 + 1) + 1, 0≤ x≤2
Rgf = [ 2,126]

sgfreepapers.com 116
Qn Solution
9 (i) V = π r 2 h
(a) V
h= 2
πr

A = 2π r 2 + 2π rh
 V 
= 2π r 2 + 2π r  2 
πr 
2V
= 2π r 2 + (shown)
r
dA 2V
(ii) For min A, = 4π r − 2 = 0
dr r
3
4π r = 2V
1
 V 3
r = 
 2π 
d2 A 4V
2
= 4π + 3 > 0
dr r
Thus, A is minimum.
Substitute V = π r 2 h ,
1
 π r 2h 3
r = 
 2π 
r 2h
r3 =
2
h = 2r
(b) y = 22 − x 2
= 4 − x2
dy 1
= ( −2 x )
dx 2 4 − x 2
x
=−
4 − x2
dy dy dx
= ×
dt dx dt
x
=− × (0.02)
4 − x2
1
=− × (0.02)
4 − 12
= −0.011547
= −0.0115
∴ y decreases at a rate of 0.0115 ms-1.

sgfreepapers.com 117
Qn Solution
10(a) dx
(i) x = e 3t ⇒ = 3e3t
dt
d y
y = t2 ⇒ = 2t
dt
dy 2t
∴ = 3t
dx 3e
dy
When = 0,
dx
2t
=0
3e3t
t=0
(ii)

(b) x 2 − 2 xy + 2 y 2 = k … (1)
Differentiate throughout w.r.t. x.
 dy  dy
2x − 2  x + y  + 4 y =0
 dx  dx
dy y−x
=
dx 2 y − x
dy
For tangents which are parallel to the line y = x , = 1.
dx
y−x
=1
2y − x
y − x = 2y − x
y=0
Subst. y = 0 into (1):
x 2 − 2 x(0) + 2(0) 2 = k
x2 = k
Given that there are 2 tangents parallel to the line y = x ,
k >0

sgfreepapers.com 118
dy
For tangents which are parallel to the y-axis, is undefined.
dx
2y − x = 0
x = 2y
Subst. x = 2 y and k = 4 into (1):
(2 y )2 − 2(2 y ) y + 2 y 2 = 4
y=± 2
x = ±2 2
The coordinates are −2 2, − 2 ( ) and (2 )
2, 2 .

Qn Solution
11(a)
(i) ∫ x 2 e x d x = x 2 e x − 2 ∫ xe x d x

= x 2 e x − 2  xe x − ∫ e x dx 

= x 2 e x − 2  xe x − e x  + c
= ex ( x2 − 2 x + 2) + c
(ii) π
1 π3
∫ sin 2 x dx = ∫ 1 − cos 4 x dx
3 2
0 2 0
π
1 1 3
=  x − sin 4 x 
2 4 0
1 π 1 4π 
=  − sin
23 4 3 
1 π 3
=  + 
23 8 
(b) 9x u +1
∫ ( 3 x − 1) 2
dx = ∫ 2 du
u
1
=∫ + u −2 du
u
1
= ln u − + c
u
1
= ln 3 x − 1 − +c
3x − 1

10

sgfreepapers.com 119
(c) x + 1 = A ( 2 x − 4) + B
= 2 Ax − 4 A + B
By comparing coefficients,
1
2A =1⇒ A =
2
−4 A + B = 1 ⇒ B = 3

x +1
∫x 2
− 4 x + 13
dx

1
( 2x − 4) + 3
=∫22 dx
x − 4 x + 13
1 2x − 4 1
= ∫ 2 dx + 3∫ dx
2 x − 4 x + 13 ( x − 2) 2 + 32
1 1  x−2
= ln x 2 − 4 x + 13 + 3   tan −1  +c
2 3  3 
1  x−2
= ln ( x 2 − 4 x + 13) + tan −1  +c
2  3 

11

sgfreepapers.com 120
2013 MJC H2 MATH (9740) JC 1 PROMOTIONAL EXAM – MARKING SCHEME

Qn Solution
1 Inequalities
2 2
 1 1
x − x +7 =  x −  +7 − 
2

 2 2
2
 1  27
=x−  +
 2 4
2 2
 1  1  27
Since  x −  ≥ 0 for all real values of x, x−  + > 0 (shown).
 2  2 4

3 −1
> , x ≠ −1, x ≠ 2
( x − 2)
2
x +1
3 1
+ >0
( x − 2)
2
x +1
3 ( x + 1) + ( x 2 − 4 x + 4 )
>0
( x + 1)( x − 2 )
2

x2 − x + 7
>0
( x + 1)( x − 2 )
2

2
 1  27
> 0 and ( x − 2 ) > 0 for all x ∈  \ {2}
2
Since x 2 − x + 7 =  x −  +
 2 4
⇒ ( x + 1) > 0
∴ x > −1 , x ≠ 2

Alternatively

2
 1  27
Since x 2 − x + 7 =  x −  + > 0 for all real values of x,
 2 4
1
>0
( x + 1)( x − 2 )
2

− + +

−1 −2

∴ x > −1 , x ≠ 2

MJC/2013 JC1 Promotional Exam Marking Scheme/H2 Math (9740)/Math Dept Page 1 of 12

sgfreepapers.com 121
Qn Solution
2 Techniques of Differentiation
2 t −t 2
x = sin −1 (1 − t ) y=e
1
dx 1 dy 1
= ( −1) =e 2t −t 2
( 2t − t ) ( 2 − 2t )
2 −2

dt 1 − (1 − t ) dt 2
2

dx
=−
1 dy e
=
2 t −t 2
(1 − t )
dt 2t − t 2 dt 2t − t 2
dy
d y dt
∴ = =e 2t −t 2
( t − 1)
dx dx
dt

Qn Solution
3 SLE
(i)
At A, b + c = a + d .
At B, a + b + c = 48.
At C, a + c = 2b.
At D, d = b + 2a.
After simplifying,
− a + b + c − d = 0.
a + b + c = 48.
a − 2b + c = 0.
2a + b − d = 0.
Using GC, a = 8, b = 16, c = 24 and d = 32.
(ii) Total amount collected = $0.50 ( 2c + b )
= $0.50 ( 48 + 16 )
= $32

MJC/2013 JC1 Promotional Exam Marking Scheme/H2 Math (9740)/Math Dept Page 2 of 12

sgfreepapers.com 122
Qn Solution
4 Vectors I

(i) 2 Q 1 B
A
3
1
P
C
a
b

O

OC = kb
Using Ratio Theorem,
 a + 3kb
OP =
4
 a + 2b
OQ =
3

(ii) Given that O, P and Q are collinear,


 
OP = λ OQ for some λ ∈ 
1 3k 1 2 
a + b = λ  a + b
4 4 3 3 

Since a and b are non-zero and non-parallel vectors,


1 λ 3 2
= ------ (1) and k = λ ------ (2)
4 3 4 3

3
From (1): λ = ------ (3)
4

Substitute (3) into (2)

2  3  4 
k =   
3  4  3 
2
=
3

2
∴k =
3

Alternatively,
Given that O, P and Q are collinear,
 
OQ = λ OP for some λ ∈ 

MJC/2013 JC1 Promotional Exam Marking Scheme/H2 Math (9740)/Math Dept Page 3 of 12

sgfreepapers.com 123
1 2 1 3k 
a + b = λ  a + b
3 3 4 4 

Since a and b are non-zero and non-parallel vectors,


1 1 2  3k 
= λ   ------ (1) and = λ   ------ (2)
3 4 3  4 
4
From (1): λ = ------ (3)
3
Substitute (3) into (2)

2 4 
k=  
3  3λ 
2  4  3  2
=    =
3  3  4  3

2
∴k =
3

Qn Solution
5* Maclaurin’s Series and Binomial Theorem [Not in topics tested for SRJC 2014 Promo]
(i) e x sin 2 x
 ( 2 x ) + ... 
3
 x 2 x3
=  1 + x + + + ...   2 x −
 2! 3!   3! 

 x 2 x3  8x3 
=  1 + x + + + ...  2 x − + ... 
 2 6  6 
3
8x
= 2x − + 2 x 2 + x3 + ...
6
1
= 2 x + 2 x 2 − x 3 + ...
3
(ii) x
e sin 2 x −
1
= e x sin 2 x  ( 4 − x ) 2
4− x
1

 1  −1  x  2
=  2 x + 2 x 2 − x3 + ...  ( 4 ) 2 1 − 
 3   4
  1  3  
1    1  x   − 2  − 2   x  2 
=  2 x + 2 x 2 − x3 + ...   1 +  −  −  +    −
1
  + ... 
2 3    2  4  2!  4 
 
 
 x 3  x  
2
1 1
=  2 x + 2 x 2 − x3 + ...   1 + +   + ... 
2 3   8 8  16  
1 2x2 3x 3 x3 2 x 3 
=  2x + + 2 x2 + − + ... 
2 8 64 3 8 
9 x 2 7 x3
= x+ − + ..
8 384

MJC/2013 JC1 Promotional Exam Marking Scheme/H2 Math (9740)/Math Dept Page 4 of 12

sgfreepapers.com 124
Qn Solution
6 Graphing Techniques 1
(i) y

y = − 5x y = 5x
( 0, 2 )

O x

y2 = 5x2 + 4
( 0, −2 )

(ii) 5x2 + 4 = h2 1 − x2( )


(
y 2 = h2 1 − x2 )
y 2 + x 2 h2 = h 2
y2
2
+ x2 = 1
h
y

y = − 5x y = 5x2 + 4 y = 5x

( 0, 2 )
( 0, h )

( 0, −h )
−1 O 1 x
y2
x2 + =1
h2
( 0, −2 )

y2
Graph to be inserted is x 2 + =1.
h2
From the graphs, 0 < h < 2 .

Qn Solution
7 Application of Differentiation (Tangent/ Normal)

MJC/2013 JC1 Promotional Exam Marking Scheme/H2 Math (9740)/Math Dept Page 5 of 12

sgfreepapers.com 125
x2
y=
x −1
dy 2 x ( x − 1) − x
2
=
( x − 1)
2
dx
x2 − 2 x
=
( x − 1)
2

8
Since gradient of tangent at A is
9
x2 − 2x 8
=
( x − 1)
2
9
Using GC, x = 4 or x = −2
Since x2 < x1 , x coordinate at point B is x2 = −2
4
Sub x2 = −2 into C we have y2 = −
3
 4
∴ coordinates of B is  −2, − 
 3
9
Since gradient of normal at B is −
8

 4 9
y −  −  = − ( x − ( −2 ) ) − − − −(*)
 3 8
9 43
y=− x−
8 12

Qn Solution
8 Transformation of graphs
(a) x −1
y= 2
3x − 5
↓ 1. Replace y by − y
x −1
−y = 2
3x − 5
↓ 2. Replace y by y − 1
x −1
1− y =
3x 2 − 5
↓ 3. Replace x by 2x
2. 2 x − 1
1− y =
12 x 2 − 5

The transformations are in the following order:


1. Reflection in the x-axis.
2. Translation of 1 unit in the positive y-direction.
3. Scaling parallel to the x-axis by factor ½.
(or 3-1-2, 1-3-2, 1-3-2)

Alternatively,
The transformations are in the following order:
MJC/2013 JC1 Promotional Exam Marking Scheme/H2 Math (9740)/Math Dept Page 6 of 12

sgfreepapers.com 126
1. Translation of 1 unit in the negative y-direction.
2. Reflection in the x-axis.
3. Scaling parallel to the x-axis by factor ½.
(or 1-3-2)
(b)(i)
y

y = f '( x)

y=0
O (3, 0) x

(b)(ii) x=0
y

y2 = f ( x )

y= 2

(−6, 0)
 3 O 3  (6, 0) x
 − ,0  ,0
 2  2 
y=− 2

x=0

Qn Solution
9 Mathematical Induction (RR) and MOD
(i) 1
Let Pn be the statement un = 2 for n ∈+ .
2n
1
When n = 1, LHS = u1 =
2
1 1
RHS = 2
= = LHS
2 (1) 2
∴ P1 is true.

Assume Pk is true for some k ∈+ ,


1
i.e. uk = 2 -------------------- (*)
2k

MJC/2013 JC1 Promotional Exam Marking Scheme/H2 Math (9740)/Math Dept Page 7 of 12

sgfreepapers.com 127
1
To prove Pk+1 is also true, i.e. uk +1 = 2
.
2 ( k + 1)
2(k + 1) − 1
LHS = uk +1 = uk − 2
(from the recurrence relation)
2k 2 ( k + 1)
2k + 1
= uk −
2k 2
( k + 1)2
1 2k + 1
= 2
− from (*)
2k 2k 2
( k + 1)2
2
=
( k + 1) − 2k − 1
2
2k 2 ( k + 1)
k2
= 2
2k 2 ( k + 1)
1
= 2
= RHS
2 ( k + 1)

Thus Pk is true ⇒ Pk +1 is true .


Since P1 is true, and Pk is true ⇒ Pk +1 is true , by Mathematical Induction, Pn is true
for all n ∈+ .

(ii) N
2n + 1 N

∑ 2n = ∑ ( un − un +1 )
( n + 1)
2 2
n =1 n =1

= u1 − u2
+ u 2 − u3

+ u N − u N +1
= u1 − u N +1

1 1 1 1 
−= = 1 − 
2 2 ( N + 1)2 2  ( N + 1)2 
 
N +1
(iii) N
2n + 3 2n + 1

n = 0 2 ( n + 1) ( n + 2 )
2 2
=∑ 2
n =1 2 n ( n + 1)
2

1 1
= −
2 2 ( N + 2 )2

MJC/2013 JC1 Promotional Exam Marking Scheme/H2 Math (9740)/Math Dept Page 8 of 12

sgfreepapers.com 128
Qn Solution
10 Vectors
(i) 2 2
      
AB = OB − OA =  −1 ⇒ a direction vector for the line is  −1
1 1
   
 1 2
   
vector equation of the line AB : r = 1 + λ  −1 , λ ∈ 
 1  
  1

To determine whether point C lies on the line:


 1
 2 = 1 + 2λ ⇒ λ =
 2  1 2 2
      
Let  1  = 1 + λ  −1 . Then 1 = 1 − λ ⇒ λ = 0
 5  1   5 = 1 + λ ⇒ λ = 4
    1 

Since the values of λ are inconsistent, i.e. no value of λ satisfies all the equations,
hence shown that point C does not lie on the line AB .
(ii) Let N be the foot of the perpendicular from C to line AB
 1 2
   
line AB : r = 1 + λ  −1 , λ ∈ 
 1  
  1
 1 + 2λ 
  
Since N lies on line AB then ON =  1 − λ  for some λ ∈ .
 1+ λ 
 

 1 + 2λ   2   −1 + 2λ 
        
CN = ON − OC =  1 − λ  −  1  =  −λ 
 1 + λ   5   −4 + λ 
     
→ →  −1 + 2λ   2 
  
CN ⊥ line AB, CN ⋅ d = 0 ⇒  −λ  .  −1 = 0
 −4 + λ   1 
  
⇒ −2 + 4 λ + λ − 4 + λ = 0 ⇒ λ = 1

Therefore, the position vector of the foot of the perpendicular from point C to line AB.
 1 + 2 (1)   3 
    
ON =  1 − (1)  =  0 
 1 + (1)   2 
   
 
Since ON = OB , the angle ABC is 90 degrees.
(iii)
The position vector of C ' , the reflection of point C in the line AB

MJC/2013 JC1 Promotional Exam Marking Scheme/H2 Math (9740)/Math Dept Page 9 of 12

sgfreepapers.com 129
 
 OC + OC '
ON =
2
  
OC ' = 2ON − OC
3 2
   
= 2 0 −  1
2 5
   
4
 
=  −1 
 −1 
 

Qn Solution
11 AP GP
(a) Let T1, T3, T6, be the first, third and sixth term of an arithmetic series with first term a and
common difference d.

T1 = a, T3 = a + 2d , T6 = a + 5d
a + 5d a + 2d
=
a + 2d a
a ( a + 5d ) = ( a + 2d )
2

a 2 + 5ad = a 2 + 4ad + 4d 2
ad = 4d 2

Since d ≠ 0 ⇒ a = 4d
T a + 2d 6 d 3
Common ratio r = 3 = = =
T1 a 4d 2
Since r > 1 , the geometric progression is not convergent.

15
S15 = [ 2a + 14d ]
2
15
= [ 2(4d ) + 14d ]
2
= 165d
165
= a
4
(b) 9
a = 2, r =
10
a
S∞ =
1− r
2
=
9
1−
10
= 20

MJC/2013 JC1 Promotional Exam Marking Scheme/H2 Math (9740)/Math Dept Page 10 of 12

sgfreepapers.com 130
Sn ≥ 15
2   9 n 
1 −  ≥ 15
9    
1 −   10  
10
  9 n 
1 −    ≥ 0.75
  10  
n
9
  ≤ 0.25
 10 
n ≥ 13.158
The minimum number of days required is 14 days.

Qn Solution
12 Applications of Differentiation
(i) h
sin α = ∴ PQ = h cosec α
PQ
QR = k − PQ − RS
= k − 2 PQ
= k − 2h cosec α (shown)

h
A= ( QR + PS )
2
h h 
=  2QR + 2 
2 tan α 
= h ( k − 2h cosec α + h cot α )
= hk + h 2 (cot α − 2 cosec α ) (shown)

(ii)
A = hk + h 2 (cot α − 2 cosec α )
dA
= h 2 (−cosec 2 α + 2cosec α cot α )

= h 2cosec α (−cosec α + 2 cot α )

dA
When = 0, h 2cosec α (−cosec α + 2 cot α ) = 0

Since h 2 cosec α ≠ 0,
−cosec α + 2 cot α = 0
−1 + 2 cos α
=0
sin α
−1 + 2 cos α = 0
1
cos α =
2
π
α=
3

MJC/2013 JC1 Promotional Exam Marking Scheme/H2 Math (9740)/Math Dept Page 11 of 12

sgfreepapers.com 131
π 
− π π 
+

α   3  
3 3
dA

Alternatively
dA
= h 2 (−cosec 2 α + 2cosec α cot α )

d2 A
= h 2 (2cosec 2 α cot α − 2cosec3 α − 2cosec α cot 2 α )
dα 2
= 2h 2 cosec α (cosec α cot α − cosec 2 α − cot 2 α )

π
When α = ,
3
d2 A π π π π π
= 2h 2 cosec (cosec cot − cosec 2 − cot 2 )
dα 2
3 3 3 3 3
2  2  1   2   1  
2 2

= 2h 2   −  −  
3  3  3   3   3  
4 2 2 4 1
= h  − − <0
3  3 3 3
4 2
=− h <0
3
π
α= gives max A
3

π
When α =
3
Max A = hk + h 2 (cot α − 2 cosec α )
π π
= hk + h 2 (cot − 2 cosec )
3 3
 1  2 
= hk + h 2  −2  
 3  3 
= hk − 3h 2

MJC/2013 JC1 Promotional Exam Marking Scheme/H2 Math (9740)/Math Dept Page 12 of 12

sgfreepapers.com 132
MERIDIAN JUNIOR COLLEGE
JC1 Promotional Examination
Higher 2

___________________________________________________________________

H2 Mathematics 9740/01
Paper 1 08 October 2013

2 Hours 30 Minutes
Additional Materials: Writing paper
List of Formulae (MF 15)
___________________________________________________________________
READ THESE INSTRUCTIONS FIRST

Write your name and civics group on all the work you hand in.
Write in dark blue or black pen on both sides of the paper.
You may use a soft pencil for any diagrams or graphs.
Do not use staples, paper clips, highlighters, glue or correction fluid.

Answer all the questions.


Give non-exact numerical answers correct to 3 significant figures, or 1 decimal place in the case of
angles in degrees, unless a different level of accuracy is specified in the question.
You are expected to use a graphic calculator.
Unsupported answers from a graphic calculator are allowed unless a question specifically states
otherwise.
Where unsupported answers from a graphic calculator are not allowed in a question, you are
required to present the mathematical steps using mathematical notations and not calculator
commands.
You are reminded of the need for clear presentation in your answers.

At the end of the examination, fasten all your work securely together.
The number of marks is given in brackets [ ] at the end of each question or part question.

___________________________________________________________________
This document consists of 6 printed pages and 2 blank page

[Turn Over
MJC/2013 JC1 Promotional Examination/9740/01

sgfreepapers.com 133
2
BLANK PAGE

*: Not in topics tested for SRJC 2014 Promotional Exam


MJC/2013 JC1 Promotional Examination/9740/01

sgfreepapers.com 134
3

1 Show that x 2 − x + 7 is always positive for all real values of x. [1]


Hence, using an algebraic method, solve the inequality
3 1
2
>− . [3]
( x − 2) x +1

2t −t 2 dy
2 The parametric equations of a curve C are x = sin −1 (1 − t ), y = e . Find in terms of t.
dx
[4]

3 The diagram below shows the traffic flow of vehicles in four traffic junctions A, B, C and D.
Each arrow indicates the direction of the vehicles entering or leaving the junction. The
unknown constants a, b, c and d indicate the number of vehicles entering or leaving a particular
junction. It is given that the total number of vehicles entering a traffic junction must be equal to
the total number of vehicles leaving that same junction. There are 48 vehicles leaving junction
B.

A D

B C

(i) Determine the values of a , b, c and d . [3]

(ii) The shaded region indicates the presence of an Electronic Road Pricing (ERP) gantry
located at that road. It is known that each gantry charges a fixed price of $0.50 per
vehicle. How much revenue will be collected in total by the gantries in these regions?
[1]

[Turn Over
MJC/2013 JC1 Promotional Examination/9740/01

sgfreepapers.com 135
4

 
4 Referred to the origin O, the points A and B are such that OA = a and OB = b, where a and b
 
are non-zero and non-parallel vectors. The point C lies on OB such that OC = kOB , where k
is a constant. P is on AC such that AP : PC = 3 : 1, and Q is on AB such that AQ : AB = 2 : 3.

 
(i) Find OP and OQ in terms of a, b and k . [2]

(ii) Given that O, P and Q are collinear, find the value of k . [3]

5 (i)* Obtain the series expansion for e x sin 2 x , up to and including the term in x 3 . [3]

e x sin 2 x
(ii)* Hence deduce the first three non-zero terms in the series expansion of . [3]
4− x

6 The curve C has equation y 2 = 5 x 2 + 4 .

(i) Sketch C, indicating clearly the axial intercepts, the equations of the asymptotes and the
coordinates of the stationary points. [3]

(ii) Hence by inserting a suitable graph, determine the range of values of h, where h is a

( )
positive constant, such that the equation 5 x 2 + 4 = h2 1 − x 2 has no real roots. [3]

7 The curve C has equation


x2
y= .
x −1
Points A ( x1, y1 ) and B ( x2 , y2 ) lie on curve C such that the tangent at A is parallel to tangent at

8 16
B where x2 < x1 . Given further that the equation of tangent at A is y = x + , find the
9 9
coordinates of B, and hence find the equation of normal at point B. [6]

*: Not in topics tested for SRJC 2014 Promotional Exam


MJC/2013 JC1 Promotional Examination/9740/01

sgfreepapers.com 136
5

x −1
8 (a) State a sequence of transformations which transform the graph of y = to the
3x 2 − 5
2x − 1
graph of 1 − y = . [3]
12 x 2 − 5

(b) The diagram below shows the graph of y = f ( x ) .

x
O

Sketch, on separate clearly labeled diagrams, the graphs of

(i) y = f '( x ) , [2]

(ii) y2 = f ( x ) . [3]

MJC/2013 JC1 Promotional Examination/9740/01

sgfreepapers.com 137
6

1
9 A sequence u1 , u2 , u3 , is such that u1 = and
2
2n + 1
un+1 = un − , for all n ≥ 1.
2n 2
( n + 1)2

1
(i) Use the method of mathematical induction to prove that un = 2
for n ∈  + . [4]
2n

N
2n + 1
(ii) Hence find ∑ 2n ( n + 1)
n =1
2 2
. [3]

N
2n + 3
(iii) Use your answer to part (ii) to find ∑ 2 ( n + 1) ( n + 2)
n =0
2 2
. [2]

10 Referred to the origin O, the position vectors of two points A and B are given by i + j + k and
3 i + 2 k respectively. Also, the position vector of C is given by 2i + j + 5k .

(i) Find a vector equation of the line AB and show that point C does not lie on the line. [3]

(ii) Find the position vector of the foot of the perpendicular from point C to line AB.
Hence write down the size of angle ABC. [5]

(iii) Find the position vector of C ' , the reflection of point C in the line AB. [2]

*: Not in topics tested for SRJC 2014 Promotional Exam


MJC/2013 JC1 Promotional Examination/9740/01

sgfreepapers.com 138
7

11 (a) The first, third and sixth terms of an arithmetic progression with non-zero common
difference d and first term a, are three consecutive terms of a geometric progression.
Determine if the geometric series is convergent, justifying your answer. Find also the
sum of the first 15 terms of the arithmetic progression in terms of a. [5]

(b) A pile driver is used to drive piles into the soil at a new condominium site. On the first
day, the depth piled into the soil is 2 m. On each subsequent day, the depth piled into the
9
soil is of the depth piled into the soil on the previous day. Find the maximum
10
theoretical depth that can possibly be piled into the soil. Find the minimum number of
days required to drive the piles to a depth of at least 15m into the soil. [5]

12 A student wants to construct a model of a roof structure of fixed height h cm from a


rectangular piece of cardboard of width k cm. The cardboard is to be bent in such a way that
the cross-section PQRS is as shown in the diagram, with PQ + QR + RS = k and with PQ and
RS each inclined to the horizontal at an angle α .

Q R

h h

P S

(i) Show that QR = k − 2h cosec α and that the area A cm 2 of the cross-section PQRS is

given by A = hk + h2 (cot α − 2 cosec α ) . [3]

(ii) Use differentiation to find, in terms of k and h, the maximum value of A as α varies. [5]

MJC/2013 JC1 Promotional Examination/9740/01

sgfreepapers.com 139
8
BLANK PAGE

*: Not in topics tested for SRJC 2014 Promotional Exam


MJC/2013 JC1 Promotional Examination/9740/01

sgfreepapers.com 140
NANYANG JUNIOR COLLEGE
JC1 PROMOTIONAL EXAMINATION
Higher 2

MATHEMATICS 9740/01

Paper 1 1st October 2013

3 Hours

Additional Materials: Cover Sheet


Answer Papers
List of Formulae (MF15)

READ THESE INSTRUCTIONS FIRST

Write your name and class on all the work you hand in.
Write in dark blue or black pen on both sides of the paper.
You may use a soft pencil for any diagrams or graphs.
Do not use staples, paper clips, highlighters, glue or correction fluid.

Answer all the questions.


Give non-exact numerical answers correct to 3 significant figures, or 1 decimal place in the case of angles in
degrees, unless a different level of accuracy is specified in the question.
You are expected to use a graphic calculator.
Unsupported answers from a graphic calculator are allowed unless a question specifically states otherwise.
Where unsupported answers from a graphic calculator are not allowed in a question, you are required to
present the mathematical steps using mathematical notations and not calculator commands.
You are reminded of the need for clear presentation in your answers.

At the end of the examination, fasten all your work securely together.
The number of marks is given in brackets [ ] at the end of each question or part question.

This document consists of 7 printed pages.

NYJC 2013 JC1 Promotional Examination 9740/01 1


sgfreepapers.com 141
1 Solve the inequality x 2 − 2 x − 3 > x + 1 . [4]

2 Differentiate the following expressions with respect to x, simplifying your answers as far as possible:
2
(a) tan −1   , [3]
x

1+ x
(b) ln . [3]
1− x

1 1
3 A sequence u1 , u2 , u3 , ... is such that u1 = and un+1 = un + + 2− n , for n ∈  + .
4 n ( n + 1)

9 1 − n+1
(i) Prove by mathematical induction that un = − −2 for n ∈  + . [5]
4 n
(ii) Explain why {un } is convergent. [1]

9
(iii) Show that u n is less than for n ∈  + . [1]
4

4 Show that r !( r 2 + 1) = ( r + 2)! − 3( r + 1)! + 2r ! where r ∈  + . [1]


Hence, using method of difference, show that the sum of the first n terms of the series

( 5)( 2!) + (10)( 3!) + (17 )( 4!) + is ( n + 2)!( n + 1) − 2 . [4]


n
Using the above result, explain why ∑ r !(r
r =1
2
) is less than ( n + 1) !n . [2]

NYJC 2013 JC1 Promotional Examination 9740/01 2

sgfreepapers.com 142
5 (a) The points A and B relative to the origin O have position vectors i + 2 j − 2k and − 4i + 5 j + 2k
AP λ
respectively. The point P lies on line AB such that = .
PB 1 − λ

(i) Show that OP = (1 − 5λ ) i + ( 2 + 3λ ) j + ( 4λ − 2 ) k . [1]

(ii) Given further that C is a point with position vector −5i + α j − 2k and that O, P and C
are collinear, find the values of λ and α . [3]

(b) The equations of three planes π 1 , π 2 , π 3 are


π 1 : 2 x − 2 y + z = −4 ,
π 2 : 2x + 3 y − 4z = 1 ,
π3 : β x − 3y + z = γ ,
respectively.

(i) The planes π 1 and π 2 intersect in a line l. Find a vector equation of l. [1]
(ii) Hence, find the values of β and γ such that there are infinitely many points of
intersection between π 1 , π 2 and π 3 . [2]

y2
6 The curve C 1 has equation x 2 − = 1 . The curve C 2 has parametric equations
4

x = a sin t , y = a cos t , where 0 ≤ t ≤ 2π and a > 0 .

(i) Write down the Cartesian equation of C 2 . Sketch C1 and C 2 on the same diagram, stating the
exact coordinates of any points of intersection with the axes and the equations of any
asymptotes. [5]

(ii) State the range of values of a such that there are 4 points of intersection between C1 and C 2 .
Show algebraically, that the x-coordinates of the points of intersection satisfy the equation
5x2 = 4 + a2 . [2]

(iii) Explain geometrically why there are only 2 values for the x-coordinates when there are 4 points
of intersection between C1 and C 2 . Find the exact values of x if a = 3 . [2]

[Turn Over
NYJC 2013 JC1 Promotional Examination 9740/01 3

sgfreepapers.com 143
7 The function f is defined by
1
f : x → x 2 − , x ∈ , 1 ≤ x < 2 .
x
(i) Show, by differentiation, that f is strictly increasing. [2]
(ii) State the range of f. [1]
Solve the equation f ( x ) = f ( x ) , giving your answer to two decimal places.
−1
(iii) [2]

The function g is defined by


π
g: x → 1 + sin x, x ∈ , 0 ≤ x < .
2
(iv) Only one of the composite functions fg and gf exists. Give a definition (including the domain)
of the composite that exists, and explain why the other composite does not exist. [3]
(v) For the composite function which exists, state its range. [1]

8 The equation of a curve is


2
y ( x + 2 ) + 2 y 2 ( x + 2 ) − 12 x = 0 , where x and y are positive variables.

dy 1
(i) Show that the value of is when x = 2 . [5]
dx 16
(ii) Find the equation of the normal to the curve at the point where x = 2 . [2]
(iii) Given that the normal in (ii) meets the line x = 2 at the point P and the line x = 0 at the
point S. Find the exact area of triangle OSP, where O is the origin. [2]

NYJC 2013 JC1 Promotional Examination 9740/01 4

sgfreepapers.com 144
9 There are 16 boys and 10 girls in a JC1 class. It so happens that within the class, the heights of all the
girls form a geometric progression, while the heights of all the boys form an arithmetic progression.
The two shortest students in the class, a boy and a girl, both have a height of 150.0 cm, while the
tallest boy in the class has a height of 180.0 cm. The fourth shortest girl in the class has a height of
157.5 cm.
1
(i) Show that the common ratio r between the heights of the girls is 1.05 3 and find the height of
the tallest girl in the class, giving your answer in cm correct to one decimal place. [2]
(ii) Find the number of girls in the class taller than 164.0 cm. [3]
(iii) Find the average height of the girls in the class, giving your answer in cm correct to one
decimal place. [3]
(iv) Find the average height of the entire class, giving your answer in cm correct to one decimal
place. [2]

10 The position vectors of the points A, B and C with respect to the origin O are a, b and a − 2b
respectively. Plane π contains the point A and has b as its normal vector. If the angle between
vectors a and b is 60o and a = 2 b , find in terms of b,
(i) the length of projection of a onto b , [2]
(ii) the distance between point C and the plane π . [3]

Given that a = i + 5j + 2k and b = i + 2j − k ,


(iii) find the position vector of the foot of perpendicular from point C to the plane π , [5]
(iv) show that the position vector of the point of the reflection of point C in the plane π is
3i + 9 j . [2]

[Turn Over
NYJC 2013 JC1 Promotional Examination 9740/01 5

sgfreepapers.com 145
11 The graphs of y = f ' ( x ) and y 2 = f ( x ) are shown in the diagrams below.

y y

y2 = f(x)

x x
–1 0 1 –1 0 1 2

y=
y= f 'f( x’(x)
) –2

(a) On separate diagrams, sketch the graphs of


(i) y = f ' (1 − x ) , [3]

(ii) y = f ( x) , [4]

showing clearly the x-intercepts and asymptotes (if any).

(b) State the set of values of x for which the graph of y = f ( x ) is concave upwards. [2]

NYJC 2013 JC1 Promotional Examination 9740/01 6

sgfreepapers.com 146
12 (a) The curve C has parametric equations
2
x = θ 2 + 4θ , y= , for θ > 0 .
θ
A point P(x, y) moves on the curve C in such a way that the x-coordinate of P decreases at a
constant rate of 4 units per second. Find the rate at which the y-coordinate of P is changing
when x = 4 . [4]

(b)
A B

D 4 C

The diagram above shows the floor plan of a storeroom. The floor plan consists of a square
ABCD of side 4 units from which a quadrant of a circle with centre A and radius 3 units has
been removed. The owner intends to store a rectangular crate with one corner of the base at C,
and the opposite corner of the base at P against the curved wall. The base of the crate has area
π
y unit2 and angle DAP is θ radians, where 0 ≤ θ ≤ .
4

dy
Show that = 3 ( sin θ − cos θ )( 4 − 3sin θ − 3cos θ ) . [2]

Hence, find the least possible value of y. [5]

-----END OF PAPER-----
[Turn Over
NYJC 2013 JC1 Promotional Examination 9740/01 7

sgfreepapers.com 147
2013 NYJC JC1 Promo 9740/1 Solutions

Qn
1
y y=x+1

y = ( x + 1)( x − 3)

O x
–1 2 3 4

x < −1 or − 1 < x < 2 or x > 4 .

2 d −2
 2  2( − x ) −2
(a) [tan −1  ] = = 2
dx  x  1 + ( 2 )2 x + 4
x

d 1+ x d 1
(b) (ln ) = [ ( ln(1 + x ) − ln(1 − x ) )]
dx 1− x dx 2
1 1 −1 1 1
= ( − )= 2
or
2 1+ x 1− x 1− x (1 + x )(1 − x )
Alternative Solution
 
d  1+ x   1  1 1 − x  1 − x + 1 + x  
 In  =   .  
dx  1− x   1+ x  2 1 + x  (1 − x )2  
   
 1− x 
1  1− x  2 
=   
2  1 + x   (1 − x )2 
 
1
=
(1 − x ) (1 + x)

3 9 1
(i) Let Pn denote the proposition un = − − 2 − n +1 for all n ∈ Z + .
4 n
1
For n = 1 , LHS = u1 =
4
9 1 −1+1 9 1
RHS = − − 2 = − 1 − 1 = = LHS.
4 1 4 4
∴ P1 is true.
Page 1 of 10

sgfreepapers.com 148
2013 NYJC JC1 Promo 9740/1 Solutions
Qn
9 1
Assume that Pk is true for some k ∈ Z + , i.e. , uk =− − 2− k +1 .
4 k
9 1
−2 ( )
− k +1 +1
To prove that that Pk+1 is true, i.e., uk +1 = −
4 k +1
For n = k + 1,
1
LHS = uk +1 = uk + + 2− k
k ( k + 1)
9 1 1
= − − 2− k +1 + + 2− k
4 k k ( k + 1)
9 1 1 
= −  −  − 2
4  k k ( k + 1) 
−k
( ) ( 2 − 1)
9 k + 1−1
= − − 2− k −1+1
4 k ( k + 1)
9 1
−2 ( )
− k +1 +1
−=
4 k +1
Hence Pk+1 is true
Since P1 is true and Pk is true ⇒ Pk+1 is true, hence by Mathematical Induction, Pn is true for
all n ∈ Z + .

1 9
(ii) As n → ∞ , → 0 , 2− n → 0 , hence un → , i.e. {un } is convergent
n 4

1 9 1 9
(iii) Since > 0 , 2− n > 0 for n ≥ 1 , un = − − 2− n+1 <
n 4 n 4

4 (r + 2)!− 3(r + 1)!+ 2r ! = r !( (r + 2)(r + 1) − 3(r + 1) + 2 )


= r !(r 2 + 3r + 2 − 3r − 3 + 2)
( )
= r ! r 2 + 1 (Shown)

n +1 n +1

∑ r !(r
r =2
2
+ 1) = ∑ [ (r + 2)!− 3(r + 1)!+ 2r !]
r =2
= 4!− 3(3!) + 2(2!)
+ 5!− 3(4!) + 2(3!)
+ 6!− 3(5!) + 2(4 !)

+ (n + 1) !− 3(n)!+ 2(n − 1)!
+ (n + 2) !− 3(n + 1)!+ 2(n)!
Page 2 of 10

sgfreepapers.com 149
2013 NYJC JC1 Promo 9740/1 Solutions
Qn
+ (n + 3) !− 3(n + 2)!+ 2(n + 1)!
= (n + 3)!− 2(n + 2)!− 3!+ 2(2!)
= (n + 2)!(n + 3 − 2) − 2
= (n + 2)!(n + 1) − 2 (Shown)

∑ r !(r
r =1
2
+ 1) = (n + 1)!(n) − 2 + (1!)(11 + 1) = (n + 1)!n

Since r !(r 2 ) < r !(r 2 + 1) for r ∈  +


n n
Therefore ∑ r !(r 2 ) < ∑ r !(r 2 + 1) = (n + 1)!n
r =1 r =1

 
5a  (1 − λ ) OA + λ OB
OP =
1− λ + λ
= (1 − λ )( i + 2 j − 2k ) + λ ( −4i + 5 j + 2k )
(1 − 5λ ) i + ( 2 + 3λ ) j + ( 4λ − 2 ) k
 
OP = µ OC
 1 − 5λ   −5 
   
 2 + 3λ  = µ  α 
 4λ − 2   −2 
   

2 1
Solving, λ = , µ= , α = 16
5 5

b π 1 : 2 x − 2 y + z = −4 ,
π 2 : 2x + 3 y − 4z = 1 ,
π3 : β x − 3y + z = γ .

 −1  1
Line of intersection of π 1 and π 2 , l: r =  1  + λ  2  , λ ∈  .
 
0  2
   

For infinite points of intersection between 3 planes, l is on π 3 .

Page 3 of 10

sgfreepapers.com 150
2013 NYJC JC1 Promo 9740/1 Solutions
Qn
1  β 
  
 2 i −3  = 0 ⇒β =4
 2  1 
  

 −1   β 
  
 1 i −3  = γ ⇒ γ = −7
01
  

6(i) x = a sin t , y = a cos t


x y
sin t = , cos t =
a a

sin 2 t + cos 2 t = 1
2 2
x  y
  +  =1
a a

x2 + y 2 = a2

y = −2 x y = 2x
y

C1 C2

(-a,0) (-1,0) (1,0)

x
( a, 0 )

(ii) a >1
y2
x2 − = 1 .........(1)
4

x2 + y2 = a2
y 2 = a 2 − x 2 .......(2)

Page 4 of 10

sgfreepapers.com 151
2013 NYJC JC1 Promo 9740/1 Solutions
Qn
 a2 − x2 
x2 −   =1
 4 
4x2 − a2 + x2 = 4

(iii) 5x2 = 4 + a2 (shown)


The points of intersection between the 2 curves are symmetrical about the x-axis, thus there are
only 2 values for the x-coordinates.

5 x 2 = 13
13
x=±
5

7(i) 1
f ′ ( x) = 2x + > 0 for 1 ≤ x < 2 ⇒ f is strictly increasing.
x2

(ii) Since f is strictly increasing, its minimum and maximum values correspond to the minimum and
maximum x values. Thus
 1  7
Rf = 1 − 1, 4 −  =  0,  .
 2  2

(iii) f ( x ) = f −1 ( x ) ⇒ f ( x ) = x
1
⇒ x2 − = x
x
⇒ x − x2 − 1 = 0
3

⇒ x = 1.47.

(iv) Since Rg = [1, 2 ) = Df, fg exists.


 7  π
Since Rf = 0,  ⊄ 0,  = Dg, gf does not exist.
 2  2

2 1
fg(x) = f( sin x + 1 ) = ( sin x + 1) − .
sin x + 1
 π
Dfg = Dg = 0,  .
 2

2 1 π
(v) fg : x → ( sin x + 1) − , x ∈ , 0 ≤ x < .
sin x + 1 2
 7
Rfg = 0,  .
 2

Page 5 of 10

sgfreepapers.com 152
2013 NYJC JC1 Promo 9740/1 Solutions
Qn
8(i) ( x + 2)
2
y + 2 ( x + 2 ) y 2 − 12 x = 0
Differentiating wrt x,
dy 2 dy
( x + 2 ) + 2 y ( x + 2 ) + 4 y ( x + 2 ) + 2 y 2 − 12 = 0 -------(1)
dx dx

When x = 2 , 16 y + 8 y 2 − 24 = 0
y2 + 2 y − 3 = 0
( y + 3)( y − 1) = 0
y = −3(rejected ∵ y > 0) or y =1

Subst (2 , 1) into equation (1),


dy dy
16 + 8 + 16 + 2 − 12 = 0
dx dx
dy
32 =2
dx
dy 1
=
dx 16

(ii) Equation of normal: y − 1 = −16 ( x − 2 )


y = −16 x + 33

(iii) Points P and S has coordinates (2 , 1) and (0 , 33) respectively


1
Area of triangle OSP = × 33 × 2 = 33
2

9(i) Let un denote the height of the nth shortest girl in the class in cm, and r denote the common ratio
between the heights of the girls.
Then un = ar n −1 where u1 = a = 150.0 and u4 = ar 3 = 157.5
1
157.5
⇒r = 3
= 1.05 ⇒ r = 1.05 3
150.0
Also, u10 = ar 9 = a (r 3 )3 = (150.0)(1.05)3 = 173.6 (to 1 d.p.)
∴ The height of the tallest girl is 173.6 cm.

ii un > 164.0
n −1
⇒ (150.0)(1.05) 3 − 164.0 > 0
Using GC,
n n−1
(150.0)(1.05) 3 − 164.0
6 -1.29
7 1.38
8 4.09

Page 6 of 10

sgfreepapers.com 153
2013 NYJC JC1 Promo 9740/1 Solutions
Qn
Hence n ≥ 7 .
Since there are 10 girls in the class, the number of girls who are taller than 164.0 cm is 10-7+1=4.
Thus there are 4 girls in the class taller than 164.0 cm.

iii Average height of girls


1 1 a (1 − r10 )
= S10 =
10 10 1 − r
10
(150.0)(1 − 1.05 3 )
= 1
10(1 − 1.05 3 )
= 161.57
= 161.6 cm (to 1 d.p.)

Average height of boys


iv 1
= S16
16
1 16
= × (150.0 + 180.0)
16 2
= 165.0 cm
Average height of class
16(165.0) + 10(161.57)
=
16 + 10
= 163.7 cm (to 1 d.p.)
10(i) length of projection = aib = a b cos 60°

1
=2b = b
2

(ii) distance between C and the plane =


( a − 2b − a ) ⋅ (b) =
−2b ⋅ b
b b

2
−2 b
=
b
=2b

 1   1   −1 
(iii)      
c =  5  − 2 2  =  1 
 2   −1   4 
     

Page 7 of 10

sgfreepapers.com 154
2013 NYJC JC1 Promo 9740/1 Solutions
Qn
 1  1  1 
π : r ⋅  2  =  5  ⋅  2  = 9,
 −1   2   −1
     

 −1 1
   
l :r =  1 +λ 2 
4  −1 
   

 −1 + λ   1 
   
 1 + 2λ  ⋅  2  = 9
 4 − λ   −1 
   
− 1 + λ + 2 + 4λ − 4 + λ = 9 ⇒ λ = 2

 −1   1   1 
     
(iv) position vector of the foot of perpendicular from c to plane =  1  + 2  2  =  5 
 4   −1   2 
     

 1   −1  3 
     
position vector of point of reflection of C in plane = 2  5  −  1  =  9 
 2  4  0
     

11a

x= 0 x=2

Page 8 of 10

sgfreepapers.com 155
2013 NYJC JC1 Promo 9740/1 Solutions
Qn
y

y = f(x)
4

–1 0 1 x
2

b (−∞, −1) ∪ ( −1,1)

2
12a x = θ 2 + 4θ , y=
θ
dy dy dx
= ⋅
dt dx dt
 dy dθ  dx
= ⋅ ⋅
 dθ dx  dt
−2 1
= 2⋅ ⋅ ( −4 )
θ 2θ + 4
4
= 2
θ (θ + 2 )
When x = 4 , θ 2 + 4θ = 4 ⇒ θ = 0.82843 since θ > 0
dy 4
= 2 = 2.06 units/sec
dt θ (θ + 2 )

Rate of change of y-coordinate is 2.06 units/sec.

Page 9 of 10

sgfreepapers.com 156
2013 NYJC JC1 Promo 9740/1 Solutions
Qn
12b y = ( 4 − 3cosθ ) ( 4 − 3sinθ )

dy
= ( 4 − 3sin θ )( 3sin θ ) + ( 4 − 3cos θ )( −3cos θ )

= 3  4sin θ − 3sin 2 θ − 4 cos θ + 3cos 2 θ 

= 3 3 ( cos 2 θ − sin 2 θ ) + 4sin θ − 4 cos θ 


= 3 3 ( cos θ − sin θ )( cos θ + sin θ ) + 4 ( sin θ − cos θ ) 
= 3 ( sin θ − cos θ )( 4 − 3sin θ − 3cos θ )

dy
=0

3 ( sin θ − cos θ )( 4 − 3sin θ − 3cos θ ) = 0
sin θ − cos θ = 0 or 4 − 3sin θ − 3cos θ = 0
4
sin θ + cos θ =
π 3
θ= or
4 π
θ = 0.44556 θ =1.1252 (rej 0 ≤ θ ≤ )
4
2
d y
= 3 ( sin θ − cos θ )( 3sin θ − 3cos θ ) + 3 ( sin θ + cos θ )( 4 − 3sin θ − 3cos θ )
dθ 2
π d2y
When θ = , < 0 ⇒ y is max
4 dθ 2
d2y
When θ = 0.44556 , > 0 ⇒ y is min
dθ 2

Min y = ( 4 − 3cos 0.44556 )( 4 − 3sin 0.44556 ) =3.50

Page 10 of 10

sgfreepapers.com 157
1

Name ( ) Class

RIVER VALLEY HIGH SCHOOL


2013 Year 5 Promotional Examination
Higher 2

MATHEMATICS 9740/01
Paper 1 19 September 2013
3 hours
Additional Materials: Answer Paper
List of Formulae (MF15)
Cover Page

READ THESE INSTRUCTIONS FIRST

Do not open this booklet until you are told to do so.


Write your name, class and index number in the space at the top of this page.

Write your name and class on all the work you hand in.
Write in dark blue or black pen on both sides of the paper.
You may use a soft pencil for any diagrams or graphs.
Do not use staples, paper clips, highlighters, glue or correction fluid.

Answer all the questions.


Give non-exact numerical answers correct to 3 significant figures, or 1 decimal place in the
case of angles in degrees, unless a different level of accuracy is specified in the question.
You are expected to use a graphic calculator.
Where unsupported answers from a graphic calculator are not allowed in a question, you are
required to present the mathematical steps using mathematical notations and not calculator
commands.
You are reminded of the need for clear presentation in your answers.
Up to 2 marks may be deducted for poor presentation in your answers.

At the end of the examination, place the cover page on top of your answer paper and fasten
all your work securely together.

The number of marks is given in brackets [ ] at the end of each question or part question.

This document consists of 7 printed pages and 1 blank page.

©RIVER VALLEY HIGH SCHOOL 9740/01/2013

sgfreepapers.com 158
2

1
Let f (xx)   x  3
 9  4x 

1. (i) 2 . Find thee series exp
pansion off f ( x ) in asscending
m in x 2 .
powerss of x, up to and includiing the term [3]
9 9
(ii) Denotee the answer to part (i)) by g( x ) . Find,
F for   x  , tthe set of values
v of
4 4
x for which
w the vaalue of g( x ) is within 0.2 of f (xx ) . [2]

2. Thhe graphs oof y 2  f ( x) and y  f  x  are giv


ven below.

D
Deduce the graphs
g of
(i) y  f ( x) , [3]
(ii) y  f '(( x) , [2]
cllearly indicaating any assymptotes, iintersection
ns with the axes
a and staationary poin
nts.

©RIVER
R VALLEY HIGH
H SCHOO
OL 9740/01/2
2013

sgfreepapers.com 159
3

3. The diagram shows the sketch of the curve C, ( y  1) 2  x x 2  1 , with the vertex at
1,1 .

(i) Write down the equation of the graph when C is translated 1 unit in the negative
y-direction. [1]
(ii) The shaded region R, bounded by C and the vertical line, x  a , is rotated
through π radians about the line y  1 . By using the substitution u  x 2  1 , or
otherwise, find the exact volume obtained in terms of a . [5]

4. (a) A theme park sells day passes at different prices depending on the age of the
customer. The age categories are senior citizens (ages 60 and above), adult (ages
13 to 59) and child (ages 4 to 12). Three tour groups visited the theme park on the
same day. The numbers in each category for each group together with the total
cost of the day passes for each group are given as follows.

Group Senior Citizens Adult Child Total Cost


1 2 19 9 $196.40
2 0 10 3 $90.20
3 1 7 4 $77.00

Write down and solve equations to find the cost of a day pass for each of the
age category. [3]

4 x2  4 x  1
(b) Without using a GC, solve  0. [4]
x2  2 x  8

©RIVER VALLEY HIGH SCHOOL 9740/01/2013

sgfreepapers.com 160
4

5. The cross section of an open container consists of a semicircle, a rectangle ABCD and
an isosceles triangle CED as shown in the diagram below. Given that AD  BC  x cm ,
5
AB  DC  FE  y cm , DE  CE and the height of the container is cm .
3
A x cm D

y cm F E

B C

The interior vertical walls of this container, ADECB, need to be painted. The time
needed to paint the walls will be 1 minute per 10 cm2 for the straight parts and 1 minute
per 8 cm2 for the semicircular part. Given that a total time of 200 minutes is required to
paint all the walls, find, by differentiation, the values of x and y which gives a
maximum cross-sectional area, giving your answers correct to the nearest integers. [7]

π π
6. It is given that the curve y3  tan 1 y  ln(cos x) , where   x  , passes through the
2 2
origin.
dy
(i) Show that  3 y 4  3 y 2  1
  1  y 2  tan x . [2]
dx
(ii) Find the Maclaurin series for y , up to and including the term x 2 . [3]
π
dy

4
(iii) Hence, find an approximation to the value of dx , in terms of  . [2]
0 dx

7. In a particular river in Brazil, a sudden surge in the number of piranhas (a type of fish
known for their sharp teeth and a voracious appetite for meat) is observed and has
affected the livelihood of the villagers living along the river. A group of fishermen is
engaged to catch these piranhas and the piranhas are caught at a rate inversely
proportional to the number of piranhas left. Furthermore, due to aggressive nature, the
number of piranhas is reduced at a rate of one-tenth of the piranhas remaining.

(i) If x (in thousands) is the number of piranhas remaining at time t (in days) after the
group of fishermen is deployed to catch the piranhas, show that x 2  10k  Ae 0.2t
where k is a positive constant. [4]
(ii) If there are 5000 piranhas at the start of the deployment of the fishermen and after
5 days, the number of piranhas remaining is 3000. Calculate the number of days
required to remove all the piranhas. [3]

©RIVER VALLEY HIGH SCHOOL 9740/01/2013

sgfreepapers.com 161
5

8. (a) Five out of the six digits, 0, 1, 2, 3, 4 and 5 are chosen and arranged randomly to
form a five-digit number. No digit is repeated.

Find the number of five-digit numbers that are


(i) greater than 10000, [2]
(ii) greater than 10000 and even. [3]

(b) An ice-cream shop has 4 different flavours of ice-cream, vanilla, chocolate,


strawberry and durian and 3 different toppings containing peanuts, raisins and
berries. Assuming Peter decides to visit the ice-cream shop and make a selection
of at least 1 flavour and at least 1 topping, find how many different selections can
he make? [3]

9. (a) The function f and g are defined by

f : x  x2  6 x  11, x 3
1
g:x  , x  k , where k is a positive constant.
x2

(i) Show that the inverse function of f exists. [1]


(ii) Find f 1  x  and state the domain of f 1 . [3]
(iii) State the greatest value of k for which the composite function gf exists and
find the range of gf for this value of k. [3]

(b) Given that h is a one-one function, determine, with reasons, if hh 1 exists. [2]

10. (a) The sum, Sn of the first n terms of a sequence u1 , u2 , u3 ,  is given by


1
Sn  ln a nb 2
 n n
2

, where 0  a  1, b  1 .
(i) Find un in terms of a and b . [2]
(ii) Prove that the sequence is an arithmetic progression. [2]
(iii) Given that 0  ab  1 when n  7 , find the sum of the negative terms of
n 1

the sequence. [1]


1 
(b) By considering sin  n  sin    , show, using the method of differences,
2 
 1 
cos  N    
N
1 1   2 
 sin  n   cot    
2 2  1 
. [4]
n 1
2sin   
2 

©RIVER VALLEY HIGH SCHOOL 9740/01/2013

sgfreepapers.com 162
6

11. (a) A and B are events such that P  B   0.3 , P  A '  B '  0.9 and
P  A  B '  0.45 .
(i) P  A , [2]
(ii) P  A ' B  . [2]

(b) In a cooking school, all students must take a theory and practical test. It is
reported that 95% of the students pass the theory test. Of those who pass, 85%
also pass the practical test. Of those who fail the theory test, 60% pass the
practical test.

Draw a tree diagram to show the above information. [2]

Find the probability that a student, randomly chosen from the cooking school,
(i) passes the practical test, [1]
(ii) passes the theory test, given that he fails the practical test. [2]

12. A curve C has parametric equations


x  et , y  t 2 .
(i) Sketch the curve C. [2]

The normal to C at point A with coordinates  e 2 , 4  is denoted by l .


(ii) Find the Cartesian equation of l , expressing y in terms of x. [3]
(iii) Find the exact area of the region bounded by l , C and the x-axis. Express your
a
answer in the form 2  be 2  c where a, b and c are constants to be determined.
e
[5]

©RIVER VALLEY HIGH SCHOOL 9740/01/2013

sgfreepapers.com 163
7

13. It is thought that the pH value of water may affect the size of pearl in pearl oyster
farming. A pearl farmer wished to investigate whether there was any correlation
between the pH value of the water and the size of the pearl cultivated. The size of the
pearls and the pH value of the water where the oysters are cultivated are shown in the
table below.

pH value of
7.7 7.8 7.9 8.0 8.1 8.2 8.3
water, x
Size of
pearl, y (in 6.82 7.28 7.61 7.79 7.91 8.02 8.05
cm)

(i) Draw a scatter diagram to illustrate the data, labeling the axes clearly. [2]
(ii) Comment on whether a linear model would be appropriate. [1]

It is thought that the size of pearl can be modeled by one of the formulae
y  a  bx 2 or y  c  dx
2

where a, b, c and d are constants.

(iii) Find, correct to 4 decimal places, the value of the product moment correlation
coefficient between
(a) x 2 and y,
(b) x and y 2 . [2]
(iv) Use your answer to parts (i) and (iii) to explain which of y  a  bx 2
or
y 2  c  dx is the better model. [2]
(v) The pearl farmer will like to have pearls which are exactly 8.00 cm. Find the
equation of a suitable regression line, and use it to find the required pH value of
the water, correct to 1 decimal place. Comment on the reliability of your answer.
[4]

END OF PAPER

©RIVER VALLEY HIGH SCHOOL 9740/01/2013

sgfreepapers.com 164
8

Blank Page

©RIVER VALLEY HIGH SCHOOL 9740/01/2013

sgfreepapers.com 165
2013 Y
Year 5 H22 Maths Promotion
P nal Exam
mination Marking
M S
Scheme

1(i) 1
f ( x )   x  3 9  4 x 

2

1
1 
 4 
 2
  x  3 9 1  x 
2

 9 
  1  1  3  
         2 
1 2   4   2  2   4 
  x  3 1     x     x   
3  1  9  2  9  
 
1  2 2 2 
  x  3 1  x  x  
3  9 27 
5 4 2
 1 x  x
9 27
(ii) 0.2  f ( x)  g( x)  0.22 or f  x   g  x   0.2
2
y

0
0.2
f g
x

0.2

U
Using GC,
 x  R, 1.877  x  1.25

2(i)

(ii)

sgfreepapers.com 166
0

3 (i) G
Graph to be translated
t 1 unit in neggative y- direction
 y  f  x 1  y 1  f  x
R
Replace y with
w y 1,
( y  1  1) 2  x x 2  1
y 2  x x2 1
(ii) V
Volume obtaained
a
   x x 2  1 dx
1

a 2 1
u
  xu   du
 x
0 u  x2 1
2
a 1
du x
 0
u 2 du 
dx u
a 2 1 dx u
 u3  
   du x
 3 0
 3

3
a 2
 1 2

4(a) L
Let x, y andd z be the cost of a dayy pass for a senior, ad
dult and
chhild respecttively.

2 x  19 y  9 z  196.4
10 y  3 z  90.2
x  7 y  4 z  77

U
Using GC,

sgfreepapers.com 167
x  3.60
y  7.40
z  5.40

Thus, the cost of a day pass for a senior is $3.60, for an adult is
$7.40 and for a child is $5.40.

(b) 4x2  4 x  1
0
x2  2 x  8
Let y  x
4 y2  4 y 1
0
y2  2 y  8
 2 y  1  0
2

 y  2  y  4 

Since  2 y  1  0,  2 y  1  0 satisfy the inequality


2 2

1
y
2
1
x 
2
1 1
x  or x  
2 2

 y  2  y  4  0
y4
y  2
or x 4
x  2
x4 or x  4
(no solution)

Answer: x  4 or x4

Alternatively(Method 2),

2
4 x  4 x 1
2
0
x  2 x 8

sgfreepapers.com 168
When x  0 ,
4x2  4x  1
 0 and x  0
x2  2x  8
 2 x  1  0
2
+ -2 - -1/2- 0 - 4 +
 x  2  x  4 
x  2 or x  4 and x  0
1
x or x  4
2 -2 -1/2 0 4

Or when x  0 ,
4x2  4x  1
 0 and x  0
x2  2x  8
 2 x  1  0
2

 x  2  x  4 
x  2 or x  4 and x  0
1
x or x  4
2

1 1
Answer: x  or x   or x  4 or x4
2 2

Alternatively( Method 3),

2
4 x  4 x 1
2
0
x  2 x 8

 2 x  1  0
2

 x  2  x  4 
1 1
 2 x  1
2
 0 satisfy the inequality  x  x
2 2
 x  2   0 for all values of x,
 x  4   0  x  4  x  4 or x  4
1 1
Answer: x  or x   or x  4 or x4
2 2

5 2
 y 5y
side of triangle     y 2 
2 2

sgfreepapers.com 169
 y 5 
 x  x  2  y
  2   2   5   200
  
10 8  3
 
 
x 5y  y
   120
5 10 16
x 5y  y
 120  
5 10 16
5 y 5 y
x  600  
2 16

Cross sectional area, W


2
 y 1 1
       xy  y 2
 2 2 2
y 2
y 2
 5 y 5 y 
   y  600   
8 2  2 16 
 y2 y2 5 y 2 5 y 2
   600 y  
8 2 2 16
3 y 2 y 2 5 y2
 600 y   
16 2 2

For maximum W,
dW
0
dy
3 y
600   y  5y  0
8
 3 
y  1  5   60
 8 
y  248.533  249
x  78.1347  78

d 2W 3
2
  1  5  2.414
dy 8

y  249 and x  78 will result in a maximum cross sectional


area.

sgfreepapers.com 170
6(i) y 3  tan 1 y  ln(cos x)
Differentiating both sides w.r.t x,
dy 1 dy  sin x
3 y2  
dx 1  y 2 dx cos x
dy
dx
 
3 y 2 1  y 2   1   1  y 2  tan x

 3 y 4  3 y 2  1 ddyx   1  y 2  tan x (shown)


(ii) Differentiating both sides w.r.t x,
2
 dy  d2 y
 
d
  x
12 y 3
 6 y  
d x 2 
3 y 4  3 y 2  1

 dy 
  1  y 2  sec 2 x   2 y  tan x
 dx 

When x  0,
dy d2 y
y  0,  0,  1
dx dx 2
0 1 1
 y  0  x  x2     x2  
1! 2! 2

(iii) π
π dy  x2  4
 dx    
4
0 dx  2 0

 4
2
π
 0
2
π2
 or  0.03125 2
32

7 (i) Due to the fishermen catching the fishes,


dx 1

dt x
dx k
 ,where k is a positive constant
dt x

Due to the aggressive nature of the fishes,


dx
 0.1x
dt

Rate of change of fishes,

sgfreepapers.com 171
dx k
   0.1x
dt x
k  0.1x 2

x
x
 k  0.1x 2 dx   1 dt
1 0.2 x

0.2 k  0.1x 2
dx   1 dt

1
ln k  0.1x 2  t  c
0.2
ln k  0.1x 2  0.2t  c1
k  0.1x 2  e 0.2t  c1
k  0.1x 2  ec1 e 0.2t
x 2  10k  10ec1 e 0.2t
x 2  10k  Ae 0.2t

Alternatively,
x
 k  0.1x 2 dx   1 dt
1 0.2 x

0.2 k  0.1x 2
dx   1 dt

1
ln  k  0.1x 2   t  c since k  0.1x 2  0
0.2
ln  k  0.1x 2   0.2t  c1
k  0.1x 2  e0.2t  c1
x 2  10k  Ae 0.2t

(ii) When t  0 , x  5
25  10k  A

When t  5 , x  3
9  10k  Ae1

Solving,
A  25.3116 and k  0.0311627

When x  0 ,
t  21.986

Number of days required = 22

sgfreepapers.com 172
8(a) No. of five-digit numbers greater than 10000
(i)  5 5 4  3 2
 600

Alternatively,
No restrictions – case where 0 is the first digit
6
P5  5 P4  600
(ii) Method 1
Case 1: First digit is 1 or 3 or 5 (odd)
3  4  3  2  3  216

Case 2: First digit is 2 or 4 (even)


2  4  3  2  2  96

No. of five-digit numbers greater than 10000 and even


 216  96
 312
Method 2
Case 1: Last digit is 2 or 4
4  4  3  2  2  192

Case 2: Last digit is 0


5  4  3  2  1  120

No. of five-digit numbers greater than 10000 and even


 192  120
 312

Method 3
No. of five digit numbers greater than 10000 – No. of five digit
numbers greater than 10000 that are odd
 600  4  4  3  2  3
 312

(b) Total number of selections


  24  1   23  1  105

Alternatively,
Method 2: Listing 12 Cases
C
4
1 C2  4C3  4C4    3C1  3C2  3C3   105
4

Method 3: Complement

sgfreepapers.com 173
No restrictions – 0 flavors or 0 toppings
 24  23   23  24  1
 128  23
 105

9(i)

Any horizontal line, y  k , k  2 cuts the graph of y  f ( x ) at


most once.
(ii) Let y  f ( x)  x 2  6 x  11
y  ( x  3) 2  2
( x  3) 2  y  2
x  3 y  2
Since x  3 , x  3  y  2
 f 1 ( x)  3  x  2, x  2

Df 1  Rf   2,  
(iii) For gf to exist, Rf  Dg i.e.  2,     k ,  
 greatest value of k  2

Df Rf Rgf
 
 3,   2,   0, 1 4 
 1
Rgf   0, 
 4
Alternative method,
gf  x   g  x 2  6 x  11
1
 ,x 3
 x 2  6 x  11
2

sgfreepapers.com 174
y  1
 3, 
 4
x
0 y  gf  x 

 1
Rgf   0, 
 4
(b) Since h is a one-one function, h 1 exists.

Since Rh 1  Dh , the rule for composite function, Rh 1  Dh is


fulfilled. Therefore hh 1 exists.

10 un  Sn  Sn 1
(a)(i) 1
 n n
2 1
 n1  n1
2

 ln a n b 2  ln a n 1b 2

 ln ab 2
1
 n  n  n 3n  2  
2 2

 ln ab n 1
(ii) un  un 1  ln ab n 1  ln ab n 11
 ln b
Since ln b is a constant, the sequence is an AP.
(iii) For n  7 , 0  ab n 1  1  ln ab n 1  0
Therefore, sum of negative terms is S6  ln a 6b 2
1
 6 6  ln a 6b15
2

(b) Using factor formula,

 1  1   1   1  
sin  n  sin      cos  n      cos  n     
2  2   2   2 
1   1   1  
sin  n    cos  n      cos  n     
1   2   2 
2sin    
 2 

10

sgfreepapers.com 175
N
1 N
  1   1  
 sin  n   1 
  cos  n      cos  n     
 2   2  
n 1
2sin    n 1 
2 
 3  
 cos  cos 
 2 2 
 5 3 
  cos  cos 
 2 2 
 7 5 
  cos  cos 
1  2 2 

 1    
2sin     
2  
 1   3 
  cos  N      cos  N     
  2   2  
 
  1   1  
  cos  N  2     cos  N  2    
     

1   1  
  cos  N      cos 
1   2  2
2sin    
2 
  1 
cos cos  N    
2   2 

1  1 
2sin    2sin   
2  2 
 1 
cos  N    
1 1   2 
 cot     (shown)
2 2  1 
2sin   
2 

11

sgfreepapers.com 176
11 P  A  B  =11  P  A  B   1  0.9  0.1
(a)(i)
P  A =P  A  B   P  A  B   0.445  0.1  0.5
55

(ii) P  A ' B   P  B   P  A  B 
 0.3  0.1
 0.2
(b)

(i) P  passes thee practical test


t 

 0.05  0.6  0.95  0.85


5  0.8375
(ii) P  passes thee theory testt | he fails thhe practical test 
0.95  0.155

1  0.83755
 0.877

12
(i)

12

sgfreepapers.com 177
(ii) dy dx
 2t ,  et
dt dt
dy
 2te  t
dx
At point A, t  2,
A
1 e 2
ggradient of nnormal  
2  2  e 2 4
E
Equation of linel l,
2
e
y  4    x  e2 
4
e 2
116  e 4
y  x
4 4
(iii)

R
Required areea
 area of triangle + areaa under curvve C
t 2
1  16  e 4 2 dx
   2
4  e   y dt

2  e  t  0 dt
2
 16 
 2  2    t 2 e t dt
e  0
32  2 t 2 
2
 2    t e   2  t e t dt 
 
e  0
0 
32  t 2 2 t 
 2  4e  2  te    e dt 
2

e 
0
0 

32
e 2 
 4e 2  2 2e 2   et 
2

0 
32
 2  4e 2  4e 2  2e 2  2
e
32
 2  2e 2  2
e

13

sgfreepapers.com 178
13(i) y

8.05

6.82
x
0 7.7 8.3

(ii) The scatter diagram shows y is increasing at a decreasing rate


and hence a linear model is not appropriate.
(iii) (a) r  0.9358

(b) r  0.9464
(iv) Since the product moment correlation coefficient between x
and y 2 is closer to 1 compared to that between x 2 and y and y
increases as x increases but at a decreasing rate, hence
y 2  c  dx is the better model.
(v) Using the GC,
y 2  176.23  29.347 x

When y  8.00 , x  8.2

From (iii), r  0.9464 is close to 1. Since y  8.00 is within


the data range of y and x is the independent variable, hence
the answer is reliable.

14

sgfreepapers.com 179
TEMASEK JUNIOR COLLEGE, SINGAPORE
JC One
Promotion Examination 2013
Higher 2

MATHEMATICS 9740

Solutions

TJC/MA9740/JC1Promo2013

sgfreepapers.com 180
2

1 Find the general solution of the following differential equation

1 dy 1
  0, where x  1. [4]
1  x dx 1  x 2
Solution:
 1  dy 1
   0
 1  x  dx 1  x
2

dy 1 x

dx 1  x2
dy 1 x
 
dx 1  x 1  x2
2

1 1 2x
y   dx   dx
1 x 2
2 1  x2
  tan1 x  ln 1  x2   c
1
2
1
(or  tan1 x  ln 1  x2  c )
2

TJC/MA9740/JC1Promo2013

sgfreepapers.com 181
3

2 (i) The first three terms of a sequence are given by u1  19 , u2  34 , u3  52 . Given


that un is a quadratic polynomial in n, find un in terms of n. [4]

(ii) Find the smallest value of n for which un is greater than 200. [2]

Solution:
(i) Let un  an 2  bn  c where a, b, c are constants.

When n  1 , a  b  c  19 ----- (1)


When n  2 , 4 a  2b  c  34 ----- (2)
When n  3 , 9 a  3b  c  52 ----- (3)

Using GC to solve the system of equations, we get


3 21
a , b , c7
2 2
3 21
 un  n 2  n  7
2 2

(ii)
Method I: For un  200 ,
3 2 21
n  n  7  200
2 2
 n  15.4 or n  8.37 (3sf)

 the smallest value of n is 9.

Method II:

For un  200 ,
By GC
U 8  187  200
U 9  223  200

 The smallest value of n is 9.

TJC/MA9740/JC1Promo2013

sgfreepapers.com 182
4

3 A wire of length L cm is cut into two pieces. One piece is used to form a circle while
the other piece is used to form an equilateral triangle.
Show that, with the total area of the circle and triangle being the smallest, the
3
proportion of the length of the smaller piece to the length of the bigger piece is .
9
[6]
Solution:
Let one of the pieces be x cm and use it for form the circle.
So the other piece is L-x and it’s used to for the equilateral triangle.

L-x x

x
For area of circle (radius r): 2 r  x  r 
2
2
 x  x2
Therefore area is    
 2  4

For area of equilateral triangle:

1 Lx  
2
3
 sin     L  x
2
Area = 
2 3   3  36
 L  x
2
x2 3 3 2
  L  x 
2
Hence total area, A = [the other form x also accepted]
4 36 4 36
dA x 3
   L  x
dx 2 18

TJC/MA9740/JC1Promo2013

sgfreepapers.com 183
5

Method I:
dA
For max/min,  0,
dx
1x 2 3 x 3 x 3
   L  x  0    L  x    1
2 36 2 18  L  x 9

3
Hence the ratio of the length of the smaller piece to the length of the bigger piece is
9
(shown)

d2 A 1 3
And    0  A is minimum.
dx 2
2 18

Method II:
dA
For max/min,  0,
dx
2x 2 3 x 3
   L  x   0    L  x   0 --- (*)
4 36 2 18
 1 3 3 3 L
 x     L x
 2 18  18 9  3

d2 A 1 3 3 L
   0  A is minimum at x 
dx 2
2 18 9  3

x 3 x 3 x 3
From (*)  ( L  x)  0   ( L  x)   ( 1)
2 18 2 18 Lx 9

3
Hence the ratio of the length of the smaller piece to the length of the bigger piece is
9
(shown)

TJC/MA9740/JC1Promo2013

sgfreepapers.com 184
6

4
y
y   x  1 , x  1
2

y  4x  4
4

y  x 1
2
R

x
1 1 3

The shaded region R in the diagram above is bounded by the y-axis, the line y   x  1
and the curves y   x  1 for x  1 and y  4 x  4 .
2

Find the volume of the solid of revolution formed when R is rotated completely about
the y-axis. [6]

Solution:

2
 y2  4 
 
 2
4 4


2
Required volume =  1  y dy    dy  1 1
0 2  4  3
 17.26666709  54.24483447  54.2 unit2

TJC/MA9740/JC1Promo2013

sgfreepapers.com 185
7

5 Given that y  ln  2  tan 1 x  , show that


2

1  x  ddxy2  2 x ddyx  1  x2   ddyx   0 .


2
2
[3]

Hence find the Maclaurin's expansion for y, up to and including the term in x2 . [3]

Solution:
y  ln  2  tan 1 x   e y  2  tan 1 x

Differentiate wrt x
dy dy
 1  x 2   e y    (1)
1
 ey 
dx 1  x 2
dx
Differentiate (1) wrt x
2
d2y dy dy  dy 
 1  x  2  2 x
2
 e  y   1  x 2    [From (1)]
dx dx dx  dx 
2
d2 y dy  dy 
 1  x  2  2 x  1  x 2     0
2

dx dx  dx 
dy 1 d 2 y 1
When x  0, y  ln 2,  , 2 
dx 2 dx 4
1  1
   
 y  ln 2   2
x
4 2 1 1
x    ln 2  x  x 2
1! 2! 2 8

TJC/MA9740/JC1Promo2013

sgfreepapers.com 186
8

n
r 9 1 3 n
6 Prove by mathematical induction 3
r 1
r 1
     for all positive integers
4 3n1  4 2 
of n. [5]
Hence show that
1 2 3 4 9
 2  3  4 . [2]
4 4 4 4 16

Solution:

n
r 9 1 3 n
Let P(n) be the statement 3 r 1
r 1
     for n  1, 2, 3, 4,
4 3n1  4 2 
1
r 9 3 1
When n = 1, LHS =  r 1 = 1 ; RHS =     1
r 1 3 4 4 2
So P(1) is true.
k
r 9 1 3 k
Assume P(k) is true for some k    , i.e.  r 1   k 1   
r 1 3 4 3 4 2
k 1
r 9 1  3 k 1 
To show P(k + 1) is true i.e.  r 1   k   
r 1 3 4 3 4 2 
k 1
r k
r k 1
LHS =  r 1 =  r 1 +
r 1 3 r 1 3 3k
 9 1  3 k  k  1
=   k 1      k
 4 3  4 2  3
9 1  9 3k 
=  k    k  1
4 3 4 2 

9 1  9 3k  2k  2  9 1  9 k  2  9 1  5 k 
=  k  =  k   =    
4 3 4 2  4 3 4 2  4 3k  4 2 

9 1  3 k 1 
=  k    = RHS
4 3 4 2 

So P ( k+1) is true.
Since P(1) is true, and P(k) is true  P(k + 1) is true.
n
r 9 1 3 n
 By mathematical induction, P(n) is true for all n    , ie. 3
r 1
r 1
    
4 3n1  4 2 

n

r 9
Since  r 1 
r 1 3 4

1 2 3 r 1  r 1  r 9
Hence  2  3     r   r 1   r 1  (deduced)
4 4 4 r 1 4 4 r 1 4 4 r 1 3 16

TJC/MA9740/JC1Promo2013

sgfreepapers.com 187
9

7 Functions f and g are defined by


2x  2
f:x , for x  , x  1 ,
x2
g : x  2  x , for x  , x  2 .

(i) Given that f has an inverse, show that the composite function gf 1 exists. Find

gf 1 and state its range. [5]

(ii) Find the value(s) of x such that f  x   f 1


 x . [2]

Solution:
(i) R f 1 = Df =  ,1

D g   , 2

Since R f 1  D g , the composite function gf 1 exists. (Shown)

2x  2
Let y .
x2
 xy  2 y  2 x  2

 xy  2 x  2 y  2

 x  y  2  2 y  2

2y  2
 x
y2
2x  2
 f 1
( x)  .
x2

 2x  2 
gf 1  x  = g  
 x2  y

 2x  2   2  2
= 2   22  
 x2   x2 x2

Dgf 1 = Df 1  R f   0, 2 

2
So, gf 1: x   , x  , 0  x  2 x
x2

TJC/MA9740/JC1Promo2013

sgfreepapers.com 188
10

For range of gf 1 :
M1 - By mapping method
g

1 2 x

Thus, R gf 1 = 1,   .
y
M2 - By direct sketching method
Dgf 1  Df 1 = R f   0, 2 

Therefore R gf 1 = 1,  

1
x
0 2

y
(ii)

From the graph,


f  x   f 1  x 

 0  x 1

x
1

TJC/MA9740/JC1Promo2013

sgfreepapers.com 189
11

8 Prove that

  r  1 r  2  
ln    ln   r  1 r    2 ln   r  r  1   ln   r  1 r  2   . [2]
 r  r  1 

Hence, find in terms of n,

 1 4   25   3 6    n  1 n  2     n  n  3 
ln    ln    ln      ln    ln   ,
 23   3 4   45    n  n  1    n  1 n  2  

leaving your answer as a single logarithmic function. [5]

Solution:

(i) RHS  ln   r  1 r    2 ln   r  r  1   ln   r  1 r  2  


  r  1 r  r  1 r  2  
 ln  
  r 
2
 r  1
2

 
  r  1 r  2  
 ln    LHS
  r  r  1 

(ii)
 1 4   25   3 6    n  1 n  2     n  n  3 
ln    ln    ln      ln    ln  
 23   3 4   45    n  n  1    n  1 n  2  
n1 
  ln
 r 1 r  2   n1 ln r 1 r  2ln r r  1   ln r 1 r  2 
         
r 2  r  r  1  r2 
 ln 1 2   2ln  2  3  ln  3 4 
 ln  2  3  2ln  3 4   ln  4  5
 ln  3 4   2ln  4  5  ln  5 6 
 ln  4  5  2ln  5 6   ln  6  7 

 ln  n  2  n  1  2ln  n  1 n   ln  n  n  1
 ln  n  1 n   2ln  n  n  1  ln  n  1 n  2 
 ln  n  n  1  2ln  n  1 n  2   ln  n  2  n  3
 ln 1 2   ln  2  3  ln  n  1 n  2   ln  n  2  n  3 [

 2  n  2  n  3   n3 
 ln    ln  
 6  n  1 n  2    3  n  1 

TJC/MA9740/JC1Promo2013

sgfreepapers.com 190
12

9 Jessie wishes to take up a loan of $20,000 on the 1st day of the Year 2014. She intends
to pay an instalment of $300 on the 1st day of each month, beginning from February
2014. She sources out two banks, XYZ Bank and ABC Bank, which offer such loans.
The two banks have different ways of charging interest. XYZ Bank charges a monthly
interest of 0.5% on the outstanding amount owed at the end of each month, while ABC
Bank charges a fixed interest of $60 at the end of each month until the loan is repaid.

(a) If Jessie takes up the loan from XYZ Bank, show that the outstanding loan at the
end of February 2014 after the interest has been added will be $19899. [2]

Hence, find the number of months Jessie will take to repay her loan. [4]

(b) Which bank should Jessie take a loan from if she wishes to clear her loan as soon
as possible? Justify your answers. [3]

Solution:

kth Outstanding loan at the beginning of kth Outstanding loan at the


month month from 2014 end of kth month from
2014
1 20000 1.005  20000 
2 1.005  20000   300 1.0052  20000   300 1.005

3 1.0052  20000   300 1.005  300


 
 20000   300 1.005  300 1.005 
n 1 n2 n 3
n 1.005
   300 1.005   300 1.005   300
2

(a) Outstanding loan at the end of February 2014  1.0052  20000   300 1.005  $19899
[Shown]
Hence
Let 1.005n 1  20000   300 1.005     300 1.005   300  0
n2

 1.005n 1  20000   300 1  1.005   1.005     1.005  0


2 n2

 
11.005 n 1  1 
 1.005 n 1
 20000   300  0
 1.005  1 

 1.005n 1  20000   60000 1.005   1  0


n 1

 
 40000 1.005 
n 1
 60000
60000
ln
  n  1  40000  n  82.29558565
ln 1.005 
 Jessie will repay her loan on the 1 day of 83 month. Therefore, she will take 82
st rd

months to repay her loan.


TJC/MA9740/JC1Promo2013

sgfreepapers.com 191
13

(b)
Method I:
For Bank ABC,

kth Outstanding loan at the beginning of kth month Outstanding loan at


month from 2014 the end of kth month
from 2014
1 20000 20000  60
2 20000  60  300 20000  60  300  60
3 20000  60  300  60  300  20000  60  2   300  2 
 20000  240  2 

 
n 20000  240  n  1

For 20000  240  n  1  0  n  84.33333


 Jessie will repay her loan on the 1 day of 85 month if she takes up bank ABC.
st th

Hence, she should take the loan from bank XYZ.

Method II:
When n = 83, 20000  240  83  1  320  0
 Jessie will not be able to clear her loan by the 83 month if she takes up bank ABC.
rd

Hence, she should take the loan from bank XYZ.

TJC/MA9740/JC1Promo2013

sgfreepapers.com 192
14

10 A curve C is given parametrically by the equations

x  2cos3  , y  2sin 3 
 
where    .
2 2

Show that the normal at the point with parameter  has equation
y sin   x cos   2  sin 4   cos 4   . [4]

The normal at the point Q where   , cuts C again at the point P, where   p .
6
Show that sin 3 p  3 cos3 p  1  0 and hence find the coordinates of P. [5]
Solution:
x  2cos3  , y  2sin 3 
dx dy
 3  2  cos 2    sin    3  2  sin 2  cos 
dt dt
 6sin  cos2   6sin 2  cos 

dy d y d t
     tan 
d x dt dx
 Gradient of normal to the curve  cot 

Eqn. of normal to the curve at  2 cos3  , 2sin 3   :


y  2sin 3  cos 

x  2cos3  sin 
 y sin   2sin 4   x cos   2 cos 4 
 y sin   x cos   2  sin 4   cos 4   (shown)

Eqn. of normal to the curve at Q , i.e.   :
6
 3    1 4  3 
4
1
y    x    2       
2  2    2   2  

 y  3x  2

When the normal to the curve at Q cuts C again at P, i.e.   p ,


2sin 3 p  3  2 cos3 p   2
 sin 3 p  3 cos3 p  1  0 (shown)
 p  0.7445633 or 0.52359878  rejected, pointQ 

 The coordinates of P is  0.795, 0.622  . (3sf)

TJC/MA9740/JC1Promo2013

sgfreepapers.com 193
15

11 A sequence of real numbers x1 , x 2 , x3 ,... satisfies the recurrence relation

2( xn2  xn )
xn1   1 , x1  k , where k  1 .
3

(a) When k = 5, state the value of x 9 and describe the behavior of the sequence. [2]

(b) Prove algebraically that, if the sequence converges, then it converges to


either 1 or 3. [3]

(c) State a value of k such that the sequence converges to 1. [1]

(d) When k  2 , state the integer m such that m  xn  m  1 for all integers n  1 . [1]
x 1
Hence, by considering n 1 , show that x n 1  x n for all integers n  1 . [3]
xn  1
Solution:
(a) x9  3.44
The sequence converges to 3 decreasingly.

(b) If the sequence converges to l. So when n   , xn1  l and xn  l .


Solving, we have
2(l 2  l )
l  1  3(l  1)2  2l 2  2l  l 2  4l  3  0  l  1 or l  3 .
3
Hence, if the sequence converges, then it converges to either 1 or 3. [Proven]

(c) The sequence converges to 1 when k = 1

(d) From GC, m = 2.

Method I:

xn 1  1 3
 xn  xn 
2 2
2 xn 2 1
   1
xn  1 xn  1 3  xn  1 3 xn  1
1 1
2  xn  3   .
xn  1 2
2 1 2 3
 1  1
3 xn  1 3 2
x 1
 n 1  1  xn 1  xn
xn  1

xn 1  1 3
 xn  xn 
2 2
2 xn
Or  
xn  1 xn  1 3  xn  1

2 xn 2 xn
Now xn  3  2 xn  3  3 xn   1  xn  1  1
3  3 xn 3  xn  1

TJC/MA9740/JC1Promo2013

sgfreepapers.com 194
16

Method II:

xn1  1 3
 xn  xn 
2 2
2 xn
 
xn  1 xn  1 3  xn  1
2x
From the graph of y  , when 2  x  3, y  1
3  x  1

1
x
0 2 3

Since 2  xn  3
xn 1  1 2 xn
  1  xn1  xn
xn  1 3  xn  1

TJC/MA9740/JC1Promo2013

sgfreepapers.com 195
17


1  1 
12 (a) Find ln   dx , leaving your answer in exact form. [4]
1 x2  x2 

t
(b) Using the substitution u  t , find  t  1 dt . [6]

Solution:
(a) Method I (simplify using Laws of Log before integration):


e
1  1 
ln   dx
1 x2  x2 


e
 2 x 2 ln x dx
1
 1 

e
e
 
 2    x ln x    x 1 dx 
1

 1
1 x 
 

e
 2   e1  0   x 2 dx 
 1 


 2 e1    x 1  
e

1

 
 2 e 1   e 1  1  2 2e 1  1  
 4e1  2

Method II (apply By Parts formula without simplification):


e
1  1 
ln   dx
1 x2  x2 
 
1 1  2 

e


e
 1  1 
   ln  2          dx
 x  x  1 1 x  1   x3 
 
 x2 


e
 1  1  2
   ln  2   ln1  dx
 e e   1 x2


e
 1  2
    2   0   dx
 e  1 x2
e
2  2
  
e  x 1
2  2 
    2
e  e 
4
 2
e

TJC/MA9740/JC1Promo2013

sgfreepapers.com 196
18

(b) u t  t  u2
dt
Diff. wrt u,  2u
du
t
 t  1 dt
u u2
  2  2u  du  2 2 du
u 1 u 1
 1 
 2 1  2  du
 u 1 
 1 u 1 
 2 u  ln C
 2 u  1 
t 1
 2 t  ln C
t 1

TJC/MA9740/JC1Promo2013

sgfreepapers.com 197
19

k 2 1
13 It is given that f  x    x  1  where k  1 .
x 1

(i) Show by differentiation that the graph of y  f  x  has no turning points. [3]

(ii) On separate diagrams, draw sketches of the graphs of

(a) y  f  x , [4]

(b) y  f ' x . [2]

You should indicate where possible, numerically or in terms of k, any asymptotes


and axial intercepts for each of the curves.

(iii) Find in terms of k, the range of x that satisfies the inequality


k f ( x)   x  k   x  k 
2
[4]
Solution:
k 2 1 k 2 1
(i) f  x  x 1   f   x   1 
x 1  x  12
Since k  1,  k 2  1  0
k 2 1
Since  x  1 is also always > 0, 1 
2
0
 x  12
 f '( x )  0 for all x  
 y  f  x  has no turning points.

Hence y  f  x  has no turning point.

TJC/MA9740/JC1Promo2013

sgfreepapers.com 198
20

k 2 1
(ii)(a) When x = 0, y  1   k 2
1
k 1
2
When y = 0, x 1 0
x 1
k 2  1   x  1 x  1
k 2 1  x2 1
x  k
y

(ii)(b)
y

TJC/MA9740/JC1Promo2013

sgfreepapers.com 199
21

(iii)
Methodd 1:
k f ( x)   x  k   x  k 
2

 f ( x) 
 xk x  k 
2

Sketcch the curvees y  f ( x) and y


 x k x k
2

k
Case 1:
y

x
k α k

 k2 y  f ( x)

To findd α and , seet

k 2 1  x  k   x  k 
2
x 1 
x 1 k


x  k
2 2

x k x  k
2

x 1 k
 x  k  1 
  x  k  x  k    0
 k x  1
 x 2   k  1 x  2k 
  x  k  x  k   0
 k  x  1 
 x   k or o x   k  1 x  2k  0
2

 k  1   k  12  8k
 x   k or
o x
2

 
 k  1  k  6k  1
2
nd  
an
 k  1  k 2  6k  1
2 2

 k  x  1 or
 k  1 
k 2  6k  1
x
 k  1  k 2  6k  1 or x  k
2 2
This case is valid if k  6k  1  0 , i.e.  k  3  8  0 , i.e. k  3  2 2 (ssince k > 1)
2 2

TJC/MA97
740/JC1Promo
o2013

sgfreepapers.com 200
22

Case 2 ( 1  k  3  2 2 ):
y y
 x  k 2  x  k 
x 1 k
y  x 1

x
k k

k2

From the diagram, we have


k  x  1 or x  k .

TJC/MA9740/JC1Promo2013

sgfreepapers.com 201
23

Method 2:
 k 2 1 
  x k x  k
2
k   x 1 
 x 1 
  x2  k 2 
  x k x  k
2
k
 x 1 
k
 x  k  x  k   
 x  k   0
 x 1 

 x 2   k  1  2k 
 x  k  x  k     0
 x  1 
 
 x  k  x  k  x  1 x   k  1  2k  0 ,
2
x 1

 
Case 1( x 2   k  1  2k can be factorized, i.e. when  k  1  4 1 2k   0 ,
2

i.e. k 2  6k  1  0 ,
6  36  4
i.e. k  ,
2
i.e. k  3  2 2 )

We have
   k  1  k  1 2  8k   
        x     k  1   k  1  8k
2
  0
 x  k  x  k  x  1  x        
  2 2
    

 k  x  1 or
 k  1  k 2  6k  1
x
 k  1  k 2  6k  1
or x  k
2 2

Case 2 ( 1  k  3  2 2 )

Since x 2   k  1  2k  0 ,
  x  k  x  k  x  1  0
k  x  1 or x  k

TJC/MA9740/JC1Promo2013

sgfreepapers.com 202
TEMASEK JUNIOR COLLEGE, SINGAPORE
JC One
Promotion Examination 2013
Higher 2

MATHEMATICS 9740
4 October 2013
Additional Materials: Answer paper 3 hours
List of Formulae (MF15)

READ THESE INSTRUCTIONS FIRST

Write your Civics Group and Name on all the work that you hand in.
Write in dark blue or black pen on both sides of the paper.
You may use a soft pencil for any diagrams or graphs.
Do not use staples, paper clips, highlighters, glue or correction fluid.

Answer all the questions.


Give non-exact numerical answers correct to 3 significant figures, or 1 decimal place in the case of
angles in degrees, unless a different level of accuracy is specified in the question.

You are expected to use a graphic calculator.

Unsupported answers from a graphic calculator are allowed unless a question specifically states
otherwise.

Where unsupported answers from a graphic calculator are not allowed in a question, you are required to
present the mathematical steps using mathematical notations and not calculator commands.

You are reminded of the need for clear presentation in your answers.

The number of marks is given in brackets [ ] at the end of each question or part question.

At the end of the examination, fasten all your work securely together.

This document consists of 5 printed pages.

© TJC 2013 [ Turn over

TJC/MA9740/JC1Promo2013

sgfreepapers.com 203
2

1 Find the general solution of the following differential equation

1 dy 1
  0, where x  1. [4]
1  x dx 1  x 2

2 (i) The first three terms of a sequence are given by u1  19 , u2  34 , u3  52 . Given


that un is a quadratic polynomial in n, find un in terms of n. [4]

(ii) Find the smallest value of n for which un is greater than 200. [2]

3 A wire of length L cm is cut into two pieces. One piece is used to form a circle while
the other piece is used to form an equilateral triangle.
Show that, with the total area of the circle and triangle being the smallest, the ratio of
3
the length of the smaller piece to the length of the bigger piece is .
9
[6]

4
y
y   x  1 , x  1
2

y  4x  4
4

y  x 1
2
R

x
1 1 3

The shaded region R in the diagram above is bounded by the y-axis, the line
y   x  1 and the curves y   x  1 for x  1 and y  4 x  4 .
2

Find the volume of the solid of revolution formed when R is rotated completely about
the y-axis. [6]

TJC/MA9740/JC1Promo2013

sgfreepapers.com 204
3

5 Given that y  ln  2  tan 1 x  , show that


2

1  x  ddxy2  2 x ddyx  1  x2   ddyx   0 .


2
2
[3]

Hence find the Maclaurin's expansion for y, up to and including the term in x2 . [3]

n
r 9 1 3 n
6 Prove by mathematical induction 3
r 1
r 1
     for all positive integers
4 3n1  4 2 
of n. [5]
Hence show that
1 2 3 4 9
 2  3  4 . [2]
4 4 4 4 16

7 Functions f and g are defined by


2x  2
f:x , for x  , x  1 ,
x2
g : x  2  x , for x  , x  2 .

(i) Given that f has an inverse, show that the composite function gf 1 exists. Find

gf 1 and state its range. [5]

(ii) Find the value(s) of x such that f  x   f 1


 x . [2]

8 Prove that

  r  1 r  2  
ln    ln   r  1 r    2 ln   r  r  1   ln   r  1 r  2   . [2]
 r  r  1 

Hence, find in terms of n,

 1 4   25   3 6    n  1 n  2     n  n  3 
ln    ln    ln      ln    ln   ,
 23   3 4   45    n  n  1    n  1 n  2  

leaving your answer as a single logarithmic function. [5]

TJC/MA9740/JC1Promo2013

sgfreepapers.com 205
4

9 Jessie wishes to take up a loan of $20,000 on the 1st day of the Year 2014. She intends
to pay an instalment of $300 on the 1st day of each month, beginning from February
2014. She sources out two banks, XYZ Bank and ABC Bank, which offer such loans.
The two banks have different ways of charging interest. XYZ Bank charges a monthly
interest of 0.5% on the outstanding amount owed at the end of each month, while ABC
Bank charges a fixed interest of $60 at the end of each month until the loan is repaid.

(a) If Jessie takes up the loan from XYZ Bank, show that the outstanding loan at the
end of February 2014 after the interest has been added will be $19899. [2]

Hence, find the number of months Jessie will take to repay her loan. [4]

(b) Which bank should Jessie take a loan from if she wishes to clear her loan as soon
as possible? Justify your answers. [3]

10 A curve C is given parametrically by the equations

x  2cos3  , y  2sin 3 

 
where    .
2 2

Show that the normal at the point with parameter  has equation

y sin   x cos   2  sin 4   cos 4   . [4]


The normal at the point Q where   , cuts C again at the point P, where   p .
6
Show that sin 3 p  3 cos3 p  1  0 and hence find the coordinates of P. [5]

11 A sequence of real numbers x1 , x 2 , x3 ,... satisfies the recurrence relation

2( xn2  xn )
xn1   1 , x1  k , where k  1 .
3

(a) When k = 5, state the value of x 9 and describe the behavior of the sequence. [2]

(b) Prove algebraically that, if the sequence converges, then it converges to


either 1 or 3. [3]

(c) State a value of k such that the sequence converges to 1. [1]

(d) When k  2 , state the integer m such that m  xn  m  1 for all integers n  1 . [1]
x 1
Hence, by considering n 1 , show that x n 1  x n for all integers n  1 . [3]
xn  1
TJC/MA9740/JC1Promo2013

sgfreepapers.com 206
5


e
1  1 
12 (a) Find ln   dx , leaving your answer in exact form. [4]
1 x2  x2 

t
(b) Using the substitution u  t , find  t  1 dt . [6]

k 2 1
13 It is given that f  x    x  1  where k  1 .
x 1

(i) Show by differentiation that the graph of y  f  x  has no turning points. [3]

(ii) On separate diagrams, draw sketches of the graphs of

(a) y  f  x , [4]

(b) y  f ' x . [2]

You should indicate where possible, numerically or in terms of k, any asymptotes


and axial intercepts for each of the curves.

(iii) Find in terms of k, the range of x that satisfies the inequality


k f ( x)   x  k   x  k 
2
[4]

***End of Paper***

TJC/MA9740/JC1Promo2013

sgfreepapers.com 207
VICTORIA JUNIOR COLLEGE
PROMOTIONAL EXAMINATION

MATHEMATICS 9740
(HIGHER 2)
Monday 8 am -11 am
23 September 2013 3 hours

Additional materials: Answer Paper


List of Formulae (MF15)

READ THESE INSTRUCTIONS FIRST

Write your name and CT group on all the work you hand in.
Write in dark blue or black pen on both sides of the paper.
You may use a soft pencil for any diagrams or graphs.
Do not use staples, paper clips, highlighters, glue or correction fluid.

Answer all the questions.


Give non-exact numerical answers correct to 3 significant figures, or 1 decimal place in the case of angles in
degrees, unless a different level of accuracy is specified in the question.
You are expected to use a graphic calculator.
Unsupported answers from a graphic calculator are allowed unless a question specifically states otherwise.
Where unsupported answers from a graphic calculator are not allowed, you are required to present the
mathematical steps using mathematical notations and not calculator commands.
You are reminded of the need for clear presentation in your answers.

At the end of the examination, fasten all your work securely together.
The number of marks is given in brackets [ ] at the end of each question or part question.

This document consists of 6 printed pages

© VJC 2013 VICTORIA JUNIOR COLLEGE [Turn over

sgfreepapers.com 208
1 A sequence with its first four terms given is shown below.

1, (1  2), (1  2  2 2 ), (1  2  22  23 ), 

Show that the n th term of this sequence is 2 n  1. [2]

Find the sum of the first n terms of the sequence. [3]

2 A sequence of positive real numbers x1 , x2 , x3 , ... satisfies the relation

3  xn
xn 1  for n  1.
2 xn  3
(i) Given that the sequence converges to  , find the exact value of  . [3]

(ii) By using a graphical approach, prove that

xn 1  xn if 0  xn   .
[2]

3 A curve is defined by the parametric equations

x  2 at 2 , y  3at ,

where a is a non–zero constant.

 17a 
Given that B is the point  ,0  , find the coordinates of the points on the curve which
 4 
are nearest to B. [5]

4 (i) Given that f  r    r  1 r 2 , show that f  r  1  f  r   r  3r  1 . [1]

N
(ii) Use the method of differences to find  r  3r  1
r 1
in terms of N. Hence find the
N
r  3r  1
limit of 
r 1 N3
as N approaches infinity. [3]

N
(iii) Use your first answer in part (ii) to find   r  1 3r  2 
r 3
in the form

aN 3  bN 2  cN  d , where a , b, c and d are constants to be found. [2]

sgfreepapers.com 209
d  x  2 1
5 (a) (i) Prove that  2  2  2 . [2]
dx  x  1   x  1 2
x 1


1
1
(ii) Find the exact value of dx. [3]
x 2  1
2
0

e2 x
 1 e
1 1
(b) Find the constant A such that  A . Hence find dx. [3]
1  e2 x 1  e2 x 2x

1 1
6 (i) Find the expansion of  in ascending powers of x, up to and
1  x2 (1  x) 2
2
including the term in x . [3]

1
Let y  sin 1 ( x)  .
(1  x)
(ii) By successively differentiating y, find the Maclaurin’s series for y, up to and
including the term in x3 . [4]

(iii) Show that the same result in part (i) can be obtained by using your answer in part
(ii). [2]

7 A sequence u0 , u1 , u2 , … is such that u0  b and un 1  run  a , for all n ൒ 0, where

a , b and r are constants.

(a) For the case where r  1 ,


1 rn
(i) prove by induction that un  r b  a for n  0,
n
[4]
1 r

(ii) write down the set of values of r for which the sequence u0 , u1 , u2 , . . .

converges, and state the limit of this sequence. [2]

N
(b) For the case where r  1 , find u1 , u2 , u3 , and hence find u
n 0
n in terms of a, b, N.

N 1
Give your answer in the form  k2b  Na  , where k1 and k2 are integers to be
k1
determined. [3]

[Turn over
3

sgfreepapers.com 210
8 y

x
O

x
The above diiagram show
ws a sketch of the curv quation y 
ve C with eq , x  0.
ex
(aa) (i) Finnd the exactt coordinate s of the max
ximum poin
nt on C. [3]

hat ln x  x – 1 for all x > 0.


(ii) Hennce show th [2]

b)
(b A partiicle is consttrained to m
move along C, starting from the orrigin O, such that its
x-coorddinate increeases at a coonstant rate. The particcle took 2 sseconds to reach
r the
 4  a
point  4, 4  . Wh
hen it is at thhe point  a, a  , the y-coordinat
y te of the paarticle is
 e   e 
decreassing at a raate of 0.25 uunit per seco
ond. Find a given that a < 2. [4]

9 (aa) m, S n 1 , off the first n  1 terms of


The sum o a sequencce u1 , u2 , u 3 , … is giv
ven by

S n 1  8n 2  19n  11 .
(i) Finnd un and sh
how that thee sequence is an arithm
metic progreession. [4]
(ii) Finnd the least value of n, such that sum of the first
fi n terms is at least 4000
4 less
thaan the sum of
o the next n terms. [3]

b)
(b

4m

A frog falls into a mud ddy drain wwith a slantt wall meassuring 4m iin length. Itt tries to
esscape from the drain by b leaping ssuccessively y on the slaant wall. T
Though it can cover
0..7 m in its first leap, the
t wall is so slippery y that for suubsequent aattempts it can only
coover 4/5 thee distance of
o its previoous leap. Determine if the frog wiill be able to
t escape
foorm the draiin, justifying your answwer. [3]

sgfreepapers.com 211
y
10 (i) y  f ( x)
 4, 1 
 
 3, 2   2
x
y0 O  3, 0 

x  2 x2

The diagram ows the grraph of y  f  x  . It has a non--stationary point of


m above sho
innflexion  0, 0  , an inteersection wiith the x-axis at  3, 0  , a minimum
m point  3,
3 2  and
 1
a maximum point  4,  . The veertical asym he graph aree x  2 an
mptotes of th nd x  2 .
 2
The horizonttal asymptotte is y  0 .

Skketch the grraph of y  f  2x  , m


making cleaar the main relevant
r feaatures and the shape
off the graph near the points where y  0 . [3]

((ii)

ws the graphh of y  g  x  . The inteersections oof the graph with the


Thhe diagram above show
axxes have cooordinates  0, 1 , 1, 0  and  3, 0  . The asymptotes of thee graph are the lines
x  2 and y   x  2 .

Skketch the grraph of y  g '  x  , makking clear th


he main releevant featurres. [3]

((iii) The funnction h is defined


d as
. g( x) for x  2,
h(xx)  
 f ( x) for x  2.
Skketch the grraphs of
(aa) y = h(x)), [1]
1
(bb) y  , making clear
c the maain relevant features. [4]
h( x)
urn over
[Tu
5

sgfreepapers.com 212
11 The function f is defined as follows.
4
f :x x for x  , x  0.
x
(i) Find f 1 ( x ) . [3]
(ii) Show that f ( x)  0 . [1]
(iii) Solve the inequality f 1 ( x)  6 , giving your answer in exact form. [2]
(iv) Sketch the graph of y  f 1f ( x ) . [1]

Functions h and g are defined by


4
h: x  x  for x   , x   2 , x  0 , x  2,
x
1
g : x  1 for x  , x  0.
x
( x 2  x  4)
(v) Show that gh( x)   . [1]
( x 2  4)
(vi) Solve the inequality gh( x )  0 , giving your answer in an exact form. [3]

 x  1
2
y2
12 The curve C1 has equation  4.
4 9
Sketch C1, making clear the main relevant features, and state the set of values that x can
take. [4]

Another curve C2 is defined by the parametric equations


2
x , y  3 t ln t , where t  1.
t 1
2

Use a non-graphical method to determine the set of possible values of x. [2]

Sketch the curve C2, labelling all axial intercepts and asymptotes (if any) clearly. [2]

Hence, without solving the equation, state the number of real roots to the equation
2
 2 
 
2
9 2  1  4 3 t ln t  144 ,
 t 1 

explaining your reason(s) clearly. [2]


Given that k > 0, state the smallest integer value of k such that the equation
2
 2 
 
2
9 2  k  1  4 3 t ln t  144
 t 1 
has exactly one real root which is positive. [2]

sgfreepapers.com 213
Victoria Junior College
Mathematics H2 (9740) – JC 1 Promotional Examination 2013
Solutions

1. (i) The nth term


 1  2  22  ...  2n 1

1  2n

1 2
 2n  1

Sn    2r  1   2r 1
n n n
 ii 
r 1 r 1 r 1

2(1  2n )
 n
1 2
 2n1  n  2

2. (i) As n   , xn   and xn1   .


3

2  3
2  4  3  0
2

4  16  24

4
  1  12 10

Since xn  0 for all n,   1  12 10 .

3 x 1 9
(ii) Sketch y    and y  x .
2x  3 2 2(2 x  3)
y
y=x

(0,1) 3 x
y
(3, 0) 2x  3
o
x
y
1 
2

3
x
2

sgfreepapers.com 214
3 x
2. When 0  x   , the graph of y  is above the graph of
2x  3
3 x
y  x.   x.
2x  3

3  xn
Hence for 0  xn   ,  xn
2 xn  3

 xn 1  xn .

3 (i) Let A be a point on the curve.


2
 17 a 
AB    2at 2    0  3at 
2 2

 4 
2
289a
  4a 2t 4  17 a 2t 2  9a 2t 2
16
289a 2
 4 a 2 t 4  8a 2 t 2 
16
289a 2
AB  4a 2t 4  8a 2t 2 
16

Let S  AB .

dS 16a 2t 3  16a 2t

dt 289a 2
2 4a 2t 4  8a 2t 2 
16

dS
Let  0 , then
dt

16a 2 t 3  16a 2t
0
289a 2
2 4 a t  8a t 
2 4 2 2

16

16a 2t 3  16a 2t  0  t (t 2  1)  0

 t  0 or t  1 or t  1
17 a
At t  0 , S  AB  .
4
15a
At t  1 , S  AB  (nearer)
4
Hence, substitute t  1 (which correspond to points nearest to B) into
x and y .
The coordinates are: (2a, 3a) and (2a, -3a).

sgfreepapers.com 215
4. (i)

f(r  1)  f(r )
 r (r  1)2  (r  1)r 2
 r (r  1)2  (r  1)r 
 r  r 2  2r  1  r 2  r 
 r (3r  1)
N
 ii   r(3r  1)
r 1
N
   f(r  1)  f(r) 
r 1

 f(2)  f(1) +
f(3)  f(2) +

f(N )  f(N  1) +
f(N  1)  f( N ) +
 f(N  1)  f(1)
 N ( N  1)2  0
 N ( N  1)2

r  3r  1 N ( N  1) 2  N  1  
2 2
N
1

r 1 N 3

N 3

 N  
  1   .
N

1 N
r  3r  1
As N   ,
N
 0.  the limit of 
r 1 N3
is 1.

N
 iii   ( r  1)(3r  2)
r 3

 2  7  3  10  ...  ( N  1)(3N  2)
N

 r(3r  1)
r 1

 1  4   2  7  ...  ( N  1)(3N  2)   N (3N  1)


N N
  ( r  1)(3r  2)   r (3r  1)  4  N (3N  1)
r 3 r 1

 N ( N  1) 2  4  N (3N  1)
 N 3  2 N 2  N  4  3N 2  N
 N3  N2  4

sgfreepapers.com 216
d  x  x 2  1  x (2 x )
5. (a)  i   
dx  x 2  1   x 2  1
2

1  x2

x  1
2 2

2  1  x2

x  1
2 2

2 1  x2
 
x  1 x  1
2 2 2 2

2 1
 
x  1 x 1
2 2 2

 1 

1 1
2  x 
 ii    2 dx   2 
  x 2  1 x  1  x  10
2
0
 


1
1 1 1
2 dx   tan 1 x  
x 2  1
2
0
0 2


1 
1
1
2 dx  
x 2  1
2
0 2 4


1 
1
1
dx  
x 2  1
2
0 4 8

e2 x
(b) RHS = A+
1  e2 x
A  Ae2 x  e 2 x

1  e2 x
Comparing the numerator to that of the LHS,

A  Ae2 x  e2 x  1
 A 1

1  e2 x 
 1  e2 x dx   1  1  e2 x  dx
1
 x  ln 1  e2 x  C
2

sgfreepapers.com 217
6 (i)

1 1 1
  (1  x 2 ) 2  (1  x) 2
1  x2 (1  x) 2

 1   (2)(3) 2 
 1  x 2     1  2 x  x  
 2   2! 
5
 2x  x2  
2
dy 1
 ii    (1  x ) 2
dx 1 x 2

d y  1
2
3
    (1  x 2 ) 2 ( 2 x )  2(1  x ) 3
 2
2
dx
3
 x (1  x 2 ) 2
 2(1  x ) 3
d3 y 3  3 5
 (1  x 2 ) 2  x    (1  x 2 ) 2 ( 2 x )  6(1  x ) 4
 2
3
dx

When x = 0,

y 1
dy
 1 1  0
dx
d2 y
 02  2
dx 2
d3 y
 1  0  6  5
dx3
5 3
Hence, y  1  x 2  x 
6

1 5
(iii) y  sin 1 ( x )   1  x2  x3  
(1  x ) 6

Differentiating both sides w.r.t x,


1 1 5
 = 2 x  x 2   (verified).
1 x 2 (1  x) 2
2

sgfreepapers.com 218
7. (a)(i)

1 rn
Let Pn be the statement: un  r nb  a for n  0.
1 r
Consider P0 :
L.H.S. of P0 = u0 = b

1 r0
R.H.S. of P0 = r 0b  a b
1 r

 P0 is true.

Assume Pk is true for some k  0.

1 rk
i.e. uk  r b  a
k
.
1 r

Consider Pk  1 :

1  r k 1
R.H.S. of Pk  1 = r k 1b  a
1 r

L.H.S. of Pk  1 = uk  1
 ruk  a
 1  rk 
 r  rkb  a a
 1 r 
k 1
ar 1  r k  a (1  r )
 r b 
1 r 1 r
k 1
ar  ar  a  ar
 r k 1b 
1 r
a 1  r k 1 
 r k 1b 
1 r

 Pk is true  Pk  1 is true.

 P0 is true
Hence, 
 Pk is true  Pk 1 is true.

1 rn
By induction, un  r nb  a for n  0.
1 r

sgfreepapers.com 219
7  ii  The sequence converges for r   : 1  r  1.
a
The limit of the sequence is .
1 r

(b)

u0  b
u1  b  a
u2  b  2a
u3  b  3a
 
uN  b  Na
N
N
  un   N  1 b   a  Na 
n 0 2
N
  N  1 b  1  N  a
2
N 1
=  2b  Na 
2

x
8(a)(i) y 
ex
dy e x  xe x

dx e2 x
1 x
 x
e
dy
 0  x 1
dx

 1
Substitute x  1 into y . Maximum point is 1,  .
 e

(ii) For x  0,
1 x 1
y  i.e. x 
e e e
Since ln is an increasing function,
 x
ln  x   ln  e 1 
e 
 ln x  ln e x  1
 ln x  x  1
 ln x  x  1

sgfreepapers.com 220
8(b) The particle took 2 seconds to move from x  0 to x  4 ,

dx
so  2.
dt

At x  a ,

dy dy dt
 
d x d t dx
1 1
 0.25  
2 8

 a  dy 1  a
At  a , a  ,  a
 e  dx e
1 a 1
 a  1 x
e 8 y
ex
From GC, a  1.65 (reject 2.45 as a  2 ).
y y

1 x
y
ex

x
o
1
y
8

9  a  i  Replacing n with n  1,
Sn  8(n  1) 2  19(n  1)  11
 8n 2  16n  8  19n  19  11
 8n 2  3n

un  Sn  Sn 1
  8n 2  3n   8n 2  19n  11
 16n  11
un  un 1  16n  11  16( n  1)  11
 16
Since the difference between 2 consecutive
terms is a constant, the sequence is an AP.

sgfreepapers.com 221
 ii   S2n  Sn   Sn  4000
8(2n) 2
 3(2n )   2  8n 2  3n   4000
32n 2  6n  16n 2  6n  4000
n 2  250
 n  15.8 (reject as n    ) or n  15.8
Thus, least n is 16.

(b) The distance covered by frog is a GP with a =0.7 and r = 0.8


Total distance covered after n leaps is given by

0.7 1  0.8n 
Sn 
1  0.8
 3.5 1  0.8n 
As n  , (0.8) n  0  Sn  3.5, that is, S   3.5
Since S  4, the frog will never be able to escape from the drain.

10 (i) y

1
(2, )
2
( 3 , 2)
2 y  f 2x
y=0 (0, 0) x
( 3 , 0)
2
x  1

(ii) y

x
o
y  1
y  g '( x )

x2

(iii) (a)

y x2
(0,1) 1
(4, )
2 y  h( x)
o (1,0) x
(3, 0) y0

y  x  2

sgfreepapers.com 222
10(b)

y
1
y
h( x)
(0,1) (4, 2)
(2, 0)
x
y0 o

x 1 x3
4 x2  4
11 (i) y  x  y
x x
x  xy  4  0
2

y  y 2  16
x
2
y  y 2  16
Since x  0 , x 
2
1 1 1 1 2
 f 1 ( y )  y  y 2  16  f 1 ( x )  x  x  16 .
2 2 2 2
4 4
(ii) f '( x )  1  2 . Since 2  0 for all real x<0, f '( x )  1
x x
Hence f '( x )  0.

(iii) Since f is an increasing function,


f 1 ( x )  6  f  f 1 ( x )   f ( 6)
4 16
x  6  x
6 3
y
(iv)

o x
yx
y  f 1f(x)

1
(v) gh( x )  g  h(x )   1
x 4 2

x
x x  ( x 2  4) ( x 2  x  4)
 2 1   
x 4 x2  4 x2  4

10

sgfreepapers.com 223
11(vi) Test Point method:

x2  x  4  0  x 
1
2

1  17 
     ( x 2  x  4)
Sign of 
2
1
(1  17) 2
1
(1  17)
x2  4
2 2
1 1
 2  x  (1  17 ) or 2  x  (1  17)
2 2
Alternatively, use graphs:
x  2
y x2

1  1 
 (1  17), 0   (1  17), 0 
2  2 
o
x
y  1 ( x 2  x  4)
y
‘ x2  4

1 1
 2  x  (1  17 ) or 2  x  (1  17)
2 2

12 y
3
y  ( x  1)
2

(1, 0)
o x
( 3, 0) (5, 0)
 x  1
2
y2
 4
4 9
3
y   ( x  1)
2
 x  1  x  1  y 2  1
2 2
y2
 4
4 9 42 62
 the set of values of x =  x   : x  3 or x  5
1 1
t 2  1  t2 1  2  0  
t 1 2
2

2
0  1, that is, 0  x  1
t 1 2

 the set of values of x =  x   : 0  x  1

11

sgfreepapers.com 224
12
x0
2
y x , y  3 t ln t , t  1.
t 1
2

(1, 0)
x
o

2
 2 
 
2
9 2  1  4 3 t ln t  144  (1)
 t 1 
2
 2 
 1
 
2
 2
 t  1   3 t ln t 4
4 9

 x  1
2
y2 2
Since C1 :   4 and C2 : x  2 , y  3 t ln t ,
4 9 t 1
the number of roots of the above equation can then be found by the
number of intersections between C1 and C2. However, since C1 is
only defined for x  3 or x  5 and C2 is defined for 0  x  1 ,
there is no point of intersection.
2
 2 
 
2
Hence 9  2  1  4 3 t ln t  144 has no real root.
 t 1 
2
 2 
 
2
9 2  k  1  4 3 t ln t  144
 t  1 

Since x is replaced with x + k in the equation of C1, C1 is


translated k units in the negative x-direction. Hence smallest
integer value of k is 5.
OR
Since x is replaced with x - k in the equation of C2, C2 is
translated k units in the positive x-direction. Hence smallest
integer value of k is 5.

12

sgfreepapers.com 225

You might also like